Инфоурок Математика Другие методич. материалыПрактические работы обучающихся по дисциплине Математика для специальности Технология машиностроения.

Практические работы обучающихся по дисциплине Математика для специальности Технология машиностроения.

Скачать материал

Выберите документ из архива для просмотра:

Выбранный для просмотра документ МЕТОД.УК. ПР-ТЕХМАШ,1 КУРС..docx












Методические указания

к выполнению практических работ обучающихся

по дисциплине Математика

для специальности 151901. Технология машиностроения.



























2015г.



Пояснительная записка


Цель методических указаний: оказание помощи обучающимся в выполнении практических работ по дисциплине Математика.

Настоящие методические указания содержат работы, которые позволят обучающимся самостоятельно овладеть профессиональными знаниями и умениями, опытом творческой деятельности при решении проблем учебного и профессионального уровня и направлены на формирование следующих компетенций:

ОК 1. Понимать сущность и социальную значимость своей будущей

профессии, проявлять к ней устойчивый интерес.

ОК 2. Организовывать собственную деятельность, выбирать типовые

методы и способы выполнения профессиональных задач, оценивать их

эффективность и качество.

ОК 3. Принимать решения в стандартных и нестандартных

ситуациях и нести за них ответственность.

ОК 4. Осуществлять поиск и использование информации,

необходимой для эффективного выполнения профессиональных задач,

профессионального и личностного развития.

ОК 5. Использовать информационно-коммуникационные технологии

в профессиональной деятельности.

ОК 6. Работать в коллективе и команде, эффективно общаться

с коллегами, руководством, потребителями.

ОК 7. Брать на себя ответственность за работу членов команды

(подчиненных), результат выполнения заданий.

ОК 8. Самостоятельно определять задачи профессионального и

личностного развития, заниматься самообразованием, осознанно

планировать повышение квалификации.

ОК 9. Ориентироваться в условиях частой смены технологий

в профессиональной деятельности.

ОК 10. Исполнять воинскую обязанность, в том числе с применением

полученных профессиональных знаний (для юношей).

В результате выполнения практических работ по дисциплине Математика обучающиеся должны:

уметь:

    • Выполнять арифметические действия, сочетая устные и письменные приемы, применение вычислительных устройств;

    • Решать уравнения и неравенства (линейные, квадратные);

    • Вычислять значения числовых и буквенных выражений, осуществляя необходимые подстановки и преобразования;

    • Находить значения корня натуральной степени, степени с рациональным показателем, логарифма, используя при необходимости вычислительные устройства;

    • Проводить по формулам и правилам преобразования буквенных выражений, включающих степени, радикалы, логарифмы, тригонометрические функции;

    • Строить графики изученных функций;

    • Решать показательные и логарифмические уравнения и неравенства; простейшие иррациональные и тригонометрические уравнения;

    • Вычислять производные и первообразные элементарных функций, используя справочные материалы;

    • Исследовать в простейших случаях функции на монотонность, находить наибольшие и наименьшие значения функций; строить графики многочленов с использованием аппарата математического анализа;

    • Распознавать на чертежах и моделях пространственные формы; соотносить объекты с их описаниями, изображениями;

    • Описывать взаимное расположение прямых и плоскостей в пространстве ;

    • Изображать основные многогранники и круглые тела;

    • Решать планиметрические и простейшие стереометрические задачи на нахождение геометрических величин (длин, углов, площадей, объемов);

    • Использовать при решении стереометрических задач планиметрические факты и методы;

    • Решать простейшие комбинаторные задачи методом перебора, а также с использованием формул;

    • Вычислять в простейших случаях вероятности событий на основе подсчета числа исходов;

    • Использовать приобретенные знания и умения для анализа числовых данных, представленных в виде таблиц, диаграмм;

знать:

    • Выполнение арифметических действий, сочетая устные и письменные приемы, применение вычислительных устройств;

    • Решение уравнений и неравенств (линейных, квадратных);

    • Вычисление значений числовых и буквенных выражений, осуществляя необходимые подстановки и преобразования;

    • Нахождение значений корня натуральной степени, степени с рациональным показателем, логарифма, используя при необходимости вычислительные устройства;

    • Преобразования буквенных выражений, включающих степени, радикалы, логарифмы, тригонометрические функции;

    • Построение графиков изученных функций;

    • Решение показательных и логарифмических уравнений и неравенств; простейших иррациональных и тригонометрических уравнений;

    • Вычисление производных и первообразных элементарных функций, используя справочные материалы;

    • Исследование в простейших случаях функций на монотонность, нахождение наибольших и наименьших значений функций; построение графиков многочленов с использованием аппарата математического анализа;

    • Вычисление в простейших случаях площадей и объемов с использованием определенного интеграла;

    • Распознавание на чертежах и моделях пространственных форм; соотношение объектов с их описанием, изображением;

    • Описание взаимного расположения прямых и плоскостей в пространстве ;

    • Изображение основных многогранников и круглых тел ;

    • Решение планиметрических и простейших стереометрических задач на нахождение геометрических величин (длин, углов, площадей, объемов);

    • Использование при решении стереометрических задач планиметрических фактов и методов;

  • Решение простейших комбинаторных задач методом перебора, а также с использованием формул;

  • Вычисление в простейших случаях вероятности событий на основе подсчета числа исходов;

  • Использование приобретенных знаний и умений для анализа числовых данных, представленных в виде таблиц, диаграмм;

  • Значение математической науки для решения задач, возникающих в теории и практике; широту и в то же время ограниченность применения математических методов к анализу и исследованию процессов и явлений в природе и обществе;

  • Значение практики и вопросов, возникающих в самой математике для формирования и развития математической науки; историю развития понятия числа, создания математического анализа, возникновения и развития геометрии;

  • Универсальный характер законов логики математических рассуждений, их применимость во всех областях человеческой деятельности;

Критерии оценки:

Оценка «5» выставляется если : выполнено задание из части 1) и 2),выборочно из части 3).

Оценка «4» выставляется , если : выполнено задание из части 2),выборочно из части 3).

Оценка «3» выставляется, если : выполнено задание из части 1),выборочно из части 2).

Требования к оформлению самостоятельной работы:

Расчетные задания должны быть выполнены в рабочей тетради №2.

Учебно-методическое и информационное обеспечение: приложение №1.

Списки практических работ по специальности:

151901. Технология машиностроения.

Списки ПР

Сроки выполнения

ПР № 1«Вычисление катетов, гипотенузы прямоугольного треугольника по теореме Пифагора . Решение треугольников».

1 семестр, 1 неделя

ПР № 2«Построение четырехугольников и вычисление их элементов. Вычисление элементов правильных многоугольников».

1 семестр, 2 неделя

ПР № 3«Вычисление площадей фигур».

1 семестр, 2 неделя

ПР №4«Построение параллельных прямых и расчет углов. Доказательство параллельности прямых и плоскостей».

1 семестр, 3 неделя

ПР № 5«Определение взаимного расположения прямых в пространстве. Построение параллельных плоскостей».

1 семестр, 3 неделя

ПР №6 «Решение задач на определение перпендикулярности прямых, прямой и плоскости».

1 семестр, 4 неделя

ПР №7«Вычисление перпендикуляра и наклонной. Вычисление угла между плоскостями».

1 семестр, 5 неделя

ПР №8 «Перпендикулярность двух плоскостей. Изображение пространственных фигур».

1 семестр, 5 неделя

ПР № 9«Построение многогранников. Вычисление элементов призмы».

1 семестр, 6 неделя

ПР № 10«Вычисление элементов пирамиды, правильной пирамиды, усеченной пирамиды.».

1 семестр, 7 неделя

ПР № 11«.Построение фигур с помощью симметрии»

1 семестр, 7 неделя

ПР № 12 «Вычисление элементов цилиндра».

1 семестр, 8 неделя

ПР № 13 «Вычисление элементов конуса, усеченного конуса».

1 семестр, 9 неделя

ПР № 14 «Вычисление элементов сферы».

1 семестр, 9 неделя

ПР № 15 «Вычисление элементов тел вращения».

1 семестр, 10 неделя

ПР № 16«Расчет по модели объёма прямоугольного параллелепипеда».

1 семестр, 11 неделя

ПР № 17 «Вычисление объёма прямой призмы. Вычисление объёма цилиндра».

1 семестр, 11 неделя


ПР № 18 «Вычисление объёма пирамиды .Расчет по модели объёма конуса».

1 семестр, 12 неделя

ПР № 19 « Расчет по модели площади цилиндра и конуса».

1 семестр, 12 неделя

ПР № 20 «Вычисление объёма шара. Расчет объёмов сегмента, слоя, сектора шара».

1 семестр, 13 неделя

ПР № 21 «Вычисление объёмов тел».

1 семестр, 13 неделя

ПР № 22 «Составление уравнения сферы».

1 семестр, 14 неделя

ПР №23 «Решение задач на вычисление длины векторов, суммы векторов».

1 семестр, 14 неделя

ПР № 24«Умножение вектора на число .Вычисление координат векторов».

1 семестр, 15 неделя

ПР № 25 « Решение задач в координатах».

1 семестр, 15 неделя

ПР № 26  « Степень числа и ее свойства».

1 семестр, 16 неделя

ПР № 27«Решение пропорций. Решение задач с помощью пропорций».

1 семестр, 17 неделя

ПР № 28«Вычисление квадратных корней. Решение задач на проценты».     

1 семестр, 17 неделя

ПР № 29«Решение квадратных уравнений . Решение неравенств ».   

2 семестр, 18 неделя

ПР № 30   « Решение систем уравнений и неравенств. Вычисления по  формулам сокращенного умножения».

2 семестр, 18 неделя

ПР № 31 «Техника умножения и деления чисел».

2 семестр, 19 неделя

ПР № 32 « Корни и степени» .  

2 семестр, 20 неделя

ПР № 33 « Вычисление логарифмов ».         

2 семестр, 21 неделя

ПР № 34« Перевод из одной меры угла в другую. Вычисление синуса, косинуса, тангенса угла ». 

2 семестр, 22 неделя

ПР № 35 « Вычисление значения тригонометрических функций по формулам». 

2 семестр, 22 неделя

ПР №36«Вычисление значения тригонометрических функций двойного и половинного угла ». 

2 семестр, 23 неделя

ПР № 37« Решение уравнений cosx = a, sinx = a, tg x = a ».

2 семестр, 24 неделя

ПР № 38«Решение тригонометрических уравнений заменой переменной».

2 семестр, 24 неделя

ПР № 39 « Решение тригонометрических уравнений способом деления» .

2 семестр, 25 неделя

ПР № 40 «Вычисление множества значений тригонометрических функций по формулам».

2 семестр, 26 неделя

ПР № 41 « Нахождение экстремумов функции. Нахождения наибольшего и наименьшего значения функции».

2 семестр, 27 неделя

ПР № 42«Построение графиков тригонометрических функций».

2 семестр, 28 неделя

ПР № 43 «Решение иррациональных уравнений».         

2 семестр, 29 неделя

ПР № 44 «Решение показательных уравнений».

2 семестр, 29 неделя

ПР № 45«Решение логарифмических уравнений».

2 семестр, 30 неделя

ПР № 46«Решение тригонометрических уравнений».

2 семестр, 30 неделя

ПР № 47«Решение показательных неравенств».

2 семестр, 31 неделя

ПР № 48«Решение логарифмических неравенств».

2 семестр, 31 неделя

ПР № 49«Решение тригонометрических неравенств».

2 семестр, 31 неделя

ПР № 50 «Решение систем уравнений и неравенств ».

2 семестр, 32 неделя

ПР № 51 «Решение неравенств с помощью метода интервалов».

2 семестр, 32 неделя

ПР № 52 «Вычисление угловых коэффициентов. Составление уравнения касательной к графику функции».

2 семестр, 33 неделя

ПР №53«Вычисление производных по правилам дифференцирования».

2 семестр, 34 неделя

ПР №54«Вычисление производных элементарных функций».

2 семестр, 34 неделя

ПР № 55«Построение графиков функций с использованием производной ».

2 семестр, 35 неделя

ПР № 56 «Вычисление второй производной. Исследование функции на выпуклость, вогнутость, определение точек перегиба ».

2 семестр, 36 неделя

ПР № 57«Вычисление первообразных элементарных функций».

2 семестр, 37 неделя

ПР № 58 «Вычисление площадей с помощью интегралов».

2 семестр, 37 неделя

ПР № 59 «Решение задач на подсчет числа размещений, перестановок, сочетаний. Решение задач на перебор вариантов».

2 семестр, 38 неделя

ПР № 60 «Вычисление числовых характеристик дискретной случайной величины».

2 семестр, 39 неделя

Всего практ.работ : 60

120 ч.



ПРИЛОЖЕНИЕ №1


Основные учебники :


  1. Алгебра и начала анализа 10-11 кл. Ш.А.Алимов и др. М., «Просвещение», 2009 г.

  2. Геометрия 10-11 кл. Л.С. Атанасян. М., «Просвещение»,2011 г.


Дополнительные учебники :



  1. Алгебра и начала математического анализа 10-11 кл. Колмогоров А.Н. и др. М., «Просвещение»,2009г.

  2. Алгебра и начала математического анализа 10-11 кл. Башмаков М.И. М., «Дрофа»,2009г.

  3. Геометрия 10-11 кл. Потоскуев Е.В., Звавич Л.И. М., «Просвещение», 2009г.




Интернет-ссылки для ВСР.

Алгебра:

  1. http://math-prosto.ru/?page=pages/library-math/alimov-10-11.php

  2. http://nashol.com/2012102467590/algebra-i-nachala-matematicheskogo-analiza-10-11-klass-alimov-sh-a-kolyagin-u-m-2012.html

  3. http://nsportal.ru/shkola/algebra/library/algebra-i-nachala-analiza-10-11-klass-po-uchebniku-sha-alimova-i-dr

  4. http://nashol.com/2014021575799/algebra-i-nachalo-matematicheskogo-analiza-10-klass-muravin-g-k-2013.html

  5. http://elkniga.ucoz.ru/load/multimedijnye_posobija/matematika/multimedijnoe_posobie_po_matematike_uroki_algebry_kirilla_i_mefodija_10_11_klass/14-1-0-15

Геометрия:

  1. http://nashol.com/knigi-po-matematike/#po_godam_2012

  2. http://nashol.com/2011102361137/geometriya-uchebnik-10-11-klass-atanasyan-l-s-butuzov-v-f-kadomcev-s-b-2009.html

  3. http://4book.org/uchebniki-rossiya/10-klass/62-geometriya-uchebnik-dlya-10-11-klassov-atanasyan-l-s-i-dr

  4. http://neovit.net/edu/math1.htm

  5. http://elkniga.ucoz.ru/publ/uchebniki/10_klass/geometrija_atanasjan_l_s_uchebnik_dlja_10_11_klassa_obshheobrazovatelnykh_uchrezhdenij/98-1-0-311





И любые другие аналогичные из интернета по разделам «Алгебра и начала анализа», «Геометрия».













Просмотрено: 0%
Просмотрено: 0%
Скачать материал
Скачать материал "Практические работы обучающихся по дисциплине Математика для специальности Технология машиностроения."

Методические разработки к Вашему уроку:

Получите новую специальность за 6 месяцев

Менеджер по управлению сервисами ИТ

Получите профессию

Технолог-калькулятор общественного питания

за 6 месяцев

Пройти курс

Рабочие листы
к вашим урокам

Скачать

Выбранный для просмотра документ МЕТОД.УК. ПР-ТЕХМАШ,2 КУРС..docx














Методические указания

к выполнению практических работ обучающихся

по дисциплине Математика

для специальности 151901. Технология машиностроения .






















2015г.

Пояснительная записка


Цель методических указаний: оказание помощи обучающимся в выполнении практических работ по дисциплине Математика.

Настоящие методические указания содержат работы, которые позволят обучающимся самостоятельно овладеть профессиональными знаниями и умениями, опытом творческой деятельности при решении проблем учебного и профессионального уровня и направлены на формирование следующих компетенций:

ОК 1. Понимать сущность и социальную значимость своей будущей

профессии, проявлять к ней устойчивый интерес.

ОК 2. Организовывать собственную деятельность, выбирать типовые

методы и способы выполнения профессиональных задач, оценивать их

эффективность и качество.

ОК 3. Принимать решения в стандартных и нестандартных

ситуациях и нести за них ответственность.

ОК 4. Осуществлять поиск и использование информации,

необходимой для эффективного выполнения профессиональных задач,

профессионального и личностного развития.

ОК 5. Использовать информационно-коммуникационные технологии

в профессиональной деятельности.

ОК 6. Работать в коллективе и команде, эффективно общаться

с коллегами, руководством, потребителями.

ОК 7. Брать на себя ответственность за работу членов команды

(подчиненных), результат выполнения заданий.

ОК 8. Самостоятельно определять задачи профессионального и

личностного развития, заниматься самообразованием, осознанно

планировать повышение квалификации.

ОК 9. Ориентироваться в условиях частой смены технологий

в профессиональной деятельности.

ОК 10. Исполнять воинскую обязанность, в том числе с применением

полученных профессиональных знаний (для юношей).

В результате выполнения практических работ по дисциплине Математика обучающиеся должны:

уметь:

  • анализировать сложные функции и строить их графики;

выполнять действия над комплексными числами;

  • производить операции над матрицами и определителями;

  • решать задачи на вычисление вероятности с использованием элементов комбинаторики;

  • решать прикладные задачи с использованием элементов дифференциального и интегрального исчислений;

  • решать системы линейных уравнений различными методами.

знать:

  • основные математические методы решения прикладных задач;

  • основные понятия и методы математического анализа, линейной алгебры, теорию комплексных чисел, теории вероятностей и математической статистики;

  • основы интегрального и дифференциального исчисления;

  • роль и место математики в современном мире при освоении профессиональных дисциплин и в сфере профессиональной деятельности.



Списки практических работ по специальности:

151901. Технология машиностроения.

Списки ПР

Сроки выполнения

ПР № 1 «Выполнение действий над матрицами». 

3 семестр, 1 неделя

ПР № 2 «Вычисление определителей второго и третьего порядка».

3 семестр, 3 неделя

ПР № 3 «Решение систем уравнений с помощью матричного метода».

3 семестр, 4 неделя

ПР № 4 «Решение систем линейных уравнений по правилу Крамера». 

3 семестр, 4 неделя

ПР № 5 «Решение систем линейных уравнений методом Гаусса». 

3 семестр, 6 неделя

ПР № 6 «Выполнение действий над комплексными числами в алгебраической форме». 

3 семестр, 7 неделя

ПР № 7 «Выполнение действий над комплексными числами в тригонометрической форме». 

3 семестр, 8 неделя

ПР № 8 «Выполнение действий над комплексными числами в показательной форме». 

3 семестр, 9 неделя

ПР № 9 «Решение простейших задач на определение вероятности». 

3 семестр, 10 неделя

ПР № 10 «Вычисление производных функций». 

3 семестр, 12 неделя

ПР № 11 «Вычисление неопределенных интегралов». 

3 семестр, 13 неделя

ПР № 12 «Вычисление определенных интегралов». 

3 семестр, 14 неделя

ПР №13 «Исследование функций и построение графика с помощью производной».

3 семестр, 15 неделя

ПР №14 «Вычисление площади плоских фигур с помощью определённого интеграла». 

3 семестр, 16 неделя

Всего практ.работ : 14

28 ч.






Критерии оценки:

Оценка «5» выставляется если : выполнено задание из части 1) и 2),выборочно из части 3).

Оценка «4» выставляется , если : выполнено задание из части 2),выборочно из части 3).

Оценка «3» выставляется, если : выполнено задание из части 1),выборочно из части 2).

Требования к оформлению самостоятельной работы:

Расчетные задания должны быть выполнены в рабочей тетради №2.

Учебно-методическое и информационное обеспечение: приложение №2.












ПРИЛОЖЕНИЕ №2

Учебники :


  1. Григорьев С.Г., Иволгина С.В. Математика:Учебник для СПО / Григорьев С.Г., Иволгина С.В., под ред.В.А.Гусева– 9-е изд., стер. – М.: Академия, 2013 – 416 с.

  2. Башмаков М. И. Математика: Учебник СПО / М. И. Башмаков – М.: Академия, 2011 – 256 с.

  3. Пехлецкий И. Д. Математика: Учебник для СПО / Пехлецкий И. Д. – 6-е изд., стер. – М.: Академия, 2011 – 304 с.

  4. Березина Н. А., Максина Е. Л. Математика: Учеб. пособие для СПО / Н. А. Березина, Е. Л. Максина – М.: РНО, 2007 – 175 с.

  5. Богомолов Н. Практические занятия по математике.- М., 2006;

  6. Выгодский М.Я. Справочник по элементарной математике, - М., 2006

  7. Дадаян А. А. Математика: Учебник для СПО / А. А. Дадаян. – М.: Форум, 2008 – 544 с.

  8. Дадаян А. А. Сборник задач по математике / А. А. Дадаян – М.: Инфра – М.: Форум, 2008 – 352 с.

  9. П.Е. Данко, А.Г. Попов, Т.Я. Кожевникова Высшая математика в упражнениях и задачах (часть 1) - М.: «Мир и Образование», 2005-304с.

  10. Д. Т. Письменный. Конспект лекций по высшей математике. - «Айрис», 2007.

  11. Д. Т.Письменный. Сборник задач по высшей математике. - «Айрис», 2007

  12. Математика и информатика: Учебник для СПО / Ю. Н. Виноградов, А. И. Гомола, В. И. Потапов и др. – М.: Академия, 2011 – 272 с.

  13. Математический анализ в вопросах и задачах: Учебное пособие для ВУЗов/ В.Бутузов и др. - М, 2005.

  14. Михеев В. С., Стяжкина О. В.Учебное пособие для СПО - «Феникс», 2009.

  15. М. С. Спирина, П. А. Спирин. Учебник для студентов СПО - «Академия», 2009.

  16. Максимова О. В. Теория вероятностей и математическая статистика. Учебное пособие для СПО изд.2-е, 2007.

  17. Старков С. Справочник по математическим формулам и графикам. - СПб., 2008.



Интернет-ссылки для ВСР.



  1. http://www.mathedu.ru/

  2. http://www.onecomplex.ru/

  3. http //matemonline.com/wp-content/uploads

  4. Cайты «Энциклопедий», например: http://www.rubricon.ru/; http://www.encyclope.com

  5. Новые технологии в образовании: http://edu.secna.ru/main/ http://festival.1september.ru

  6. http://www.fxyz.ru

  7. http://referat.ru

  8. http://math.immf.ru/

  9. http://integraly.ru/

  10. http://www.alleng.ru/edu/math.htm

(и другие сайты, связанные с математикой)







Просмотрено: 0%
Просмотрено: 0%
Скачать материал
Скачать материал "Практические работы обучающихся по дисциплине Математика для специальности Технология машиностроения."

Получите профессию

Интернет-маркетолог

за 6 месяцев

Пройти курс

Рабочие листы
к вашим урокам

Скачать

Выбранный для просмотра документ ПР-ТЕХМАШ-1КУРС.docx

Инструкционная карта

ПР №1«Вычисление катетов, гипотенузы прямоугольного треугольника по теореме Пифагора . Решение треугольников».

Задание:

1)Перепишите и заполните пропуски:

Пример 1. а = 5, b = 12, найти с. Решение: с2 = а2 + b2 = 52 + 122 = 25 + 144 = …, с = …;

Ответ: 13.

Пример 2. с = 41, а = 40, найти b. Решение: b 2 = с2 а2 = 412 – 402 = 1681 – 1600 = …, b = …;

Ответ: 9.

Пример 3. а = 10, b = , найти с. Решение: с2 = а2 + b2 = 102 + 2 = 100 + 44 = … ,с = …;

Ответ: 12.

Пример 4. а = 10, b = 10, с = 12, найти h1, h2, h3.

Решение: p = (а + b + с) : 2 = (10 + 10 + 12) : 2 = 32 : 2 = …,

S = = = =

= 642 = …,

h1 = 2S : a = 2 48: 10 = 96 : 10 = … , h2 = 2S : b = 2 48:10 = …, h3 = 2S : с = 2 48 : 12 =

= 96 : 12 = …;

Ответ: 9,6;9,6;8.

Пример 5. а = 12, b = 18, С = 50°, найти с, А, В.

Решение: с2 = а2 + b2 – 2 а b cos C = 122 + 182 -2 12 18 cos 50° = = 144 + 324 - 2 12 18 0,6428 = 144 +3 24 – 278 = …, с ≈ 14,

cos А = (b2 + с2 а2) : (2 b с) = (182 + 142 – 122) : (21814) = 0,7460, А = 41°45 / ,

В = 180° - (50° + 41°45 /) = 180° – 91°45 / = 89° – 45 / = 88°15 /; Ответ: 14, 41°45 / , 88°15 / .

Пример 6. а =24,6, В = 45°,С = 70°, найти А, b, с.

Решение: А = 180° - (45° + 70°) = …°,

b = = 24,6 = 24,6 = 19,2;

с = = 24,6 = 24,6 = 25,6;

Ответ: 65°,19,2; 25,6 .

Пример 7. а = 14, b = 18, с = 20, найти А, В, С .

Решение: cos А = (b2 + с2 а2) : (2 b с) = (182 + 202 142) : (2 18 20) = 0,7333;

А = 42°50 / ≈ 43°, cos В = (а2 + с2 b2) : (2ас) = (142 + 202 182) : (21420) = 0,4857;

В = 60°56 / ≈ 61°, С = 180° (43° + 61°) = …°;

Ответ: 43°,61°,76°.

2)Решить задачи ( по примерам):

  1. а = 8, b = 15, найти с.

  2. с = 41, а = 9, найти b.

  3. а = 10, b = , найти с.

  4. а = 10, b = 10, с = 16, найти h1, h2, h3.

  5. а = 6,3, b = 6,3, <С = 54°, найти с, А, В.

  6. а =14, <В = 40°,<С = 60°, найти А, b, с.

  7. а = 6, b = 7,3, с = 4,8, найти А, В, С .

3)Решить задачи :

  1. Найдите периметр прямоугольного треугольника, катеты которого равны 3см и 4см.

  2. Найдите диагонали прямоугольника со сторонами 9см и 12см.

  3. Найдите катет прямоугольного треугольника, если его гипотенуза равна 8см, а

проекция искомого катета на гипотенузу – 2см.

  1. а = 9, b = 12, найти с.

  2. с = 17, а = 15, найти b.

  3. а = 6, b = 8, с = 10, найти h1, h2, h3.

  4. а = 16, b = 10, <С = 80°, найти с, А, В.

  5. а =32, <В = 45°,<С = 87°, найти А, b, с.

  6. а = 12, b = 14,6, с = 9,6, найтиА, В, С .

  7. На какое расстояние надо отодвинуть от стены дома нижний конец лестницы длиною 17 м, чтобы верхний конец ее достал до слухового окна, находящегося на высоте 15 м от поверхности земли.

  8. Две башни в равнине находятся на расстоянии 60 локтей одна от другой. Высота первой башни 50 локтей, высота второй 40 локтей. Между башнями находится колодец, одинаково удаленный от вершин башен. Как далеко находится колодец от основания каждой башни.hello_html_4a8a8a80.jpg

  9. В треугольнике АВС  А = 45°, ВС = 13, а высота ВD отсекает на стороне АС отрезок DС, равный 12 см. Найти площадь Δ АВС и высоту, проведенную к стороне ВС.

  10. В параллелограмме АВСD ВK делит сторону АD на отрезки АK и KD. Найдите стороны параллелограмма, если ВK = 12, АK = 5, ВD = 15.

  11. Диагональ  прямоугольника  равна  52  мм,  а  стороны  относятся как 5 : 12. Найти его периметр.

  12. Дан  прямоугольный  треугольник  ОМK  ( K = 90°).  Запишите теорему  Пифагора  для  этого  треугольника  и  найдите  сторону  МK, если ОK = 15 см,
    ОМ = 17 см.

  13. В прямоугольнике проведена диагональ. Найдите длину диагонали, если известны стороны прямоугольника – 8 см и 15 см.

  14. Записать теорему Пифагора для треугольников и найдите х.

1)

hello_html_m36d3c9b1.jpg

2)

hello_html_1200601f.jpg

3) АВСD – ромб,ВD = 16,АС = 12, АВ = ?

hello_html_m71523c39.jpg

4) АВСD – прямоугольник,АВ = 12,

ВС = 16,ВD = ?

hello_html_5d1935fd.jpg

5)ВС = 26,ВD = 24, DС = ?

hello_html_m408baaf4.jpg

6) DЕ – высота, ВЕ = ВD = 12, ЕD = ?

hello_html_e92519b.jpg


  1. ВЕ =?

hello_html_mf3d8d67.jpg

20


Инструкционная карта

ПР №2«Построение четырехугольников и вычисление их элементов. Вычисление элементов правильных многоугольников».

Задание:

1)а) Построение четырехугольников .hello_html_m233e7e65.gif

б)Перепишите и заполните пропуски:

Пример 1. Дано: ABCD – ромб, AC = 1,5 BD,SABCD = 27 см2.

Найти: AC, BD.

Решение:

  1. SABCD = 1/2 AChello_html_1cc4939.gifBD.

Пусть BD = x см, тогда AC = 1,5x см. Т.к. по условию задачи SABCD = 27 см2, то получаем уравнение: 1/2hello_html_1cc4939.gif1,5xhello_html_1cc4939.gifx = 27,

1,5x2=54, x2=36, x= … ( x = – 6 не подходит по смыслу задачи).

  1. BD = 6 см, AC = 1,5hello_html_1cc4939.gif6 = см.

Ответ : 6 см и 9 см.

Пример 2. Стороны четырехугольника, взятые последовательно, пропорциональны

числам 2: 5: 2: 5. Периметр четырехугольника равен 42 см. Найти стороны.

Решение: х- коэффициент пропорциональности, тогда стороны будут 2х, 5х, 2х,5х; периметр будет

2х + 5х + 2х + 5х = 42, 14х = 42 , х = …, 2х = 2 3 = …, 5х = 53 = …

Ответ : 6 и 15 см.
Пример 3. В параллелограмме ABCD одна сторона больше другой в два раза. Периметр параллелограмма равен 42 см. Найти стороны.

Решение: х- коэффициент пропорциональности, тогда стороны будут х и 2х, периметр будет

х + 2х + х + 2х = 42, 6х = 42 , х = …, 2х = 2 7 = …

Ответ : 7 и 14 см.

Пример 4. Найти углы параллелограмма АВСD, если известно, что угол А больше угла В в 3 раза.

Решение:В = х,А= 3х, А + В = 180° , 3х + х = 180° ,4х = 180° , х = …, В = …,

А= 180° – 45°= … ,

Ответ : 45° и 135° .
Пример 5. Найти диагонали прямоугольника АВСD, еслиАBD = 30° , АD= 6 см.
hello_html_5d1935fd.jpg

Решение: Δ АBD : АBD = 30° , АD= 6 см, ВD = 62 =…

Ответ : 12 см.

Пример 6. Диагональ  прямоугольника  равна  52  мм,  а  стороны 
относятся как 5 : 12. Найти его периметр.

Решение: х- коэффициент пропорциональности, тогда стороны будут 5х, 12х; ΔАBD – прямоугольный, диагональ ВD найдем по теореме Пифагора:

ВD2 = АD2 + АВ2, ВD2 = (5х)2 + (12х)2 = 25х2 + 144х2 = 169 х2 = 522 = 2704, х2 = 16, х = …,

периметр Р = 2D + АВ) = 2 (5 4 + 12 4) = 2 (20 + 48) = 2 68 = …

Ответ : 136 мм2.

Пример 7. Дано: правильный треугольник , n = 3,

а) R = 3 см, Найдите r, а3, P , S. ;б) S = 48 , Найдите r, R, а3, P; в) r = 2 см, Найдите R, а3, P , S.

Решение: а) r = R : 2 = 3 : 2 = …, а3 = R = … , Р = 3а3 = 33 = … ,

S = 3R2 /4 = 3 9 / 4 = … / 4,

б) ( а3)2 = 4S : = 4 48 : = 4 48 = …,а3 = 8 , R = а3 : = 8 : = …,r = R : 2 = 8 : 2 = …,

Р = 3 а3 = 3 8 = … .

в) R = 2 r = 22 =…, а3 = R = … , Р = 3 а3 = 3 4 = … , S = 3R2 /4 = 3 16 / 4 =

= 3 4 = … .

Ответ : а) r = 1,5, а3 = 3 , Р = 9 , S = 27 / 4,б) а3 = 8 , R = 8, r = 4, Р = 24 ,

в) R = 4, а3 = 4 , Р = 12 , S = 12 .

Пример 8. Дано: правильный четырехугольник , n = 4,

а) а4 = 6 см, Найдите r, R, P , S. ;б) r = 2 см, Найдите R, а4, P , S. в) S = 64, Найдите r, R, а4, P;

Решение: а) Р = 4а4 = 46 = …, S = (а4)2 = 62 = …, R = а4 : = 6 : = 3 ,

r = R : = 3 : = …, б) R = r = … , а4 = 2r = 22 = …, Р = 4а4 = 44 = …,

S = (а4)2 = 42 = …, в) а4 = = …, R = а4 : = 8 : = 4 , r = R : = 4 : = …,

Р = 4а4 = 4 8 = …

Ответ : а) Р = 24, S = 36, R = 3 , r = 3, б) R = 2 , а4 = 4,Р = 16, S = 16,

в) а4 = 8, R = 4 , r = 4, Р = 32.

2)Решить задачи ( по примерам):

  1. Дано: ABCD – ромб, AC = 0,75 BD,SABCD = 54 см2. Найти: AC, BD.

  2. Стороны четырехугольника, взятые последовательно, пропорциональны

числам 3: 4: 3: 4. Периметр четырехугольника равен 70 см. Найти стороны.

  1. В параллелограмме ABCD одна сторона больше другой в два раза. Периметр параллелограмма равен 66 см. Найти стороны.

  2. Найти углы параллелограмма АВСD, если известно, что угол А больше угла В в 5 раз.

  3. Найти диагонали прямоугольника АВСD, еслиАBD = 30° , АD= 8 см.

  4. Диагональ  прямоугольника  равна  26  мм,  а  стороны  относятся как 5 : 12. Найти его периметр.

  5. Дано: правильный треугольник , n = 3,

а) R = 6 см, Найдите r, а3, P , S. ;б) S = 27 , Найдите r, R, а3, P; в) r = 4 см, Найдите R, а3, P , S.

  1. Дано: правильный четырехугольник , n = 4,

а) а4 = 8 см, Найдите r, R, P , S. ;б) r = 4 см, Найдите R, а4, P , S. в) S = 16, Найдите r, R, а4, P;

3)Решить задачи :

  1. Стороны параллелограмма пропорциональны числам 3 и 7. Найдите наименьшую сторону, если периметр параллелограмма равен 18 см.

  2. Один из углов ромба равен 120° , а его меньшая диагональ равна 4,5 см. Найдите периметр ромба.

  3. В прямоугольнике СКМN проведена биссектриса угла С, которая пересекает сторону КМ в точке Е, причем длинна отрезка КЕ на 3 см меньше длинны МЕ. Найдите МN, если периметр СКМN равен 51 см.

  4. Найти стороны параллелограмма АВСД, если его периметр равен 40 см, а сторона АВ больше ВС на 4 см.

  5. Найти углы равнобедренной трапеции, если один из них равен 75º.

  6. Найти стороны параллелограмма АВСД, если его периметр равен 54 см, а сторона АВ больше ВС в 2 раза.

  7. Найти углы параллелограмма АВСД, если известно, что угол А меньше угла В на 40 °.

  8. Найти углы прямоугольной трапеции, если больший из них равен 120º.

  9. Радиус окружности, вписанной в квадрат, равен 1 см. Найдите радиус R описанной окружности около этого квадрата.

  10. Дано: правильный треугольник , n = 3, а) а3 = 6, Найдите r, R, P , S; б)Р = 6 , Найдите r, R3, S;

  11. Дано: правильный треугольник , n = 4, а) R = 4 см, Найдите r, а4, P , S ;б) Р = 32 ,
    Найдите
    r, R, а4, S;

  12. Периметр правильного шестиугольника, описанного около окружности, равен 6  см. Чему равен радиус этой окружности?

  13. Периметр квадрата, вписанного в окружность, равен 4  см. Найдите радиус r вписанной окружности.

  14. В правильный шестиугольник ABCDEF, со стороной 6 см, вписан правильный треугольник A1B1C1. Найдите отношение радиуса окружности, вписанной в треугольник A1B1C1, к радиусу окружности, вписанной в шестиугольник ABCDEF.

  15. В правильный треугольник MNP вписана окружность. Отрезок NR перпендикулярен отрезку MP и пересекает его в точке K. Угол KMR=300. Найдите радиус вписанной окружности в треугольник MNP и её длину.

  16. Ширина кольца, образованного двумя концентрическими окружностями, равна 2. Хорда большей окружности, касательная к меньшей, равна 8. Найти радиусы окружностей.

  17. В окружность радиуса R = 12 вписан правильный n-угольник. Определите его сторону и периметр, если: а) n = 3; б) n = 4; в) n = 6.

  18. Около окружности радиуса r = 6 описан правильный n-угольник. Определите его сторону и периметр, если: а) n = 3; б) n = 4; в) n = 6.

  19. Для правильного n-угольника со стороной а = 6 см найдите радиус описанной около него окружности, если: а) n = 3; б) n = 4; в) n = 6.

  20. Правильный треугольник АВС вписан в окружность с центром О и радиусом 8 см. На стороне этого треугольника построен квадрат. Определите радиус окружности, описанной около квадрата.

Инструкционная карта

ПР №3«Вычисление площадей фигур».

Задание:

1)Перепишите и заполните пропуски:

Пример 1. Периметр участка земли имеющего форму прямоугольника составляет 80 см, а отношение сторон 2:3. Найдите площадь участка.

Дано: ABCD-прямоугольник, P = 80cм, AD : AB = 2 : 3 .Найти: S.
Решение: х - коэффициент пропорциональности, AD = 2x (см), АВ = 3х (см), P = (AD+AB)2,
(2х+3х) 2 = 80, 5х = 40, х = ...,
AD = 82 = … cм, АВ = 83= …см, S = ab, S = 1624 = …cм2
Ответ: 384 см2.

Пример 2. Дано: АВСD-ромб, A=150º ,AB = 6 см. Найти: S.
Решение: ABCD- ромб (по условию),A+В=180º (внутренние односторонние углы при параллельных прямых AD и ВС и секущей АВ). В=180º150º = …º .

Δ АВК- прямоугольный. Катет, лежащий против угла в 30º равен половине гипотенузы.

АК=1/2АВ = 6 : 2= … см.  S = ah = 63= … см².
Ответ: S=18 см².
hello_html_m7e207ca8.jpg

Пример 3. Дано: ABCD-параллелограмм, BH= 15 см , AD =21 см. Найти: S.

Решение: S = BH AD = 15 21 = …

Ответ : 315 см2.

Пример 4. В параллелограмме ABCD стороны равны 14 и 8см., высота проведенная к большей стороне, равна 4 см. Найдите площадь параллелограмма и вторую высоту.

Дано: ABCD-параллелограмм, BH= 4 см , AD = 14 см, АВ = 8 см. Найти: S, H.

Решение: S = BH AD = 4 14 = …, H = S : АВ = 56 : 8 = …

Ответ : S = 56 см2, H = 7 см.

Пример 5. Площадь параллелограмма равна 48 см2 , а его периметр 40 см. Найдите стороны параллелограмма, если высота, проведенная одной из них, в 3 раза меньше этой стороны.

Решение: х- коэффициент пропорциональности, тогда высота и сторона будут х,3х;

S = х 3х = 48, х2 = 16, х = …, BH = 4 см, AD = 3 4 = …, P = (AD+AB)2 = 40, AD+AB = 20,

AВ = 20 – 12 = …

Ответ : 8 и 12 см.
Пример 6. а) Найти площадь ромба, если его диагонали равны 3см и 6см.

Решение: S=1/2 hello_html_1cc4939.gif3hello_html_1cc4939.gif6 = 18 : 2 = … (см2). Ответ : 9 см2 .

б) Найти площадь квадратного участка земли, если его диагональ равна 10м.

Решение: S = 1/2 hello_html_1cc4939.gif102 = 100 : 2 = … (м2). Ответ : 50 м2 .

в) Найти одну из диагоналей ромба, если его площадь равна 20 см2, а вторая диагональ 8 см.

Решение: d1= 2S/d2, d1=220/8 = 40 : 8 = … (см). Ответ : 5 см .

г)Найти диагональ квадрата, если его площадь равна 18 см2.

Решение: d2 = 2S, d2 = 36, d = …(см). Ответ : 6 см .

Пример 7. Вычислить площадь ромба, если одна из его диагоналей равна 5 см, а другая в 4 раза больше.

Решение: d2 = 4 d1 = 45 = …, S = 5 20 : 2 = 5 10 = … Ответ : 50 см2 .
Пример 8. Площадь трапеции равна 320 см2, а высота трапеции равна 8 см. Найдите основания трапеции, если одно и них составляет 0,6 длины другого. 

Решение: S = (a + b)h : 2, a + b = 2S : h = 2 320 : 8 = 2 40 =…, a = 0,6 b, 0,6 b + b = 80,

1,6b = 80, b = …, a = 80 – 50 = …hello_html_m7ca91367.png

Ответ : 30 и 50 см.

Пример 9. Дано: АВСD- прямоугольник, CKD = 80º .Найти: CBK.

Решение: ABCD –прямоугольник, АС = ВD ,

ΔАВК – равнобедренный, ВКА = CKD = 80º .

АВК = КАВ = ( 180° – 80°) : 2 = 100° : 2 = …,

CBK = 90°АВК = 90° – 50° = … Ответ :CBK = 40°.hello_html_m713f329.jpg

Пример 10.





Решение:

hello_html_20e18f82.jpg

2)Решить задачи ( по примерам):

  1. Дано: ABCD-прямоугольник, P = 60cм, AD : AB = 2 : 3 .Найти: S.

  2. Дано: АВСD-ромб, A=150º ,AB = 12 см. Найти: S.

  3. Дано: ABCD-параллелограмм, BH= 13 см , AD = 21 см. Найти: S.

  4. Дано: ABCD-параллелограмм, BH= 4 см , AD = 28 см, АВ = 8 см. Найти: S, H.

  5. Площадь параллелограмма равна 48 см2 , а его периметр 60 см. Найдите стороны параллелограмма, если высота, проведенная одной из них, в 3 раза меньше этой стороны.

  6. а) Найти площадь ромба, если его диагонали равны 3см и 12см.

б) Найти площадь квадратного участка земли, если его диагональ равна 12 м.

в) Найти одну из диагоналей ромба, если его площадь равна 40 см2, а вторая диагональ 8 см.

г)Найти диагональ квадрата, если его площадь равна 32 см2.

  1. Вычислить площадь ромба, если одна из его диагоналей равна 6 см, а другая в 4 раза больше.

  2. Площадь трапеции равна 120 см2, а высота трапеции равна 8 см. Найдите основания трапеции, если одно и них составляет 0,5 длины другого. 

  3. Дано: АВСD- прямоугольник, CKD = 40º .Найти: CBK.

  4. Параллелограмм и прямоугольник имеют одинаковые стороны. Найдите острый угол параллелограмма, если площадь его равна площади прямоугольника.

3)Решить задачи :

  1. Найти: S по рис.

hello_html_744b8681.png.

  1. Как изменяется площадь квадрата, если каждую сторону увеличить в 2 раза?

  2. Во сколько раз надо уменьшить сторону квадрата, чтобы площадь уменьшилась в 25 раз?

  3. Параллелограмм и прямоугольник имеют равные основания и равные периметры. Площадь, какой фигуры больше.hello_html_m5558a9b3.png

  4. Поле имеет фигуру параллелограмма, основание которого равны 250 м, а высота 100 м. Через это поле прямым углом к основанию проходит проселочная дорога шириной 5 м. Найдите посевную площадь этого поля. hello_html_m7e207ca8.jpg

  5. Дано: ABCD-параллелограмм,S = 187 см2 , AD =17 см.  Найти: ВН.

  6. В параллелограмме ABCD высоты равны 10 и 5 см., площадь параллелограмма равна 60 см2 .Найдите стороны параллелограмма.

  7. Площадь параллелограмма равна 50 см2, а его периметр 34 см.
    Найдите стороны параллелограмма, если одна из них в 2 раза больше проведенной к ней высоты.


Инструкционная карта

ПР №4«Построение параллельных прямых и расчет углов. Доказательство параллельности прямых и плоскостей».

Задание:

1)а) Построение параллельных прямых, параллельных прямой и плоскости.

б) Перепишите и заполните пропуски:hello_html_69440865.png

Пример 1. Дано: а || α, b α, а || b, с - секущая, 2 1= 30°,

Найти: 1, 2.

Решение: Углы 1 и 2 внутренние односторонние,

их сумма равна 180°, т. е.  l + 2 = 180°. (1)

Обозначим градусную меру угла 1 через х.

По условию 2 х = 30°, или 2 = 30° + x.

Подставим в равенство (1) значения углов 1 и 2, получим 
х + 30° + х = 180°, 2х = 150°,

Решая это уравнение, получим х = …°, т. е. 

1 = 75°, a 2 = 180° 75° = …°.

Ответ: 1 = 75°, a 2 = 105°.hello_html_11aaf2e3.png

Пример 2. Две параллельные прямые, одна из которых лежит в плоскости α, пересечены третьей. Известно, что сумма двух внутренних накрест лежащих углов равна 150°. Чему равны эти углы и остальные шесть?

Дано: а || α, bα, а || b, с - секущая, l + 2 = 150°,

Найти: 1, 2, 3, 4, 5, 6, 7, 8.

Решение: Углы 1 и 2 внутренние накрест лежащие, следовательно, они равны.

Сумма этих углов по условию задачи равна 150°, тогда 1 = 2 = 150° : 2 = ...

Найдем остальные углы . 1 = 3 = 75° и 2 = 7 = 75° (вертикальные). Углы 4 и 5, 6 и 8 равны как вертикальные, a 5 = 6 как внутренние накрест лежащие. Все перечисленные углы 4, 5, 6 и 8 равны между собой , так как 4 + 3 = 180°, hello_html_2ed53250.png

то 4 = 180° 3 = 180 ° – 75 ° = ...

Получили четыре угла по 75°, четыре угла по 105°.

Ответ: 1 = 2 = 3 = 7 = 75°, 4 = 5 = 6 = 8 = 105°.hello_html_m40fa69e3.jpg

Пример 3. Дано: а || α, bα, а || b, с - секущая, 1 = 150°,

Найти: 2, 3, 4, 5, 6, 7, 8.

Решение: 3 = 1 = 150°(верт.), 3 = 5 = 150°(н.леж.),

5 = 7 = 150°(верт.), 1 + 2 = 180°(смежные),

2 = 180° 1 = 180° 150° = …°,

2 = 4 = 30°(верт.), 4 = 6 = 30°(н.леж.), 6 = 8 = 30°(верт.).

Ответ: 3 = 5 = 7 = 150°, 2 = 4 = 6 = 8 = 30°.

Пример 4. Дано: а || α, bα, а || b, с - секущая, 4 = 70°,

Найти: 1, 2, 3, , 5, 6, 7, 8.

Решение: 2 = 4 = …°(верт.), 4 = 6 = 70°(н.леж.),

6 = 8 = …°(верт.), 4 + 3 = 180°(смежные),

3 = 180° 4 = 180° 70° = …°,

3 = 1 = 110°(верт.), 3 = 5 = …°(н.леж.), 5 = 7 = …°(верт.).

Ответ: 1 = 3 = 5 = 7 = 110°, 2 = 6 = 8 = 70°.

Пример 5.Плоскости α и β пересекаются по прямой АВ. Прямая а || α, а || β. Докажите, что а || АВ.

Док-во: Через точку А проведем АМ || α. Так как а || α, а || β, то АМ α, АМ β. Таким образом, αβ =АМ, т.е. она совпадает с АВ. Следовательно, АВ || а.

Пример 6. Дано: АВСD- прямоугольник, М ( АВСD), (СВМ) = α. Докажите, что АD || α.

Док-во: АD || ВС, ВС α. Следовательно, АD || α.




2)Решить задачи ( по примерам):

  1. Дано: а || α, bα, а || b, с - секущая, 2 1= 40°,Найти: 1, 2.

  2. Дано: а || α, bα, а || b, с - секущая, l + 2 = 150°,Найти: 1, 2, 3, 4, 5, 6, 7, 8.

  3. Дано: а || α, bα, а || b, с - секущая, 1 = 145°,Найти: 2, 3, 4, 5, 6, 7, 8.

  4. Дано: а || α, bα, а || b, с - секущая, 4 = 50°,Найти: 1, 2, 3, , 5, 6, 7, 8.

  5. Плоскости α и β пересекаются по прямой МС. Прямая а || α, а || β. Докажите, что а || МС.

  6. Дано: АВСD- прямоугольник, К ( АВСD), (СВК) = α. Докажите, что АD || α.

3)Решить задачи :

  1. Дано: а || α, bα, а || b, с - секущая, 2 1= 50°,Найти: 1, 2.

  2. Две параллельные прямые, одна из которых лежит в плоскости α, пересечены третьей. Известно, что сумма двух внутренних накрест лежащих углов равна 110°. Чему равны эти углы и остальные шесть?

  3. Дано: а || α, bα, а || b, с - секущая, 1 = 135°,Найти: 2, 3, 4, 5, 6, 7, 8.

  4. Дано: а || α, bα, а || b, с - секущая, 4 = 20°,Найти: 1, 2, 3, , 5, 6, 7, 8.

  5. Плоскости α и β пересекаются по прямой МК. Прямая а || α, а || β. Докажите, что а || МК.

  6. Дано: ОРТЕ- прямоугольник, К ( ОРТЕ), (РТК) = α. Докажите, что ОЕ || α.

  7. Дано: АВСD- трапеция, М ( АВСD), (СВМ) = α. Докажите, что АD || α.

  8. Дано: АВСD- трапеция, АВ || α, С α, MN – средняя линия трапеции. Докажите, что MN || α.

  9. Дано: АВСD- параллелограмм, К ( АВСD), (АВК) = α. Докажите, что СD || α.hello_html_m477473c3.png

  10. Дано: А α, В α, С α, М – середина АС, К – середина ВС. Докажите, что MК || α.

  11. hello_html_me6b84c4.png


  1. Плоскость α проходит через середины боковых сторон АВ и CD трапеции ABCD- точки M и N. а) Докажите, что AD׀׀α.
    б) Найдите ВС, если
    AD=10 см, MN=8см.
    hello_html_m33a48410.gif

  2. Плоскость α проходит через основание АD трапеции ABCD. Точки M и Nсередины боковых сторон трапеции. а) Докажите, что MN׀׀α.

б) Найдите AD, если BC = 4 см, MN = 6см.

  1. На модели куба укажите плоскости, параллельные прямой DC, прямой DD1.

  2. По готовому рисунку: а) докажите, что: KMEF;
    б) найдите
    KM , если EF=8 см. hello_html_m3e0c4470.jpg
    hello_html_5a486f25.jpg

  3. ABCD – трапеция, AD , E и F – середины AB и CD соответственно.
    Докажите, что
    EF || α.

  4. Дан ΔВСЕ. Плоскость, параллельная прямой СЕ, пересекает BE
    в точке Е1,а ВС - в точке С1. Найдите ВС1, если С1Е1 : СЕ = 3 : 8, ВС = 28 см.

  5. Через основание AD трапеции ABCD проведена плоскость α. ВС α. Докажите, что прямая, проходящая через середины сторон АВ и CD,
    параллельна плоскости α.
    hello_html_m1a1d6694.jpg

  6. Дано: А, В, С, D; В (ACD). Е, F, М, К- середины сторон АВ, ВС, CD, AD;
    AC = 6 см, BD = 8 см. Доказать: EFMK - параллелограмм. Найти: P(EFMK).  

  7. Дано: ΔАВК, М (АВК); E.D- точки пересечения медиан ΔМВК и ΔАВМ;
    АК = 14 см. Доказать: ADEK - трапеция. Найти: DE


Инструкционная карта

ПР №5«Определение взаимного расположения прямых в пространстве. Построение параллельных плоскостей».

Задание:

1)Перепишите и заполните пропуски:

Пример 1. Два отрезка длин а и b упираются концами в две параллельные плоскости. Проекция первого отрезка (длины а) на плоскость равна с. Найдите проекцию второго отрезка, если а = 17 , b = 10, с = 15 см.

Дано: α || β, а = 17 , b = 10, с = 15 см. Найти: х

Решение:

а2 – с2 = b2 – х2, х2 = b2а2 + с2 , х2 = 102 – 172 + 152 =

= 100 – 289 + 225 = …, х = … см.
Ответ: х = 6 см.

Пример 2.

Две параллельные плоскости расстояние между

которыми 2 дм, пересечены прямой, составляющей с каждой из

плоскости угол в 300. Найти длину отрезка этой прямой, заключенной

между плоскостями.

Дано: α || β, АВα = А, АВβ = В, АВС = 30°, АС = 2 дм.

Найти: АВ

Решение: Δ АСВ – прямоугольный, АВС = 30°, АС = 2 дм.

АВ = 2 АС = 2 2 = … дм.
Ответ: АB = 4 дм.

Пример 3. Расстояние между параллельными плоскостями равно 8 см. Отрезок прямой длина которого 17 см расположен между ними так, что его конец принадлежит плоскости. Найти проекцию этого отрезка на другую плоскость.

Дано: α || β, АВα = А, АВβ = В, АВ = 17 см, АС = 8 см.

Найти: ВС

Решение: Δ АСВ – прямоугольный, ВС2 = АВ2 – АС2 = 172 – 82 = 289 – 64 = …, ВС = … см.

Ответ: BС = 15 см.

Пример 4. На параллельных плоскостях α и β, выбрано по паре точек А12 и В12 соответственно так, что прямые А1В1 и А2В2 пересекаются в точке S Вычислите SА1 и SВ2, если А1В1= 6см;
2 = 2,5см; SВ2 : SА2 = 3 : 1 . S

Дано: α || β, А1 А2В1 В2 = S, А1, А2 α, В12 β,

А1В1= 6см; SА2 = 2,5см; SВ2 : SА2 = 3 : 1

Найти: 1, SВ2

Решение: Δ SА1 А2 ~ Δ SВ1В2 , (α || β), SВ2 : SА2 = 3 : 1, SА2 = 2,5см,

2 = 3 2,5 = … см. 1 : SА1 = 3 : 1, А1В1= 6см, SА1 = х ,

( х + 6 ) : х = 3 : 1, 3х = х + 6 , 2х = 6, х = …, SА1 = … см.

Ответ:1 = 3 см, SВ2 = 7,5 см .

Пример 5.

Дано: α || β, а α, bβ, а || b, с - секущая, 1 = 150°,

Найти: 2, 3, 4, 5, 6, 7, 8.

Решение: 3 = 1 = 150°(верт.), 3 = 5 = 150°(н.леж.),hello_html_m40fa69e3.jpg

5 = 7 = 150°(верт.), 1 + 2 = 180°(смежные),

2 = 180° – 1 = 180° – 150° = …°,

2 = 4 = 30°(верт.), 4 = 6 = …°(н.леж.), 6 = 8 = …°(верт.).

Ответ: 3 = 5 = 7 = 150°, 2 = 4 = 6 = 8 = 30°.

2)Решить задачи ( по примерам):

  1. Два отрезка длин а и b упираются концами в две параллельные плоскости. Проекция первого отрезка (длины а) на плоскость равна с. Найдите проекцию второго отрезка, если а = 13 , b = 15, с = 5 см.

  2. Две параллельные плоскости расстояние между которыми 6 дм, пересечены прямой, составляющей с каждой из плоскости угол в 300. Найти длину отрезка этой прямой, заключенной между плоскостями.

  3. Расстояние между параллельными плоскостями равно 10 см. Отрезок прямой длина которого 26 см расположен между ними так, что его конец принадлежит плоскости. Найти проекцию этого отрезка на другую плоскость.

  4. На параллельных плоскостях α и β, выбрано по паре точек А12 и В12 соответственно так, что прямые А1В1 и А2В2 пересекаются в точке S Вычислите SА1 и SВ2, если А1В1= 12см;
    2 = 4,5см; SВ2 : SА2 = 3 : 1.

  5. Дано: α || β, а α, bβ, а || b, с - секущая, 1 = 140°. hello_html_m48661333.jpg

Найти: 2, 3, 4, 5, 6, 7, 8.

3)Решить задачи :

1. Концы двух пересекающихся отрезков AC и BD лежат на двух параллельных плоскостях, причем расстояния между точками одной плоскости равны.
а) Докажите, что АВ ǁ
CD;
hello_html_3cafc7c0.jpg

б) Один из углов четырехугольника ABCD равен 65° . Найдите остальные углы.
2.
Через вершины А и С параллелограмма АВСD проведены параллельные прямые АА1 и СС1, не лежащие в плоскости параллелограмма.

Докажите параллельность плоскостей А1АВ и С1СД.

3.Через точку O, которая находится между параллельными плоскостями α и β, проведены прямые c и d, пересекающие плоскости так, что точки A и B находятся в плоскости α, а точки C и D - в плоскости β ,

AB=15 см, DO=29 см и AC=3AO.Вычислить: BD;CD.                                         

4.Через точку О, лежащую между параллельными плоскостями α и β, проведены прямые l и m. Прямая l пересекает плоскости α и β в точках А1 и А2 соответственно, прямая m – в точках В1 и В2. Найдите длину отрезка А2В2, если А1В1 = 12 см, В1О:ОВ2 = 3 : 4.

5.Дан треугольник АВС. Плоскость, параллельная прямой АВ, пересекает сторону АС этого треугольника в точке А1, а сторону ВС в точке В1.

Найдите длину отрезка А 1В 1,если АВ = 8 см, АА1 : А1С = 5 : 3.

6.Параллелограммы ABCD и ABC1D1 лежат в разных плоскостях.

Докажите, что четырёхугольник CD C1D1 тоже параллелограмм.

7.Три прямые, проходящие через одну точку О, пересекают плоскость α в точках A,B,C, а параллельную ей плоскость β в точках A1,B 1,C 1. Найдите площадь треугольника АВС,hello_html_49c55645.jpg

если угол В1 равен 900, А1 В1= 3 см, В1С1= 4 см, а точка А1 делит ОА пополам .

8.Дано: α || β,  α .Доказать: что m || β.

9.Три прямые, проходящие через одну точку О, пересекают плоскость α в точках A,B,C, а параллельную ей плоскость β в точках A 1,B 1,C 1..
Докажите подобие треугольников ABC и A
 1B 1C 1.
hello_html_m2d5bd222.jpg

10.Две стороны треугольника параллельны плоскости α.

Докажите, что и третья сторона параллельна плоскости α.

Дано: ΔАВС, АВ || α, ВС || α. Доказать: АС || α.

9. 10.

hello_html_79dd18b.gifhello_html_m694c6cb2.gif

Инструкционная карта

ПР №6«Решение задач на определение перпендикулярности прямых, прямой и плоскости».

Задание:

1)Перепишите и заполните пропуски:

Пример 1. Дан параллелепипед АВСDА1В1С1D1 , у которого основание квадрат. Докажите, что а) СD В1С1 , б) С1D1 АD . hello_html_75ed7ad2.jpg

Доказательство: а) СD || A1B1, A1B1 В1С1 СD В1С1 ( по лемме),
б) С
1D1 || ВС , ВС АD С1D1 АD ( по лемме) .
hello_html_m66ec6653.jpg

Пример 2. Прямые АВ, АС и AD попарно перпендикулярны . Найдите отрезок CD, если: АВ = 3 см, ВС = 7 см, AD = 1,5 см;

Дано: АВ, АС и AD попарно перпендикулярны, АВ = 3 см, ВС = 7 см, AD = 1,5 см; Найти: CD

Решение: Δ САВ – прямоугольный, АС2 = СВ2 – АВ2,

АС2 = 72 – 32 = 49 – 9 = …, Δ САD – прямоугольный, СD2 = АС2 + АD2,

СD2 = 40 + 1,52 = 40 + 2,25 = …,

СD = … см. Ответ: СD = 6,5 см.

Пример 3. Через точки А и В проведены прямые, перпендикулярные плоскости α, пересекающие ее в точках С и D соответственно. Найдите расстояние между точками А и В, если АС = 2 м, BD = 3 м, CD = 2,4 м и отрезок АВ не пересекает плоскость α.

Дано: АС α , BD α , АС = 2 м, BD = 3 м, CD = 2,4 м. Найти: A B

Решение: BK = BD – АС = 3 – 2 = …,

Δ BKА – прямоугольный, AB2 = АK2 + BK 2 = СD2 + BK2,

AB2 = 2,42 + 12 = 5,76 + 1 = …, AB = … см.

Ответ: AB = 2,6 см.

Пример 4. Телефонная проволока длиной 15 м протянута от телефонного столба, где она прикреплена на высоте 8 м от поверхности земли, к дому, где ее прикрепили на высоте 20 м. Найдите расстояние между домом и столбом, предполагая, что проволока не провисает.

Дано: AB = 15 м, АС = 8 м, BD = 20 м,

Найти: CD

Решение: BK = BD – АС = 20 – 8 = …,

Δ BKА – прямоугольный, АK2 = AB2 BK 2 = 152 122 = 225 – 144 = …, АK = … см.

CD = АK = 9 см. Ответ: CD = 9 см.

Пример 5. К плоскости треугольника из центра, вписанной в него окружности радиуса 0,7 м восставлен перпендикуляр длиной 2,4 м. Найдите расстояние от конца этого перпендикуляра до сторон треугольника.

Дано: Δ АВС, О – центр , вписанной в него окружности,

ОК = r = 0,7 м, ОМ (АВС), ОМ = 2,4 м, Найти: МК

Решение: ΔМОК - прямоугольный,

МК2 = ОК2 + ОМ2 = 0,72 + 2,42 = 0,49 + 5,76 = …,

МК = … м.

Ответ: МК = 2,5 м.hello_html_m634b7c0d.jpg

Пример 6. Дано: ΔАВС; АВ = АС = ВС; CD  (ABC); AM = MB, 

DM = 15, CD = 12 Найти: SΔADB.

 Решение:1) CD  (ABC)  CD  AC и CD  ВС, тo есть 

ACD = BCD = 90° и ΔADC, ΔBDC -прямоугольные.

2) ΔADC = ΔBDC (по двум катетам): DC - общий, AC = ВС (по условию). Значит, AD =BD (как соответствующие в равных треугольниках), тогда ΔADB - равнобедренный (по определению) и DM - медиана. Следовательно, DM - высота (по свойству медианы равнобедренного треугольника).

3) DC  МС  DCM = 90° и ΔMCD - прямоугольный. По теореме Пифагора:  MD2 = DC2 + МС2. Тогда  152 = 122 + МС2 , МС2 = 225 –144 = …, МС = …

4) ΔМСВ - прямоугольный (CMB = 90°, так как СМ - медиана и высота в ΔАВС - равностороннем),

В = 60°,sinВ = МС : ВС, тогда ВС = МС : sin60° = 9 : = 18: = 6,АВ = ВС (по условию),

5)  Ответ: .hello_html_4318143f.jpg

Пример 7. Дано: АВ - отрезок; α; АВ  α; О - середина АВ, О  α;

ХА = ХВ. (рис. 2). Доказать: X  α.

Доказательство:

1) Если X  АВ, то Х = О, и поэтому X  а.

2) Если X  АВ, то ХО - медиана ΔАХВ. ΔАХВ - равнобедренный (по определению), значит, ХО - высота (по свойству медианы равнобедренного треугольника), то есть ХО АВ. Таким образом, О  ХО, О  АВ и ХО  АВ, следовательно, ХО  а и X  а.

Пример 8. Дано: α; ABCD - параллелограмм; BD α; А α, C α, AB = 7 см .hello_html_7046f1aa.jpg

Найти: РABCD.

Решение: 1)Так как А  α, С  α, то AC α;BD α;=> BD AC;

  (по определению прямой, перпендикулярной плоскости). Значит, ABCD - ромб (по признаку). Тогда АВ = ВС = CD =AD = 7 см (по определению ромба).

2) РABCD = 4 · 7 = …(см). Ответ: 28 см.

2)Решить задачи ( по примерам):

  1. Дан параллелепипед АВСDА1В1С1D1 , у которого основание квадрат.

Докажите, что а) С1D1 ВС, б) СD А1D1 .

  1. Прямые АВ, АС и AD попарно перпендикулярны . Найдите отрезок CD, если: АВ = 6 см, ВС = 14 см, AD = 3 см;

  2. Через точки А и В проведены прямые, перпендикулярные плоскости α, пересекающие ее в точках С и D соответственно. Найдите расстояние между точками А и В, если АС = 8 см, BD = 20 см, CD = 5см и отрезок АВ не пересекает плоскость α.

  3. Телефонная проволока длиной 26 м протянута от телефонного столба, где она прикреплена на высоте 6 м от поверхности земли, к дому, где ее прикрепили на высоте 30 м. Найдите расстояние между домом и столбом, предполагая, что проволока не провисает.

  4. К плоскости треугольника из центра, вписанной в него окружности радиуса 1 м восставлен перпендикуляр длиной 2,4 м. Найдите расстояние от конца этого перпендикуляра до сторон треугольника.

  5. Дано: ΔАВС; АВ = АС = ВС; CD  (ABC); AM = MB, DM = 17, CD = 8. Найти: SΔADB.

  6. Дано: АВ - отрезок; α; АВ  α; О - середина АВ, О  α; CА = CВ.
    Доказать:
    C  α.
    hello_html_22068a07.jpg

  7. Дано: α; ABCD - параллелограмм; BD α; А α, C α,

AB = 10 см .Найти: РABCD.

3)Решить задачи :

  1. Дано: ABCD квадрат; AM - прямая; АМ  (ABCD); АС  BD = О. Доказать: a) BD  (АМО); б) МО  BD.

  2. Прямые АВ и CD перпендикулярны некоторой плоскости и пересекают ее в точках В и D соответственно. Найдите AС, если АВ = 9, CD = 15, BD = 8.

  3. Отрезок МН пересекает некоторую плоскость в точке К. Через концы отрезка проведены прямые HP и ME, перпендикулярные плоскости и пересекающие ее в точках Р и Е. Найдите РЕ, если HP = 4 см, НК = 5 см, ME = 12 см.

  4. Треугольник ABC правильный, точка О - его центр. Прямая ОМ перпендикулярна плоскости AВС. Докажите, что МА = MB = МС. Найдите МА,
    если АВ = 6 см, МО = 2 см.
    hello_html_m754188c5.jpg

  5. ABCD - квадрат . АЕ - перпендикулярно плоскости квадрата, К  BE. Найти: (ВС; АК).

  6. ABCD прямоугольник. Отрезок АЕ перпендикулярен к плоскости ABC. ЕВ = 15, ЕС = 24, ED = 20. Докажите, что треугольник EDC прямоугольный, и найдите АЕ.

  7. Точка А принадлежит окружности, АК - перпендикуляр к ее плоскости, АК = 1 см, АВ - диаметр, ВС — хорда окружности, составляющая с АВ угол 45°. Радиус окружности равен 2 см. Докажите, что треугольник КСВ прямоугольный, и найдите КС.

  8. Дано: α; АВ - отрезок;AB α = O,AD α; BC α; AD α = D,BC α = C, AD = 6 см, BC = 2 см, OC = 1,5 см. Найти: АВ.


Инструкционная карта

ПР №7«Вычисление перпендикуляра и наклонной. Вычисление угла между плоскостями».

Задание:

1) а) Записать по рисунку:

  • какой отрезок является перпендикуляр, наклонная, проекция наклонной,

  • угол между наклонной и плоскостью α.

АС - …, АВ - …, СВ – …, АВ2 = ВС2 + АС2.

- угол между наклонной и плоскостью α.

б)Перепишите и заполните пропуски:

Пример 1. Из точки, не принадлежащей данной плоскости , проведены к ней
две наклонные, равные 10см и 18см. Сумма длин их проекций на

плоскость равна 16см. Найти проекцию каждой наклонной.(рис.1)

Дано: ОС - перпендикуляр, АС и ВС - наклонные, АО и ОВ – их проекции, рис.1

АС = 10 см, СВ = 18 см, АО + ОВ = 16 см,

Найти: АО, ОВ

Решение: АС = 10, СВ = 18, АО + ОВ = 16, АО = х, ОВ = 16 – х,

АС2 АО2 = ВС2 – ОВ2 , 102 х2 = 182 – (16 х)2, 100 х2 = 324 – 256 + 32 х х2 ,

32 х = 32, х = … , АО = 1, ОВ = 16 – 1 = .... Ответ: 1 и 15 см.

Пример 2. Из точки к плоскости проведены две наклонные. Одна из них длиной 12см наклонена к плоскости под углом 60°, проекция другой на эту плоскость равна 6 см.

Найти длину этой наклонной.

Дано: ОС - перпендикуляр, АС и ВС - наклонные, АО и ОВ – их проекции,

СА = 12 см , САО = 60°, ОВ = 6 см ,

Найти: СВ

Решение: Δ АОС- прямоугольный, АСО = 90 ° – 60 ° = 30°, АО = СА : 2 = 12: 2 = … ,

СО2 = СА2 –АО2 = 122 – 62 = 144 – 36 = … ,

СВ2 = СО2 + ОВ2 = 108 + (6 )2 = 108 + 36 6 = 108 + 216 = … , СВ = … см. Ответ: 18 см.

Пример 3. Из точки С к данной плоскости проведены перпендикуляр СО = 6см и две наклонные. Каждая из наклонных образует с плоскостью угол 60°. Угол между наклонными 120°. Найти расстояние между основаниями наклонных.

Дано: ОС - перпендикуляр, АС и ВС - наклонные, АО и ОВ – их проекции,

СО = 6см, САО = СВО = 60°, АСВ = 120°,

Найти: АВ
Решение: sin САО = СО : АС, АС = ВС = СО : sin САО = 6: sin60 ° = 6 : = 12 : = 4 ,

Δ АВС – равнобедренный, АВ2 = АС2 + ВС2 – 2АС ВС cos АСВ =

= (4)2 + (4)2 – 24 cos 120° = 16 3 + 16 3 - 216 3( – ) = 48 + 48 + 48 = … ,

АВ = … см. Ответ: АВ = 12 см.

Пример 4. Из точки С к данной плоскости проведены перпендикуляр СО и две наклонные СВ и АС. ОВ= 4,САО = 30°, СВО = 60°, а угол между наклонными 90°. Найти расстояние между основаниями наклонных.

Дано: ОС - перпендикуляр, АС и ВС - наклонные, АО и ОВ – их проекции,

ОВ= 4,САО = 30°, СВО = 60°, АСВ = 90°,

Найти: АВ

Решение: ΔСОВ – прямоугольный, СВО = 60°, ОСВ = 90 ° - 60 ° = 30 °,

ВС= 2 ОВ = 24 = … , СО2 = ВС2 – ОВ2 = 82 – 42 = 64 – 16 = … , СО = = 4,

АС = 2 СО = 24 = … , ΔАСВ - прямоугольный, АВ2 = АС2 + ВС2 = (8)2 + 82 =

= 64 3 + 64 = … , АВ = … см. Ответ: АВ = 16 см.
Пример 5. Диагонали квадрата АВСD пересекаются в точке О.


Из точки О проведён к плоскости квадрата перпендикуляр ОМ. Найти расстояние от точки М до

стороны ВС, если AD = 6см, ОМ = 4см. (рис.2)

Дано: АВСD - квадрат, ОМ - перпендикуляр,
О - точка пересечения диагоналей квадрата,

МК - расстояние от точки М до стороны ВС, AD = 6см, ОМ = 4см.

Найти: МК

Решение: ОК = АВ : 2 = AD : 2 = 6 : 2 = … , ΔМОК - прямоугольный, Рис.2

МК2 = ОМ2 + ОК2 = 42 + 32 = 16 + 9 = … , МК = ... Ответ: МК = 5 см.
Пример 6. Прямые АВ, АС и AD попарно перпендикулярны . Найдите отрезок CD, если: АВ = 3 см, ВС = 7 см, AD = 1,5 см;

Дано: АВ, АС и AD попарно перпендикулярны, АВ = 3 см, ВС = 7 см, AD = 1,5 см; Найти: CD

Решение: Δ САВ – прямоугольный, АС2 = СВ2 – АВ2, АС2 = 72 – 32 = 49 – 9 = … ,

Δ САD – прямоугольный, СD2 = АС2 + АD2, СD2 = 40 + 1,52 = 40 + 2,25 = … ,

СD = … см. Ответ: СD = 6,5 см.hello_html_75ed7ad2.jpg

Пример 7. Дан параллелепипед АВСDА1В1С1D1 , у которого основание квадрат.

Докажите, что а) СD В1С1 , б) С1D1 АD .

Доказательство: а) СD || A1B1, A1B1 В1С1 СD В1С1 ( по лемме),

б) С1D1 || ВС , ВС АD С1D1 АD ( по лемме) .

2)Решить задачи ( по примерам):

  1. Из точки, не принадлежащей данной плоскости, проведены к ней две наклонные,
    равные 20 см и 36 см. Сумма длин их проекций на плоскость равна 32 см.
    Найти проекцию каждой наклонной.

  2. Из точки к плоскости проведены две наклонные. Одна из них длиной 24 см наклонена к плоскости под углом 60°, проекция другой на эту плоскость равна 12 см. Найти длину этой наклонной.

  3. Из точки С к данной плоскости проведены перпендикуляр СО = 12 см и две наклонные. Каждая из наклонных образует с плоскостью угол 60°. Угол между наклонными 120°.
    Найти расстояние между основаниями наклонных.

  4. Из точки С к данной плоскости проведены перпендикуляр СО и две наклонные СВ и АС.
    ОВ= 8,
    САО = 30°, СВО = 60°, а угол между наклонными 90°. Найти расстояние между основаниями наклонных.

  5. Диагонали квадрата АВСD пересекаются в точке О. Из точки О проведён к плоскости квадрата перпендикуляр ОМ. Найти расстояние от точки М до стороны ВС, если AD = 12 см, ОМ = 8 см.

  6. Прямые АВ, АС и AD попарно перпендикулярны.
    Найдите отрезок CD, если: АВ = 6 см, ВС = 14 см, AD = 3 см;

  7. Дан параллелепипед АВСDА1В1С1D1 , у которого основание квадрат.
    Докажите, что
    а) С1D1 ВС, б) СD А1D1 .

3)Решить задачи :

  1. Дано: АС - перпендикуляр, АВ - наклонная,
    а) АВ = 10 см, ВС = 6 см, АС = ?, б) АС = 12 см, ВС = 5 см, АВ = ? (Указание:
    АВ2 = ВС2 + АС2 )

  2. Дано: Δ АВС – равнобедренный, АК(АВС), АК = 12 см, АВ = АС = 5 см, ВС = 6 см,
    КМ
    ВС. Найти: КМ, АМ.

(Указание: АВ = АС => КВ = КС => Δ СКВ – равнобедренный, КМ ВС => ВМ- медиана,

ВМ = МС = ВС : 2, КС2 = АК2 + АС2 , КМ2 = КС2 - МС2 , АМ2 = АС2 - МС2 )

  1. Дано: АО - перпендикуляр, АВ и АС - наклонные, АВ = АС, ОАВ = ВАС = 60°,
    АО = 2,5 см.
    Найти: ВС. (Указание: Δ ВАС – равносторонний, ВС = АВ = АС = 2АО)

  2. Телефонная проволока длиной 15 м протянута от телефонного столба, где она прикреплена на высоте 8 м от поверхности земли, к дому, где ее прикрепили на высоте 20 м. Найдите расстояние между домом и столбом, предполагая, что проволока не провисает.

Дано: AB = 15 м, АС = 8 м, BD = 20 м, Найти: CD.

(Указание: Δ BKА – прямоугольный, АK2 = AB2 - BK 2)

  1. Дан куб АВСDА1В1С1D1 . Найдите следующие двугранные углы: а) АВ В1С , б) АDD1В,
    в) А
    1ВВ1К, где К- середина А1D1.

  2. Из вершины равностороннего треугольника АВС проведен перпендикуляр АК к плоскости треугольника. Найдите длину АК, если ВС = 3 см, КС = 3 см.

  3. Найдите тангенс угла между диагональю куба и плоскостью одной из его граней.

Инструкционная карта

ПР №8 «Перпендикулярность двух плоскостей.Изображение пространственных фигур».

Задание:hello_html_41ca87ca.jpg

1)Перепишите и заполните пропуски:

Пример 1. Из точек А и В, лежащих в двух перпендикулярных плоскостях, опущены перпендикуляры АС и ВD на прямую пересечения плоскостей. Найдите длину отрезка АВ если:

а) АС = 6 м, ВD = 7 м, СD = 6 м, б) АD = ВС = 5 м, СD = 1 м.

Решение: а) Пусть плоскости α и β перпендикулярны. СD – прямая пересечения плоскостей , тогда АС СВ и ВD АD. Тогда в Δ АСВ: АВ2 = АС2 + ВС2, но из Δ СDВ следует ,что: ВС2 = СD2 + ВD2 , так что АВ2 = АС2 + СD2 + ВD2.

АВ2 = 62 + 72 + 62 = 36 + 49 + 36 = …, АВ = …

б) АВ2 = АС2 + ВС2, но из Δ СDА следует ,что: АС2 = АD2 СD2 ,

так что АВ2 = АD2 СD2 + ВС2. АВ2 = 52 12 + 52 = 251 + 25 = …, АВ = …

Ответ: а) 11 м, б) 7 м.hello_html_552bcaf5.jpg

Пример 2. Точка А находится на расстоянии а = 24 см и b = 10 см от двух перпендикулярных плоскостей α и β. Найдите расстояние от этой точки до прямой пересечения плоскостей.

Решение: Пусть α β и α β = с. Проведем перпендикуляры АВ, АD, АС. Тогда четырехугольник АВСD – прямоугольник. АС2 = а2 + b2 ,

АС –искомое расстояние. ВС - проекция АС на плоскость α, поэтому по теореме о 3 – х перпендикулярах ВС с, ВС β. Так как АD β, то по теореме АD||ВС, а, значит, АD и ВС лежат в одной плоскости.

Итак , АС2 = 242 + 102 = 576 + 100 = … , АС = …hello_html_mf971f4e.jpg

Ответ: АС = 26 см.

Пример 3. Плоскости α и β перпендикулярны. В плоскости α взята точка А, расстояние от которой до прямой с ( линия пересечения плоскостей ) равно 0,5 м. В плоскости β проведена прямая b, параллельная прямой с и отстоящая от нее на 1,2 м. Найдите расстояние от точки А до прямой b.

Решение: Пусть α β , b || с, ВС = 1, АВ = 0,5м , где АВ с и ВС b.

Тогда по теореме о 3 – х перпендикулярах АС b. Так что

АС – искомое расстояние и АС2 = АВ2 + ВС2 = 1,22 + 0,52 = 1,44 + 0 ,25 = …, АС = …

Ответ: АС = 1,3 м.

Пример 4. Перпендикулярные плоскости α и β пересекаются по прямой с. Плоскости α проведена прямая а|| с, в плоскости β – прямая b || с. Найдите расстояние между прямыми а и b , если расстояние между прямыми а и с равно 1,5 м, а между прямыми b и с – 0,8 м. hello_html_m209580fa.jpg

Решение: Возьмем в плоскости α точку А на прямой а. По теореме о 3 – х параллельных прямых получаем, что а || b (так как а || с, b ||с). Проведем

АС с и СВ b. Тогда по теореме о 3 – х перпендикулярах АВ b.

Так что АВ – искомое расстояние и АВ СВ, так как α β
( по условию), из прямоугольного треугольника АВС по теореме Пифагора имеем: АВ
2 = СВ2 + АС2 = 1,52 + 0,82 = 2,25 + 0,64 = … , АВ = …

Ответ: АВ = 1,7 м.

Пример 5. Построить изображение правильного шестиугольника .

Решение: Разобьем правильный шестиугольник на три части: прямоугольник FBCE и два равнобедренных треугольника Δ FAB и Δ CDE. Построим вначале изображение прямоугольника FBCE – произвольный параллелограмм FBCE. Находим на изображении точку О и проводим

через нее прямую, параллельную ВС и FE, получим при этом точки N и К

(точки пересечения прямой и сторон параллелограмма). Откладываем от точек N и К

на прямой такие же отрезки, как от центра О до точек N и К, получаем две оставшиеся

вершины правильного шестиугольника А и D.

ABCDFE – искомое изображение.

hello_html_m5af7163d.gif

2)Решить задачи ( по примерам):

  1. Из точек А и В, лежащих в двух перпендикулярных плоскостях, опущены перпендикуляры АС и ВD на прямую пересечения плоскостей. Найдите длину отрезка АВ если:

а) АС = 3 м, ВD = 4 м, СD = 12 м, б) АD = 4 м, ВС = 7 м, СD = 1 м.

  1. Точка А находится на расстоянии а = 17 см и b = 8 см от двух перпендикулярных плоскостей α и β. Найдите расстояние от этой точки до прямой пересечения плоскостей.

  2. Плоскости α и β перпендикулярны. В плоскости α взята точка А, расстояние от которой до прямой с ( линия пересечения плоскостей ) равно 0,9 м. В плоскости β проведена прямая b, параллельная прямой с и отстоящая от нее на 1,2 м. Найдите расстояние от точки А до прямой b.

  3. Перпендикулярные плоскости α и β пересекаются по прямой с. Плоскости α проведена прямая а|| с, в плоскости β – прямая b || с. Найдите расстояние между прямыми а и b , если расстояние между прямыми а и с равно 4,5 м, а между прямыми b и с – 2,4 м.

  4. Построить изображение правильного пятиугольника .

3)Решить задачи :

  1. Треугольник AM В и прямоугольник ABCD расположены так, что их плоскости взаимно перпендикулярны. Докажите, что угол MAD — прямой.

  2. В прямоугольном параллелепипеде ABCDA1B1C1D1 точки Е, F и К — середины ребер

А1В1, A1D1 и AD соответственно; АВ = 4,АА1 = 6, A1C =. 1) Постройте сечение параллелепипеда плоскостью, проходящей через точки Е, F и К, и докажите, что плоскости сечения и основания взаимно перпендикулярны. 2) Найдите AD.

  1. Прямоугольник ABCD и параллелограмм ВЕМС расположены так, что их плоскости взаимно перпендикулярны. Докажите, что L MCD — прямой.

  2. В прямоугольном параллелепипеде ABCDA1B1C1D1 точка Е — середина С1 D1, AD = 5, АВ= 4, В1D = . 1)Постройте сечение параллелепипеда плоскостью, проходящей через AD и точку Е и докажите, что плоскость сечения перпендикулярна плоскости боковой грани DD1C1C.

2)Найдите АА1.

  1. Два правильных треугольника ABC и DBC расположены так, что их плоскости взаимно перпендикулярны. Найдите тангенс двугранного угла, образованного плоскостями ADC и ABC.

  2. В прямоугольном параллелепипеде ABCDA1B1C1D1 основание ABCD — квадрат,
    AD = 2; АС1 = 2. 1)Найдите СС1. 2)Докажите, что плоскости АСС1 и BB1D1 взаимно перпендикулярны.

  3. Сторона правильного треугольника ABC равна 4. Треугольник DBC — равнобедренный
    (DB = DC). Их плоскости взаимно перпендикулярны. Плоскость ADC составляет с плоскостью АВС угол 60°. Найдите площадь треугольника DBC.

  4. В прямоугольном параллелепипеде ABCDA1B1C1D1 боковая грань DD1C1C — квадрат,
    DC = 3; BD1 = . 1)Найдите ВС. 2)Докажите, что плоскости BCD1 и DC1B1 взаимно перпендикулярны.

  5. Дана параллельная проекция треугольника. Как построить проекции медиан этого треугольникаhello_html_56a3268d.png

  6. а) Докажите, что параллельная проекция центрально - симметричной фигуры также является центрально – симметричной. б) по рис.











Инструкционная карта

ПР № 9«Построение многогранников. Вычисление элементов призмы».

Задание:

1) а) Л.С. Атанасян. Геометрия 10-11 класс, стр. 57,59. Построить многогранники.
б) Перепишите и заполните пропуски:
Пример 1. Основанием прямой призмы является равнобедренный треугольник, в котором высота проведенная к основанию равняется 8см.Высота призмы равняется 12см. Найдите полною поверхность призмы если боковая грань что содержит основание треугольника - квадрат. 
Решение:  Площадь поверхности призмы будет равна сумме площадей оснований и сумме площадей боковых поверхностей, то есть  S = 2S1 + S2 + 2S3 , где S1 - площадь основания призмы, S2 - площадь боковой поверхности, содержащей основание, S3 - площадь боковой поверхности, содержащей стороны равнобедренного треугольника. (Они равны, так как стороны основания равны в следствие того, что треугольник равнобедренный, а вторые стороны равны высоте призмы) .
Поскольку боковая грань, содержащая основание треугольника, является квадратом, то основание треугольника также равно 12 см. (основание треугольника одновременно является стороной грани).
 
Таким образом, зная высоту и основание равнобедренного треугольника можно найти его остальные стороны и площадь.
  S1 = 1/2ah = 1/2 12 8 = 6 8 = … см2 . 
Катеты, соответственно равны (у нас высота, являющаяся в равнобедренном треугольнике одновременно и медианой 12 /2 = 6 см, с каждым из катетов образует прямоугольный треугольник) по теореме Пифагора
  62 + 82  = 102 , Таким образом
S
2 = 1212 = … см2 . S3 = 1012 =… см2 . S = 2S1 + S2 + 2S3 = 2 48 + 144 + 2 120 = … см2 . 
Ответ: … см2.

Пример 2. В основании прямой треугольной призмы лежит прямоугольный треугольник с катетами 8 и 6 см. Найти боковое ребро призмы, если ее боковая поверхность равна 120 квадратных сантиметров. 
Решение: Сначала найдем гипотенузу основания призмы. AB2 = AC2 + BC2 , AB2 = 82 + 62 ,
AB
2 = 64 + 36 = …, AB = … .
Обозначим боковое ребро призмы как  h . Боковое ребро одновременно является и высотой призмы, поскольку по условию задачи призма является прямой. Тогда площадь боковой поверхности призмы является суммой площадей трех прямоугольников - ACC
1A1, CBB1C1 и ABB1A1 или, если подставить известные значения катетов основания призмы, то 10h + 6h + 8h = 120,  24h = 120, h =…, 
Ответ: ребро прямоугольной призмы с прямоугольным треугольником в основании равно 5 см. 

Пример 3 . В правильной четырёхугольной призме площадь основания 144 см2, а высота 14 см. Найти диагональ призмы и площадь полной поверхности. 
Решение: Правильный четырехугольник - это квадрат, сторона основания равна а = = … см. 
Откуда диагональ основания правильной прямоугольной призмы будет равна 
d
2 =122 + 122  = …, d = 12 ,
Диагональ правильной призмы образует с диагональю основания и высотой призмы прямоугольный треугольник. Соответственно, по теореме Пифагора диагональ заданной правильной четырехугольной призмы будет равна: d
12 = ( 12)2 + 142 = 288 + 196 = …, d1 = … см.
Ответ: 22 см .

Пример 4. Основанием прямой призмы ABCD A1B1C1D1 является параллелограмм ABCD со сторонами 4 см и 4 см и углом, равным 30 °. Диагональ AC1 призмы образует с плоскостью основания угол в 60 °. Найдите площадь боковой поверхности призмы.
Решение: Поскольку сумма соседних углов параллелограмма равна 180 градусам, то углы B и D. будут равны 180° – 30° = 150 °. 
Диагональ параллелограмма AC, таким образом, образует треугольник ACD с углом C равным 150
°. Применим теорему косинусов, при этом обозначив диагональ параллелограмма как d, а  стороны параллелограмма как a и b.
Учтем, что
  cos( 150° ) = – / 2. Получим: 
d
2 = a2 + b2 – 2abcos 150° , d2 = 16 + 48 – 2 4 4 (/ 2 ) = 16 + 48 + 48 = …,   
d = 4 , AC = 4 .
Зная величину диагонали параллелограмма, найдем высоту параллелограмма. Треугольник, который образует диагональ AC
1 ( AC1С ) с основанием призмы, согласно условию задачи (призма - прямая)
hello_html_a20ca67.png

является прямоугольным. Угол C1AC по условию равен 60 градусов. Для прямоугольного треугольника тангенс угла C1AC равен отношению противолежащего катета к прилежащему, то есть tg ( C1AC ) = C1С / AC . Учтем, что тангенс 60 градусов равен tg 60° = . 
Соответственно,
 C1С  AC tg ( C1AC ) , C1С = 4 tg 60° , C1С = 4 .
Зная высоту призмы, определим площадь ее боковой поверхности:
  S = 2ha + 2hb, 
S = 2
 4 4  + 24 = 96+ ≈ 327,31   
Ответ: 96 + 32 ≈ 327,31.

Пример 5. Определите полную поверхность правильной четырехугольной призмы, если ее диагональ равна 5 см, а диагональ боковой грани равна 4 см. 
Решение:  Поскольку в основании правильной четырехугольной призмы лежит квадрат, то сторону основания (обозначим как a) найдем по теореме Пифагора: 
a
2 + a2 = 52 , 2a2 = … , a =
Высота боковой грани (обозначим как h) тогда будет равна: h
2 + 12,5 = 42 , h2 + 12,5 = 16 ,h2 = … ,
h = .
Площадь полной поверхности будет равна сумме площади боковой поверхности и удвоенной площади основания S = 2a
2 + 4ah , S = 25 + 4 ,S = 25 + 4 ,S = 25 + 4
S = 25 + 4  , S = 25 + 10≈ 51,46 см
2 . 
Ответ: 25 + 10≈ 51,46 см2 .

2)Решить задачи ( по примерам):

  1. Основанием прямой призмы является равнобедренный треугольник, в котором высота проведенная к основанию равняется 6см. Высота призмы равняется 16 см. Найдите полною поверхность призмы если боковая грань что содержит основание треугольника - квадрат.

  2. В основании прямой треугольной призмы лежит прямоугольный треугольник
    с катетами 4 и 3 см. Найти боковое ребро призмы, если ее боковая поверхность
    равна 120 квадратных сантиметров.
     

  3. В правильной четырёхугольной призме площадь основания 36 см2, а высота 7см.
    Найти диагональ призмы и площадь полной поверхности. 

  4. Основанием прямой призмы ABCD A1B1C1D1 является параллелограмм ABCD
    со сторонами 2 см и 2
    см и углом, равным 30°. Диагональ AC1 призмы образует с плоскостью основания угол в 60°. Найдите площадь боковой поверхности призмы.

  5. Определите полную поверхность правильной четырехугольной призмы, если ее диагональ равна 10 см, а диагональ боковой грани равна 8 см. 

3) Решить задачи:

  1. Дана прямая призма, в основании которой прямоугольный треугольник Δ АВС, В = 90º,
    ВDD 1В1 – сечение , ВD АС, АА1 = 10 см, АD = 27 см, DС = 12 см.
    Найти площадь сечения Sсеч.
    hello_html_m57d4dae2.jpg

  2. Дана прямая треугольная призма со сторонами a=5,b=12 ,c=13 см и высотой h= 8 см. Найдите площадь боковой и полной поверхности призмы.

  3. Основанием прямой призмы АВСА1В1С1 является прямоугольный треугольник АВС с прямым углом В. Через ребро ВВ1 проведено сечение ВВ1D1D перпендикулярное к плоскости грани АА1С1С.

Найдите площадь сечения, если АА1 =14 см, АD =25 см, DС =36 см.

  1. Основание прямой призмы - треугольник со сторонами AB=5 и BC=12 см и углом в 90° между ними. Наибольшая из площадей боковых граней равна S наиб.=39 см2. Найдите площадь боковой поверхности призмы.hello_html_m5af2f329.jpg

  2. Боковое ребро наклонной четырехугольной призмы

равно l = 12 см, а перпендикулярным сечением является ромб со стороной a=8 см. Найдите площадь боковой поверхности призмы.



Инструкционная карта

ПР № 10«Вычисление элементов пирамиды, правильной пирамиды, усеченной пирамиды.».

Задание:

1) Перепишите и заполните пропуски:
Пример 1. В основании пирамиды лежит прямоугольный треугольник, один из катетов которого 8см, а радиус описанной около него окружности равен 5 см. Основанием высоты этой пирамиды является середина гипотенузы. Высота пирамиды равна 12см. Вычислить боковые ребра пирамиды. 
Решение: В основании пирамиды лежит прямоугольный треугольник. Центр окружности, описанной около прямоугольного треугольника, лежит на его гипотенузе. Соответственно, AB = 10 см, AO = 5 см. 
Поскольку высота ON = 12 см, то величина ребер AN и NB равна
 
AN
2 = AO2 + ON2 , AN2 = 52 + 122 = …, AN = = …. 
Поскольку нам известна величина AO = OB = 5 см и величина одного из катетов основания (8 см), то высота, опущенная на гипотенузу, будет равна
  CB2 = CO2 + OB2 , 64 = CO2 + 25 , CO2 = 39 ,
CO = . Соответственно, величина ребра CN будет равна :CN2 =  CO2 + NO2 , CN2 = 39 + 144 = …,
CN = .
Ответ: 13, 13 , .
hello_html_3a511f3b.png

Пример 2. Боковая грань правильной треугольной пирамиды представляет собой правильный треугольник, площадь которого 16 корней из 3 см2 (16). Вычислить периметр основания пирамиды. 
Решение
: Правильный треугольник - это равносторонний треугольник. Соответственно, боковая грань пирамиды представляет собой равносторонний треугольник. 
Площадь равностороннего треугольника равна:
 . 
Соответственно:
 16 = a2 / 4 , 16 = a2 / 4 , a2 = 64 ,a = … см .
Основанием правильной треугольной пирамиды является правильный (равносторонний) треугольник. Таким образом, периметр основания пирамиды равен
  Р = 83 = … см .
Ответ: 24 см. 

Пример 3. Высота правильной треугольной пирамиды 4 см, а ее апофемы 8 см. Вычислите площадь боковой поверхности пирамиды. 
Решение:  Исходя из того, что MK = 8, MO = 4, синус угла OKM равен  MO/MK = 1/2 , откуда угол равен arcsin 1/2 = 30 °. Откуда  KO / MK = cos 30° , KO / 8 = cos 30° , KO = 8 cos 30° .
 KO = 8/2 = 4 .
Тогда по свойству равностороннего треугольника
  КО = r = a/6. 4 = a /6 , a = 24. 
Теперь, зная размер основания боковой грани и ее апофему, найдем площадь боковой грани как площадь равнобедренного треугольника:
 Sт = 1/224 8 = 12 8 = … см2 .
Откуда площадь боковой поверхности пирамиды
 S = 3 Sт = 3 96 = … см2 . 
Ответ: 288 см2.

Пример 4. Высота и сторона основания правильной четырехугольной пирамиды соответственно равны 24 и 14. найдите апофему пирамиды. 
Решение: Поскольку пирамида правильная, то в ее основании лежит правильный четырехугольник - квадрат. Кроме того, высота пирамиды проецируется в центр квадрата. Таким образом, катет прямоугольного треугольника, который образован апофемой пирамиды, высотой и отрезком, их соединяющим, равен половине длины основания правильной четырехугольной пирамиды.  Откуда по теореме Пифагора длина апофемы будет найдена из уравнения:  72 + 242 = x2 , x2 = …,  x = ….  Ответ: 25 см .
hello_html_m105d4bfc.png

Пример 5. Дано: усеченная правильная пирамида, n = 3, h = 4, a1= 16 , a2= 10 . Надо найти площадь полной поверхности усеченной пирамиды .

Решение: r1= a1 / 2  = 16  : 2  = 16 : 2 = …, r2= a2 / 2  = 10  : 2  = 10 : 2 = … ,

l2 = h2 + (r2 r1)2, l2 = 42 + (5 8)2 = 16 + 9 = …, l = … Sn =  /4 (a12 + a22) + 1,5 l(a1 + a2) .

Sn =  /4 ((16 )2 + (10 )2) + 1,5 5(16  + 10 ) =  /4 (768 + 300) + 1,5 5 = =267 + 195  =   .

Ответ: 462 

Пример 6. Дано: усеченная правильная пирамида, n = 4, h = 3, a1= 16, a2= 8 . Надо найти площадь полной поверхности усеченной пирамиды .

Решение: r1= a1 / 2= 16: 2= …, r2= a2 / 2= 8  : 2  = …,

l2 = h2 + (r2 r1)2, l2 = 32 + (4 8)2 = 9 + 16 = …, l = ….

Sn = (a12 + a22) + 2 l(a1 + a2) .Sn = (162 + 82) + 2 5(16 + 8) = 320 + 240 = … .

Ответ: 560

Пример 7. Дано: усеченная правильная пирамида, n = 6, h = 2, a1= 2 , a2= 6 . Надо найти площадь полной поверхности усеченной пирамиды .

Решение: r1= a1 / 2  = 2  : 2  =  , r2= a2 / 2  = 6  : 2  = 3 ,

l2 = h2 + (r2 r1)2, l2 = 22 + ( )2 = 4 + 12 = …, l = ….

Sn =3  /2 (a12 + a22) + 3 l(a1 + a2) .Sn =3  /2 (22 + 62) + 3 4(2 + 6) = …   + .

Ответ: 60   + 96

Пример 8. Дано: усеченная правильная пирамида, n = 4, h = 3, r1=2, r2= 6 . Надо найти площадь полной поверхности усеченной пирамиды .

Решение: l2 = h2 + (r2 r1)2, l2 = 32 + (6 2)2 = 9 + 16 = …, l = ….

Sn = 4 (r12 + r22) + 4 l(r1 + r2) . Sn = 4 (22 + 62) + 2 5(2 + 6) = 160 + 80 = … .

Ответ: 240.

2)Решить задачи ( по примерам):

  1. В основании пирамиды лежит прямоугольный треугольник, один из катетов которого 16 см, а радиус описанной около него окружности равен 10 см. Основанием высоты этой пирамиды является середина гипотенузы. Высота пирамиды равна 24 см. Вычислить боковые ребра пирамиды. 

  2. Боковая грань правильной треугольной пирамиды представляет собой правильный треугольник, площадь которого 64 корней из 3 см2 (64). Вычислить периметр основания пирамиды. 

  3. Высота правильной треугольной пирамиды 8 см, а ее апофемы 16 см. Вычислите площадь боковой поверхности пирамиды. 

  4. Высота и сторона основания правильной четырехугольной пирамиды соответственно равны 24 и 20. Найдите апофему пирамиды. 

  5. Дано: усеченная правильная пирамида, n = 3, h = 8, a1 = 14 , a2 = 2 . Надо найти площадь полной поверхности усеченной пирамиды .

  6. Дано: усеченная правильная пирамида, n = 4, h = 8, a1 = 16, a2 = 4 . Надо найти площадь полной поверхности усеченной пирамиды .

  7. Дано: усеченная правильная пирамида, n = 6, h = 2, a1 = 4 , a2 = 8 . Надо найти площадь полной поверхности усеченной пирамиды .

  8. Дано: усеченная правильная пирамида, n = 4, h = 3, r1 = 5, r2 = 9 . Надо найти площадь полной поверхности усеченной пирамиды .

3)Решить задачи :hello_html_m3eb51a34.png

  1. Основанием пирамиды является ромб, сторона которого равна AB=15 см, а одна из диагоналей равна BD = 18 см. Найдите боковые ребра пирамиды, если высота ее проходит через точку пересечения диагоналей основания и равна

SO = .

  1. Дана правильная четырехугольная пирамида со стороной основания a=12 см и
    высотой
    h=8 см. Найдите площадь полной поверхности пирамиды.
    hello_html_m3367cdaa.jpg

  2. Дана пирамида со сторонами основания a = 10,b = 24,c = 26 см и апофема равна l=10 см. Найдите площадь полной поверхности пирамиды.

  3. Дана пирамида со сторонами основания a=10,b=13,c=13 см и высотой h2=. Найдите площадь полной поверхности пирамиды.hello_html_m1aa89a30.jpg

  4. Дана усеченная правильная треугольная пирамида со сторонами
    a1 = 26 и а2 = 14 см и высотой h = 8 см. Найдите площадь полной поверхности усеченной пирамиды.

Инструкционная карта

ПР № 11 «Построение фигур с помощью симметрии».
Задание:

1)Перепишите и заполните пропуски:

Пример 1. а)Может ли фигура иметь более одного центра симметрии?

Ответ: Да, например, прямая, плоскость и т.д. имеют бесконечно много центров симметрии.

б) Приведите примеры центрально-симметричных фигур.
Ответ: Центрально-симметричные: куб, прямоугольный параллелепипед, шар и др.;

Пример 2.а) Построить точки А1 и А2 симметричные относительно точки О,

б) Построить треугольники АВС и А1В1С1 симметричные относительно точки О,
Ответ: а) б)

hello_html_27d1ce35.gif

hello_html_2c7a64c8.gif







Пример 3. а) Построить точки А1 и А2 симметричные относительно прямой а.

б) Построить прямоугольники симметричные относительно прямой b.
Ответ: а) б)
hello_html_me2c259c.gif

hello_html_73eae775.gif


Пример 4. Найди подходящую правую часть робота.

hello_html_m69d1ea91.gifhello_html_m43e930ed.jpg

Ответ:2 и 5; 1 и …; … и 4.

Пример 5. Можно ли назвать ножницы симметричной фигурой?

Ответ: Да.

Пример 6. На рисунке укажите буквы латинского алфавита имеющие одну ось симметрии;

hello_html_m79939fd7.png

Ответ: A, …, …, D, …, …, .., U, V, W, Y;

Пример 7. Построить рисунок.

hello_html_17f4ab15.jpg

а

Пример 8. Сколько осей симметрии имеет прямоугольный параллелепипед, гранями которого не являются квадраты?hello_html_76dbeb1f.png

Ответ: 3.hello_html_18121dc3.png

Пример 9. Сколько плоскостей симметрии имеет правильная шестиугольная призма?

Ответ:7.

Пример 10.

Тест по теме «Зеркальная симметрия в призме ».

1)Сколько плоскостей симметрии имеет правильная четырехугольная призма?

а)2 б)4 в)3 г)5 д)12,

2)Сколько плоскостей симметрии имеет прямая призма, в основании которой лежит прямоугольник?

а)2 б)3 в)1 г)4 д)8,

3)Сколько плоскостей симметрии имеет правильная треугольная призма?

а)4 б)3 в)1 г)2 д)5.

2)Решить задачи ( по примерам):

  1. а)Может ли фигура иметь ровно два центра симметрии?

б)Приведите примеры не центрально-симметричных фигур.

  1. а) Построить точки В1 и В2 симметричные относительно точки О,

б) Построить прямоугольные треугольники АВС и А1В1С1 симметричные относительно точки О,hello_html_m575e1189.png

  1. а) Построить точки В1 и В2 симметричные относительно прямой а.

б) Построить квадраты симметричные относительно прямой b.hello_html_m3970ee64.jpg

  1. Дорисуй рисунки так, чтобы они стали симметричными.

  2. Можно ли назвать стрекозу насекомым, у которого имеется ось симметрии?

  3. На рисунке укажите буквы латинского алфавита имеющие две оси симметрии;

hello_html_m79939fd7.png

  1. Построить рисунок.

hello_html_3cfea8dd.jpgа аhello_html_m304f2795.jpg

  1. Сколько осей симметрии имеет прямоугольный параллелепипед, две грани которого являются квадратами?

  2. Сколько плоскостей симметрии имеет прямоугольный параллелепипед, гранями которого не являются квадраты?

  3. Определить фигуры:1) обладающие центральной симметрией и указать их центр;
    2) обладающие осевой симметрией и указать ось симметрии;3) имеющие обе симметрии.




3)Решить задачи :

  1. Какие две цифры при центральной симметрии переходят одна в другую?

  2. Даны слова: ЛЕС, ОНО, ОКО, ПНИ, СЫР. Какие из них при центральной симметрии переходят сами в себя?

  3. Одно число очень любило любоваться своим отражением в зеркале. Однажды мимо проходил гном и увидел в зеркале число 18. Какое число смотрелось в зеркало?

  4. Приведите примеры зеркальной симметрии в природе.

  5. Приведите примеры симметрии в живой природе.

  6. Приведите примеры осевой и центральной симметрия в растительном мире.

  7. Сколько осей симметрии имеет прямоугольник?





  1. Построить фигуру, симметричной данной относительно оси симметрии.

hello_html_675df776.png


  1. Построить фигуру, симметричной данной относительно точки .

hello_html_ba2bd7e.gif

  1. Укажите оси симметрии: а) прямоугольника; б) квадрата.

hello_html_m43d30e22.gif


  1. Сколько осей симметрии имеет правильный шестиугольник?

hello_html_m182ed06f.png


  1. Изобразите треугольник ABC, симметричный треугольнику ABC, относительно прямой c.

а) б)

hello_html_3e4e60ee.pnghello_html_m12c7999c.png












  1. Треугольник ABC симметричен треугольнику ABC относительно некоторой прямой c. Изобразите эту прямую.

а) hello_html_2f13032b.png б) hello_html_m1cd5de4f.png

  1. Сколько осей симметрии имеет шестиугольник, изображенный на клетчатой бумаге, клетками которой являются квадраты?

hello_html_m595f5a19.png









  1. Сколько осей симметрии имеет восьмиугольник, изображенный на клетчатой бумаге, клетками которой являются квадраты?

hello_html_7b57a1eb.png








  1. Верно ли высказывание: правильная четырехугольная пирамида имеет четыре плоскости симметрии?

hello_html_m6929a8d3.gif

Инструкционная карта

ПР № 12 «Вычисление элементов цилиндра».
Задание:

  1. Перепишите и заполните пропуски:

Пример 1. В цилиндре параллельно его оси проведено сечение, диагональ которого равна 17 см, высота цилиндра равна 15 см., а радиус основания 5 см. На каком расстоянии от оси проведено это сечение? 
Решение. Сечение цилиндра в плоскости представляет собой прямоугольник. Таким образом, BM также представляет собой высоту цилиндра. Треугольник BMK - прямоугольный. Таким образом, можно найти длину стороны MK = BC:
BK
2 = BM2 + MK2 , MK2 = BK2 - BM2 ,MK2 = 172  152 = …, MK = … 
Таким образом, MK = BC = 8 см.
 
AD - диаметр цилиндра, проведенный как сечение, параллельное заданному в условии задачи. BC - прямая, принадлежащая сечению, параллельному оси цилиндра. Поэтому ABCD - трапеция. Если трапеция равнобедренная, то вокруг нее можно описать окружность. Таким образом, ABCD - равнобедренная трапеция. Найдя высоту трапеции, получим расстояние от проведенного по условию задачи сечения до оси цилиндра.
AD = 2R = 2 5 = … см,
 OC = OD = R = 5 см .
В равнобедренной трапеции высота, опущенная из вершины на большее основание, делит его на два отрезка, один из которых равен полусумме оснований, другой - полуразности оснований. Таким образом,
 
AN = DP = ( 10
8 ) / 2 = … см , тогда OP = OD DP = 5 – 1 = … см .
Треугольник CPO - прямоугольный, так как CP - высота трапеции. Откуда
 
CP
2 + OP2 = OC2 ,CP2 = OC2 OP2, CP2 = 52  42 ,CP2 = 25 16 = …,
CP = … . 
Ответ: Проведенное сечение цилиндра находится на расстоянии 3 см от его оси.
hello_html_m518413cd.pnghello_html_m49b1fd63.png

Пример 2. Найдите радиус цилиндра, если диагональ его осевого сечения, равная 8см, составляет с образующей цилиндра угол величиной 30 градусов.  hello_html_2a26c44d.png

Решение:

Поскольку AC = 8 см, а угол ACD = 30°, то 
CD = AC cos 30°  . CD = 8  /2 = 4. Аналогично,  AD = AC sin 30° , AD = 8 1/2 = 8 : 2 = … , 

Откуда радиус основания цилиндра равен R = 4 : 2 = … см. hello_html_1ffef818.jpg

Ответ:  2 см.

Пример 3. Высота цилиндра 20см, радиус основания 10см. Найдите площадь сечения, проведённого параллельно оси цилиндра на расстоянии 6см от неё.

Решение:

r = 10, d = 6, АС 2 = r2d2 = 102 – 62 = 100 – 36 = …,

АС =…, АВ1 = 2АС = 2 8 = … ,

Sсеч. = АВ1 h , h = 20, Sсеч. = 16 20 = …

Ответ:  320 см2 .

Пример 4. Найдите высоту цилиндра, если радиус основания 5см и площадь сечения равна 128 см2 , проведённого параллельно оси цилиндра на расстоянии 3см от неё.

Решение: : r = 10, d = 6, АС 2 = r2d2 = 52 – 32 = 25 – 9 = …,

АС =…, АВ1 = 2АС = 2 4 = … ,

h = Sсеч. : АВ1 = 128 : 8 = …

Ответ: 16 см.

Пример 5. Дано: цилиндр, АВ1 = 16 см, B1AB = 30° (рис.).Найти: hRосн. 

Решение:hello_html_m41d996ff.jpg

1) hк. = BB1;

2)Из ΔАВВ1 находим AB: AB = 16 cos 30° = 16 /2 = 8
R = 1/2 AB = 8 : 2 = 4 .

3) Из ΔВ1АВ находим BB1: BB1 = 16 sin 30 ° = 16 1/2 = 16 : 2 = … см.

Ответ: = 8 см; R = 4 см.

  1. Решить задачи ( по примерам):

  1. В цилиндре параллельно его оси проведено сечение, диагональ которого равна 34 см, высота цилиндра равна 30 см., а радиус основания 10 см. На каком расстоянии от оси проведено это сечение?

  2. Найдите радиус цилиндра, если диагональ его осевого сечения, равная 16 см, составляет с образующей цилиндра угол величиной 30 °

  3. Высота цилиндра 20см, радиус основания 15 см. Найдите площадь сечения, проведённого параллельно оси цилиндра на расстоянии 12 см от неё.

  4. Найдите высоту цилиндра, если радиус основания 13 см и площадь сечения равна 144 см2 , проведённого параллельно оси цилиндра на расстоянии 5 см от неё.

  5. Дано: цилиндр, АВ1 = 8 см, B1AB = 30° (рис.). Найти: hRосн.

3)Решить задачи :hello_html_1ffef818.jpg

  1. Осевое сечение цилиндра – прямоугольник, стороны которого диаметр и образующая цилиндра соответственно. Диагональ осевого сечения цилиндра равна АС = 24 см. Угол α между этой диагональю и диаметром цилиндра равен 30°. Найдите высоту, радиус, площадь основания цилиндра.

  2. Концы отрезка АВ лежат на окружностях оснований цилиндра. Радиус цилиндра равен r, его высота-h, а расстояние между прямой АВ и осью цилиндра равно d. Найдите h, если r = 5, d = 4, АВ = 10 см.

  3. Концы отрезка АВ лежат на окружностях оснований цилиндра. Радиус цилиндра равен r, его высота-h, а расстояние между прямой АВ и осью цилиндра равно d. Найдите d, если r = 10, h = 5, АВ = 13 см.

  4. Диагональ осевого сечения цилиндра равна см, а радиус основания – 3 см. Найдите высоту цилиндра.

  5. Плоскость , параллельная оси цилиндра, отсекает от окружности основания дугу AmD с градусной мерой . Радиус цилиндра равен a, высота равна h, расстояние между осью цилиндра ОО1 и плоскостью равно d. hello_html_45b595c1.gif

1) Докажите, что сечение цилиндра плоскостью есть прямоугольник.

2) Найдите AD, если a =10 см, = 60.

  1. Диагональ осевого сечения цилиндра равна см, а высота – 5 см. Найдите радиус цилиндра.

  2. В цилиндре параллельно его оси проведено сечение. Диагональ сечения, равная 16, составляет угол 60° с плоскостью основания. Радиус основания цилиндра равен 5. Найдите расстояние от оси цилиндра до плоскости сечения.

  3. Диаметр окружности основания цилиндра равен 26, а его образующая – 21. Плоскость пересекает его основания по хордам длины 24 и 10. Найти угол между этой плоскостью и плоскостью основания цилиндра.

  4. В цилиндре проведена плоскость, параллельная оси и отсекающая от окружности основания дугу в 120°. Диагональ сечения равна 20 см и удалена от оси на 3 см. Найдите площадь боковой поверхности цилиндра.

  5. Осевое сечение цилиндра - квадрат, площадь основания цилиндра равна 16π см2. Найдите площадь боковой поверхности цилиндра.

  6. Высота цилиндра 8 дм, радиус основания 5 дм. Цилиндр пересечен плоскостью параллельно оси так, что в сечении получился квадрат. Найдите расстояние от этого сечения до оси цилиндра.

  7. Дано: цилиндр; CBD = 120°; CD1 = 20 см; OK = 3 см. Найти: Sб.п.ц.hello_html_361456b7.jpg

  8. Плоскость, проходящая через центр нижнего основания цилиндра под углом α к основанию, пересекает верхнее основание по хорде, равной b и стягивающей дугу β. Найдите высоту цилиндра.

  9. Отрезок CD равен 25 см, его концы лежат на разных окружностях основания цилиндра. Найдите расстояние от отрезка CD до основания цилиндра, если его высота 7 см, а диаметр основания 26 см.



Инструкционная карта

ПР № 13 «Вычисление элементов конуса, усеченного конуса».
Задание:

  1. Перепишите и заполните пропуски:

Пример 1. Осевое сечение конуса равносторонний треугольник со стороной 10см. Найти радиус основания и высоту конуса.
Решение:  Так как ΔАВС - равнобедренный, то АВ = ВС = АС = 10 см. r = АС : 2 = 10 : 2 = … ,
Из ΔВОС по теореме Пифагора: h2 = OB2 = BC2OC2, h2 = 102 – 52 = 100 – 25 = …, h = = 5

Ответ: r = 5 см, h = 5
Пример 2. Дано: конус, ОР = 15 см, ОВ = r = 8 см (рис.). Найти: РВ. 
Решение: Из ΔОРВ по теореме Пифагора:PB2 = PO2 + OB2,
PB2 = 152 + 82 = 225 + 64 = … , PB = …
hello_html_m6d6bd24b.jpg

Ответ: 17 см.

Пример 3. Дано: Конус, ABC = 120°, АВ = 6 (рис.). Найти: R, h. hello_html_m6bcb7613.jpg

Решение:1) ΔАВС - равнобедренный, угол при основании  С = 30°.

  1. Из ΔАВО : h = ВО = AB : 2 = 6 : 2 = ... 

  2. R = AO = AB · cos 30° = 6 ·  : 2 = … .
    Ответ: H = 3, R = 3.

Пример 4. Дано: Конус. ΔАВС - равносторонний, АВ = 12, = 10 (рис.). Найти: OK, h. hello_html_m2d0d5103.jpg

Решение: 1)Из ΔВОС по теореме Пифагора: h2 = OB2 = BC2OC2, h2 = 122 – 102 = =144 – 100 = …, h = = 2

2)ΔABC - равносторонний, АС = 12, СК = 6. Из ΔСОК по теореме Пифагора
ОК
2 = ОС2 – СК2, ОК2 = 102 – 62 = 100 – 36 = …, OK = ...

Ответ: h = 2, ОК = 8.

Пример 5. Дано: конус, h = OP = 1,2 см, Sосев. = 0,6 см2 (рис.). Найти: l.hello_html_28ad1e01.jpg

Решение:

  1. Осевое сечение - треугольник: высота 1,2 см и основание 2r.

Sосев. =  · 2r h = r h, r = Sосев. : h = 0,6 : 1,2 = … см.

  1. Из ΔАОР по теореме Пифагора: l2 = h2 + r2  = OP2 + OA2. l2 = 1,22 + 0,52 = 1,44 + 0,25 = …, l = … см.

Ответ: 1,3 см.

Пример 6. Дано: усеченный конус, O1С = 3см, OD = 6 см, OO1 = 4 см (рис. ). Найти: So.сеч., CD . hello_html_m28dc10d1.jpg

Решение: Осевым сечением усеченного конуса является равнобедренная трапеция. Sсеч.= (BC + AD ) · OO1 : 2 ,
BC = 2O
1C = 2 · 3 = … см. AD = 2 OD = 2 · 6 = … см .
S
сеч.= (6 + 12 ) · 4 : 2 = 18 ·  2 = … см2,

ΔCKD - прямоугольный, по теореме Пифагора:
CD
2 = CK2 + KD2, CK = OO1 = 4 см, KD = OD – OK = OD – O1C = 6 – 3 = … см. CD2 = 42 + 32 = 16 + 9 = … ,CD = …

Ответ: Sсеч. = 36 cм2, CD = 5 см. hello_html_m5f94854e.jpg

Пример 7. Дано: усеченный конус, r 1 = 5 см, r 2 = 11 см, CD = 10 см,
Найти:
 So.сеч., h.
Решение: Осевым сечением усеченного конуса является равнобедренная трапеция.
Sсеч.= (BC + AD ) · OO1 : 2 , BC = 2O1C = 2 r 1= 2 · 5 = … см.
AD = 2 OD = 2
r 2 = 2 · 11 = … см .
ΔCKD - прямоугольный, по теореме Пифагора: CD2 = CK2 + KD2,
KD = OD – OK = OD – O1C = r2 r 1= 11 5= … , h = OO1 = CK, CK 2 = CD2 – KD2,
CK2 = 102 – 62 = 100 – 36 = … , CK = … , h = 8 см, Sсеч.= (10 + 22 ) · 8 : 2 = 32 · 4 = … см2. Ответ: Sсеч. = 128 cм2h = 8 см.

Пример 8. Дано: усеченный конус, АС = 40 см, AC  CDCD = 30 см (рис. ). Найти: Sсеч.. 

Решение: Сечение усеченного конуса является равнобедренная трапеция
 
Sсеч.= (BC + AD ) · OO1 : 2 ,

ΔADC - прямоугольный, по теореме Пифагора: AD2 = AC2 + CD2, AD2 = 402 + 302 = 1600 + 900 = … , AD = … см. Так как СН - высота прямоугольного треугольника, то СН2 = АН · HD.

ΔCHD - прямоугольный; CH2 = CD2HD2 , HD = ADAH = 50 – AH, АН · HD = CD2HD2,
AH · ( 50 AH ) = 900 – ( 50 AH)2 , 50AHAH2 = 900 – 2500 + 100 AHAH2, 50 AH = 1600, AH = … см. HD = 50 – 32 = … , OD = AD : 2 = 50 : 2 = … см , OH = OD – HD = 25 – 18 = … см, CH2 = 32· 18, CH = 24 см, Sсеч.= (2OH + 2OD ) · CH : 2 = (14 + 50) · 24 : 2 = … см2,

Ответ: Sсеч. = 768 см2.

Пример 9. Дано: усеченный конус, O1С = 16 см, OD = 25 см. Окружность, вписанная в сечение (осевое) (рис. ). Найти: Sсеч... hello_html_658e6bdc.jpg

Решение:  Осевым сечением усеченного конуса является равнобедренная трапеция. Так как в трапецию вписана окружность, то O1С = CF = 16 (см) и OD = DF = 25 (см)
(как отрезки касательных к окружности, проведенных из одной точки).
CD = CF + DF = 16 + 25 = … см, HD = OD O1C = 25 – 16 = … ,
ΔCHD - прямоугольный:  CH2 = CD2HD2, CH 2 = 412 – 9 2 = 1681 81 = … , CH = … см.

Sсеч. = (OD O1C) CH = (16 + 25) 40 = 41 40 = …

 Ответ: Sсеч..  = 1640 см2.

Пример 10. Дано: усеченный конус, r 1 = 3 см, r 2 = 6 см, h = 4 см, Найти: l.
Решение:
  l2 = h2 + (r2r1)2 , l2 = 42 + ( 6 3)2  = 16 + 9 = … , l = … см.
Ответ:
l = 5 см.

2)Решить задачи ( по примерам):

  1. Осевое сечение конуса равносторонний треугольник со стороной 8 см. Найти радиус основания и высоту конуса.

  2. Дано: конус, ОР = 12 см, ОВ = r = 9 см (рис.). Найти: РВ. 

  3. Дано: Конус, ABC = 120°, АВ = 8 (рис.). Найти: R, h. 

  4. Дано: Конус. ΔАВС - равносторонний, АВ = 24, = 20 (рис.). Найти: OK, h.

  5. Дано: конус, OP = 2,4 см, Sосев. = 2,4 см2 (рис.). Найти: l.

  6. Дано: усеченный конус, O1С = 6 см, OD = 12 см, OO1 = 8 см (рис. ). Найти: So.сеч., CD. 

  7. Дано: усеченный конус, r1 = 3 см, r2 = 11 см, CD = 10 см, Найти: So.сеч., h.

  8. Дано: усеченный конус, АС = 20 см, AC  CDCD = 15 см .Найти: Sсеч.. 

  9. Дано: усеченный конус,  O1С = 3см, OD = 12 см. Окружность, вписанная в сечение (осевое). Найти: Sсеч.. hello_html_m3219ea58.jpg

  10. Дано: усеченный конус, r 1 = 3 см, r 2 = 9 см, h = 8 см, Найти: l.

3)Решить задачи :

  1. а) Высота конуса равна h = 24 см, а радиус основания равен r = 10 см. Найдите образующую конуса l.hello_html_775d711f.jpg

б) Осевое сечение конуса – прямоугольный треугольник Δ АВС. Найдите площадь этого сечения, если радиус основания конуса равен r = 6 см.

в) Радиусы оснований усеченного конуса равны r 1 = 6 и r 2 = 11 см, а образующая равна l = 13 см. Найдите высоту и площадь осевого сечения усеченного конуса, если его осевое сечение-трапеция.hello_html_m45781634.jpg

г) Разверткой боковой поверхности конуса является сектор с дугой α. Найдите α , если его радиус основания равен 6 см, а образующая равна 20 см.hello_html_1754fa31.jpg

д) Разверткой боковой поверхности конуса является сектор с дугой α. Найдите радиус основания конуса, если α = 90°, а образующая равна 12 см.

  1. Образующая конуса l наклонена к плоскости основания под углом в 30°. Найти высоту конуса и площадь осевого сечения.

  2. Радиус основания конуса равен 3 м, а высота 4 м. Найти образующую и площадь осевого сечения.hello_html_415082a2.jpg

  3. Высота конуса равна 6 см, угол при вершине осевого сечения равен 90°. Найдите площадь боковой поверхности конуса.

  4. Дано: конус; SO = 6 см; ASB = 90°; CSD = 35°.Найти: S6.п.конуса.

  5. Через вершину конуса проведена плоскость, пересекающая основание по хорде, длина которой равна 5 см, и стягивающей дугу 90°. Плоскость сечения составляет с плоскостью основания угол 60°. Найдите площадь боковой поверхности конуса.

  6. Высота конуса равна 15 см, а радиус основания равен 8 см.
    Найдите образующую конуса.

Инструкционная карта

ПР № 14 «Вычисление элементов сферы».hello_html_m6a79015f.jpg

Задание:

  1. Перепишите и заполните пропуски:

Пример 1. Дано: шар, BAO = 30°Sсеч. = 75π см2 (рис. ). Найти: АС. 

Решение: S сеч. =r2 , 75 = r2 , r2 = …, r = 5 , AO1= 5 .
Из ΔАО
1О: cos 30° = AO1 / AO , AO = / () = 5 2 = … см.

AC = 2 · AO = 2 · 10 = … см. hello_html_65ab0256.jpg

Ответ: 20 см.

Пример 2. Дано: Rш. = 8 смOAB = 45° (рис.).Найти: Sceч

Решение: S сеч. =r2 , cos 45° = AO1 / AO , AO1 = r = 8 · : 2 = … ,

S сеч. = 16 · 2 = … см2.

Ответ: 32π см2.

Пример 3. Дано: шар с центром в точке О, Sсеч. = 16π см2, расстояние от точки О до сечения OA=3 см (рис. ). Найти: Sсф. hello_html_35dd295d.jpg

Решение: S сеч. =r2 = 16 π, r2 = 16, r = … . 

Рассмотрим ΔОАВ : OA = - расстояние, значит, = 90°.

OB2 = R2 = r2 + OA2 = 42+ 32 = 16 + 9 = …, S сф. =R2 = 4π 25 = … π см2

Ответ: 100π см2.

Пример 4. Сколько кожи пойдет на покрышку футбольного мяча радиуса 10 см? (На швы добавить 8% от площади поверхности мяча.)

Решение: S сф. =R2 = 4π102 = 4100 π = … π см2.

1 способ.

1% составляет 0,01 400 π = 4 π см2.

8% составляет 84 π = 32 π см2. S = 400 π + 32 π = … π 1357 см2.

2 способ. 8% составляет 1,08 400 π = 432 π см2.

Ответ: 432π см2.

Пример 5. Дано: сфера с центром в точке О, АВ  CD, АВ - диаметр сечения, hello_html_68cdf4b6.jpg

CD - диаметр сечения MN – общая хорда. MN = 6 см, ОК = 4, ОО1 = ОО2 (рис.). Найти: Sсф. 

Решение: Рассмотрим прямоугольный ΔONK с OKN = 90°;

NK = MN : 2 = 3см, NO2 = R2 =  OK2 + NK2 = 32 + (4)2 = 9 + 32 = …,

S сф. =R2 = 4π41 = … π см2.

Ответ: 164π см2.

Пример 6. Дано: сфера с центром в точке О и радиусом Rr1 и r2 - радиусы параллельных сечений сферы, r1 = 9 см, r2 = 12 см, l = 3 см - расстояние между секущими плоскостями (рис.). Найти: Sсф. hello_html_1a919b33.jpg

Решение: Проведем диаметры перпендикулярно к данным параллельным сечениям. Через диаметр проведем секущую плоскость, которая пересечет сферу по окружности, радиус которой равен радиусу сферы 

ND = r1 = 9см, MB = r2 = 12 см, NM = 3 см, OD = ОВ = R в ΔOВМ:

OM2 = R2 – 122 = R2 – 144, в ΔODN: ON2 = R2 – 92 = R2 – 81,

MN = NO – MO = – , – = 3,

= 3 + , R2 – 81= 9 + 6 + R2 – 144, 6 = 54 , = 9, R2 – 144 = 81, R2 = 144 + 81 , R2 = 225, R = …, S сф.=R2 = 4π152 = 4π 225 = … π см2. Ответ: 900π см2.

Пример 7. Стороны треугольника 13, 14, 15 см. Найти расстояние от плоскости треугольника до центра шара, касающегося всех сторон треугольника. Радиус шара 5 см.

Решение:

  1. Рассмотрим треугольник АВС со сторонами 13, 14, 15 см

S=, p = ; p = = 42 : 2 = (см)

S== 84 (см)

  1. SАВС = pr , где r – радиус вписанной окружности. S= 21r , 84 = 21r r = … см


  1. h = Rr - т. Пифагора, h = = … (см). Ответ: h = 3 (см)

2)Решить задачи ( по примерам):

  1. Дано: шар, BAO = 30°; Sсеч. = 48π см2 (рис. ). Найти: АС.

  2. Дано: Rш. = 10 см, OAB = 45° (рис.).Найти: Sceч. 

  3. Дано: шар с центром в точке О, Sсеч. = 25π см2, расстояние от точки О до сечения OA= 12 см (рис. ). Найти: Sсф. 

  4. Сколько кожи пойдет на покрышку футбольного мяча радиуса 5 см? (На швы добавить 8% от площади поверхности мяча.)

  5. Дано: сфера с центром в точке О, АВ  CD, АВ - диаметр сечения, CD - диаметр сечения MN – общая хорда. MN = 8 см, ОК = 6, ОО1 = ОО2 (рис.). Найти: Sсф. 

  6. Дано: сфера с центром в точке О и радиусом Rr1 и r2 - радиусы параллельных сечений сферы, r1 = 3 см, r2 = 4 см, l = 1 см - расстояние между секущими плоскостями (рис.). Найти: Sсф.

  7. Стороны треугольника равны 5, 5, 6 см. Найдите расстояние от плоскости треугольника до центра шара, касающегося всех сторон треугольника. Радиус шара равен 2,5 (см).

3)Решить задачи :

  1. В сферу вписан конус, образующая которого равна l = 3 см, а угол при вершине осевого сечения равен 60 градусов. Найдите площадь сферы. hello_html_50087654.jpg

  2. Точка М- середина отрезка АВ, концы которого лежат на сфере радиуса R с центром О.(рис.) Найдите ОМ, если R = 10 дм, АВ = 12 дм.

  3. Все стороны треугольника ABC касаются сферы радиуса R = 7 см. Найдите расстояние от центра сферы до плоскости треугольника, если ВС = а = 10 см, АС = в = 10 см, АВ = с = 12 см.

  4. Сечение шара плоскостью имеет площадь 36(м). Радиус шара 10м. Найти расстояние от центра шара до плоскости сечения.

  5. На поверхности шара даны три точки, кратчайшее расстояние между которыми равно 6 см. Определить площадь сечения, проходящего через эти точки.

  6. Найдите площадь сферы, если радиус сферы равен 3 см.

  7. Найдите радиус сферы, если площадь сферы равна 16π см2.

  8. Найдите площадь центрального сечения сферы, если радиус сферы равен 5 см.

  9. Найдите расстояние от точки касания плоскости и сферы, до точки на касательной плоскости, если радиус сферы равен 5 см, а расстояние от центра сферы до точки на касательной плоскости равно 13 см.

  10. Площадь сечения проходящего через центр шара, равна 16π см2. Чему равен радиус шара?

  11. Через точку, делящую радиус сферы пополам, проведена секущая плоскость, перпендикулярная к этому радиусу. Радиус сферы равен 16 см. Найдите площадь сечения.hello_html_m10715efe.jpg

  12. Шар с центром в точке О касается плоскости в т очке В. Точка А лежит в этой плоскости, ОА = 20 см, АВ = 12 см. Найдите радиус шара.

  13. Дано: шар, AC = 4; BAO = 45°.Найти: Sсеч.

  14. Дано: шар, BAO = 30°; Sсеч. = 75π см2 .Найти: АС.

  15. Радиус шара равен 17 см. Найдите площадь сечения шара, удаленного от его центра на 15 см.

  16. Радиус сферы равен 15 см. Найдите длину окружности сечения, удаленного от центра сферы на 12 см.

  17. Сфера w проходит через вершины квадрата CDEF, сторона которого равна 18 см. Найдите расстояние от центра сферы - точки О до плоскости квадрата, если радиус сферы ОЕ образует с плоскостью квадрата угол, равный 30°.

  18. Стороны треугольника MKN касаются шара. Найдите радиус шара, если МК = 9 см,
    MN = 13 см; KN = 14 см и расстояние от центра шара О до плоскости MNK равно см.

  19. Шар радиусом 41 дм пересечен плоскостью, находящейся на расстоянии 9 дм от центра. Найдите площадь сечения.

  20. Найдите площадь сферы, радиус которой равен 6 см.



Инструкционная карта

ПР № 15«Вычисление элементов тел вращения».

Задание:

1)Перепишите и заполните пропуски:hello_html_m4f14473a.jpg

Пример 1. а)Радиус цилиндра равен 10 см. Сечение, параллельное оси цилиндра и удаленное от нее на 8 см, имеет форму квадрата. Найти площадь сечения.

Решение: 1) ABCD - квадрат. 2) АО = 10 см, ОК = 8 см. OK  AD, АК = KD.

3) ΔАКО прямоугольный. АК =  

4) AD = 6 · 2 = …. (см),5) SABCD = 122 = … (см2). Ответ: 144 см2.

б) Диагональ осевого сечения цилиндра равна 8 дм и образует с плоскостью основания цилиндра угол 45°. Найдите площадь полной поверхности цилиндра.

Решение: 1) ABCD - осевое сечение.2) АС = 8дм, ΔACD - прямоугольный,
AD = DC, 2AD2 = AC2, AD2 = 64·2 : 2 =… (дм), AD= …
hello_html_m7dc30bd2.jpg

3) AD = CD = h = 8 (дм). r = АО = CD : 2 = h : 2 = 8 : 2 = …,

4) S пол .= · r· ( h + r )  = · 4 · ( 8 + 4 ) = ·48 = …. Ответ: 96 дм2.

Пример 2. а) Высота цилиндра равна 16 см. На расстоянии 6 см от оси цилиндра проведено сечение, параллельное оси цилиндра и имеющее форму квадрата. Найдите радиус цилиндра.hello_html_561073fc.jpg

Решение: 1) ABCD - квадрат. 2) ОО1 = АВ = 16 см, KО1 = 6см,
так как КО
1 расстояние от ОО1 до ABCD, К - середина ВС.

3) ВС = 16 см  ВК = 16 : 2 = … см.

4). ΔВКО1 - прямоугольный.ВО1 = r, ВО1 = Ответ: 10 см.hello_html_304428aa.jpg

б) Диагональ осевого сечения цилиндра равна 8 дм и составляет с образующей
угол 60°. Найдите площадь полной поверхности цилиндра.

Решение: 1) ΔACD - прямоугольный, ACD = 60°  CAD = 30°.

2) Так как CAD = 30°, то CD = 1/2AC, CD = 8 : 2 = … см = h.

3) AD = r =  

4) S пол .= r ( h + r )  = ··( + 4) = …·( + 4).hello_html_43dd25de.jpg

Ответ: 4·( + 4).

Пример 3. Дано: АВ = 4 см; ВС = 3 см. Найти: Sп.п.

Решение: Sп.п. = πRl + πR2; l = см; R = 3 см.
Sп.п. = π · 3 · 5 + π · 32 = (15 + 9 )π = … (см2). Ответ: 24π см2.
hello_html_415082a2.jpg

Пример 4. Дано: конус; SO = 6 см; ASB = 90°; CSD = 35° .

Найти: S6.п.конуса.  

Решение: В ΔASВ, SO - высота и биссектриса,
тогда
 ASO = 45°  AO = SO, R = H = 6 см. 

l

Sб.п. = πRl = . Ответ: 72 π см2.

Пример 5. Дано: шар, BAO = 30°Sсеч. = 12π см2 (рис. ). Найти: АС. hello_html_m590d8763.jpg

Решение: S сеч. =r2 , 12 = r2 , r2 = …, r = 2, AO1= 2 .
Из ΔАО
1О: cos 30° = AO1 / AO , AO = = 2 2 = … см.

AC = 2 · AO = 2 · 4 = … см. Ответ: 8 см.

Пример 6. Дано: шар; Сокр.сеч. = 6π см; BAO = 60° .Найти: АС.
Решение: С = r = ,r = 6 : 2 =…, Из ΔАО1О: О1ОA = 90°60° = …; 

, Ответ: 12 см.

2)Решить задачи ( по примерам):

  1. а)Радиус цилиндра равен 15 см. Сечение, параллельное оси цилиндра и удаленное от нее на 12 см, имеет форму квадрата. Найти площадь сечения.

б) Диагональ осевого сечения цилиндра равна 6 дм и образует с плоскостью основания цилиндра угол 45°. Найдите площадь полной поверхности цилиндра.

  1. а) Высота цилиндра равна 32 см. На расстоянии 12 см от оси цилиндра проведено сечение, параллельное оси цилиндра и имеющее форму квадрата. Найдите радиус цилиндра.

б) Диагональ осевого сечения цилиндра равна 6 дм и составляет с образующей
угол 60°. Найдите площадь полной поверхности цилиндра.

  1. Дано: АВ = 8 см; ВС = 6 см. Найти: Sп.п.

  2. Дано: конус; SO = 8 см; ASB = 90°; CSD = 35° .Найти: S6.п.конуса.  

  3. Дано: шар, BAO = 30°Sсеч. = 147π см2 (рис. ). Найти: АС. 

  4. Дано: шар; Сокр.сеч. = 8π см; BAO = 60° .Найти: АС.

3)Решить задачи :

1. Осевое сечение цилиндра - квадрат, длина диагонали которого равна 36 см. Найдите радиус основания цилиндра.

2. Площадь осевого сечения цилиндра 12 дм2, а площадь основания равна 64 дм2. Найдите высоту цилиндра.

3. Отрезок CD равен 25 см, его концы лежат на разных окружностях основания цилиндра. Найдите расстояние от отрезка CD до основания цилиндра, если его высота 7 см, а диаметр основания 26 см.

4. Высота конуса равна 4см, а угол при вершине осевого сечения равен 120°. Найдите площадь основания конуса.

5. Радиус основания конуса равен 7 см. Найдите наибольшую возможную площадь осевого сечения данного конуса.

6. Отрезок DE - хорда основания конуса, которая удалена от оси конуса на 9 см. КО - высота конуса, причем КО = 3см. Найдите расстояние от точки О (центр основания конуса) до плоскости, проходящей через точки D,E и K.

7. Сфера w проходит через вершины квадрата CDEF, сторона которого равна 18 см. Найдите расстояние от центра сферы - точки О до плоскости квадрата, если радиус сферы ОЕ образует с плоскостью квадрата угол, равный 30°.

8. Стороны треугольника MKN касаются шара. Найдите радиус шара, если МК = 9 см,
MN = 13 см; KN = 14 см и расстояние от центра шара О до плоскости MNK равно  см.

  1. На поверхности шара выбраны точки А и В так, что АВ = 40 см, а расстояние от центра шара до прямой АВ равно 15 см. Найдите площадь сечения шара, проведенного через точки А и В на расстоянии 7 см от центра шара.

  2. Плоскость, параллельная оси цилиндра, пересекает основание цилиндра по хорде, которая видна из центра этого основания под углом 30°. Диагональ образовавшегося сечения наклонена к плоскости основания под углом 60°. Радиус цилиндра равен R = 4. Найдите: а) площадь данного сечеиия;
    б) площадь осевого сечения.

  3. Осевое сечение цилиндра - квадрат, диагональ которого равна 12 см. Найдите площадь боковой поверхности цилиндра.

  4. В сферу вписан конус, образующая которого равна l=5, а угол при вершине осевого сечения равен 60°. Найдите площадь сферы.

  5. Вычислите полную поверхность тела вращения, которое получается в результате вращения прямоугольника вокруг его стороны AD, если АВ = 3 см, АС = 5 см.

  6. Сечение цилиндра плоскостью, параллельной оси, отсекает от окружности основания дугу в 90°. Найдите площадь сечения, если высота цилиндра равна 6 см, а расстояние между осью цилиндра и секущей плоскостью равно 3 см.

  7. Около шара радиуса R= 6 описан правильный конус. Найдите площадь поверхности конуса.

  8. Радиус шара равен 8 см. Через конец радиуса, лежащего на сфере, проведена плоскость под углом 45° к радиусу. Найдите площадь сечения шара этой плоскостью.

  9. Высота конуса равна 6 см, а образующая наклонена к плоскости основания под углом в 30°. Найдите площадь сечения конуса плоскостью, проходящей через две образующие, угол между которыми равен 60°.

  10. Какая фигура образуется при вращении ΔAВС вокруг оси (достроить). Вычислите полную поверхность тела вращения, которое получается в результате вращения ΔABC вокруг его стороны АС, если АС = 8 см, ВС = 5 см.

  11. Высота цилиндра 8 дм, радиус основания 5 дм. Цилиндр пересечен плоскостью параллельно оси так, что в сечении получился квадрат. Найдите расстояние от этого сечения до оси цилиндра.

  12. Радиус сферы равен 15 см. Найдите длину окружности сечения, удаленного от центра
    сферы на 12 см.

Инструкционная карта

ПР № 16«Расчет по модели объёма прямоугольного параллелепипеда».

Задание:

  1. Перепишите и заполните пропуски:

Пример 1. Площадь грани прямоугольного параллелепипеда равна 12. Ребро,

перпендикулярное этой грани, равно 4. Найдите объем параллелепипеда.
Решение: Каждая грань прямоугольного параллелепипеда –прямоугольник.

Пусть SABCD= a b = 12 , тогда АА1= h = 4, т.к. АА1 АВСD

Используем формулу объема прямоугольного параллелепипеда: V = a b h , V = 12 4 = ...

Ответ: 48 см3.

Пример 2. Объем прямоугольного параллелепипеда равен 12. Одно из его ребер равно 3. Найдите площадь грани параллелепипеда, перпендикулярной этому ребру.

Решение: Пусть АА1 АВСD, V = 12 , АА1= h = 3.

Найдём SABCD. Используем формулу объема прямоугольного параллелепипеда V = a b h, где SABCD= a b, S ABCD 3 = 12,S ABCD = 12 : 3 = ...

Ответ: 4 см2.

Пример 3. Два ребра прямоугольного параллелепипеда, выходящие из одной вершины, равны 2, 4. Диагональ параллелепипеда равна 6. Найдите объем параллелепипеда.

Решение: a = 4, b = 2, d = 6. Найдем V.

Формула диагонали прямоугольного параллелепипеда:

d2 = a2 + b2 + h2 , 16 + 4 + h2 = 36, h2 = … , h = ...

Формула объема прямоугольного параллелепипеда: V = abh , V = 4 2 4 = ...

Ответ: 32 см3.

Пример 4. Два ребра прямоугольного параллелепипеда, выходящие из одной вершины, равны 2, 3. Объем параллелепипеда равен 36. Найдите его диагональ и высоту.

Решение: a = 3, b = 2. Формула объема прямоугольного параллелепипеда: V = abh , 3 . 2 . h = 36,

6h = 36, h = ..., V = 36. Найдем d. d2 = 9 + 4 + 36, d2 = 49, d = ...

Ответ: 7 и 6 см.
Пример 5. Дано: ABCDA1B1C1D1 - прямоугольный параллелепипед, диагональ 
D1= 18 составляет угол в 30° с плоскостью боковой грани, и угол в 45° с боковым ребром (рис. ). Найти: V.
hello_html_m49e41dc8.jpg

 Решение: BC1 - проекция D1на плоскость боковой грани BB1С1С,
поэтому 
D1BC1 = 30°D1BB1= 45°.
Рассмотрим Δ
D1C1BD1C1= 90° (рис.). ∠В = 30°. => D1C1 = 18 : 2 = … см.
Рассмотрим Δ
D1B1- прямоугольный: BB1= 18 cos 45° = 18 : 2 = … см.
Диагональ (d) и измерения (а, b, с) прямоугольного параллелепипеда связаны соотношением:
d2 = a2 + b2 + h2 , 182 = 92 + (9)2 + B1C12 ,(ΔD1B1B: B1B =D1 B1).
B1C12 = 182 92 (9)2 = 324 – 8181 2 = 81, B1C1 = …см. V = 99 9 = … см3.   
Ответ:
V = 729см3.

Пример 6. Стороны основания прямоугольного параллелепипеда 3 и 4. Найти его объём, если высота равна длине диагонали его основания.

Решение: BD - диагональ основания прямоугольного параллелепипеда. BD2 = АВ2 + АD2,
BD2 = 32 + 42 = 9 + 16 = …, BD = …, h = 5. V = 345 = … см3.
Ответ:
60 см3.

Пример 7. Найти объём прямоугольного параллелепипеда, если стороны основания 2 и 3, а диагональ параллелепипеда .

Решение: d2 = a2 + b2 + h2 , ()2 = 22 + 32 + h2 , h 2 = 38 – 49 = 25, h = ...

V = 23 5 = … см3.
Ответ: 30 см3.

2)Решить задачи ( по примерам):

  1. Площадь грани прямоугольного параллелепипеда равна 15. Ребро, перпендикулярное этой грани, равно 6. Найдите объем параллелепипеда.

  2. Объем прямоугольного параллелепипеда равен 24. Одно из его ребер равно 3. Найдите площадь грани параллелепипеда, перпендикулярной этому ребру.

  3. Два ребра прямоугольного параллелепипеда, выходящие из одной вершины, равны 3, 4. Диагональ параллелепипеда равна 13. Найдите объем параллелепипеда.

  4. Два ребра прямоугольного параллелепипеда, выходящие из одной вершины, равны 3, 6. Объем параллелепипеда равен 108. Найдите его диагональ и высоту.

  5. Дано: ABCDA1B1C1D1 - прямоугольный параллелепипед, диагональ  D1= 12 составляет угол в 30° с плоскостью боковой грани, и угол в 45° с боковым ребром. Найти: V.

  6. Стороны основания прямоугольного параллелепипеда 6 и 8. Найти его объём, если высота равна длине диагонали его основания.

  7. Найти объём прямоугольного параллелепипеда, если стороны основания 4 и 6, а диагональ параллелепипеда .

3)Решить задачи :

1. Объём параллелепипеда равен 60 см3.

Вычислите и проставьте недостающий размер.

? 4 смhello_html_m5cd8b5bc.png

5 см

2. Каковы измерения параллелепипеда на рис. б), сложенного из 3 одинаковых брусков, изображённых на рис. а). Каков его объём?

3. Найдите объем прямоугольного параллелепипеда с ребрами 3 см, 5 см и 8 см.

а) 120 см3; б) 60 см3; в) 32 см3; г) другой ответ.

4. Длина прямоугольной комнаты в 2 раза больше ширины и на 2 м больше высоты. Найдите объем комнаты, если ее длина равна 6 м.

а) 432 м3; б) 144 м3; в) 72 м3; г) другой ответ.

5. Найдите объем куба, если площадь его развертки равна 96 см2.

а) 16 см3; б) 64 см3; в) 80 см3; г) другой ответ.

6. Найдите ребро куба, если его объем равен  512  м3

а) 4 м; б) 8 м; в) 16 м; г) другой ответ.hello_html_m115c4d1e.jpg

7. Как изменится объем параллелепипеда, если его длину увеличить в 4 раза, ширину увеличить в 6 раз, а высоту уменьшить в 8 раз?

а) увеличится в 3 раза; б) уменьшится в 12 раз; в) не изменится; г) другой ответ.

8. Три ребра прямоугольного параллелепипеда, выходящие из одной вершины, равны 1; 0,5 и 16. Найдите ребро равновеликого ему куба.

9. Прямоугольный параллелепипед описан около цилиндра, радиус основания и высота которого равны 1,5. Найдите объем параллелепипеда. hello_html_54b46307.png

10.Площадь грани прямоугольного параллелепипеда равна 12. Ребро, перпендикулярное этой грани, равно 5. Найдите объем параллелепипеда.

11. Дано: ABCDA1B1C1D1 - прямоугольный параллелепипед. B1= 10 .

Найти: V.

12. По готовым чертежам найти: V.hello_html_m523d2727.jpg

а) б) hello_html_763e6c8a.jpg





13. Измерения прямоугольного параллелепипеда равны 2,5 см, 5 см, 5 см. Найти ребро куба, объем которого в два раза больше объема данного параллелепипеда.

14.Найдите площадь поверхности прямоугольного параллелепипеда по трем его измерениям,

равным 3 см, 4 см, 5 см. 

15.Найдите площадь сечения куба ABCDA1B1C1D1 плоскостью, проходящей через ребро АВ и середину ребра В1С1, если ребро куба равно 2 см.

16.Найдите площадь сечения куба ABCDA1B1C1D1 плоскостью, проходящей через

ребра АВ и C1D1, если ребро куба равно 3 см. 

17.Найти боковую поверхность прямоугольного параллелепипеда, если его высота h,

площадь основания Q, а площадь диагонального сечения М.

4) Выполнить расчет по модели прямоугольного параллелепипеда:
измерить длину, ширину, высоту и найти объем.

Инструкционная карта

ПР № 17 «Вычисление объёма прямой призмы. Вычисление объёма цилиндра».

Задание:

  1. Перепишите и заполните пропуски:

Вычисление объёма прямой призмы. hello_html_m47ca6280.jpg

Пример 1. Дано: АВСА1В1С1 - прямая призма, АС = ВС, ACB = 90°BN NACNC1 = 45°CC1 = 6 (рис.). Найти: V. Решение: V = Sh , S = BC2 : 2, BC2 = BN2 + CN2 , BN =CN
(
ΔABC – прямоугольный,AC =BC), ΔC1CN – прямоугольный,CNC1 = 45°
CC1 = CN= 6, BC2 =2CN2 = 2 62 = 236 = …, BC = 6 ,
V = (62 6 : 2 = 36 6 = … см3.    
Ответ:216см3.     Пример 2. Дано: ABCDА1В1С1D1 - прямая призма,  ABCD - ромб, BAD = 60° (рис.). ВВ1 = 2, B1DB = 45°. Найти: V. РешениеSp = AB AD sin 60°. ΔABD – равносторонний( AB = AD,BAD = 60° ).
AB = BD = AD. ΔB1DB –прямоугольный ,
B1DB = 45°. => ΔB1DB – равнобедренный, ВВ1 = ВD = 2,
V = AB AD sin 60° BB1= BB13 sin 60° = 23 / 2 = … см3.
hello_html_6c1a9bbb.jpg

Ответ: 4 см3

Пример 3. Дано: ABCDFM...M1 - правильная шестиугольная призма. AD1 = 8 см - наибольшая диагональ.AD1= 30°(рис.).hello_html_72355ff0.jpg

Найти: V. 
Решение: V= S0 · h. h = DD1 в ΔADD1, = 90°. D1 = 30°,

DD1 = AD1 · cos 30°. DD1 = 8 / 2 = … , AD = AD1 : 2 = 8 : 2 = … см,
OD = OC = CD = AD : 2 = 4 : 2 = …
см,
S
0 = 6S ΔOCD = 6 / 4) a2 = 6 / 4) 22 = 6 см. V = 6 = 6 43 = … см3.    

Ответ: 72 см3.   hello_html_m62762a7f.jpg

Пример 4. Дана трапеция, S(BB1C1C) = 8 см2, S(AA1D1D) = 12см2, BH = 5 см (рис.).Найти: Vnp. 
Решение:1)Расстояние между параллельными плоскостями ВВ1С1 и AA1D1 есть длина перпендикуляра ВН, который является высотой трапеции ABCD.

2) Обозначим верхнее основание трапеции - а, нижнее - b, высоту призмы h, тогда S(BB1C1C) = ah, 8 = ah, a = 8 / h, S(AA1D1D) = bh , 12 = bh, b = 12 / h,

3) S0 = (AD + BC)BH : 2 =( a + b ) BH : 2 = (8 / h + 12 / h) 5 : 2 = … / h,

4) V= S0 · h. V= 50 / · h = … см3.  Ответ: 50 см3.

Вычисление объёма цилиндра.
Пример 1
. Дано: цилиндр, r = 2см, h = 3 см. Найти: V.

Решение: V= S0 · h. V= πr2 · h = π()2 3 = π 8 3= … π см3.
Ответ: 24π см3.

Пример 2. Дано: цилиндр, r = h= 8π см3.Найти: h.

Решение: V= S0 · h. V= πr2 · h, так как r = h, то V = πh3 => h3 = V / π, h3 = 8 π / π = 8, h = … см.
Ответ: 2 см.
hello_html_2cd2a3a8.jpg

Пример 3. Дано: цилиндр, ABCD - осевое сечение, ABCD - квадрат, 

АС = 8см. (рис.). Найдите: Vцил. 

Решение:1) V= S0 · h. 

2)Рассмотрим ΔАВС - прямоугольный, так как ABCD квадрат.
Пусть АВ = ВС =
x см(x >0), тогда x2 + x2 = (8)2, 2x2 = 642,x2 = 64, x = ....
Итак: АВ = ВС = 8 см, т.е. 
= 8 (см).

3) Найдем радиус основания: = 1/2AD = h / 2 = 8 : 2 = … см, тогда S0 = πr2 , S0 = 16π см2. 

4) V= S0 · h. V= 16 π · 8 = … π см3.  

Ответ: 128 π см3.

Пример 4. Дано: цилиндр, ABCD - осевое сечение, ABCD - квадрат, АС = 6см
(рис.
Пример 3.). Найдите: Vцил. Решение:

1) V= S0 · h.  2)Рассмотрим ΔАВС - прямоугольный и равнобедренный, так как ABCD – квадрат.

Обозначим АВ = ВС = х см (x >0), тогда x2 + x2 = (6)2, 2x2 = 362,x2 = 36, x = … ,
т. е. АВ = ВС = 6 см, и так = 6 см. 3) Найдем радиус основания r = AD : 2 = AB : 2 = 6 : 2 = …см. S0  = πr2 = 9πсм2. 

4) V= S0 · h. V= 9π · 6 = … πсм3.  
Ответ: 54π см3.
hello_html_60118f26.jpg

Пример 5. Дано: цилиндр (MNKL) || OO1, ОН =15 см, МК = 20 см, r = 17 см (рис.). Найдите: Vцил. 

Решение:1) MN || OO1 и KL || OO1, т.е. MN || KL; ОО1 основанию  MN  основанию и КО  основанию, кроме того NK ||ML - лежат в параллельных плоскостях, таким образом четырехугольник MNKL - прямоугольник.

2)  V= S0 · h. V= πr2 · h = 172πh = … πh см3

3) Рассмотрим ΔMOL: проведем ОН  ML; ОН и есть расстояние от плоскости сечения до оси цилиндра, т. е. ОН = 15 см. ОН - высота, медиана и биссектриса равнобедренного ΔMOL,

HL = ML : 2 , HL2 = OL2OH2 = 172 – 152 = 289 – 225 = … , HL = … см, ML = 28 = … см.

4) Находим высоту цилиндра из прямоугольного ΔMKL:
h2 = KL2 = MK2ML2 = 202 – 162 = 400 – 256 = … , h = … см.

5) V =289π 12 = … π см3. Ответ: 3468π см3.

2)Решить задачи ( по примерам): Призма.

  1. Дано: АВСА1В1С1 - прямая призма, АС = ВС, ACB =90°BN NACNC1 = 45°CC= 8 (рис.). Найти: V.

  2. Дано: ABCDА1В1С1D1 - прямая призма,  ABCD - ромб, BAD = 60° (рис.). ВВ1 = 4, B1DB = 45°. Найти: V.

  3. Дано: ABCDFM...M1 - правильная шестиугольная призма. AD1 = 16 см - наибольшая диагональ.AD1= 30° (рис.). Найти: V. 

  4. Дана трапеция, S(BB1C1C) = 10 см2, S(AA1D1D) = 14см2, BH = 10 см (рис.). Найти: Vnp.  Цилиндр.

  1. Дано: цилиндр, r = 4см, h = 3 см.Найти: V.

  2. Дано: цилиндр, r = h= 27π см3.Найти: h.

  3. Дано: цилиндр, ABCD - осевое сечение, ABCD - квадрат, АС =10см.(рис.). Найдите: Vцил. 

  4. Дано: цилиндр, ABCD - осевое сечение, ABCD - квадрат, АС = 4 см (рис. Пример 3.). Найдите: Vцил.

  5. Дано: цилиндр (MNKL) || OO1, ОН =30 см, МК = 40 см, r = 34 см (рис.). Найдите: Vцил. 

3)Решить задачи :hello_html_m47ca6280.jpg

  1. Дано: АВСА1В1С1 - прямая призма, АВ = ВС = 20 см, АС = 24 см, К - середина ребра,  KDB =60° (рис.). Найти: Vпр.

  2. В сосуд, имеющий форму правильной треугольной призмы, налили воду. Уровень воды достигает 27 см. На какой высоте будет находиться уровень воды, если ее перелить в другой такой же сосуд, у которого сторона основания в 3 раза больше, чем у первого?

  3. Основанием прямой треугольной призмы служит прямоугольный треугольник с катетами 6 и 5. Объем призмы равен 60. Найдите ее боковое ребро.

  4. Основанием прямой треугольной призмы служит прямоугольный треугольник с катетами 6 и 5, боковое ребро равно 4. Найдите объём призмы.

  5. Стор она основания правильной треугольной призмы равна 3см, а высота – 4 см. Найдите объём призмы.

  6. Найдите объем цилиндра с высотой, равной 3 см, и диаметром основания, равным 6 см.

  7. Объем цилиндра равен 27π. Найдите диаметр основания цилиндра, если площадь полной его поверхности в два раза больше площади боковой поверхности.

  8. Диагональ осевого сечения цилиндра составляет с плоскостью основания цилиндра угол 60°. Найдите объем цилиндра, если площадь осевого сечения равна 16 см².

  9. Площадь осевого сечения цилиндра равна 21 см², площадь основания - 18π см². Найдите объем цилиндра.

  10. Параллельное оси цилиндра сечение отсекает от окружности основания дугу в 120°. Радиус основания цилиндра равен R, угол между диагональю сечения и осью цилиндра равен 30°. Найдите объем цилиндра.


Инструкционная карта

ПР № 18 «Вычисление объёма пирамиды .Расчет по модели объёма конуса».

Задание:

  1. Перепишите и заполните пропуски:

Пирамида.

Пример 1. В правильной четырехугольной пирамиде высота равна 9 см. Сторона основания 4 см. Найдите объем пирамиды.

Решение: V= 1/3 S0 · h. V= 1/3 a2 · h = 1/3· 42·9 = 1/3 · 16 · 9 = 16 · 3 = … см3. Ответ: 48см3. 

Пример 2. a) Объем правильной четырехугольной пирамиды равен 27 см3, высота 9 см. Найти сторону основания.

Решение: V= 1/3 S0 · h. V= 1/3 a2 · h, a2  = 3V : h = 3 · 27 : 9 = 3 · 3 = ... , a = … см.

Ответ: 3 см.hello_html_m12ec6366.jpg

б) Объем пирамиды равен 56 см3, площадь основания 14 см2. Чему равна высота?

Решение: V= 1/3 S0 · h.  h = 3 V : S0  = 3 · 56 : 14 = 3 · 4 = … см.

Ответ: 12 см.

Пример 3. Дано: ABCD - правильная пирамида.

АВ = a = 3; AD = 2 (рис.).Найти: aSocн.; б) АО; в) DO; г) V.

 Решение:

а) S0 = 0,25 · a2  = 0,25 · 32 = 2,25 (используем формулу для вычисления площади правильного треугольника). 

б) AO = R = 2/3h = 1/3 a  (формула радиуса описанной окружности через сторону правильного треугольника). AO = 1/3 · 3 = .

в) DO2 = AD2AO2, (по теореме Пифагора).

DO2 = (2)2 – ()2 = 4 · 3 – 3 = … , DO = h = 3.hello_html_5fef969e.jpg

г) V= 1/3 S0 · h. V= 1/3 · 2,25 · 3 = … см3.

Ответ: aSocн. = 2,25 см2; б) АО = см; в) DO = 3см; г) V = 2,25 см3 .

Пример 4. Дано: ABCDF - правильная пирамида. 

FCO = 45°FO = 2 (рис.). Найти: a) Socн.; б) V. 

Решение:

1) Рассмотрим ΔFOC= 90°= 45°, значит, = 45°. Следовательно, ΔFOC - равнобедренный, ОС ≈ FO = h= 2.

2) АС = 2OС = 4. AC = AD (по свойству диагонали квадрата, d2 = 2а2).

Тогда  AD = AC / = 4 / = 2 .

3) ABCD - квадрат (пирамида правильная). S0 = AD2 = (2)2 = 2 · 4 = ...

4) V= 1/3 S0 · h. V= 1/3 · 8 · 2 = 16/3 5,3. Ответ: a) 8; б) 5,3.hello_html_6d91a8cf.jpg

Пример 5. Дано: ABCA1B1C1 – усеченная пирамида. ΔАВС – прямоугольный,
AB = 18 дм, BC = 24 дм, AA1 = BB1 = СС1 = 12,5 дм, k = 0,5. Найти V.

Решение: S1 = SABC = 1/2 · AB · BC = 1/2 · 18 · 24 = 9 · 24 = … ,
S
2 = S(A1B1C1) = 1/2· A1B1 · B1C1 = 1/2 (k · AB) · (k · BC) =
= 1/2· 0,5 · 18 · 0,5 · 24 = 6 · 9 = … ,
S = S
1 + S2 + = 216 + 54 + = 216 + 54 + 54 = … ,
V = 1/3 · h · S = 1/3 378 h = 126 h, R
1 = abc/4S1 ,

c = = = … , R1 = = = …, hello_html_m78f1984a.jpg

R2 = R1 : 2 = 7,5; h2 = 12,52 – (15 – 7,5)2 = 12,52 – 7,52 = (12,5 – 7,5) · (12,5 + 7,5) =

= 5 · 20 = … , h = … ,

V = 126 h = 126 · 10 = … (дм3).
Ответ: 1260 (дм3).

Пример 6. усеченная пирамида а) n = 3, а1 = 2, а2 = 5, h = 12, V =?
Решение: A = 22 + 52 + 2 · 5 = 39, V = · h · A = · 12 · 39 = … . Ответ: 39 .

б) n = 4, a1 = 3, a2 = 8, h = 6, V = ?
Решение: A = 32 + 82 + 3 · 8 = 97, V = 1/3 · 6 · 97 = 2 · 97 = ...

Ответ: 194.


в) n = 6, a1 = 4, a2 = 9, h = 8, V = ?

Решение: A = 42 + 92 + 4 · 9 = 133, V = · 8 ·133 = 4 · 3 · 133 = ... Ответ: 1596.

Конус.

Пример 1. a) Вычислите объем конуса, если его высота 6 см, а площадь основания 42 см2.

Решение: V= 1/3S0 · h. V= 1/3· 42 · 6 = 42 2 = … см3.

Ответ: 84 см3. 

б) Найти объем конуса с радиусом основания 4 м и высотой 6 м .hello_html_22f040f9.jpg

Решение: V= 1/3 πr2 · h. V= 1/3 · π ·42 · 6 = … π м3. 

Ответ: 32 π м3. 

Пример 2. Образующая конуса равна 60 см, высота 30 см. Найдите Vк (рис.).

Решение: Из ΔАOР (O = 90°): Так как РО = 1/2АР, то = 30°, 
R = AO = 60· cos 30° = 60 · / 2 = …  см,

V= 1/3 πr2 · h. V= 1/3 π(30)2 · 30 = 900 3 10 π = … π см3. Ответ: V = 27000π см3.

Пример 3. Образующая конуса, равна 12 см, наклонена к плоскости основания под углом 30° (рис.).

Найдите объем конуса.hello_html_77625222.jpg

Решение: V= 1/3 π ·AO2 · SO.  Из ΔАSO (O = 90°): h = SO = 1/2 AC = 12 : 2 = … см.

R = AO = 12 · cos 30° = 12 · / 2 = …  см.

V= 1/3 π(6)2 · 6 = 2 π · 36 · 3 = … π см3. Ответ: V= 216π см3.

Пример 4. Образующая конуса 8 см, а угол при вершине осевого сечения 60°.hello_html_6c88cf6d.jpg

Найдите объем конуса. 

Решение: (рис.) V= 1/3 πr2 · h. r = 8 : 2 = … см.

h = 8 · sin 60° = 8 · / 2 = …  см.

V= 1/3 π · 42 · 4 = 64 / 3 21,3π см3. Ответ: 21,3π см3.

2)Решить задачи ( по примерам): Пирамида.

  1. В правильной четырехугольной пирамиде высота равна 6 см. Сторона основания 5 см. Найдите объем пирамиды.

  2. a)Объем правильной четырехугольной пирамиды равен 48 см3, высота 4 см. Найти сторону основания. б) Объем пирамиды равен 28 см3, площадь основания 4 см2. Чему равна высота?

  3. Дано: ABCD - правильная пирамида. АВ = a = 6; AD = 4 . Найти: aSocн.; б) АО; в) DO; г) V.

  4. Дано: ABCDF - правильная пирамида.  FCO = 45°FO = 4 . Найти: a) Socн.; б) V. 

  5. Дано: ABCA1B1C1усеченная пирамида. ΔАВС прямоугольный, AB = 12 дм,BC = 16 дм, AA1 = BB1 = СС1 = 13 дм, k = 0,5. Найти V.

  6. а) n = 3, а1 = 2, а2 = 5, h = 24, V =?, б) n = 4, a1 = 3, a2 = 8, h = 3, V = ?,
    в) n = 6, a1 = 4, a2 = 9, h = 4 , V = ?

Конус.

  1. a)Вычислите объем конуса, если его высота 3 см, а площадь основания 12 см2.

б) Найти объем конуса с радиусом основания 5 м и высотой 9 м .

  1. Образующая конуса равна 4 см, высота 2 см. Найдите Vк (рис.).

  2. Образующая конуса, равна 6 см, наклонена к плоскости основания под углом 30° (рис.).
    Найдите объем конуса.

  3. Образующая конуса 4 см, а угол при вершине осевого сечения 60°.Найдите объем конуса. 

3)Решить задачи :

  1. Дано: ABCDEKF – прав. пирамида. FO  (ABC), FM  AK, FO = 8, FM = 10.
    Найти:
    a) Socн.; б) V. 

  2. В треугольной усеченной пирамиде с высотой, равной 10, стороны одного из оснований равны 27, 29 и 52. Определите объем усеченной пирамиды, если периметр другого основания равен 72.

  3. Дано: конус, АР = см, PAB = 45°. Найти: V. 

  4. Найдите объем конуса, осевое сечение которого представляет собой равнобедренный прямоугольный треугольник с гипотенузой, равной 6 см.

  5. Найдите объем конуса, полученного в результате вращения вокруг большего катета прямоугольного треугольника с гипотенузой, равной 2 см, и углом 30°.

4) Выполнить расчет по модели конуса: измерить диаметр основания и высоту конуса , найти радиус основания и объем конуса.

Инструкционная карта

ПР № 19 « Расчет по модели площади цилиндра и конуса».

Задание:

  1. Перепишите и заполните пропуски:

Пример 1. Найдите площадь полной поверхности цилиндра, если диагональ его осевого сечения, равная 8см, составляет с образующей цилиндра угол величиной 30° . Решение: Поскольку AC = 8 см, а угол ACD = 30°, то CD = AC cos 30°  . Треугольник ACD - прямоугольный. Соответственно, CD / AC = cos ACD по свойству тригонометрических функций в прямоугольном треугольнике.
Значение
  cos 30 найдем из таблицы значений тригонометрических функций. CD = 8  /2 = 4. Аналогично,  AD = AC sin 30° , AD = 8 1/2 = … , Откуда радиус основания цилиндра
равен
R = 4/2 = ... см.
Площадь основания цилиндра, соответственно, равна
  S1 = πR2 = 4π. Площадь боковой поверхности цилиндра равна площади его развертки - произведению длины окружности основания и высоты цилиндра. То есть: S2 = 2πRh = 2π 2 4= … π. Общая площадь поверхности цилиндра равна: 
S =S1 + S2 =   4π +  16π.
Ответ:  4π +  16π.
Пример 2. Дано: цилиндр; ABCD - квадрат; АС = 4 см (рис. ). Найти: Sб.п.ц. Решение: Sб.п.ц. = 2πRH. Пусть АВ = х, тогда х2 + х2 = 42; 2х2 = 16; х2 = 8;
х = 2. 
= ; Н = 2. . Sб.п.ц. = 2π · · 2= 2 2 2 π = …π (см2).
Ответ: 8π см2. Пример 3. Дано: цилиндр, ABCD - квадрат; Sосн.ц. = 16π см2 (рис.). Найти: Sб.п.ц. Решение: πR2 = 16π; R2 = 16; R = ... , АВ = ВС = 4 · 2 = … (см). 
Sб.п.ц. = 2πRH, где R = 4; Н = 8.Sб.п.ц. = 2π · 4 · 8 = … π (см2).
Ответ:
64π см2.
Пример 4. Высота конуса равна 5см, а радиус основания 12см.
Найдите площадь полной поверхности конуса. 
Решение: Для нахождения площади поверхности конуса воспользуемся следующими формулами: S1 = rl - площадь боковой поверхности конуса, где r - радиус конуса, а l - длина образующей, S2 = r2 - площадь круга, то есть основания конуса. Таким образом, площадь поверхности конуса составит  S = S1 + S2 . Поскольку S1 = rl , найдем образующую. Поскольку высота конуса, радиус основания конуса и образующая являются сторонами прямоугольного треугольника, то l2 = h2 + r2 , l2 = 52 + 122 = 25 + 144 = … , l = ....
Тогда
 S = S1 + S2 = + 144 = 156+ 144 = … ≈ 942,48 
Ответ: 300 ≈ 942,48 см2 .
Пример 5. Дано: конус, h = OP = 1,2 см, Sосев. = 0,6 см2 (рис.). Найти: Sполн. . Решение: 1) Осевое сечение - треугольник: высота 1,2 см и основание 2r.
Sосев. =  · 2r h = r h, r = Sосев. : h = 0,6 : 1,2 = 0,5 см.
Из ΔАОР по теореме Пифагора:
l2 = h2 + r2  = OP2 + OA2.
l2 = 1,22 + 0,52 = 1,44 + 0,25 = …, l = … см.
Sполн. = · (r + l) , Sполн. = 0,5 · (0,5 + 1,3) = · 0,5 · 1,8 = …
hello_html_m6d6bd24b.jpghello_html_2cd2a3a8.jpghello_html_28ad1e01.jpg

Ответ: 0,9π см2.


2)Решить задачи ( по примерам):


  1. Найдите площадь полной поверхности цилиндра, если диагональ его осевого сечения, равная 16 см, составляет с образующей цилиндра угол величиной 30 °.

  2. Дано: цилиндр; ABCD - квадрат; АС = 16 см . Найти: Sб.п.ц.

  3. Дано: цилиндр, ABCD - квадрат; Sосн.ц. = 25π см2 . Найти: Sб.п.ц.

  4. Высота конуса равна 10 см, а радиус основания 24 см.
    Найдите площадь полной поверхности конуса.

  5. Дано: конус, OP = 2,4 см, Sосев. = 2,4 см2 .Найти: Sполн..




3)Решить задачи :

  1. Найдите площадь полной поверхности цилиндра, если диагональ его осевого сечения,
    равная 4 см, составляет с образующей цилиндра угол величиной 30
    °. 

  2. Дано: цилиндр; ABCD - квадрат; АС = 8см (рис.). Найти: Sб.п.ц.

  3. Дано: цилиндр, ABCD - квадрат; Sосн.ц. = 36π см2 (рис.). Найти: Sб.п.ц.

  4. Площадь боковой поверхности цилиндра вдвое больше площади основания, а площадь полной поверхности равна 256π см². Найдите радиус r и высоту цилиндра h.

  5. Площадь основания равностороннего цилиндра равна 2π см². Найдите площадь осевого сечения цилиндра.

  6. Найдите угол между высотой и образующей конуса, если площадь боковой поверхности конуса равна 2, а площадь полной его поверхности равна 3.

  7. Образующая конуса, равная 4 см, наклонена к плоскости основания под углом 60°. Найдите площадь осевого сечения конуса.

  8. Угол при вершине осевого сечения конуса равен 60°, сумма длин его высоты и образующей равна 2 см. Найдите площадь полной поверхности конуса.

  9. Радиус основания конуса равен 10 см, а высота равна 15 см. Найдите площадь сечения конуса плоскостью, параллельной основанию и находящейся на расстоянии 2 см от вершины конуса.

  10. Высота конуса равна 6 см, а радиус основания 8 см. Найдите площадь полной поверхности конуса.

  11. Образующая конуса наклонена к плоскости основания под углом 60° и равна 4   см. Найдите площадь осевого сечения конуса, 

  12. Радиус основания конуса равен 7 см, а высота — 7 см. Найдите площадь сечения конуса плоскостью, параллельной основанию и находящейся на расстоянии 4 см от его вершины, 

  13. Найдите боковую поверхности цилиндра с высотой, равной 3 см, если осевое сечение цилиндра плоскостью - квадрат,

  14. Найдите боковую поверхность конуса, в осевом сечении которого равнобедренный прямоугольный треугольник с гипотенузой 6 см. 

  15. Радиус основания конуса равен 2см, а образующие наклонены к плоскости основания под углом 60°. Найдите боковую поверхность и объем конуса,

  16. Боковая поверхности цилиндра равна 48π см2, радиус основания - 6 см. Найдите площадь осевого сечения,

  17. Найдите боковую поверхность конуса, осевое сечение которого равнобедренный треугольник с углом при вершине 120° и боковой стороной 6см. 

  18. Осевое сечение цилиндра - квадрат, диагональ которого 4 см. Найдите площадь боковой поверхности цилиндра.

  19. Радиус основания конуса равен 6 см, а образующая наклонена к плоскости основания под углом 60°. Найдите площадь сечения, проходящего через две образующие, угол между которыми равен 45° и площадь боковой поверхности конуса.

  20. В цилиндре проведена плоскость, параллельная оси и отсекающая от окружности основания дугу в 120°. Диагональ сечения равна 20 см и удалена от оси на 3 см. Найдите площадь боковой поверхности цилиндра.

4) Выполнить расчет по модели цилиндра и конуса:
измерить диаметр основания и высоту цилиндра и конуса ,
найти площадь основания цилиндра и конуса,
найти площадь боковой поверхности цилиндра и конуса,
найти площадь полной поверхности цилиндра и конуса.
5) Формулы для расчета площадей (результат округлить до целого числа, принять
при расчете
π = 3) .

Цилиндр:

S осн.= r2 = 0,25 d2 π , S бок.= rh= , S пол .= r ( h + r ) = 0,25 d2 π;

r = 0,5; S пол .= S осн. + S бок.

Конус:

S осн.= r2 = 0,25 d2 π , S бок.= r l = 0,5 l; S пол .= r (r + l ) = 0,25 d2 π + 0,5 l;

l2= h2+ 0,25 d2, r = 0,5; S пол .= S осн. + S бок.




Инструкционная карта

ПР № 20 «Вычисление объёма шара. Расчет объёмов сегмента, слоя, сектора шара».

Задание:

  1. Перепишите и заполните пропуски:

Пример 1. a)Вычислите объем шара, если его радиус = 6 см.
Решение: Vшара = 4/3 R3 = 4/3216 = 72 4 = ….

  • Ответ: 288 см3.

  • б)Вычислите диаметр шара, если его объем V = 36π.

  • Решение: Vшара = 1/6 d3 = 1/6 d3, d3 = 36 6 = … , d = 6.

  • Ответ: 6 см.hello_html_66b842ba.jpg

  • Пример 2.Диаметр основания конуса равен 6 м, образующая наклонена к плоскости основания под углом 60° (рис.). Найти: объем шара описанной около конуса сферы. 

  • Решение:

  • 1) Центр O1  ОС, OBC = 60°  ΔАВС - равносторонний.

  • 2) R = O1C = AB / 3 = 6 /3 = … .

  • 3) Vшара = 4/3 R3 = 4/3 (2)3 = 4/3 8 3 = … .

  • Ответ: 32  м2.

  • Пример 3. Диаметр свинцового шара равен 30 см. Сколько шариков, диаметр которых 3 см, можно сделать из этого свинца?

  • Решение: n = V1 / V2 = /6 d1 3) / (6 d23) = d1 3 / d2 3 = (d1 / d2)3 = (30 / 3) 3 = 103 = ...

  • Ответ: 1000 шариков.

Пример 4. Какую часть шара составляет объем шарового сегмента, у которого высота равна 0,1 диаметра шара?
Решение: Десятая часть диаметра есть пятая часть радиуса. Значит, высота сегмента h= R/5 ,
V сегм. = (R/5)2 (RR /15) = (R2/25) 14R/15 = 14 R3/375,
V сегм.: V = ( 14/375) : (4/3) = 7/250 100 % = 28 : 10 = … % .
Ответ:  2,8%.
Пример 5. Плоскость, перпендикулярная диаметру шара, делит его на части 3 см и 9 см. На какие части делится объем шара?
Решение: = (3 + 9) : 2 = … см. Высота меньшего сегмента h равна 3 см.
Его
V1 = h2 (Rh / 3) = 32 (61) = 5 9 = … см2.
V = 4/3 R3 = 4/3 63 = 4/3 216 = 72 4 = … см3.
Значит, 
V2 = VV1 = 288 - 45 = … см3.
Ответ: 45 , 243 см3. Пример 6. Дано: шар, DС — диаметр секущей плоскости, АМ = 6 см, MB = 12 см (рис.). V1 - объем меньшего шарового сегмента, V2 - объем большего шарового сегмента. Найти: V1, V2. 
Решение: СD  АВ, АМ = 6 см, MB = 12 см. На рисунке: DС - диаметр круга, который является плоскостью, перпендикулярной к диаметру шара, делящей шар на два шаровых сегмента.
Диаметр шара АВ = АМ +
 MB = 6 + 12 = … (см),R = 18 : 2 = … см.
Объем шарового сегмента вычисляется по формуле:
V = h2 (Rh / 3) ,  где h = AM - высота меньшего сегмента. V1 = AM2 (RAM / 3) = 62 (9 – 6/3) = 36 7 = … см3. Объем шара равен:   Vшара = 4/3 R3 = 4/3 93= 4 81 3 = … см3.
V2 = VV1 =  972 252 = … см3.
Ответ: 252π см3 и 720π см3.
Пример 7. Чему равен объем шарового сектора, если радиус окружности основания равен 60 см, а радиус шара - 75 см.
Решение: Пусть R - радиус шара, r - радиус основания сегмента. Вычислим высоту сегмента Н = РО1, OP = R. Из прямоугольного ΔОО1М: 
OO12 = OM2O1M2 = R2r2 = 752602 = 5625 – 3600 = …, OO1 = … см. h = PO1 = OPOO1 = 7545 = … см.
V = 2/3 R2 h = 2/3 75230 = 20 5625 = … см3.
Ответ: 112 500 см3.
Пример 8. Дано: шар, h = 30, R = 45 см. Найти: V1, V2, V3.
Решение:
Объем шарового сегмента вычисляется по формуле: V1 = h2 (Rh / 3) ,  V1= 302 (45 – 30:3) = 900 35 = … см3.
V2 = 4/3R3 2 h2 (Rh / 3)
V2 = 4/3453 2 302 (45 – 30 / 3) = 121500 63000 = …см3.
V3= 2/3 R2h = 2/3452 30 = 2025 20 = … см3.
Ответ: 31500 58500 40500см3.
hello_html_m45f5722.jpghello_html_m4c44f677.jpg

2)Решить задачи ( по примерам):

  1. a)Вычислите объем шара, если его радиус = 3 см.
    б)Вычислите диаметр шара, если его объем V= 32π/3.

  2. Диаметр основания конуса равен 6 м, образующая наклонена к плоскости основания под углом 60° .Найти: объем шара описанной около конуса сферы. 

  3. Диаметр свинцового шара равен 30 см. Сколько шариков, диаметр которых 3 см, можно сделать из этого свинца?

  4. Какую часть шара составляет объем шарового сегмента, у которого высота равна 0,2 диаметра шара?

  5. Плоскость, перпендикулярная диаметру шара, делит его на части 6 см и 12 см. На какие части делится объем шара?

  6. Дано: шар, DС — диаметр секущей плоскости, АМ = 3 см, MB = 9 см . V1 - объем меньшего шарового сегмента, V2 - объем большего шарового сегмента. Найти: V1V2. 

  7. Чему равен объем шарового сектора, если радиус окружности основания равен 12см, а радиус шара - 15 см.

  8. Дано: шар, h = 30, R = 42 см. Найти: V1V2, V3.

3)Решить задачи :

  1. Вычислите объем шара, если его радиус R = 6 см.

  2. Вычислите диаметр шара, если его объем V = 36π.

  3. Объем шара равен 135. Найти объем другого шара, диаметр которого в 3 раза больше, чем у данного.

  4. Площадь сечения шара плоскостью равна 16π. Найти расстояние от плоскости сечения до центра шара, если объем шара равен 500/3 .

  5. Шаровой сегмент, R = 10 см, h = 6 см. Найти объем сегмента V.

  6. Шаровой слой, R = 36 см, h = 12 см, V = ?

  7. Шаровой сектор, R = 6 см, h = 2 см, V = ?

  8. Шаровой сегмент, R = 75 см, r = 60 см, (h > 100). Найти V.

  9. Шар, плоскость α делит его на две части и перпендикулярна AB, AB – диаметр шара,
    AO1 = 6 см, O1В = 12 см, O1 – центр сечения плоскостью шара.
    Найти объемы частей шара
    V1 и V2.

  10. а) Шаровой сегмент, h = 6 см, V = 720 π см3. R - ? б) Шаровой сегмент, r = 5 см, h = 1 см. R - ?

  11. а) Шаровой сектор, h = 15 см, V = 4000 π см3. R - ?
    б) Шаровой сектор,
    R = 10 см, V = 400 π см3. h - ?

  12. а) Шаровой сектор, r = 60 см, R = 75 см. V = ?
    б) Шаровой сектор,
    h = 30 см, V = 112500 π см3. R - ?

  13. Шаровой слой, h = 30 см, R = 45 см. V = ?

  14. Какую часть объема шара составляет объем шарового сегмента, у которого высота равна 0,1 диаметра шара, равного 20 см?

  15. Чему равен объем шарового сектора, если радиус окружности основания равен 60 см, а радиус шара - 75 см.

  16. Диаметр шара радиуса 15 см разделен на 3 части, длины которых относятся как 2:3:5. Через точки деления проведены плоскости, перпендикулярные диаметру. Найдите объем образовавшегося шарового слоя.

  17. Радиусы трех шаров 3, 4 и 5 см. Найдите радиус шара, объем которого равен сумме их объемов.

  18. В шаре радиуса 15 см проведено сечение, площадь которого равна 81 см2. Найдите объем меньшего шарового сегмента, отсекаемого плоскостью сечения.

  19. Найдите объем шарового сектора, если радиус шара равен 6 см, а высота соответствующего сегмента составляет шестую часть диаметра шара.

  20. Радиусы оснований шарового слоя равны 3 см и 4 см, а радиус шара - 5 см. Найдите объем слоя, если его основания расположены по одну сторону от центра шара. 


Инструкционная карта

ПР № 21 «Вычисление объёмов тел».

Задание:

1) Перепишите и заполните пропуски:
Пример 1. Дано: цилиндр, ABCD - осевое сечение, ABCD - квадрат, АС = 8 см. (рис.).
Найдите: 
Vцил.
Решение: 1) V= Sосн · h.
hello_html_2cd2a3a8.jpg

2) Рассмотрим ΔАВС - прямоугольный, так как ABCD квадрат.
Пусть АВ = ВС = х см, тогда х
2 + х2 = (8)2 , 2х2 = 64· 2 , х2 = 64,  х = …,
х = - 8 не удовлетворяет условию задачи. Итак: АВ = ВС = 8 см, т.е. 
= 8 (см).

3) Найдем радиус основания: = 1/2AD = ВС : 2 = 8 : 2 = … см,
тогда
S осн. =  r2 = … ,

4) V= 16 8 = …см3.
Ответ: 128 см3.
Пример 2. Цилиндр имеет диаметр основания 14 см, а высоту 5 см. Найдите объем и площадь полной поверхности цилиндра.

Решение: = 14 : 2 = … см, = 5 см, V= r2 · h = 49 · 5 = … см3.
S пол .= r ( h + r ) = 2 · 7 · · ( 5 + 7) = 14 ·12 = … см2.
Ответ:  245 см3 и 168 см2.
Пример 3. Образующая конуса равна 12 см. Угол между образующей и плоскостью основания равен 30 градусов. Найти объем конуса.
hello_html_m120fbaaf.png

Решение: Объем конуса найдем по формуле: 
 V= 1/3S0 · h.

Поскольку образующая вместе в высотой конуса и радиусом его основания образуют прямоугольный треугольник, то необходимые размеры конуса вычислим исходя из того, что нам известен угол этого прямоугольного треугольника между основанием и образующей конуса.

h / OB = sin 30 , h = OB sin 30 , h = 12 sin 30 , h = 12 · 1/2  = 12 : 2 = … ,
R / OB = cos 30
 , R = OB cos 30 , R = 12 cos 30 ,
R = 12 /2
 , R = 6   

Откуда объем конуса будет равен: 
V = 1/3π ( 6 )2 · 6  = 1/3π· 36 · 3 · 6 = 36 · 6 = …   см3.

Ответ: объем конуса равен   216π см3 .  
Пример 4. Объем конуса равен 27. На высоте конуса лежит точка и делит её в отношении 2:1 считая от вершины. Через точку проведено сечение, которое является основанием меньшего конуса с той же вершиной. Найдите объем меньшего конуса. 
Решение: Обратим внимание, что треугольники AOB и COD - подобны. Из условия задачи определим коэффициент подобия как 2:3. 
Объем конуса находится по формуле:  Vконуса = 1/3πR
2h = 27 (по условию) 
Тогда объем малого конуса будет равен 
Vмал.конуса = 1/3π
· (2/3R)2 · (2/3h) , то есть  Vмал.конуса = 1/3π· 4/9 R2 ·2/3 h, 
Vмал.конуса = 8/27
·1/3π R2 h 
а так как мы знаем, что 1/3π R
2 h= 27 (см. выше), то  Vмал.конуса = 8/27 · 27 = … 
Ответ:  объем малого конуса равен 8 см3.
Пример 5.Сторона основания правильной четырехугольной пирамиды равна 10, а угол боковой грани с плоскостью основания равен 60°. Найдите объем шара, вписанного в пирамиду (рис.). 
hello_html_m1caaa07f.jpg

Решение: Рассмотрим сечение, проведение через высоту пирамиды и две апофемы. В сечении получается ΔАВС - равносторонний. Радиус вписанной в него окружности будет равен  r = a / 2, r = 10/2 = 10 : 2 = … ,
Vшара= 4/3R3 = 4/3 53 = 500/3 167.
hello_html_7ac6054e.jpg

Ответ: 167.

Пример 6.В шар вписана правильная треугольная призма так, что ее высота вдвое больше стороны основания. Найдите объем шара, если объем призмы равен 27/π (рис.) 

Решение:

1) Пусть х - сторона основания. Тогда высота призмы 2х. Ее объем Sосн. · h.
V=/4 x2 x = /2x3. По условию V=27/ , /2 x3 = 27/, x3 = 54/, x = 3 .

2) Радиус найдем из ΔOO1A1O1A1 - радиус описанной окружности около треугольника A1B1C1O1A1 = a / ,

O1A1 = = ,
OO1= x = 3 , так как О - середина О1О2.

R2 = OA12 = O1A12 + OO12 = ()2 + ()2 = ( )2 = (2 )2. R = OA1 = 2 .

3)Объем шара Vшара= 4/3R3 = 4/3 8 = 4 · 8 · 2 = ...

 Ответ: = 64.

2)Решить задачи ( по примерам):

  1. Дано: цилиндр, ABCD - осевое сечение, ABCD - квадрат, АС = 6 см. Найдите: Vцил.

  2. Цилиндр имеет диаметр основания 16 см, а высоту 5 см. Найдите объем и площадь полной поверхности цилиндра.

  3. Образующая конуса равна 20 см. Угол между образующей и плоскостью основания равен 30 градусов. Найти объем конуса.

  4. Объем конуса равен 54. На высоте конуса лежит точка и делит её в отношении 2:1 считая от вершины. Через точку проведено сечение, которое является основанием меньшего конуса с той же вершиной. Найдите объем меньшего конуса.

  5.  Сторона основания правильной четырехугольной пирамиды равна 12 , а угол боковой грани с плоскостью основания равен 60°. Найдите объем шара, вписанного в пирамиду.

  6. В шар вписана правильная треугольная призма так, что ее высота вдвое больше стороны основания. Найдите объем шара, если объем призмы равен 54/π .

3)Решить задачи :


  1. Пусть V, r и h соответственно объем, радиус и высота цилиндра.
    Найдите
    V, если r = 5 см, h = 6 см.

  2. Пусть V, r и h соответственно объем, радиус и высота цилиндра. Найдите h,
    если
    r = 10 см, V = 400 см3.

  3. Радиус основания конуса равен 12 см, а его образующая равна 13 см. Найдите ребро куба, объем которого равен объему данного конуса.

  4. Пусть V, r и h соответственно объем, радиус основания и высота конуса. Найдите h,
    если
    r = 6 см, V = 288 см3.

  5. Радиусы оснований усеченного конуса равны r 1 и r 2 , а высота равна h. Найдите объем усеченного конуса V, если r 1 = 3 м, r 2 = 4 м, h = 3 м.

  6. Радиусы оснований усеченного конуса равны r 1 и r 2 , а объем равен V. Найдите высоту усеченного конуса h, если r 1 = 3 м, r 2 = 5 м, V = 294 м3.

  7. Пусть V, R соответственно объем и радиус шара. Найдите объем шара V, если R = 6 см.

  8. Пусть V, d соответственно объем и диаметр шара. Найдите диаметр шара d, если
    V = см3.

  9. Пусть V1, V 2 , V 3 соответственно объем шарового сегмента, объем шарового слоя, объем шарового сектора, R- радиус шара, h – высота шарового сегмента. Найдите V1, V 2 , V 3 , если R = 42 см и h = 30 см.

  10. Объемы двух шаров относятся как 8 : 1. Найдите отношение их радиусов.



Инструкционная карта

ПР № 22 «Составление уравнения сферы».
Задание:

1)Перепишите и заполните пропуски:
Пример 1. Сфера задана уравнением x 2 + (y + 3)2 + (z – 2)2 = 25.

Найдите координаты центра и радиуса сферы.

Решение: О - центр сферы, О(0,3,2), R = = ...
Ответ:
О(0,3,2), R = 5.

Пример 2. Напишите уравнение сферы радиуса = 7 с центром в точке А(2; 0; 1). Решение: (x …)2 + y 2 + (z + …)2 = 72. (x2)2 + y 2 + (z + 1)2 = …
Ответ: (x2)2 + y 2 + (z + 1)2 = 49.

Пример 3. Лежит ли А(2; 1; 4) на сфере, заданной уравнением  (x + 2)2 + (y 1) 2 + (z 3)2 = 1. Решение: Подставим координаты точки А в уравнение сферы (2 + 2)2 + (1 1) 2 + (4 3)2 = 1, 1 = 1(верно), точка А лежит на сфере.
Ответ:
точка А лежит на сфере.

Пример 4. Найти координаты центра и радиус сферы x2 + y2 + z2 + 4y - 2z = 4. Решение: x2 + y2 + z2 + 4y 2z = 4 выделим квадрат двучлена:
х
2 + у2 + 4у + 4 4 + z2  4z + 1 1 = 4, х2 + (у + 2)2 + (z 1)2 = 9, центр окружности С(…; …; …), радиус R = ...
Ответ:
С(0; 2; 1), R = 3.

Пример 5. Дано: уравнение сферы, х2 + у2z2 + 2у 4= 4.

Найти: а) О(х0; у0z0), R; б) m, при котором А(0; m; 2) и В(1; 1; m2) принадлежат сфере.

Решение: а) x 2 + y 2 +2у + z 2 – 4z = 4, x 2 + y 2 +2у +11 + z 2 – 4z + 4 4 = 4,
x 2 + (y + 1)2 + (z – 2)2 = 9. О(...,…,…), R = = ...
б) А(0; 
m; 2) и В(1; 1; m2)


 , , ,

, m = 2. При m = … точки A и В принадлежат сфере. Ответ: а) О(0; 1; 2), R = 3; б) при m = 2.

2)Решить задачи ( по примерам):

  1. Сфера задана уравнением (x – 1)2 + y 2 + (z – 2)2 = 9.

Найдите координаты центра и радиуса сферы.

  1. Напишите уравнение сферы радиуса = 4 с центром в точке А(2; 1; 0).

  2. Лежит ли А(5; 1; 4) на сфере, заданной уравнением  (x 3)2 + (y+ 1) 2 + (z 4)2 = 4.

  3. Найти координаты центра и радиус сферы x2 – 6x + y2 + z2 = 0.

  4. Дано: уравнение сферы, х2 + у2z2 + 4у 2= 4.

Найти: а) О(х0; у0z0), R; б) m, при котором А(0; m; 1) и В(1; 0; m2) принадлежат сфере.

3)Решить задачи :

  1. Точки А(3; – 5; 6) и В(5; 7; – 1) являются концами одного из диаметров сферы. Составьте уравнение этой сферы.

  2. Дана сфера x2 + y2 + z2 = 450  . Найти координаты точек пересечения сферы с прямой, проходящей через начало координат и точку А(4; 5; 3).

  3. Даны точки А(– 1; 3; 2), В(0; 3; 1), С(2; – 2; 0), D(– 4; 2; 2), Е(5; 7; 8). Какие из этих точек принадлежат сфере с центром О(– 2; 1; 0) и радиусом 3?

  4. Составьте уравнение сферы с центром О (2; 3; 4) и радиусом R=5.

  5. Точки А(7; – 2; 4) и В(9; – 8; 6) лежат на поверхности сферы и на прямой, проходящей через её центр. Составьте уравнение сферы.

  6. Сфера задана уравнением x 2 + y 2 + z 2 + 2y – 4z = 4. a)Найдите координаты центра и радиуса сферы. б) Найдите значение m, при котором точки А(0; m; 2) и В(1; 1; m – 2) принадлежат данной сфере.

  7. Диаметр сферы – отрезок АВ с концами А(2; – 1; 4) и В(2; 7; 10). a) Составьте уравнение сферы. б) Найдите кратчайшее расстояние от точки данной сферы до плоскости Оxy.

  8. Сфера задана уравнением (x – 1)2 + y 2 + (z – 2)2 = 9. а)Найдите координаты центра и радиуса сферы. б)Определите, принадлежат ли данной сфере точки А(1; 3; 1) и В(2; 2; 1).


  1. Составить уравнение сферы в каждом из следующих случаев:

а) сфера имеет центр С(0; 0; 0) и радиус r = 9;

б) сфера имеет центр С(5; 3; 7) и радиус r = 2;

в) сфера проходит через начало координат и имеет центр С(4; 4;2);

г) сфера проходит через точку А(2; 1; 3) и имеет центр С(3; 2; 1);

д) точки А(2; 3; 5) и В(4; 1; 3) являются концами одного из диаметров сферы;

  1. Сфера задана уравнением x2 + у2 + z2 + 2у 4z = 4.

а) Найдите координаты центра и радиус сферы.

б) Найдите значение m, при котором точки А(0; m; 2) н В (1; 1; m2) принадлежат данной сфере.

  1. Найдите координаты центра и радиус сферы, заданной уравнением

(x – 2)2 + (y + 3) 2 + z2 = 25. 

  1. Напишите уравнение сферы радиуса R = 7 с центром в точке А(2; 0; 1).

  2. Лежит ли А(2; 0; 3) на сфере, заданной уравнением (x + 2)2 + (y 1) 2 + (z 3)2 = 1. 

  3. Могут ли все вершины прямоугольного треугольника с катетами 4 см и 2см лежать на сфере радиуса см?

  4. Найти координаты центра и радиус сферы x2 + 6х + y2 + z 2 = 0. 

  5. Найдите координаты центра и радиус сферы, заданной уравнением

(x + 3)2 + y 2 + (z 1)2 = 16. 

  1. Напишите уравнение сферы радиуса R = 4 с центром в точке А(2; 1; 0).

  2. Лежит ли точка А(5; 1; 4) на сфере, заданной уравнением

(x –3)2 + (y + 1) 2 + (z 4)2 = 4. 

  1. Могут ли все вершины прямоугольного треугольника с катетами 4 см и 2см лежать на сфере радиуса см?

  2. Найти координаты центра и радиус сферы x2 + y2 + 6у + z2 = 0. 

  3. Составить уравнение сферы с центром в точке А (– 3; 4; – 9) и проходящую через

точку N (– 2; 6; 1).

  1. Составить уравнение сферы которая касается каждой из координатных плоскостей и проходит через точку M (2;1;3).

  2. Составьте уравнение сферы с центром в точке О(– 1;0;2), если известно, что этой сфере принадлежит точка А(3;1;1).

  3. Даны точки А(2; – 5;8) В(8; – 2;5) С(5; – 8:2)и Д(– 2; – 8; – 5).Составьте уравнение сферы, если известно, что эти точки лежат на её поверхности.

  4. Точка А лежит на сфере с центром О(3; 0; 0).

  1. Напишите уравнение сферы.

  2. Принадлежат ли этой сфере точки с координатами и (4; – 1; 0)?

  1. Составьте уравнение сферы, радиус которой равен 2, если известно, что центр сферы лежит в плоскости ОХZ, а сама сфера проходит через начало координат и точку А(1; 1; 0).

  2. Центр сферы имеет координаты (0; 0; 4). Сфера проходит через точку .

  1. Напишите уравнение сферы.

  2. Принадлежат ли сфере точки с координатами (3; 1; 5) и (0;6)?

  1. Составьте уравнение сферы с радиусом, равным 3, если известно, что центр сферы лежит на оси OZ и сфера проходит через точку К(– 2; – 2; 1).

  2. Найти уравнение сферы, проходящей через точки  (0;0;0), (4;0;0),(0;6;0) и (0;0;8).

  3. Определить координаты центра С и радиус r сферы, заданной уравнением

х 2 + у 2 + z 2 6х + 4z – 3 = 0.

  1. Определить координаты центра С и радиус r сферы, заданной уравнением

х 2 + у 2 + z 2 – 2х + 2у – 10z + 2 = 0.

  1. Найдите координаты центра и радиус сферы, заданной уравнением:

1)x² + y² + z² = 49,

2)(х 3)² + (у + 1)² + (z + 3)² = 1,

3)х² + (y 4)² + z² = 3,

4)(x 1)² + y² + (z + 2)² = 25.

  1. Найти координаты центра и радиус сферы, заданной уравнением
    x2 + y2 + z2 – x + 2y + 1 = 0.

Инструкционная карта

ПР № 23«Решение задач на вычисление длины векторов, суммы векторов».

Задание:

1)Перепишите и заполните пропуски:

Пример 1. Даны точки А(2;3; –1) и В (–5;3; 0) . Найти длину отрезка АВ.

Решение: По соответствующей формуле: А(2;3; –1) и В (–5;3; 0). 


Ответ:

Пример 2. Даны точки: А(5;7; 2), В (5;4; 6) , С (9;4; 9) 

Выяснить, равнобедренный ли треугольник, построенный на этих точках.

Решение: По формуле длины вектора найдём длины сторон и установим, есть ли среди них две равные:


Две равные стороны нашлись, следовательно необходимость искать длину третьей стороны отпадает, а заданный треугольник является равнобедренным.

Ответ: треугольник является равнобедренным.

Пример 3. а) Даны два вектора:  и .Найти .

Решение: .

б) Даны четыре вектора: .

Найти координаты векторов  

Решение:


Ответ:

Пример 4. В кубе АВСDА1В1С1D1, сторона которого равна 3, на диагоналях граней АD1 и D1В1 взяты точки Е и К так, что D1Е : АD1 = 1 : 3, D1K : D1B1 = 2 : 3. hello_html_m36f43c90.png

Найдите длину отрезка DK.
Решение:

hello_html_m14849d6b.gif

hello_html_3365f952.gif

hello_html_3b804b4.gif

hello_html_61eeff54.gif

Ответ:

Пример 5. Дано:

Решение:

Ответ:

Пример 6. Найдите длину вектора КА АС.

hello_html_m436d4048.gif

Ответ: 20 см.

2)Решить задачи ( по примерам):

  1. Даны точки А(4;6; –2) и В (–10;6; 0) . Найти длину отрезка АВ.

  2. Даны точки: А(10;14; 4), В (10;8; 12) , С (18;8; 18) 

Выяснить, равнобедренный ли треугольник, построенный на этих точках.

  1. а) Даны два вектора:  и .Найти .

б) Даны четыре вектора: .

Найти координаты векторов  

  1. В кубе АВСDА1В1С1D1, сторона которого равна 3, на диагоналях граней АD1 и D1В1 взяты точки Е и К так, что D1Е : АD1 = 2 : 3, D1K : D1B1 = 1 : 3. Найдите длину отрезка DK.

  2. Дано:

  3. Найдите длину вектора КА АС, диагонали ромба 6 и 8 см.

3)Решить задачи :

  1. Даны векторы и   Найдите координаты вектора .

  2. Даны векторы,. Найдите координаты вектора 

  3. На каком расстоянии от плоскости (хОу) находится точка А(2; 3; -5).

  4. На каком расстоянии от начала координат находится точка А(3; 4; 0).

  5. Найти длину вектора  если А(5; 3; 2), В(3; 1; 4).

  6. На каком расстоянии от плоскости (yOz) находится точка В(3; 2; 4).

  7. Даны векторы и  . Найдите  

  8. Изобразить систему координат Оху: и построить точку А(1; 2; 4). Найти расстояние от этой точки до координатных плоскостей.

  9. Вершины ΔАВС имеют координаты А(2; 0; 1), В(1; 2; 3), С(8; 4; 9). Найдите координаты вектора  если ВМ - медиана ΔABC.

  10. Даны точки А(1; 5; 3) В(7; 1; 3) С(3; 2; 6). Доказать, что ΔАВС - прямоугольный.

  11. Даны точки А(2; 1; 2), В(6; 3; 2) на оси аппликат.
    Найти точку С, равноудаленную от точек А и В.

  12. Дано: А(2; 5; 8), В(6; 1;0).На оси ординат найти точку С, равноудаленную от точек А и В.

Найти: площадь ΔABC.

Инструкционная карта

ПР № 24«Умножение вектора на число .Вычисление координат векторов».

Задание:

1)Перепишите и заполните пропуски:
Пример 1. Дано:



Решение:

  1. Находим координаты вектора

;

  1. Затем находим координаты вектора


  1. Теперь находим аналогично координаты вектора


  1. Теперь находим сумму данных векторов, складывая соответствующие координаты:


Ответ:
Пример 2. Дано: , . Найдите  

Решение: Первый случай

  1. Находим координаты вектора

;

  1. Затем находим разность векторов

;

  1. Теперь находим длину вектора :

Второй случай

  1. Находим координаты вектора

;

  1. Находим координаты вектора

;

  1. Затем находим сумму векторов

;

  1. Теперь находим длину вектора : =


Ответ:

Пример 3. Даны векторы   и . Найти

Решение: Для действий с векторами справедлив обычный алгебраический приоритет: сначала умножаем, потом складываем:

=  3 - 2= - =
= = .

= + 4 {7; -9 ;1 } = + = =

=
Ответ:  ,

Пример 4. Найдите сумму векторов: и .

Решение: , .

Ответ:

Пример 5. Даны векторы , Найдите координаты векторов

Решение: , , с ,

, . Ответ:

2)Решить задачи ( по примерам):

  1. Дано: А(2;-1;6), В (2;0; -1), С(1; - 5; 0)

  1. Дано: , , ; 2).

  2. Даны векторы   и ,  . Найти

  3. Найдите сумму векторов: и .

  4. Даны векторы , , . Найдите координаты векторов

и

3)Решить задачи :

  1. Найдите координаты вектора , если

  2. Даны векторы {1;3; 3} и . Найдите координаты и длину вектора.

  3. Даны векторы {3;1; 2} и . Найдите координаты вектора

  4. Найдите длину вектора , , если {2;1; 5} и .

  5. Из точки А построен вектор . Найдите координаты точки В , если:

А(3;1;2), .

  1. Упростите выражение:  

  2. Точка К - середина ребра В1С1 куба ABCDA1B1C1D1. Разложите вектор  по векторам  и найдите длину этого вектора, если ребро куба равно 2.

  3. Дан параллелограмм KLMN.  Выразите вектор через векторы  и



Инструкционная карта

ПР № 25 « Решение задач в координатах».

Задание:

1)Перепишите и заполните пропуски:
Пример 1. Дано: ΔАВС, А(2; 0; 1), В(1; 2; 3), С(8; 4; 9). ВМ - медиана.

Найти: координаты вектора .

Решение: По определению медианы, М - середина отрезка АС. Следовательно, координаты М найдем по формулам координат середины отрезка  M ((82)/2, ( 4 + 0)/2,(9 + 1)/2), M(…,…,…).{3 + 1,22,53}, {…,… ,…}. Ответ: {4; 4; 2}.

Пример 2. Дано: А(1; 5; 3), В(7; 1; 3), С(3; 2; 6). Доказать: ΔABC - прямоугольный.

Решение: По формуле расстояния между двумя точками найдем длины отрезков АВ, АС, ВС.
AB2 = (7 + 1)2 + (5 + 1)2 + (3 – 3)2, AB2 = 64 + 36 = … , BC2 = (7– 3)2 + (– 2 + 1)2 + (6 – 3)2,
BC2 = 16 + 1 + 9 = … , AC2 = (3 + 1)2 + (5 + 2)2 + (6 – 3)2, AC2 = 16 + 49 + 9 = ...

Проверим равенство АВ2 = ВС2 + АС2, 100 = 26 + 74 верно.

По теореме обратной теореме Пифагора делаем вывод, что ΔABC - прямоугольный
с гипотенузой АВ.

Пример 3. Дано: ΔАВС; М, N, К - середины сторон соответственно АВ, ВС, АС. М(3; 2; 5), 
N(3,5; 1; 6), К(1,5; 1; 2). Найти: координаты А, В, С.

Решение: Пусть A (х1; у1z1), В(х2; у2z2), С(х3; у3z3). По формулам координат середины отрезка составим системы для абсцисс, ординат и аппликат. Пользуясь методом сложения, решим эту систему:

Ответ: А(2; 0; 1), В(8;4; 9), С(1; 2; 3).

Пример 4. Дано: А(-2; 1; 2), B(-6; 3; -2), С  оси OZ; АС = ВС. Найти: координаты точки С.

Решение: По условию С  оси OZ, значит она имеет координаты С(0; 0; z) и АС = ВС. Составим уравнение, пользуясь формулой расстояния между двумя точками: 4 + 1 + (z 2)2 = 36 + 9 + (z + 2)2, 5 + z2 – 4z + 4 = 45 + z2 + 4z + 4, 8z = 40, z = … Ответ: (0; 0;5).

Пример 5. Дано: А(2; 1; 2), B(6; 3; 2), С (0; 0; 5); АС = ВС. Найти: SABC).

Решение: По формуле координат середины отрезка АВ найдем координаты точки М — середины:
M ((62)/2, (1 + 3)/2,(22)/2), M(4,2,0). AB2 = (6 + 2)2 + ( 31)2 + (2 + 2)2 = 16 + 4 + 16 = …, AB = ... СМ-высота равнобедренного ΔABC.
CM2 = (40)2 + (20)2 + (0 (5))2 = 16 + 4 + 25 = … , CM = 3 ,
SABC) = AB · CM : 2 = 6 · 3 : 2 = … . Ответ: 9.

2)Решить задачи ( по примерам):

  1. Дано: ΔАВС; А(1; 2; 3), B(1; 0; 4), С(3; 2; 1). AM - медиана.Найти: координаты вектора

  2. Дано: А(1; 5; 3), В(1; 3; 9), С(3; 2; 6).Доказать: ΔAВС - прямоугольный.

  3. Дано: ΔАВС, М, N, К - середины сторон соответственно ABBС, AС. М(3; 2; 4), 
    N(6; 4; 10), К(7; 2; 12).Найти: координаты вершин А, В, С.

  4. Дано: A(4; 5; 4), B(2; 3; 4); С  оси  OXAC = ВС. Найти: координаты точки С.

  5. Дано: А(4; 5; 4), B(2; 3; 4), С(1; 0; 0), АС = ВС. Найти: S(ΔABC).

3)Решить задачи :

  1. Дано: A (10, 4, –3), B (– 6, 2, 1). Найти координаты точки M – середины отрезка AB.

  2. Дано: A (5, 4, 7), B (10, 10, 0). Найти координаты вектора .

  3. Дано: {0, 5, 0}, {2, – 2, 1}. Найти длину векторов.

  4. Даны точки А (1,5; 1; – 2), B (2; 2; – 3); и C (2; 0; – 1). Найдите: периметр треугольника ABC.

  5. Дано: М(–4; 7; 0) N(0; –1; 2).Найти: расстояние от начала координат до середины отрезка MN.

  6. На каком расстоянии от плоскости (хОу) находится точка А(2; 3; 5).

  7. Дано: ΔАВС; М, N, К - середины сторон соответственно АВ, ВС, АС.

М(3; 2; 5),N(3,5; 1; 6), К(1,5; 1; 2).Найти: координаты А, В, С.

  1. На каком расстоянии от начала координат находится точка А(3; 4; 0).

  2. Найдите координаты середины отрезка, если его концы имеют координаты А(5; 3; 2), В(3; 1; 4).

  3. Найти длину вектора  если А(5; 3; 2), В(3; 1; 4).

  4. Записать координаты вектора , если .

  5. На каком расстоянии от плоскости (yOz) находится точка В(3; 2; 4).

  6. На каком расстоянии от начала координат находится точка В(3; 0; 4).

  7. Найдите координаты середины отрезка, если концы его имеют
    координаты А(
    3; 2; 4), В(1; 4; 2).

  8. Найти длину вектора  если А(3; 2; 4), В(1; 4; 2).

  9. Записать координаты вектора   если .

  10. Найдите координаты вектора  если А(5; 1; 3), В(2; 2; 4).

  11. Изобразить систему координат Оху: и построить точку А(1; 2; 4). Найти расстояние от этой точки до координатных плоскостей.

  12. Найдите координаты вектора , если С(6; 3; 2), D(2; 4; 5).

  13. Даны векторы и  . Найдите  

  14. Изобразить систему координат oxyz и построить точку В(2; 3; 4). Найти расстояние от этой точки до координатных плоскостей.

  15. Дан вектор   Найти координаты ,  если   и векторы и  сонаправлены.

  16. Вершины ΔАВС имеют координаты: А(1; 2; 3), В(1; 0; 4), С(3; 2; 1). Найдите координаты вектора , если AM - медиана ΔАВС.

  17. Дан вектор   Найти координаты ,  если   и векторы и   противоположно направлены.

  18. Даны точки А(1; 5; 3), В(1; 3; 9), С(3; 2; 6). Доказать, что ΔАВС - прямоугольный.

  19. Середины сторон ΔАВС имеют координаты: М(3; 2; 5), (3,5; 1; 6), К(1,5; 1; 2).
    Найдите координаты вершин ΔАВС.

  20. Даны точки А(2; 1; 2), В(6; 3; 2) на оси аппликат. Найти точку С, равноудаленную от
    точек А и В. Найти площадь ΔАВС.

  21. Середины сторон ΔАВС имеют координаты: М(3; 2; 4). N(6; 4; 10), К(7; 2; 12).
    Найдите координаты вершин ΔАВС.

  22. Даны точки А(4; 5; 4), В(2; 3; 4) на оси абсцисс. Найти точку С, равноудаленную от точек А и В. Найти площадь ΔABC

  23. Даны точки А(3; 1; 2) и В(1; 1;2). Найдите: а) координаты середины отрезка АВ;

  24. б) координаты и длину вектора  в) координаты точки С, если .

  25. Даны точки А(0; 4; 0), В(2; 0; 0), С(4; 0; 4) и D(2; 4; 4). Докажите, что ABCD - ромб.

  26. Даны точки А(0; 1; 2), В(√2 ; 1; 2), С(; 2; 1) и D(0; 2; 1). Докажите, что ABCD- квадрат.

  27. Даны точки А(2; 1; 8), В(1; 5; 0), С(8; 1; 4). Докажите, что ΔАВС - равнобедренный и найдите длину средней линии треугольника, соединяющей середины боковых сторон.

  28. Даны координаты трех вершин параллелограмма ABCD: А(6; 4; 0), В(6; 6; 2), С(10; 0; 4). Найдите координаты точки D и угол между векторами  и .

  29. Даны точки А(2; 5; 8) и В(6; 1; 0). Найдите: а) на оси ординат точку С, равноудаленную от точки А и В; б) площадь треугольника АВС.

  30. Дано: А(1; 2; 3), B(2; 1; 2), С(0; 1; 1), D  (OYZ), AD = BD = CD.Найти: координаты точки D.

  31. Дано: A(4; 4; 0), В(0; 0; 0), C(0; 3; 4), D(1; 4; 4). Доказать: ABCD - равнобедренная трапеция.hello_html_793497f7.jpg

  32. Дано: О(0; 0; 0), А(4; 0; 0), В(0; 6; 0), С(0; 0; 2). ΔAОВ - вписанный в окружностьW(D; r).Найти: а) координаты центра окружности D;
    б)
     r- радиус окружности.

  33. Дано: ΔАВС - прямоугольный; АС, ВС - катеты;AC = b = 9 ;BC = a = 12;
    CD = m = 4; CD  (ABC); М - середина гипотенузы АВ. Найти: DM.

Инструкционная карта

ПР № 26 «Вычисление выражений, содержащих степень».

Задание:

1)Перепишите и заполните пропуски:

Пример 1. Упростить выражение:


в) (– 4)3 = (– 4) · (– 4) · (– 4) = – …

Ответ: а) 0,25; б) 3; в) – 64.

Пример 2. Найдем значение выражения  а) 6 · 33, б) 0,5482.

Решение: а) 1)33 = 3 · 3 · 3 = …, 2) 6 · 27 = ..., б) 0,5 · 482 = 0,5 · 2304 = …

Ответ: а)162, б)1152.

Пример 3. Заполнить таблицы:

а)

64

1

0

0,81

2,25

196

х2







х2+3,5







х2+3,5: 64+3,5=…, 1+3,5=…, 0+3,5=…, 0,81+3,5=…, 2,25+3,5=…, 196+3,5=…

б)

64

0,027

1

0

27

729

0,1 х3







х3+10













х3+10: – 64 + 10 =…, – 0,027 + 10=…, – 1 + 10 =…, 0 + 10 = …, 27 + 10 = …, 729 + 10 = …

Пример 4. 71.7 · 7 - 0.9 = 71.7+( - 0.9) = 71.7 - 0.9 =  70.8, 133.8 / 13 -0.2 = 13(3.8 +0.2) = 13

(23)2 = 2 3·2 = 26 = …, (2 / 5)3 = (2 / 5) · (2 / 5) · (2 / 5) = 23 / 53= 8/125 = ...

Пример 5. Определить знак результата:

Решение:

Ответ:

Пример 6. Вычислить значение:

Решение:

Ответ:

Пример 7. Какие числа надо возвести в квадрат, чтобы получить:25,36,49,81,121,144,196,225,361,400,576,625,729,841,900.

Решение: 5,…,7,…,11,…,14, …, 15,…,20,…,25,…,29,…

Пример 8. Представить в виде степени:

Решение:

Ответ:

Пример 9. Упростить выражение:

Решение: Ответ:

Пример 10. Упростить:

Решение:


Ответ:
2)Решить задание ( по примерам):

  1. Упростить выражение: в) (– 5)3;

  2. Найдем значение выражения  а) 5 · 33, б) 0,5452.

  3. Заполнить таблицы:

а) б)


  1. Упростить выражение: 71.7 · 7 - 0.7 , 133.8 / 13 -1.2 , (25)2 , (1 / 5)3 .

  2. Определить знак результата:

  3. Вычислить значение:

  4. Какие числа надо возвести в квадрат, чтобы получить:16,64,100,8169,256,289,324,441,529,676,784,961,1225,2025,2500.

  5. Представить в виде степени:

  6. Упростить выражение:

  7. Упростить:

3)Решить задание :

х

6

1

0,2

0

1

8

х3







0,5 х3







х3- 10







  1. Заполнить таблицы: а) б)

х

5

2,5

0

0,3

1

12

х2







х2







х2- 4








  1. Даны числа 3; 1/3; 2; 1/2. Сколько получится, если:

а) возвести каждое из этих чисел в квадрат;

б) сначала удвоить каждое из данных чисел, затем полученный результат возвести в квадрат?

  1. Определить знак результата:

  2. Вычислить:

  3. Вычислить:

  1. Каким числом, положительным или отрицательным, является значение выражения:

(3)25; (6)8; (9)14; (1)13?

  1. Упростить:

  2. Упростить:

  3. Вычислить: .

  4. Упростить:


4)Решить задание : 1-в

  1. Представить произведение в виде степени: :

1)(7х)4 ; 2)74х ; 3) 7х4; 4) 7х.

  1. Представить в виде произведения степень (-3)4 х2 :

1) ; 2) ; 3) ; 4) -3х.

  1. Вычислить: 34.

  2. Вычислить: (0,6)2.

  3. Вычислить: )3.

  4. Вычислить: (-8)3.

  5. Вычислить: .

  6. Найти значение выражения: 10а3 при а= – 0,5.

  7. Найти значение выражения: 1–5х2 при х= – 4.

  8. Упростите выражения и выберите верную таблицу.

А=.

A

B

C

D

E

a10

x2

c12

x3y3



A

B

C

D

E

a7

x9

c12

x3y3



A

B

C

D

E

a7

x3

c12

x3y3



  1. Соотнесите значение выражения из нижней строки с числовым выражением из верхней строки:

А)

1) 9 , 2) , 3) 32 , 4) 729 , 5) 36.


A

B

C

D

E







  1. Вычислить: .

  2. Представить в виде степени произведение: 36а2b2.

  3. Представить в виде степени произведение: – 32а5b5.

  4. Замените * так, чтобы равенство было верным: .

  5. Замените * так, чтобы равенство было верным: .

  6. Замените * так, чтобы равенство было верным: .

  7. Найдите значение выражения: .

  8. Сравните значения выражений: (– 6,5)4 и (– 2,4)3.

  9. Сравните значения выражений: .

  10. Представьте в виде степени с основанием 2: 85.

  11. Представьте в виде степени с основанием 2: ((16)2 )3.

  12. Вычислить: . 24.Вычислить: .

  1. Упростить выражение: .

  2. Упростить выражение: .

  3. Упростить выражение: ).


2-в.

  1. Представить произведение в виде степени: :

1) (6у)3 ; 2)63у ; 3) 6х3; 4) 6у.

  1. Представить в виде произведения степень (-2)5 х3:

1) ; 2) ; 3) ; 4) -2х3.

  1. Вычислить: 52

  2. Вычислить: (0,5)3

  3. Вычислить: )5

  4. Вычислить: (-4)3

  5. Вычислить:

  6. Найти значение выражения: 0,5b3 при а= – 0,4

  7. Найти значение выражения: 7– 3х2 при х= – 5

  8. Упростите выражения и выберите верную таблицу.

А=

A

B

C

D

E

x15

y2

z15

m2n2



A

B

C

D

E

x15

y2

z8

mn2



A

B

C

D

E

x8

y4

z15

m2n2



  1. Соотнесите значение выражения из нижней строки с числовым выражением из верхней строки:

А)

1) 5 2) 3) 36 4) 1024 5) 27


A

B

C

D

E







  1. Вычислить: .

  2. Представить в виде степени произведение: 8а3b3.

  3. Представить в виде степени произведение: – 0,00001а5b5.

  4. Замените * так, чтобы равенство было верным: .

  5. Замените * так, чтобы равенство было верным: .

  6. Замените * так, чтобы равенство было верным: .

  7. Найдите значение выражения: .

  8. Сравните значения выражений: (-0,2)6 и (-0,2)10.

  9. Сравните значения выражений: .

  10. Представьте в виде степени с основанием 3: 272.

  11. Представьте в виде степени с основанием 3: ((-81)2 )3.

  12. Вычислить: .24.Вычислить: .

  1. Упростить выражение: .

  2. Упростить выражение: .

  3. Упростить выражение: ).


Инструкционная карта

ПР № 27 «Решение пропорций. Решение задач с помощью пропорций».

Задание:

1)Перепишите и заполните пропуски:
Пример 1. В городе Жуковском на площади Громова находится магазин Навигатор. Хозяин магазина настолько строг, что за опоздание вычитает из зарплаты по 70 рублей за один день. В одном отделе работали две девушки Юля и Наташа. Их зарплата зависела от числа рабочих дней.

Юля за 20 рабочих дней получила 4100 рублей, а Наташа за 21 день получить должна бы больше, но получила меньше, т.к. опаздывала 3 дня подряд. Узнайте, сколько денег получила Наташа?

Решение:

Рабочие дни

Зарплата (руб.)


Юля

20

4100

Наташа

21

х

Т.к. зависимость прямо пропорциональная, то составляем уравнение:


Значит 4305 рублей зарплата Наташи за 21 день без штрафов.

1) 4305 – 3×70 = …(руб.) - Зарплата Наташи

Ответ: 4095 рублей

Пример 2. У Изабеллы было двенадцать братьев. Жили они очень дружно, пока не позавидовала злая колдунья. Она превратила братьев в лебедей. Изабелле надо связать 12 рубашек из крапивы, чтобы расколдовать братьев. За 3 дня она свяжет 4 с половиной рубашки. За сколько дней Изабелла свяжет 12 рубашек?

Решение: Пусть х дней понадобится Изабелле, чтобы связать 12 рубашек.

Дни Рубашки

  1. 4,5

х 12

Т. к. зависимость прямо пропорциональная составляем пропорцию.

, , Ответ: за 8 дней.

Пример 3. В магазин привезли поровну яблок и груш. Яблоки разложили в 25 ящиков по 18 кг в каждом. а груши – в 30 ящиков. Сколько килограммов груш в каждом ящике?

Решение: Пусть х кг груш в каждом ящике.

25ящ. – 18 кг

30ящ.—х кг

;Ответ: 15кг.

Пример 4. Для нового дельфинария строят бассейн. Необходимо выложить пол и стены бассейна керамической плиткой. На складе имеется плитка двух видов: площадью 1,2 дм2 и площадью 3,8 дм2. Сколько потребуется плитки площадью 1, 2 дм2, если плитки площадью 3, 8 дм2 требуется
2400 упаковок?

Решение: 3, 8 дм² – 2400 упаковок
1,2 дм² – х упаковок

, , , Ответ: 7600 упаковок

Пример 5. Рабочие бригады, состоящей из 8 человек, могут выложить бассейн плиткой за 6 дней. Сколько человек в другой бригаде, если они могут выполнить эту работу на 2 дня быстрее?

(Производительность бригад одинакова)

Решение:

1) 6 – 2 = … (дня) время работы второй бригады

2) 8 человек – 6 дней

х человек – 4 дня

Ответ: 12 человек.


2)Решить задачи ( по примерам):

  1. Для перевозки арбузов с юга на оптовую базу города Жуковского приехали 24 машины грузоподъемностью 7,5 тонн. Сколько нужно машин грузоподъемностью 4,5 тонн, чтобы перевести тот же груз?

  2. Для приготовления 4 порций салата потребуется 50г майонеза. Сколько майонеза потребуется для приготовления 10 порций салата?

  3. Маленькое колесо повозки, имеющее окружность 2,4м, обернулось на некотором расстоянии 1250 раз. Сколько раз обернулось на этом расстоянии большое колесо, имеющее колесо, имеющее окружность 3м?

  4. В магазин привезли поровну яблок и груш. Яблоки разложили в 35 ящиков по 18 кг в каждом. а груши – в 45 ящиков. Сколько килограммов груш в каждом ящике?

  5. Рабочие бригады, состоящей из12 человек, могут выложить бассейн плиткой за 6 дней. Сколько человек в другой бригаде, если они могут выполнить эту работу на 2 дня быстрее?

3)Решить задачи :

  1. Решить пропорцию: а) , б) 2 : 3,4 = х : 17,

  2. Из 15 т руды получено 3 т меди. Сколько тонн меди получится из 20 т этой руды?

  3. Все сваренное варенье разложили в 60 баночек вместимостью 350 мл. Сколько для этого понадобилось бы баночек вместимостью 200 мл, 300 мл? Какой вместимостью понадобилось бы баночки, если их было 50?

  4. Из 5 ц молока получается 40 кг сыра. Сколько центнеров молока потребуется для изготовления 80 кг сыра, 160 кг сыра? Сколько килограммов сыра получится из 1 ц молока?

4)Решить задачи и получите ответ на вопрос.

Во вновь выстроенный и заполненный морской водой бассейн запустили дельфинов – обитателей дельфинария. Как называются виды дельфинов, обитающих в дельфинариях, вы узнаете верно выполнив следующие задания:1)

1 вариант:

2 вариант:

У , Е

Н , Ф

2)Л В начале 50-х годов 20 века в Черном море обитало 2,5 млн дельфинов (название которых зашифровано в таблице 2-го варианта), а ныне из-за загрязнения моря, их поголовье сократилось до 0,1 млн. На сколько процентов меньше стало их в Черном море? Теперь этот вид дельфинов отнесен к редким и включен в Красную книгу России и Украины.

3) А Один из самых быстрых обитателей моря – дельфин-белобочка. Он способен развивать скорость 50 км/ч. А самый быстрый из всех обитателей моря – свирепая косатка, которая тоже относится к семейству дельфинов. Она может плыть со скоростью 55 км/ч. Расстояние между двумя островами дельфин белобочка проплывает за 11 мин. За какое время проплывет это же расстояние косатка?

4)Серый дельфин, или грампус, - самый крупный в семействе дельфинов после косаток и гринд. Длина его тела до 4 м, вес до 0,5 т. плавают серые дельфины в Атлантическом и Тихом океанах, в Средиземном и Красном морях. Один из этих дельфинов по кличке Пелорус-Джек, резвясь и играя, сопровождал корабли как лоцман, указывая им путь в непогоду между двумя островами Новой Зеландии. За это новозеландский парламент даровал ему охранную грамоту, запрещающую убивать и обижать этого дельфина.

В каком году Пелорус-Джек начал сопровождать корабли вы узнаете, если количество букв «о» в слове «пропорция» увеличите в 1000 раз, а из результата вычтете 104 (это задание выполните устно).

У первого варианта под ответом к этому заданию зашифрована буква Б, а у второго варианта буква И.

5)Известно также, что «работал» этот дельфин лоцманом 20 лет. В каком году он ушел на «пенсию»? (Это тоже устное задание).

У первого варианта под ответом к этому заданию зашифрована буква Х, а у второго варианта буква А.

1 вариант. 2 вариант.


1896

10,5

96

0,5

1916

10







1916

10,5

10

96

1896

0,5

1916













Инструкционная карта

ПР № 28 «Вычисление квадратных корней. Решение задач на проценты».

Задание:

1)Перепишите и заполните пропуски:

Вычисление квадратных корней.

Пример 1. Найдем значение выражения:

Решение: а) Используем теорему о корне из произведения: 

б) Представим подкоренное выражение в виде произведения множителей, каждый из которых является квадратом целого числа. Применим также теорему о корне из произведения. Имеем:

 

в) В подкоренном выражении разложим разность квадратов чисел на множители и используем теорему о корне из произведения. Получаем:

 

Ответ:

Пример 2. Найдем значение выражения

Решение: а) По теореме о корне из дроби имеем:
б)Используя указанные теоремы, получим:

Ответ:

 Пример 3. Найдите значение выражения: .

Решение: а) Учтем свойства квадратного корня и формулу разности квадратов. Тогда получаем: 

б)

в)

г) В подкоренном выражении выделим полный квадрат суммы: 


Ответ: а) 5; б) 6; в) 46; г) 6.

Пример 4. Вычислить:

Решение:


Ответ:

Пример 5. . а)Найдите значение выражения
б) Упростите выражение

Решение: а) Используем формулы квадрата суммы и квадрата разности, выполним действия и получим:


б)

Ответ: а)60 ; б) 0.

Решение задач на проценты.

Пример 1. а) Найдите: 48% от 250. б) Найдите: число, 8% которого равны 12.

в) Сколько процентов составляет 15 от 60?

Решение: а) 48% = 0,48, 250 0,48 =…, б) 8% = 0,08, 12 : 0,08 =...,

в) 15 : 60 100% = 1500 : 60 = 50 : 2 = … %.

Ответ: а) 120,б) 150,в)25%.

Пример 2. Курящие люди сокращают себе жизнь на 15 %. Определите какова продолжительность жизни курильщиков, если средняя продолжительность жизни в России 56 лет.

Решение: 100% - 15% = … % , 85 % от 56, 85 % = 0, 85, 0,85 56 = … ( лет)

Ответ:47,6 лет.

Пример 3. Бронза является сплавом олова и меди. Сколько процентов сплава составляет медь в куске бронзы, состоящем из 6 кг олова и 34 кг меди?
Решение: 1) 6+ 34 = … (кг) масса всего сплава.
2) 34 : 40
100% = 3400 : 40 = …% сплава составляет медь.

Ответ: 85%.

Пример 4. Из хлопка-сырца получается 24% волокна. Сколько надо взять хлопка-сырца, чтобы получить 480кг волокна?
Решение: Запишем 24% десятичной дробью 0,24 и получим задачу о нахождении числа по известной ему части (дроби). 480 : 0,24= … кг = 2 т. Ответ: 2 т.

Пример 5. Сколько кг белых грибов надо собрать для получения 1 кг сушеных, если при обработке свежих грибов остается 50% их массы, а при сушке остается 10% массы обработанных грибов?
Решение: 1кг сушеных грибов – это 10% или 0, 01 часть обработанных, т.е. 1 кг : 0,1= 10 кг обработанных грибов, что составляет 50% или 0,5 собранных грибов, т.е. 10 : 0,5 = … кг
Ответ: 20 кг
Пример 6. а) Сколько кг соли в 10 кг соленой воды, если процентное содержание соли 15%.

Решение: 10 · 0,15 = … (кг) соли.

Ответ: 1,5 кг.
б) Сплав содержит 10 кг олова и 15 кг цинка. Каково процентное содержание олова и цинка в сплаве?
 
Решение: Процентное содержание вещества в сплаве - это часть, которую составляет вес данного вещества от веса всего сплава. 
1) 10 + 15 = … (кг) - сплав;
2) 10/25
·100% = 10 4 = …% - процентное содержание олова в сплаве;
3) 15/25
· 100% = 15 4 = …% - процентное содержание цинка в сплаве;
Ответ: 40%, 60%.

Пример 7. К 15 л 10%-ного раствора соли добавили 5%-ный раствор соли и получили 8%-ный раствор. Какое количество литров 5%-ного раствора добавили? 
Решение: Пусть добавили х л 5%-ного раствора соли. Тогда нового раствора стало (15 + х) л, в котором содержаться 0,08 (15 + х) л соли. В 15 л 10%-ного раствора содержится 15 0,1 = … (л) соли, в х л 5%-ного раствора содержится 0,05х (л) соли.
Составим уравнение: 1,5 + 0,05х = 0,08
(15 + х); 1,5 + 0,05х = 1,2 + 0,08х ; 0,03х = 0,3;
х = 0,3 : 0,03 = 30 : 3 = … л.
Ответ: добавили 10 л 5%-ного раствора.

Пример 8. Предприятие изготовило за квартал 500 насосов, из которых 60 % имели высшую категорию качества. Сколько насосов высшей категории качества изготовило предприятие?

Решение: Найдем 60 % от 500 (общее количество насосов). 60 % = 0,6,
500 · 0,6 = 50
6 = … насосов высшей категории качества.

Ответ: 300 насосов высшей категории качества.

Пример 9. Цена товара понизилась на 40%, затем еще на 25%. На сколько процентов понизилась цена товара по сравнению с первоначальной ценой?

Решение: Обозначим первоначальную цену товара через х. После первого понижения цена станет равной х — 0, 4х = 0,6x. Второе понижение цены составляет 25% от новой цены 0,6х, поэтому после второго понижения будем иметь цену 0,6х 0,25  0,6x = (0,6 – 015)x = ... х.

После двух понижений суммарное изменение цены составляет: х 0,45x = ( 1 0,45)х= …х.

Так как величина 0,55x; составляет 55% от величины x, то цена товара понизилась на 55%.

Ответ: 55%.

Пример 10. Взяли 120 г раствора, содержащего 80% соли, смешали с 480 г раствора, содержащего 20% соли. Получили новый раствор. Найти процентное содержание соли полученного раствора.

Решение:

I

120

80% = 0,8 от 120

0,8 120 = 8 12=

II

480

20% = 0,2 от 480

0,2 480 = 2 48 =…

III

600

 

96 + 96 = …

1) 192:600 100% = 19200 : 600 = 192 : 6 = … %,содержание соли в полученном растворе.

Ответ: 32%.

2)Решить задание ( по примерам):

Вычисление квадратных корней.

  1. Найдем значение выражения:

  2. Найдем значение выражения

  3. Найдите значение выражения: .

  4. Вычислить:

  5. а)Найдите значение выражения

б) Упростите выражение

Решение задач на проценты.

  1. а) Найдите: 58% от 250. б) Найдите: число, 10% которого равны 12.

в) Сколько процентов составляет 45 от 60?

  1. Курящие люди сокращают себе жизнь на 15 %. Определите какова продолжительность жизни курильщиков, если средняя продолжительность жизни в России 60 лет.

  2. Бронза является сплавом олова и меди. Сколько процентов сплава составляет медь в куске бронзы, состоящем из 16 кг олова и 34 кг меди?

  3. Из хлопка-сырца получается 24% волокна. Сколько надо взять хлопка-сырца, чтобы получить 360кг волокна?

  4. Сколько кг белых грибов надо собрать для получения 2 кг сушеных, если при обработке свежих грибов остается 50% их массы, а при сушке остается 10% массы обработанных грибов?

  5. а) Сколько кг соли в 20 кг соленой воды, если процентное содержание соли 15%. б) Сплав содержит 20 кг олова и 30 кг цинка. Каково процентное содержание олова и цинка в сплаве? 

  6. К 15 л 10%-ного раствора соли добавили 5%-ный раствор соли и получили 6%-ный раствор. Какое количество литров 5%-ного раствора добавили? 

  7. Предприятие изготовило за квартал 300 насосов, из которых 60 % имели высшую категорию качества. Сколько насосов высшей категории качества изготовило предприятие?

  8. Цена товара понизилась на 60%, затем еще на 25%. На сколько процентов понизилась цена товара по сравнению с первоначальной ценой?

  9. Взяли 120 г раствора, содержащего 70% соли, смешали с 480 г раствора, содержащего 20% соли. Получили новый раствор. Найти процентное содержание соли полученного раствора.

3)Решить задание : Вычисление квадратных корней.

  1. Упростите выражение: .


  1. Вычислите значение выражения

  2. Избавьтесь от иррациональности в знаменателе выражения а) ,б) .

  3. Сократите дробь а) ,б) в)

  4. Упростите выражение

  5. Сравните числовые выражения: а) и , б) и .

  6. Найдите значение выражения.

  7. Вычислите значение выражения

  8. Сравните числа и

  9. Найдите значение выражения

  10. Упростите выражение и найдите его значение при  и

  11. Известно, что  и .Найдите значение выражения .

Решение задач на проценты.

  1. Токарь вытачивал за час 40 деталей. Применив резец из более прочной стали, он стал вытачивать на 10 деталей в час больше. На сколько процентов повысилась производительность труда токаря?

  2. При плановом задании 60 автомобилей в день завод выпустил 66 автомобилей. На сколько процентов завод выполнил план?

  3. Найти число, зная, что 25% его равно 45% от 640.

  4. Товар стоил тысячу рублей. Продавец поднял цену на 10%, а через месяц снизил её на 10%.Сколько стал стоить товар?

  5. Собрали 100 кг грибов. Оказалось, что их влажность 99%. Когда грибы подсушили, влажность снизилась до 98%. Какой стала масса этих грибов после подсушивания?

  6. Матроскин продает молоко через магазин и хочет получать за   него 25 рублей за литр. Магазин удерживает 20% стоимости проданного товара. По какой цене будет продаваться молоко в магазине?

  7. Из сосуда, доверху наполненного 94% -м раствором кислоты, отлили 1,5 л жидкости и долили 1,5 л 70% -го раствора этой же кислоты. После этого в сосуде получился 86% раствор кислоты. Сколько л раствора вмещает сосуд?

  8. В цистерну налили 37,4 т бензина, после чего осталось незаполненным 6,5% вместимости цистерны. Сколько бензина нужно долить в цистерну для ее заполнения?

  9. В городе в настоящее время 48400 жителей. Известно, что население этого города увеличивается ежегодно на 10%. Сколько жителей было в городе два года назад?

  10. Влажность купленного арбуза составила 99%. В результате длительного хранения влажность снизилась до 98%. Как изменилась влажность арбуза?

  11. На сколько процентов число 200 меньше, чем число 250?

  12. Толщина протектора 7мм. За полгода стирается 10%. Сколько миллиметров протектора останется после года эксплуатации машины.

  13. Первое число составляет 50% от второго. Сколько процентов от первого составляет второе?

  14. При оплате услуг через платежный терминал взымается комиссия 5%. Терминал принимает суммы кратные 10 рублям. Аня хочет положить на счет своего мобильного телефона не меньше 300 рублей. Какую минимальную сумму она должна положить в приемное устройство данного терминала?

  15. На покупку планшета взяли кредит 20000 р на 1 год под 16 % годовых. Вычислите, сколько денег необходимо вернуть банку, какова ежемесячная сумма выплат?

  16. Мобильный телефон стоил 5000 рублей. Через некоторое время цену на эту модель снизили до 3000 рублей. На сколько процентов была снижена цена?


Инструкционная карта

ПР № 29 «Решение квадратных уравнений . Решение неравенств ».   

Задание:

1)Перепишите и заполните пропуски:
Пример 1. 5– =0, D = 49 – 40 = …, х1= (7+3):10=…, х2 = (7–3):10=…,

Ответ: 1; 0,4

Пример 2 . а2 –5а +4 =0, D = 25 – 16 = …, а1= (5+3):2=…, а2 = (5–3):2=…

Ответ: 4 и 1.

Пример 3. 2t2 + 3 t – 2= 0,

D = 9 – 4 2 (–2) = 9 + 16 = …, t1= (–3 + 5 )/ 4 = 2 : 4= …, t2= (–3 – 5) / 4 = – 8 : 4=…,

Ответ: 0,5 и (– 2).

Пример 4. 3 а2 – 5 а – 2 = 0, D = 25 – 4 3 (– 2) = 25 + 24 = …,

а1= (5 + 7) / 6= 12 : 6 = …, а2= (5 – 7) / 6= – 2 : 6 =…,

Ответ: 0,5 и (– 0,3) .

Пример 5. х – 3 2 , х 2+3, х …,

Ответ: х 5.

Пример 6. 0, =0,

D = 16 – 12 = …, х1= (4+2):2=…, х2 = (4–2):2=…,

+ – + х

1 3 х …, х …,

Ответ: х 1, х 3.

2)Решить задание ( по примерам):

  1. а) х2 + 4х –21 = 0, б) х2 + 4х + 3 = 0, в) х2 – 7х + 10 = 0,

  2. а) 5– =0, б) 2t2 + 3 t – 5= 0, в) 2t2 – 7 t + 3= 0,

  3. а) – х2 + 2х + 8 = 0, б) – х2 + 7х – 10 = 0, в) – х2 + 4х + 12 = 0,

  4. а) х – 5 2 , б) х – 3 2, в) х – 6 2,

  5. а) х2 + 4х –21 0, б) х2 + 4х + 3 0, в) х2 – 7х + 10 0,

  6. а) – х2 + 2х + 8 0, б) – х2 + 7х – 10 0, в) – х2 + 4х + 12 0,

3)Решить уравнения и неравенства :

  1. а) х · (23 – х) = 120; б) х · (х + 8) = 884;

  2. а) х 2 – 12х = 0, б) 7 х 2 – 1 = 0, в) х 2 – х = 0 . г) 3 х 2 – 120 х = 0.

  3. а) 6х2 – 4х + 32 = 0; б) х2 + 5х – 6 = 0.

  4. а) –5х2 – 4х + 28 = 0; б) 2х2 – 8х – 2=0.

  5. а) (х + 4)2 = 3х + 40; б) (х + 1)2 = (2х–1)2.

  6. а) – 2х2 + Зх + 9 < 0б) 2  – 4х + 1 < 0. 

  7. а) 2х2  – х + 4 >0;       б) – х2 + Зх – 8 > 0. 

  8. а) – 2x2  + 3x + 9 ≤ 0 , б) 4x2  – 4x + 1 ≤ 0, в) 2x2 – x + 4 > 0, г) – x2 + 3x – 8 ≥ 0.

  9. а) 15x2 + 5x = 0, б) x2 – 25 = 0, в) 3x2 – 15 = 0,

г) x2 – 7x + 10 = 0, д) x2 + 2x = 16x – 49, е) x · (2x – 3) = 4x – 3,

ж) (6x + 3) · (9 – x) = 0, з) x2 + 2 = x + 2, и) (10x – 4) · (3x + 2) = 0.

  1. а) x · (x +2) = 3, б) x · (x + 3) = 4, в) x · (x –5) = – 4, г) x · (x – 4) = – 3,

д) x · (2x + 1) = 3x + 4, е) x · (2x – 3) = 4x – 3, ж) = , з) = .

  1. а) 2x2 + 3x – 5 = 0, б) 5x2 – 7x + 2 = 0, в) 2x2 – 7x + 3 = 0, г) 5x2 – 3x – 2 = 0,

д) 6x2 + x – 1 = 0, е) 2x2 – 9x + 4 = 0, ж) – x2 + 2x + 8 = 0, з) – x2 + 7x – 10 = 0,

и) 9x2 – 6x + 1 = 0, к) 4x2 + 4x + 1 = 0, л) x2 + 2x + 3 = 0, м) x2 x + 1 = 0.

  1. а) = , б) = , в) 3 + = x, г) x – = 4, д) + = 4,

е) – = 1, ж) = 0, з) = 0.


  1. а) (x2 + 4x) · (x2 + 4x –17) + 60 = 0, б) (x2 – 3x)2 – 2 · (x2 – 3x) = 8.

  2. а) x2 – 169 > 0, б) x2 – 49 ≤ 0, в) x2 ≥ 0,36, г) x2 ≤ 0,81, д) x2 + x < 0, е) x2 – 3x ≤ 0.

  3. а) x + 2 < 5x – 2 · (x – 3), б) 3 · (1 – x) – (2 – x) ≤ 2, в) 4 · (x – 1) – (9x – 5) ≥ 3,

г) 3 · (x – 2) – 5 · (x + 3) > 27, д) 5x – 2 · (x – 4) ≥ 9x + 23, е) 6x – 3 · (x – 1) ≤ 2 + 5x.

  1. а) x2 – 1 ≤ 0, б) x2 – 9 ≥ 0, в) x2 – 144 > 0, г) x2 – 121 < 0, д) x2 – 25 ≤ 0, е) x2 – 36 ≥ 0.

  2. а) x2 + x – 6 ≤ 0, б) x2 + 4x – 5 ≤ 0, в) x2 + 3x + 2 < 0, г) x2 + 7x + 12 < 0,

д) 2x2 – 9x + 4 ≤ 0, е) 3x2 – 4x + 1 ≤ 0, ж) – x2x + 12 > 0, з) – x2 + 10x – 16 ≥ 0,

и) – x2 + 3x – 2 ≤ 0, к) – x2 + 6x – 8 < 0.

  1. а) – 4 < 2x – 1 < 2, б) – 6 ≤ 5x – 1 ≤ 5, в) 0 < 4x + 3 < 1, г) – 2 ≤ 6x + 7 ≤ 1,

д) – 1 < 2x + 2 < 0, е) – 1 ≤ 2x + 1 ≤ 1.

4)Решите задачи:

  1. Прямоугольный участок земли обнесён забором, длина которого 40 м. Площадь участка 96 м2. Найдите длины сторон участка.

  2. Одно из двух положительных чисел на 4 больше другого. Найдите эти числа, если известно, что их произведение равно 96.

  3. Периметр прямоугольника равен 34 см, а его диагональ равна 13 см. Найдите стороны прямоугольника.

  4. Гипотенуза прямоугольного треугольника равна 10 см, а один из катетов на 2 см больше другого. Найдите катеты треугольника.

  5. При каких значениях параметра р квадратное уравнение х2 –5х + р2 = 0: 
    а) имеет два различных корня; б) имеет один корень; в) не имеет -корней?

  6. Два последовательных чётных числа таковы, что квадрат большего из них в 9 раз больше меньшего числа. Найдите эти числа.

  7. Одну сторону квадрата уменьшили на 2 см, а другую – на 1 см и получили прямоугольник с площадью 6 см2. Найдите длину стороны квадрата.

  8. Один из катетов прямоугольного треугольника на 6 см меньше гипотенузы, а другой на 3 см больше первого. Найдите гипотенузу, если площадь треугольника равна 54 см2.

  9. Произведение двух последовательных натуральных нечетных чисел равно 575. Найдите эти числа.

  10. Одно из чисел на 12 больше другого, а их произведение равно 315. Найдите эти числа.

  11. Площадь прямоугольника, одна из сторон которого на 3 см больше другой, равна 54 см². Найти стороны и периметр прямоугольника.

  12. Известно, что один из катетов прямоугольного треугольника на 4 см. меньше другого, а гипотенуза этого прямоугольного треугольника равна 20 см. Найти длины катетов.

Инструкционная карта

ПР № 30   « Решение систем уравнений и неравенств. Вычисления по  формулам сокращенного умножения».
Задание:

1)Перепишите и заполните пропуски:
Пример 1.а) Решить систему уравнений .

Решение: Значения х и у можно рассматривать как корни квадратного уравнения

z ² 5 z + 4 = 0. Имеем: z ₁ =1, z  = 4. Оба уравнения системы симметричны относительно х и у , поэтому получаем две пары решений: если одно решение х  = 1, y  = 4, то второе будет, наоборот: х  = 4, y  = 1.

Ответ: (1;4),(4;1).

б) Решить систему уравнений .

Решение: Здесь коэффициенты при у по абсолютному значению равны между собой, но противоположны по знаку. Для получения уравнения с одним неизвестным уравнения системы почленно складываем:



_________________________________

5х = 20;

х = ...


Полученное значение х = 4 подставляем в какое-нибудь уравнение системы (например, в первое) и находим значение у : 2 · 4 + у = 11, y = 11 8, y = ...

Ответ: (4;3).

Пример 2. Решить систему уравнений .

Решение: .

Составляем уравнение: t ²41 t  400 = 0.

Откуда t  = 25, t  = 16. Значит х ² = 25, у ² = 16 и, наоборот, у ² = 25; x ² = 16.

1, 2 = ±…; x 3, 4 = ±…;

1, 2 = ±…; y 3, 4 = ±….

Учитывая, что ху > 0, получаем всего четыре решения данной системы.

= 5, у  = 4; х  = -5, y  = -4; x  = 4, y  = 5; x  = -4, y  = -5.

Ответ:(5;4),(),(4;5),().


Пример 3. Решить систему .


Решение: Пусть  , тогда  . Имеем:

z ; 15z234z 15 = 0, D = b2 – 4ас = – 4 · 15 · 15 = 1156 900 = …,



Значит, получаем две системы уравнений:






Решим 1 систему, для этого из 2 уравнения выразим х и подставим в 1 уравнение :

x = 0,6y,


Решим 2 систему, для этого из 2 уравнения выразим у и подставим в 1 уравнение :

у = 0,6х,

Откуда находим четыре решения: x  = 3, у  = 5; х  = 3, y  = 5; x  = 5, y  = 3;

= 5, y  = 3.

Ответ: (3;5),(),(5;3),().

Пример 4. Решить систему неравенств: а) , б) ,

Решение: а) ;


Ответ: (…; …].

б) .


Ответ: (–1,25; 0,25].

Пример 5. Решить систему неравенств:

Решение:



Решим 1 неравенство: , , D = 289 - 4 1 16 = …,

х1= (17+15):2=…, х2 = (17-15):2=…, х  = , х  = ...


hello_html_5bd89a69.pngх


Получаем, что .
Решим 2 неравенство:х  = , х  = ...

hello_html_5bd89a69.pngх


Получаем, что .

Общее решение системы будет являться пересечением полученных промежутков, то есть .

Ответ:.

Формулы сокращенного умножения(ф.с.у):


(a + b)2 = a2 + 2ab + b2 , (a - b)2 = a2  2ab + b2 , a2  b2 = (a b) (a+b), (a + b)3 = a3 + 3a2b + 3ab2 + b3,

(a b)3 = a3  3a2b + 3ab2  b3 , a3 + b3 = (a + b) (a2  ab + b2) , a3  b3 = (a b) (a2 + ab + b2) ,

a3  b3 = (a b) (a2 + ab + b2).


Пример 6. Применение формул сокращенного умножения при решении примеров.

1.а) (а – 3)2 = a2 6a + …, б) (2у + 5)2 = 4y2 + 20y + …,

в) (4а – b) ( 4а + b) = …a2 - b2 , г) с2 – 0,25 = (c0,5) (c + …), 

д) 2 (3х – 2у) (3х + 2у) = 2 (…x2 …y2) = 18x2 8y2 ,

2. Найдите значение выражения: (х + 4)2 – (х 2) (х + 2) при х = 0,125

x2 + 8x + 16 (x2 4) = x2 + 8x + 16 – x2 + 4 = …x + … = 8 0,125 + 20=…



2)Решить задание ( по примерам):


1.Решить систему уравнений :

а) ,б) , в) , г) .

2.Решить систему неравенств: а) , б) , в)


3. (ф.с.у):1) а) (а + 4)2 , б) (3у с)2 ,в) (2а – 5) ( 2а + 5) , г) (х2 + у) ( х2 – у) ,

д) 0,36 с2 ,е) 3(1 + 2ху) ( 1 2ху) , ж) (а + b)2 – (а b)2 , з) ( х2  у3)2 ;
2) Найдите значение выражения: (а
2 b)2 + 4 b( а – b) при а = 0,12 .

3)Решить задание :


1.Решить системы уравнений:

а) способом подстановки:

1) ; 2) ; 3) ;

б) способом сложения:

1) ; 2) ; 3) ; 4) ;

5) ; 6) ; 7) ; 8) ;

9) ; 10) ; 11) ; 12) ;

2. Решить системы неравенств:

а) ; б) ; в) ;

г) ; д) ; е) ;

ж) ; з) ; и) ; к) ;

3. Разложить на множители:

1) а) b2 c2 ; б) x2y2; в) a2 – 9 ; г) b2 – 16 ; д) x2 – 1;

2) а) 9x2 4 ; б) 4a2 – 25 ; в) 16 – 49y2 ; г) 16m2 – 9n2 ; д) 4x2 1 ;

4. Разложить на множители:

1) а) 0,25a2 1; б) 0,16 – 4b2 ; в) 0,09x2y2 ; г)1,44a2 – 1,21; д) a2 b2 ;

2) а) x2 y2z2 ; б) a2 b2 – 16 ; в) 9 – m2 n2 ; г) a2 b2 c2 – 36; д) y6 – 9 ;

5. Вычислить:

а) 372 – 132 ; б) 722 – 282 ; в) 42,42 42,32 ;

г) 6,82 – 3,22 ; д) 19 21 ; е) 99 101;

6. Сократить дробь:

а) ; б) ; в) ; г) ;


7. Выполните умножение:

а) (y – 3)(y + 3) ;

б) (x + y) (xy) ;

в) (1 + 3m) (1 – 3m) ;

г) (2xy) (2x + y) ;

д) (4x + 3y) (3y – 4x) ;

е) (x2 + 2) (x2 – 2) ;

8. Выполните возведение в квадрат:

а) (m n)2 ;

б) (y + 2)2 ;

в) (2x – 1)2 ;

г) (3a + 2)2 ;

д) (2x + 3y)2 ;

е) (4z3)2 ;

9. Представьте трехчлен в виде квадрата двучлена:

а) a2 + 2a + 1 ;

б) 4 – 20c + 25c2 ;

в) 81x2 – 18ax + a2 ;

г) 9n2 +12mn + 4m2;

10. Упростите выражение:

а) (x + 4)2 – 7x ; б) (x y)2 – x (y x) ;

в) 9m2 – (n – 3m)2 ; г) (a + b)2 – 2b (a b) ;

д) (a + 1) (a 1) + a (a 2) ;

е) (2x y) (y + 2x) + x (4 – 3x) ;

ж) 5c (c + 1) – (b – 3c) (b + 3c) ;

11. Представить в виде многочлена:

а) (a – 5) (a – 4) – 3a (2a – 3);

б) (х – 3)2 – 3х (х – 2);

в) 5 (a + 1)2 –10a .

12. Разложить на множители: а) 3x3 – 75х ; б) 3x2 + 6ax + 3a2 ; в) x3 + 8.

13. Упростить выражение (у2 + 6у)2 – у2 (6 + 5у) (6 – 5у) – у2 (12у – у2).

14. Разложить на множители : а) (а – с)2 – а2; б) x3 + у3 + 2xу (х + у).

15. Доказать, что если из квадрата нечетного числа вычесть 1, то результат будет делиться на 8.

16. Представить в виде многочлена:

а) (с – 9) (с – 3) – 6с (3с – 2);

б) (х – 10)2 – 4х (х – 5);

в) (a + 2)2 –12a .

17. Разложить на множители: а) 7x3 – 28х ; б) 5x2 – 10ax + 5a2 ; в) x3 – 8.

18. Упростить выражение (х2 2х)2 – (х – 2) (х + 2) (х2 – 4) – 4х (7х – х2).

19. Разложить на множители : а) (х + с)2 – х2; б) x3 – у3 – 5x2 + ху + у2).

20. Доказать, что произведение двух последовательных четных чисел делится на 8.


Инструкционная карта

ПР № 31 «Вычисление примеров на умножение и деление чисел».  

Задание:

1)Перепишите и заполните пропуски:

Пример 1. Найдем значение выражения 800-625 + 331 +87 – 119.

Решение: 800 – 625 = …,   175 + 331 = …,    506 + 87 = …,     593 – 119 = ...

Ответ: 474.

Пример 2. Найдем значение выражения 780 : 39 • 212 : 106 • 13.

Решение: Это выражение не содержит скобок, и в нем имеются действия только второй ступени, поэтому их следует выполнять по порядку слева направо:

780 : 39 = …,    20 • 212 = …,   4240 : 106 = …, 40 • 13 = ...

Ответ: 520.

Пример 3. Найдем значение выражения 5781 –28 • 75 : 25 + 156 : 12.

Решение: Это выражение не содержит скобок, и в нем есть действия первой и второй ступени. Поэтому вначале выполним действия второй ступени:

28 • 75 = …,     2100 : 25 = …,     156 : 12 = …, а потом действия первой ступени:

5781 – 84 = …,   5697 + 13 = ...

Ответ: 5710.

Пример 4. Найдем значение выражения 36 000 : (62+ 14 • 2) – 23 • 5.

Решение: Это выражение содержит скобки. Поэтому выполним сначала действия в скобках:

62 + 14 • 2 = 62 + 28 = ... Подставив это значение, получим: 36 000 : 90 – 23 • 5 = 400 – 115 = ... 

Ответ: 285.
Пример 5.  Запишите выражение по следующей программе вычислений:

1. Сложить числа 215 и 748. 
2. Вычесть из 591 число 318. 
3. Перемножить результаты команд 1 и 2. 
Найдите значение этого выражения.

Решение: 1)215 + 748 = … ,2)591 – 318 = …,3)963 273 = …

Ответ: 262899.

2)Решить задание ( по примерам):

  1. Найдите значение выражения:

а) 48 – 29 + 37 – 19; 
б) 156 + 228 – 193 – 66; 
в) 39 • 45 : 65 • 2; 
г) 1024 : 128 • 15 : 10; 
д) 245 : 7 – 224 : 16 + 35 • 11; 
е) 322 : 23 • 70 – 161 • 9 : 69; 

  1. а) 315 : (162 + 12 • 24 - 11 • 39) + 558 : 31; 
    б) (24 • 7 - 377 : 29) • (2378 : 58 – 38); 
    в) (120 + 16 • 7) • 240 : (300 – 5 • 44); 
    г) (372 + 118 • 6) : (38 • 35 – 34 • 37) - 12; 
    д) 3124 : (3 • 504 – 4 • 307) + 10 403 : 101; 
    е) 15 + (12 322 : (24 + 37) – 12 • 15) : (35 • 2 – 59).

  2. Измените порядок действий на основании свойств сложения, вычитания и умножения для удобства вычислений:

а) 348 + 54 + 46;                      г) 54 • 2 • 50; 
б) 543 + 89 – 43;                       д) 34 • 8 + 66 • 8; 
в) 427 – 33 – 67;                        е) 135 • 12 – 35 • 12.

  1. Выполните действия по схеме .

hello_html_m7f78617e.jpg

  1. Найдите частное:

а) 1 989 680 : 187;                            в) 9 018 009 : 1001; 
б) 572 163 : 709;                               г) 533 368 000 : 83 600.

3)Решить задание :

Вычислить:

  1. а) (– 2,35 – 4,65) · 5 : (16,9 – 2,9),
    б) (7,63 + (– 5,13)) · 0,4 : (3,17 + 6,83),

  2. а) 30,3 · (124,9 – (48,96 : 6,8 + 36,04) : 9,2),

б) 73, 2 · 48, 3 – 37,4 · (166,02 + 219,38) : 1,64,

  1. а) 3,44 : 0,4 + 24,56 , б) 684 · 245 – 675 · 246,

  2. а) (93 · 7 + 141) : 72 , б) 7091 + 9663 – (243916 + 75446) : 527 : 3,

в) (15,964 · 5,2 – 12) · 0,1 , г) (96,6 + 98,6) : 6,4 · 1,2 – 0,2,

  1. а) ((27,12+ 43,08) · 0,007 – 0,0314) · 100,
    б) 1,53 · 54 – 0,42 · (512 – 491,2) + 1,116,

в) (867000 : 2125 – 396,4) · 2,15,

  1. а) 51,6 + (70,2 – 4,4 · (73,73 : 7,3)) · 1,6,
    б) 18,305 : 0,7 – 0,0368 : 0,4 + 0,492 : 1,2,

в) (0,6739 + 1,4261) · 557, 55 : (16,7 · 2,9 – 42,13),

г) 702,3 – (59 – 389,64 : 6,8) · (59,3 – 5,64 : 9,4),

  1. а) 316219 – (27090 : 43 +16422 : 119), б) 565,3 – 465,3 : ((1,25 + 5,8) · (55,8 – 49,2)),

в) 74 : 100 – 0,4 : 10 + 17,8 : 1000, г) 0,35 · 10 + 0,0237 · 100 – 0,00087 · 1000,

  1. а) 0,7 : 0,1 + 0,0474 : 0,01 – 0,00174 : 0,001, б) 12,3 + 7,7 · 187,2 : 4,5 : 6,4 – 3,4,

в) 10,1 + 9,9 · 107,1 : 3,5 · 6,8 – 4,85, г) 37 · 0,01 – 0,2 · 0,1 + 8,9 · 0,001.

  1. Найди значения выражений:
    а) (18370+23679):7, 156-96:(12:4):2,
    б) (800035
    784942)∙6,

в) 98560:7, 83216:4 , 8656:4, 91620:4,
г) 73170:9, 3726:9 , 91728:9 , 705355:5.

  1. Найди значения выражений:
    а) (10283+16789):9, 5∙(125+75):20+80,
    б) (200496
    134597)∙2,

в) 54663:7 , 80395:5 , 6543:9 , 860073:3,
г) 1836:4, 7542:9 , 3906:6 , 9150:3,

д)795 ·504 248.952:492,

  1. Реши примеры на деление:

    114595 : 215 =

    200064 : 384 =

    404758 : 922 =

    5370 : 358 =

    396204 : 548 =

    263082 : 978 =

    181116 : 387 =

    118956 : 276 =

    115419 : 487 =

    140070 : 435 =

    223925 : 689 =

    420210 : 435 =

  2. а)1098 + (1453 – 564) · 176  + 195 539 – 352 004,

б)30257 · 8 + 72804 · 5 5897 · 63504 : 8.


Инструкционная карта

ПР № 32 «Преобразование выражений ,содержащих корни и степени».  

Задание:

1)Перепишите и заполните пропуски:

Пример 1. Вычислите: . Решение: .

Ответ: 1.

Пример 2. Вычислите: .

Решение: .

Ответ: 0,2.

Пример 3. Упростите выражение: .

Решение: .

Ответ: 3.

Пример 4. Вычислите: а) б)

в) г)д)

е) ж)

з)

Ответ: а) 6, б) hello_html_15da1ba3.gif8, в) 3,5; г) 40, д) 2, е) 10, ж) 1,25; з) 250.

Пример 5. Выполнить действия: Решение: Ответ:

Пример 6. Решите уравнения:

Решение: Ответ: а) х = 4, б) х = 2.

Пример 7. а) Вынести множитель из-под знака корня: ,

внесите множитель под знак корня: .

Ответ: а) б) .

2)Решить задание ( по примерам):

  1. Вычислите: .

  2. Вычислите: .

  3. Упростите выражение: .

  4. Вычислите:

а) б) в) г)д)

е) ж) з)

  1. Выполнить действия:

  2. Решите уравнения:

  3. а) Вынести множитель из-под знака корня: ,

внесите множитель под знак корня: .


3)Решить задание :

  1. Вычислите: а) ,б) ,в) ,г) ,д) ,

е) ,ж) , з) .

  1. Решите уравнения: а) , б) , в) , г)

  2. Упростите выражение: .

  3. Вычислите: а) б) в)

  4. а) вынести множитель из-под знака корня:

б) внесите множитель под знак корня:

  1. Вычислите а) ,б) ,в) г) ,д) .

  2. Упростите выражение


  1. Найдите значения выражения при у = 16.

  2. Найдите значение выражения:

  3. Сократите дробь: .

  4. Найдите значения выражения при а = 4, b = 5.

  5. Найдите значение выражения при р = 49.

  6. Упростите выражение .

  7. Упростите выражение .

  8. Вычислите

  9. Упростите выражение .

  10. Найдите значение выражения:

  11. Найдите значение выражения: .

  12. Упростите выражение .

  13. Упростите выражение


Инструкционная карта

ПР № 33 « Вычисление логарифмов ».  

Задание:

1)Перепишите и заполните пропуски:
 
Пример 1. log3 9 = 2, так как 32 = 9, log5 25 = 2, так как 52 = 25, log3 81 = 4, так как 34 = 81,

Ответ: 2,2,4.

Пример 2. Вычислите : а) log2 16, б) log3 3, в) , г) , д) log2 2 log3 81, е) log12 2 + log12 72, ж) log5 75 – log5 3.
Решение: а) log2 16 = 4, б) log3 3 = …, в) = 16, г) = = …,

д) log2 2 log3 81= 1· 4 = …, е) log12 2 + log12 72 = log12 (2 ·72) = log12 144 = …,

ж) log5 75 – log5 3= log5 (75:3) = log5 25 = …

Ответ: а) 4, б) 1, в) 16, г) 8, д) 4, е) 2, ж) 2.

Пример 3. Найдите х, если logx 36 = 2 и log2 x = - 2.

Решение: logx 36 = 2, х2 = 36, х = log2 x = - 2, х = 2 -2 = 1 / 4 = …

Ответ: 0,25
Пример 4. Вычислите: а) , б) , в) .

Решение: а) = - log2 16=…, б) = 5 · = 5 · 3 = … ,

в) = = 17 = 1296 – 17 = …

Ответ: - 4, 15, 1279.

Пример 5. Упростите выражение :

а) ;

б)

;

в) ;


2)Решить задание ( по примерам):


  1. Вычислите а) log3 27, б) log4 1,в) log1/2 4,

  2. Вычислите а) log2 32, б) log3 9, в) , г) , д) log3 3 log2 8, е) lg 5 + lg 2,

ж) log3 15 – log3 5.

  1. Найдите х, если log2 4 = x и log6 x = 2.

  2. Вычислить а) , б) , в) .

  3. а) , б) , в) ,

г) , д) ,

е) .

3)Решить задание :

1. Вычислите (по свойству степени):

1) , 2) , 3) , 4) , 5) ,

6) , 7) , 8) ,9) , 10) , 11) ,

12) , 13) , 14) , 15) , 16) .

2. Вычислите (по основному лог. тождеству):

1) , 2) , 3) , 4) , 5) ,

6) ; 7) , 8) , 9) , 10) ,

11) , 12) , 13) , 14) ,15) ,

16) , 17) , 18) , 19) , 20) .

3. Вычислите (по свойствам логарифмов):

1) + , 2) + , 3) , 4) ,
5) , 6) , 7) + , 8) ,

9) 3, 10) + ,

4. Вычислите: а),б) , в) ,

г), д).

5. Упростите выражение :

а) ;б) ,в);

6.Вычислить логарифмы: log381,ln e, lg1000, log7343,ln7,29, lg0,001.

7.Вычислить логарифмы: log432 + log42, log552, log2(8 128), log654 + log64, log3108 – log34.

8.Вычислить логарифмы:



9.Упростите выражение 
10. Найдите значение выражения 




Инструкционная карта

ПР № 34 «Перевод из одной меры угла в другую. Вычисление синуса, косинуса, тангенса угла».  

Задание:

1)Перепишите и заполните пропуски:

Пример 1. Найти радианную меру угла, выраженного в градусах: а) α = 40°, б) α = 120°, в) α = 150°.

Решение:

Ответ:

Пример 2. Найти градусную меру угла, выраженного в радианах:

Решение:

Ответ:

Пример 3. Вычислите:

Решение:



Ответ:

Пример 4. Вычислите:

Решение:



Ответ:

Пример 5.

Решение:



Ответ:

2)Решить задание ( по примерам):

  1. Найти радианную меру угла, выраженного в градусах: а) α = 75°, б) α = 32°, в) α = 140°.

  2. Найти градусную меру угла, выраженного в радианах:

  3. Вычислите:

  4. Вычислите:

3)Решить задание :

  1. Вычислите:

  2. Радианная мера двух углов треугольника равна   и  . Найдите градусную меру каждого из углов треугольника. 

  3. Выразите в градусной мере величину угла: .

  4. Выразите величину угла в радианах: .

  5. Найдите знак произведения, используя правило знаков по четвертям:

  6. Вычислите значение выражения:

  7. Вычислите:


  1. Вычислите:

  2. Найдите знак произведения:

  1. Вычислить значения и ,если α =120°.

  2. Вычислите значение тригонометрических функций:

sin π/3;cos 7π/6;tg π;sin π/4;tg 2π/3;ctg π/2;sin 3π/2;cos 5π/4.

  1. Найдите радианную меру углов треугольника, если их величины относятся как 2:3:4.

  2. Может ли косинус быть равным:  

  3. Может ли синус быть равным:

  4. Вычислите:

  5. Вычислите :


  1. Известно, что Вычислите значение выражения:

  2. Известно, что .

Вычислите значение выражения:


  1. Вычислите : .

  2. Известно, что . Вычислите: .

  3. Известно, что Вычислите:

  4. Найдите значение выражения: 5sin²3х – 6,если cos²3х = 0,6.



Инструкционная карта

ПР № 35 «Вычисление значения тригонометрических функций по формулам».  

Задание:

1)Перепишите и заполните пропуски:

Пример 1.Вычислить : а) cos 18° cos 12° sin 18° sin 12°; б) cos 107° cos 17°sin 107° sin 17°;

в) sin 17° cos 13° sin 13° cos 17°; г) sin 43° cos 13° sin 13° cos 43°;

д) , е) .

Решение: а) cos 18° cos 12° sin 18° sin 12° = cos(18°12°) = cos 30° = …,

б) cos 107° cos 17° sin 107° sin 17° = cos(107°17°) = cos 90° = …,

в) sin 17° cos 13° sin 13° cos 17° = sin(17°13°) = sin 30° = …,

г) sin 43° cos 13° sin 13° cos 43° = sin(43°13°) = sin 30° = …,

д) = tg (9°51°) = tg 60° = …, е) = tg (65°20°) = tg 45° = … .

Ответ: а); б) 0; в) 0,5; г) 0,5; д) ; е) 1.

Пример 2.Вычислить : а) cos π /7 cos /21 sin π/ 7sin /21;

б) sin π /3 cos π /12  cos π /3sin π /12; в) .

Решение: а) cos π /7 cos /21 sin π /7sin /21 = cos /7 4π /21) = cos (3π /21 4π /21) =

= cos /21 = cos π /3 = …,

б) sin π /3 cosπ /12 cos π /3 sin π /12 = sin /3 π /12) = sin (4π /12π /12) = sin /12 =

= sin π /4 = …,

в) = tg (π /7 4π /21) = tg π /3 = …

Ответ: а) 0,5; б) /2; в).

Пример 3. Упростить: а) cos α cos 3α sinα sin3α; б) sin 2α cos α cos 2α sin α;

в) sin α cos 3α cos α sin 3α; г) .

Решение: а) cos α cos 3α sinα sin3α = cos (α 3α) = cosα;

б) sin 2α cos α cos 2α sin α = sin (2α α) = sin α;

в) sin α cos 3α cos α sin 3α = sin (αα) = sinα; г) = tg (x 3x) = tgx.

Ответ: а) cos 4α; б) sin α; в) sin 4α; г) tg 4x.

Пример 4. Упростить : а) cos α cos β sin α sin β, если α = 42 °, β = 18 °;

б) cos(x y) cos(x + y) + sin(x y) sin(x + y).

Решение: а) cos α cos β sin α sin β = cos (α β) = cos (42 ° 18 °) = cos 60 ° = …,

б) cos(x y) cos(x + y) + sin(x y) sin(x + y) = cos ((x  y) – (x + y)) = cos (–2y) = cos 2y.

Ответ: а) 0,5; б) cos 2y .

Пример 5. Упростить выражение:

Решение: Ответ: 1.

Пример 6. Вычислите: cos630°– sin1470°– сtg1125°.

Решение: cos630°– sin1470°– сtg1125° = cos(360° + 270°)– sin(4360° + 30°)сtg(3360 ° + 45°) =

= cos270°– sin30°– сtg45° = 0 – 0,5 – 1= … Ответ: – 1,5.

2)Решить задание ( по примерам):

  1. Вычислить : а) cos 38° cos 22° sin 38° sin 22°; б) cos 55° cos 10°sin 55° sin 10°;

в) sin 47° cos 13° sin 13° cos 47°; г) sin 103° cos 13° sin 13° cos 103°;

д) , е) .

  1. Вычислить : а) cos π /5 cos π /20 sin π/ 5sin π /20;

б) sin π /4 cos π /12  cos π /4sin π /12; в) .

  1. Упростить: а) cos 2α cos 6α – sin 2α sin 6α; б) sin 3α cos α cos 3α sin α;

в) sin 2α cos 3α cos 2α sin 3α; г) .

  1. Упростить : а) cos α cos β sin α sin β, если α = 42 °, β = 48 °;

б) cos(2x y) cos(2x + 3y) + sin(2x y) sin(2x + 3y).

  1. Упростить выражение:

  2. Вычислите: cos450°– sin750°сtg765°.

3)Решить задание :

  1. Упростите выражение: sin(3π/2 – αcos(π/2 + α) + sin(2 π – α) + cos(3π/2 + α) + cosα ·sinα.

  2. Найдите cosß, если tgß = 7/24 и ß є(π; 3π/2).

  3. Найдите значение выражения: 2sin²2х – 9cos²2х, если cos2х = – 0,9.

  4. Вычислите:3ctg60º· (sin310ºcos70º sin70ºcos310º).

  5. Найдите значение выражения:5 cos(3π/2 + α) , если α = 7π/6.

  6. Найдите значение выражения: 4 + 5tg²х • cos²х, если sinх = 0,4.

  7. Найдите значение выражения:7 cos(π + α) – sin(3π/2 + α), если cosα = 0,6.

  8. Упростить выражение 4⋅(tg(π t) + ctg(π t) + ctg(3π/2 t))ctg(π t).

  9. Упростите выражение: .

  10. Вычислите:

  11. Докажите тождество: .

  1. Упростите выражение: .

  2. Вычислите

  1. Найди значение выражения  sin1050° + cos4620° + tg1035°.

  2. Вычислите:

  3. Упростите выражение  

  4. Вычислите:

  5. Упростите выражение:

  6. Вычислите: а) sin810°cos900o + tg585octg l845o + cos l35osin405°;
    б)
    cosl05°sinl95° + sin(-135°);

  7. Найдите значение выражения sin (х + у), если sin х= 9/41; cos у =40/41; х - угол II четверти.

  8. Найдите , если  и.

  9. Найдите значение выражения, если .

  10. Упростить выражение: .

Инструкционная карта

ПР № 36 «Вычисление значения тригонометрических функций двойного и половинного угла».  

Задание:

1)Перепишите и заполните пропуски:

Пример 1. Вычислить

Решение: Ответ:

Пример 2. Вычислить

Решение: Ответ:

Пример 3. Вычислить sin2α, если sinαcosα =

Решение: Возведем обе части равенства в квадрат: (sinαcosα)2 = ,

sin2α – 2sinαcosα + cos2α = , 2sinαcosα = – 1, 2sinαcosα = , sin2α = …

Ответ: .

Пример 4. Вычислить sin2α, если sinα = 0,6,

Решение: sin2α = 2sinα cosα . Т.к. ,то cosα < 0,

cos α =

sin2α = 2() () = ...

Ответ: 0,96.

Пример 6. Вычислить sinα/2, cosα/2, tgα/2, ctgα/2, если cosα = 0,8,

Решение: cos2 α/2 = (1 + cosα) : 2 = 1,8 : 2 = 0,9, cosα/2 = .

sin2 α/2 = (1 cosα) : 2 = 0,2 : 2 = 0,1, sinα/2 = .

tgα/2 = sinα/2 : cosα/2 = 0,33 : 0,95 = 33/95, ctgα/2 = cosα/2 : sinα/2 = 0,95 : 0,33 = 95/33.

Ответ: 0,33; 0,95;33/95; 95/33.

Пример 5. Пусть Найдем sin2, cos2, tg2.

Решение:


Ответ:

2)Решить задание ( по примерам):

  1. Вычислить

  2. Вычислить

  3. Вычислить sin2α, если sinαcosα = 1/4.

  4. Вычислить cos 2α, если sinα = 0,8,

  5. Вычислить sinα/2, cosα/2, tgα/2, ctgα/2, если sinα = 0,8,

  6. Пусть Найдем sin2, cos2, tg2.

3)Решить задание :

  1. Упростите выражение:


  1. Вычислить

  2. Дано: cos х =-12/13; 180 º < х < 270 º. Найти: cos х/2,tg x/2.

  3. Упростите выражение 

  4. Упростите выражение 

  5. Найдите ctg 2α, если

  6. Найти значение выражения: 2sin150 cos150.

  7. Найти значение выражения: cos2150 sin2150).

  8. Вычислить: sin330º  и ctg315º.

  9. Упростите выражение

  10. Найти значение выражения:

  11. Найдите значение выражения

  12. Найдите sin 2α, cos 2α,tg2α,  если и  .

  13. Найдите 24cos2α, если sinα = - 0,2 .

  14. Найдите tgα/2 , если .

  15. Найдите – 16cos2α , если sinα = – 0,6.

  16. Найдите 22cos2α  , если cosα = – 0,8.

  17. Найдите , если sin2α = 0,6.

  18. Найдите , если cos2α = 0,8.

  19. Упростите выражение

  20. Упростите выражение: сtg²х · sin²х cos2х.

  21. Найдите sin 2, если 3sin + 3cos = 1.

  22. Найдите cos 2, если 

  23. Вычислите без помощи таблиц:1) sin 75°; 2) cos 75°; 3)tg75°; 4) ctg 75°.

  24. Вычислите без помощи таблиц и калькулятора: 1) sin 15°; 2) tg22,5°.

  25. Упростите выражение:

Инструкционная карта

ПР № 37 «Решение уравнений cosx = a, sinx = a, tg x = a».  

Задание:

1)Перепишите и заполните пропуски:

Пример 1.Решите уравнение sin4xcos2x = 0.

Решение: sin4x – cos2x = 0 , 2sin2x cos2x – cos2 x = 0 , сos2x(2sin2x – 1)=0 ,

сos2x=0 или sin2x=1/2 .

2x = π/2 + π k, k , 2x = (1) n π/6 + π n, n Z .

х1= π/…+ π k/2 , k Z, x2 = (1)n π/…+ π n/2 , n Z .

Ответ: х1= π/4+ π k/2 , k Z; x2 = (1)n π/12+ π n/2 , n Z .

Пример 2. Решите уравнение (2 sin x – 1) (tg x) = 0.

Решение: ( 2 sin x – 1) hello_html_m61765ba1.gif (tg x) = 0,

2 sin x – 1= 0 или tg x = 0,

sin x = 1/2 tg x = ,

х1= (–1) n π/… + π n, n Z , х2 = π/… + π k, k .

Ответ: х1= (–1) n π/6 + π n, n Z , х2 = π/3 + π k, k .

Пример 3. Решите уравнение ( ctg x – 1) hello_html_m61765ba1.gif (2sin + 1) = 0.

Решение: ( ctg x – 1) hello_html_m61765ba1.gif (2sin + 1) = 0,

ctg x – 1 = 0 или 2sin + 1 = 0,

ctg x = 1 sin = – 1/2, х/2 = (–1) n +1 π/6 + π n, n Z,

х1 = π/… + π k, k , х2 = (–1) n +1 π/… + 2π n, n Z.

Ответ: х1 = π/4 + π k, k , х2 = (–1) n +1 π/3 + 2π n, n Z.

Пример 4. Решите уравнение . Решение: , cos (4x 3x) =, cos x =, x = 2πn, nZ. Ответ: x = 2πn, nZ.

Пример 5. Решите уравнение 2cos( х + π/3) = .

Решение: 2cos( х + π/3) = , cos( х + π/3) = –, х + π/3 = ± 5π/6+2πn, nZ, x = – π/3 ± 5π/6 + 2πn, nZ. x1 = – π/3 + 5π/6+2πn, nZ, x1 = π/… +2πn, nZ,

x2 = π/3 – 5π/6+2πn, nZ, x2 = 7π/… +2πn, nZ.

Ответ: x1 = π/2 +2πn, nZ, x2 = –7π/6 +2πn, nZ.

Пример 6. Решите уравнение sin( 2х + π/2) = 0.

Решение: sin( 2х + π/2) = 0, 2х + π/2 = πn, nZ,2х = – + πn, nZ,х = = – , nZ.

Ответ: х = = – , nZ.

Пример 7. Решите уравнение a) arccos б)arcsin (5x+2) = 0.

Решение: a) arccos cos(arccos ,

б) sin(arcsin (5x+2)) = sin0, 5x + 2 = 0, 5x = – 2,x = …

Ответ: a) 2,5, б) – 0,4.

Пример 8. Решите уравнение arctg (4x – 1) =

Решение: tg(arctg (4x – 1)) = tg 4x – 1 = 1, 4x = 2, x = … Ответ: 0,5.

2)Решить задание ( по примерам):

  1. Решите уравнение sin6xco3x = 0.

  2. Решите уравнение (2 sin x ) hello_html_m61765ba1.gif (tg x – ) = 0.

  3. Решите уравнение (ctg x ) hello_html_m61765ba1.gif (2sin +) = 0.

  4. Решите уравнение cos 4xcos3x + sin4xsin3x =

  5. Решите уравнение 2cos(х + π/4) =

  6. Решите уравнение sin( 2х + π/3) = 0.

  7. Решите уравнение a) arccos б)arcsin (5x + 4) = 0.

  8. Решите уравнение arctg (3x – 1) =

3)Решить задание :

  1. Решите уравнения:

  2. Решите уравнения:

  3. Решите уравнения:

  4. Решите уравнения:

  5. Решите уравнения:

  6. Решите уравнения:

  7. Решите уравнения:

  8. Решите уравнения:

  9. Решите уравнения:

  10. Решите уравнения:



Инструкционная карта

ПР № 38 «Решение тригонометрических уравнений заменой переменной».  

Задание:

1)Перепишите и заполните пропуски:

Пример 1. a)Решите уравнение sin2 x + 3sin x – 4 = 0.

Решение: sin2 x + 5sin x – 6 = 0.

Данное уравнение соответствует (1) таблицы, поэтому делаем замену  sin x = t, t ,

получаем квадратное уравнение: t2 + 3t – 4 = 0, D = 32 41 (4 ) = 9 + 16 = …,

t 1= ( 3 + 5) : 2 = 2 : 2 = …, t 2 = ( 3 5) : 2 = 8 : 2 = ...

находим корни  t1 = 1,t2 = – 4,

замечаем, что t2 = – 4 посторонний корень, поскольку t  ,

делаем обратную замену, т.е. решаем уравнение sin x = 1 , у которого корнями будут числа

x = π/2 +2πn, nZ .

Ответ: x = π/2 +2πn, nZ.

б) Решите уравнение tg2 x 4tg x + 3 = 0.

Решение: Данное уравнение соответствует (5) таблицы, поэтому делаем замену  tg x = t,

получаем квадратное уравнение: t2 – 4t + 3 = 0, D = (4)2 41 3= 16 12 = …,

t 1= (4 + 2) : 2 = 6 : 2 = …, t 2 = (4 2) : 2 = 2 : 2 = ... ,

находим корни  t1 = 3,t2 = 1, делаем обратную замену: tg x = 3, x 1= arctg 3 + πk, kZ или

tg x = 1, x2 = π/4 + πn, nZ  .

Ответ: x 1= arctg 2 + πk, kZ, x2 = π/4 + πn, nZ  .

Пример 2. Решите уравнение 2sin2 x + 3cos x – 3 = 0.

Решение: 2sin2 x + 3cos x – 3 = 0.

Данное уравнение соответствует (3) таблицы, поэтому cделаем замену cos x = t, t . Из основного тригонометрического тождества следует, что sin2x = 1 –cos2x, sin2x = 1 – t2 ,  подставим в уравнение : 2 (1 – t2) + 3 t – 3 = 0,  2 – 2t2 + 3 t – 3 = 0,

2t2 + 3 t – 1 = 0,получим квадратное уравнение: 2t2 – 3t + 1 = 0 , находим корни:

D = (3)2 42 1 = 9 – 8 = …, t1= (3 + 1) : 4 = 4 : 4 = …, t2 = (3 1) : 4 = .

делаем обратную замену: cos x = …, x 1= 2πn, nZ или cos x = 1 / 2, x2 =

Ответ: x 1= 2πn, nZ  , x2 =  .

Пример 3. Решите уравнение cos 2x = 4cos x – 1.

Решение: cos 2x = 4cos x – 1.

Данное уравнение непосредственно не имеет вид, описанный в таблице. Как правило, легко классифицировать уравнения, если привести тригонометрические функции в него входящие к одному аргументу. Поскольку cos 2x = cos2xsin2 x = 2 cos2x –1 , то уравнение 2 cos2x = 4 cos x сведено к (2) виду таблицы. Поэтому делаем замену cos x = t, t  и получаем неполное квадратное уравнение t2 – 2t = 0, откуда t1 = 0, t2 = 2.

t2 = 2 посторонний корень, поскольку t  .

Делаем обратную замену: cos x = 0, x = …πn, nZ .

Ответ: x = 2πn, nZ .

Пример 4. Решить уравнение – 4 – cos2x = 4 sin x.

Решение: Вместо cos2x подставим тождественное ему выражение 1 – sin2x . Тогда исходное уравнение примет вид

4 – (1 –sin2x) = 4 sin x, – 5 + sin2x = 4 sin x, sin2– 4 sin x – 5 = 0.

Если положить y = sin x, получим квадратное уравнение y2 – 4y – 5= 0.

D = (4)2 41 (– 5) = 16 20 = …,

у 1= (4 6) : 2 = 2 : 2 = …, у 2 = (4 + 6) : 2 = 10 : 2 = ... ,

находим корни  y1= 1 и y2 = 5. Значит, исходное уравнение равносильно совокупности уравнений

sin x = – 1 или  sin x = 5.

Уравнение sin x = … имеет решение . Уравнение sin x = … решений не имеет. Ответ: .

Пример 5. Решите уравнение 4 sin2 x + 3 cos x– 3 = 0 .


Решение: 4 sin2 x + 3 cos x– 3 = 0 .Вместо sin2x подставим тождественное ему выражение 

1 – cos2x . Тогда исходное уравнение примет вид 4(1 – cos2 x) + 3 cosx – 3 = 0;

4 – 4cos2 x + 3 cosx – 3 = 0;– 4 cos2 x + 3 cos x + 1=0; 4 cos2 x – 3 cos x – 1=0;

Замена cos x = t, |t|≤1, 4t2 – 3t – 1 = 0;

D = (3)2 44 () = 9 + 16 = … , t1= (3 – 5) : 8 = , t2 = (3 5) : 8 = …,

Делаем обратную замену cos x = ; x 1= ±arccos() + 2πk, kZ ,

cos x = …; .

Ответ: x 1= ±arccos() + 2πk, kZ , .

2)Решить задание ( по примерам):

  1. a)Решите уравнение sin2 x + 6sin x – 7 = 0.

б) Решите уравнение tg2 x 6tg x + 5 = 0.

  1. Решите уравнение 2sin2 x cos x – 1 = 0.

  2. Решите уравнение cos 2x = 6cos x – 1.

  3. Решить уравнение 4 – cos2x = 4 sin x.

  4. Решите уравнение 6 sin2 x + 5 cos x– 7=0 .

3)Решить задание :

  1. Решите уравнение 5sin2 x +21sin x + 4 = 0 .

  2. Решите уравнение 2tg2 x – 11tg x + 5 = 0.

  3. Решите уравнение 5sin2 x – 7cos x + 1= 0.

  4. Решите уравнение cos 2x = 8cos x – 1.

  5. Решить уравнение 3 – cos2x = 3 sin x.

  6. Решите уравнение 6 sin2 x + 5 cos x– 7=0 .

  7. Решите уравнение 6cos2 x – 19cos x +3 = 0.

  8. Решите уравнение 8tg2 x +10tg x + 3 = 0.

  9. Решите уравнение 8sin2 x + 10cos x – 5 = 0.

  10. Решить уравнение 5 – cos2x = 5 sin x.

  11. Решите уравнение .

  12. Решите уравнение .

  13. Решите уравнение 2sin2 x +5 sin x – 3 = 0.

  14. Решите уравнение 3 cos2 x -5 cos x – 2 = 0.

  15. Решите уравнение tg2 x +3tgx – 4 = 0.

  16. Решите уравнение 4 sin2 x – 12 cos x+12 = 0.

  17. Решите уравнение 2 cos2 x + 5 cos x – 3 = 0.

  18. Решите уравнение 3 sin2 x -5 sin x – 2 = 0.

  19. Решите уравнение tg2 x + tgx – 12 = 0.

  20. Решите уравнение 4 cos2 x – 12sin x+12 = 0.

  21. Решите уравнение sin2 x + sin x – 2 = 0.

  22. Решите уравнение 2 cos2x – 5 sin x + 1 = 0.

  23. Решите уравнение cos2 x + cos x – 2 = 0.

  24. Решите уравнение 6cos2 x +5 cos x – 7 = 0.

  25. Дано уравнение tg2 x + 5tgx + 6 = 0.а) Решите уравнение.

б) Укажите корни, принадлежащие отрезке .

  1. Дано уравнение 6cos2 x –7cos x – 5 = 0. a) Решите уравнение.

б) Укажите корни, принадлежащие отрезку .

  1. Решите уравнение

  1. Решите уравнение 3cos2 x – 5cos x + 2 = 0.

  2. Решите уравнение 3cos2x = 5 – 7 sin x .

  3. Решите уравнение 2cos2x + 3 sin x = 0.

  4. Решите уравнение 2 sin 2x = 5 cos x – 1.

  5. Решите уравнение 2 sin 2x + 5 cos x = – 1.

  6. Решите уравнение 8cos2x + 6 sin x – 3 = 0 .

  7. Решите уравнение 3sin2 x –8 sin x + 4 = 0.

Инструкционная карта

ПР № 39 «Решение тригонометрических уравнений способом деления».

Задание:

1)Перепишите и заполните пропуски:
Пример 1. Решить уравнение .

Решение: .

Поделим обе части уравнения на cos x или sin x. Но предварительно надо доказать, что это выражение никогда не обращается в нуль. Предположим, что cos x= 0. Тогда 5sin x3∙0 = 0 , sin x = 0. Получается, что если sin x = 0, то и cos x = 0 , чего быть не может ввиду равенства . Значит можно поделить уравнение на cos x:

. Получим уравнение 5tg x 3 = 0, tg x = 3/5= ...

Отсюда .

Ответ: . Пример 2. Решить уравнение : а) ; б) .

Решение: a) , , , ,

, .

б) ,,

, можно поделить уравнение на , , , a = 5, c = 3 , k = 4, D1 = k2ac = 16 – 15 = …, , , , .

, .

Ответ: ,.

Пример 3. Решить уравнение 4 sin x cos x - cos2 x = 0.

Решение: 4 sin x cos x - cos2 x = 0, обе части уравнения можно поделить на .

Получим 4tg x – 1 = 0, tg x = 1/4, tg x = …; x = arctg 0,25 + πn, n Z.

Ответ: x = arctg 0,25 + πn, n Z.

Пример 4. Решить уравнение .

Решение: Данное уравнение не является однородным. Но его можно превратить в однородное, заменив 3sin2x на 6sin x cos x и число 2 на .

Приведя подобные слагаемые, получим уравнение . Тогда можно обе части уравнения поделить на . Получим , a = 10, c = – 4 , k = 3,

D1 = k2ac = 9 – (– 40) = 9 + 40 = …, , или .

Отсюда .

Ответ: .

Пример 5. Решите уравнение sin2 x 5 sin x cos x + 6 cos2 x = 0..

Решение: sin2 х 5 sinх cos х + 6 cos2 х = 0 .

sin2 х 5 sinх cos х + 6 cos2х = 0 | : cos2х ≠ 0,

tg 2x 5 tg x + 6 = 0, замена tg x = t, t2 – 5t + 6 = 0,

D = (5)2 41 6 = 25 – 24 = …, t1= (5 1) : 2 = 4 : 2 = …, t2 = (5 1) : 2 = ...

t1 = 2; t2 = 3.

Решением уравнения tg х = 2 являются числа вида х1 = arctg 2+ πk , k Z.

Решением уравнение tg х = 3 являются числа вида х2 = arctg …+ πn, n Z.

Ответ: х1 = arctg 2+ πk , k Z, х2 = arctg 3+ πn, n Z.

2)Решить задание ( по примерам):

  1. Решить уравнение .

  2. Решить уравнение : а) ; б) .

  3. Решить уравнение 5 sin x cos x – 3cos2 x = 0.

  4. Решить уравнение 3.

  5. Решите уравнение hello_html_5b73cee4.gif.

3)Решить задание :

  1. Решите уравнение .

  2. Решить уравнение:12sin2x + 3sin2х 2 cos2x = 0

  3. Решить уравнение:sin2 x +2 sin x cos x – 3 cos2 x = 0.
    Решить уравнение:sin2 x – 3 sin x cos x + 1 = 0;
    Решить уравнение:5sin2 x – 4 cos x sin x + 3 cos2 x = 2.

  4. Решить уравнение: 2sin2(2x) 5 sin(2x) cos(2x) + 2 cos2(2x) = 0.

  5. Решить уравнение: sin2x + 2 3 cos2x = 0.

  6. Решить уравнение:2sin2 x = 9 sinxhello_html_m61765ba1.gifcosx –7 cos2 x.

  7. Решить уравнение:2sin2 x sinxhello_html_m61765ba1.gifcosx = cos2 x.

  8. Решить уравнение: 5 cos2 x sinxhello_html_m61765ba1.gifcosx = 1.

  9. Решить уравнение:4sin2 x – 3 sinxhello_html_m61765ba1.gifcosx = – 1.

  10. Решить уравнение: 8 cos2 x – 3 sinxhello_html_m61765ba1.gifcosx = 1.

  11. Решить уравнение: 1 – 5 sinxhello_html_m61765ba1.gifcosx + cos2 x = 0.

  12. Решить уравнение:7sin2 x – 2 sinxhello_html_m61765ba1.gifcosx = 1.

  13. Решить уравнение:sin2 xsin2 x = 3 cos2 x .

  14. Решить уравнение:3sin2 x – 2 sin 2x + 5 cos2 x = 2.

  15. Решить уравнение:sin2 x = 6 + 4 cos2 x 10 cos2 x .

  16. Решить уравнение: 5 sin x + 2 cos2 x 4= 0.

  17. Решить уравнение:5sin2 x + 5 cos x 4= 0.

  18. Решить уравнение:sin2 xcos xhello_html_m61765ba1.gifsin x 2 cos2 x = 0.

  19. Решить уравнение:2sin2 x + 5 cos xhello_html_m61765ba1.gifsin x + 5 cos2 x = 1.

  20. Решить уравнение:2sin2 x – 3 cos x hello_html_m61765ba1.gifsin x + cos2 x = 1.

  21. Решить уравнение:6sin2 x + cos xhello_html_m61765ba1.gif sin x cos2 x = 2.

  22. Решить уравнение:2sin2 x – 3 cos xhello_html_m61765ba1.gif sin x + 3 cos2 x = 1.

  23. Решить уравнение: .

  24. Решить уравнение: .

  25. Решить уравнение: .

  26. Решить уравнение: .

  27. Решить уравнение: .

  28. Решить уравнение: .

  29. Решить уравнение: .

  30. Решить уравнение: .

  31. Решить уравнение: .

  32. Решить уравнение: .

  33. Решить уравнение: .

  34. Решить уравнение: .

  35. Решить уравнение: .

  36. Решить уравнение: .

  37. Решить уравнение: 7sin2 x – 8 cos x sin x + cos2 x = 0.

  38. Решить уравнение: 3sin2 x + 2 cos x sin x = 2.

  39. Решить уравнение: 6sin2 x + 3 cos x sin x 2cos2 x = 3.

  40. Решить уравнение: 5sin2 x + 3 cos x sin x = 4.



Инструкционная карта

ПР № 40 «Вычисление множества значений тригонометрических функций по формулам».

Задание:

1)Перепишите и заполните пропуски:
Пример 1. Найти множество значений функции y = 5 – 4sinx.

Решение: Из определения синуса следует, 1  sinx  1. Далее воспользуемся свойствами числовых неравенств.

4   4sinx  4, (умножили все три части двойного неравенства на -4);

1  5 4sinx  9 (прибавили к трем частям двойного неравенства 5);

Так как данная функция непрерывна на всей области определения, то множество ее значений заключено между наименьшим и наибольшим ее значением на всей области определения, если таковые существуют. В данном случае множество значений функции y =5 - 4sinx есть множество [1; 9].

Ответ: E(y) = [1; 9].

Пример 2. Найти множество значений E(y) тригонометрической функции

y = 2sin2x cos2x.

Решение: y = 2sin2x cos2x = a , 2sin2x (1 2 sin2x) = 4 sin2x 1 = a, 4 sin2x = a 1,
2(1cos 2x) =
a 1, 2 2cos 2x = a 1, 2 cos 2x = a 1, cos 2x = (a) : (), cos 2x = (1) : ,

E(y) = [ 1; 3]. Ответ: E(y) = [ 1; 3].

Пример 3. Найти множество значений E(y) тригонометрической функции

y = 3 cos 2x 4sin2x.

Решение: y = 3 cos 2x 4sin2x = g, a = 3, b = , k2 = a2 b2 = 32 ()2 = 9 16 = 25,
k = 5, 3/5∙ cos 2x 4/5∙ sin 2x = g /5, sin(φ ) = g/5, E(y) = [; …].
Ответ: E(y) = [ 5; 5].

Пример 4. Найти множество значений E(y) тригонометрической функции

y = 10cos2x – 6sin x cos x 2sin2x.

Решение: y = 10cos2x – 6sin x cos x 2sin2x = a.

Oбе части уравнения поделим на cos2x. Получим, 10 6 tg x 2tg2x = a∙ (1 tg2x),

10 6 tg x 2tg2 x a tg2x = 0, (2) ∙ tg2 xtg x (10) = 0, tg x = t, (2) ∙ t2t (10) = 0, D = ()2 4∙ (2)∙ (10) = 36 4∙ (20 2) = =3680 48a 2 = 48a 2 = 4∙ (2 , 2

2a1 = …, a2 = ...

hello_html_5bd89a69.png


E(y) = [1; 11]. Ответ: E(y) = [1; 11].

Пример 5. Найти множество значений E(y) тригонометрической функции

y = sinx cosx.

Решение: Преобразуем выражение sinx + cos x  = sinx +sin( – x) =
= 2sin((x  +  – x)/2)cos((x + + x)/2) = 2sin()cos(x + ) = cos(x + ).

Из определения косинуса следует – 1  cosx  1;  – 1  cos(x + )  1;

 cos( x + )  ;

Так как данная функция непрерывна на всей области определения, то множество ее значений заключено между наименьшим и наибольшим ее значением, если таковые существуют, множество значений функции y =  cos(x + ) есть множество [; ]. Множество значений  функции

y = sinx + cosx есть множество чисел [; ].

Ответ: E(y) = [; ].


2)Решить задание ( по примерам):

  1. Найти множество значений E(y) тригонометрической функции y =7 - 4sinx

  2. Найти множество значений E(y) тригонометрической функции y = 4sin2x cos2x.

  3. Найти множество значений E(y) тригонометрической функции y = 6 cos 2x 8sin2x.

  4. Найти множество значений E(y) тригонометрической функции

y = 5cos2x – 2sin x cos xsin2x.

  1. Найти множество значений E(y) тригонометрической функции y = 2(sinx cosx).

3)Решить задание :

  1. Найдите множество значений функции y = 1 – 8sin2x cos2x.

  2. Найдите множество значений функции y = 10 10sin23x.

  3. Найдите множество значений функции y =6 5sinx

  4. Найдите множество значений функции y =4 + 3sinx

  5. Найдите множество значений функции y = 5 +7sinx

  6. Найдите множество значений функции y = 3(sinx cosx).

  7. Найдите множество значений функции y = (sinx cosx).

  8. Найдите множество значений функции y = (sinx cosx).

  9. Найдите множество значений функции y = 2 (sinx cosx).

  10. Найдите множество значений функции y = 1 – 8 cos2x.

  11. Найдите множество значений функции y = 0,25 + 2 cos2x.

  12. Найдите множество значений функции y = 6sin2x cos2x.

  13. Найдите множество значений функции y = 9 cos 2x 12sin2x.

  14. Найдите множество значений функции y = 6cos2x 8sin x cos x 6sin2x.

  15. Найдите множество значений функции y = cos x – 10.

  16. Найдите множество значений функции y = 0,2sin 5x.

  17. Найдите множество значений функции y = cos2x, если .

  18. Найдите множество значений функции y = sin x + 5.

  19. Найдите множество значений функции y = 6cos 3x.

  20. Найдите множество значений функции y = sin 2x, если .

  21. Найдите множество значений функции .

  22. Найдите множество значений функции .

  23. Найдите множество значений функции .

  24. Найдите множество значений функции

  25. Найдите множество значений функции .

  26. Найдите множество значений функции

  27. Найдите множество значений функции

  28. Найдите множество значений функции

  29. Найдите множество значений функции

  30. Найдите множество значений функции

  31. Найдите множество значений функции

  32. Найдите множество значений функции

  33. Найдите множество значений функции

  34. Найдите множество значений функции .

  35. Найдите множество значений функции y = 8cos2x – 6sin x cos x 8sin2x.

  36. Найдите множество значений функции y =16cos2x – 10sin x cos x 16sin2x.


Инструкционная карта

ПР № 41 « Нахождение экстремумов функции. Нахождения наибольшего и наименьшего значения функции».

Задание:

1)Перепишите и заполните пропуски:

Пример 1. Найти точку максимума функции y= x3 3x2 24x 5.

Решение: Требуется найти критическую точку, в которой знак производной меняется с плюса на минус. Область определения функции: hello_html_4fb18fd6.gif Найдем критические точки функции:



- Критические точки.

Исследуем знак производной на интервалах, разделенных критическими точками:

х

- 4 2

max min

Ответ: x = ...

Пример 2. Найдите точки экстремума функции и определите их характер y= x 4 8x2.

Решение: y = x 4 8x2 , D(y) = R , y = (x 4 8x2) = 4x 3 – 16x, y = 0,

4x 3 – 16x = 0, 4x(x2 4) = 0, 4x(x2) (x2) = 0,

x1= 0 или х2=0 или х2=0

х2 = … х3 =…

х1= 0, х2 = 2, х3 = 2 – это стационарные точки.


2 0 2 х

Функция убывает на ( ;2, на 0; 2. Функция возрастает на 2; 0, на 2; +).

х3 = …, х2 = … – это точки минимума. х1= … – это точка максимума.

Ответ: х3 = 2, х2 = 2– это точки минимума, х1= 0 – это точка максимума.

Пример 3. Найдите точки экстремума функции и определите их характер.

y= x2 6x1.

Решение: y = x2 6x1, D(y) =R,

y = ( x3 x2 6x1) = x25x6 = (х3)(х2) , y = 0, x 2 5x6 = 0,

x1 = 3, x2 = 2 – это стационарные точки.

х

2 3

Функция возрастает на ( ; 2, на 3; +).Функция убывает на 2; 3.

x2 = … – это точка максимума, х1 = ... – это точка минимума.

Ответ: х2 = 2 – это точка максимума, х1 = 3 – это точка минимума.

Пример 4. Найдите точки экстремума функции и определите их характер.

y= 2x5 5x4 10x3 3.

Решение: y = 2x5 5x4 10x3 3, D(y) = R, y = (2x5 5x4 10x3 3) = 25x4 54x3 103x2 = =…x4 …x3 …x2 = 10х2 (х1)(х3), y = 0 , 10x4 20x3 30x2 = 0, 10x2 (x2 + 2x 3) = 0,

x 2 = 0 или х2 2х3=0, х1 = 0, х2 = 1, х3 = 3 – это стационарные точки.


3 0 1 х

Функция возрастает на ( ; 3, на 1; ). Функция убывает на 3; 1.

х3 = – это точка максимума. х2 = … – это точка минимума.

Ответ: х3 = 3 – это точка максимума, х2 = 1 – это точка минимума.

Пример 5. Найти наибольшее и наименьшее значения функции у(x) = 2x3 12x2 18x  3  на отрезке [– 1;2] .

Решение: 1) Вычислим значения функции в критических точках, принадлежащих данному отрезку:

 

Полученное квадратное уравнение имеет два действительных корня:

х1= 1, х2 = 3 – критические точки. Первая критическая точка принадлежит данному

отрезку: х1= 1 .
А вот вторая – нет: х2= 1 , поэтому про неё сразу забываем.

Вычислим значение функции в нужной точке:

2)Вычислим значения функции  на концах отрезка:


3) Дело сделано, среди чисел выбираем наибольшее и наименьшее.

Ответ:  

Пример 6. Число 24 представьте в виде суммы двух неотрицательных слагаемых так, чтобы сумма квадратов этих чисел была наименьшей.

Решение: Пусть х – первое слагаемое, тогда (24-х) – второе слагаемое. Сумма квадратов этих чисел По условию задачи Рассмотрим функцию Она на интервале (0;24) непрерывна и дифференцируема. Найдем критические точки.

Это значение единственное, поэтому первое число – 12, второе – 12. Ответ: 24=12+12.
Пример 7. Найдите размеры участка прямоугольной формы, имеющего наибольшую площадь, если его периметр равен 200 м.

Решение: A B

x

D C Так как функция S(x) непрерывная на всей числовой прямой, b

то будем искать ее наибольшее значение на отрезке .

Значит, наибольшей будет площадь участка 2500 м2, а стороны участка равны 50 м и 50 м.

Ответ: 50 м и 50 м.

2)Решить задание ( по примерам):

  1. Найти точку максимума функции y = x3 6x2 15x 3.

  2. Найдите точки экстремума функции а) y = x 4 2x2 , б) y = x2 4x3 ,
    в) y = 2x
    5 10x4 40x3 5 и определите их характер.

  3. Найти наибольшее и наименьшее значения функции f(x) = x3 3x2 – 72x  90 на отрезке [– 4;5] .

  4. Число 12 представьте в виде суммы двух неотрицательных слагаемых так, чтобы сумма квадратов этих чисел была наименьшей.

  5. Найдите размеры участка прямоугольной формы, имеющего наибольшую площадь, если его периметр равен 120 м.

3)Решить задание : 1.Найдите точки экстремума функции

2. Найти наибольшее и наименьшее значения функции f(x) = x4 4x2  8 на отрезке [– 1;2] .

  1. Найти максимальное и минимальное значения функции f(x) = на отрезке [– 8;0] .

  2. Найти наибольшее и наименьшее значения функции f(x) = 2x312x2 – 30x 9 на отрезке [– 4;2] .

  3. Найти наибольшее и наименьшее значение функции  на отрезке [1;4];

  4. Найдите наибольшее значение функции   на отрезке .

Инструкционная карта

ПР № 42 «Построение графиков тригонометрических функций».

Задание:

1)Перепишите и заполните пропуски:

Пример 1.Определить возрастает или убывает функция: а) y = cos x при ,

б) y = sin x при ,в) у = tg x при .

Решение: а) убывает,…,б) …, возрастает, в) возрастает.

Ответ: а) …, возрастает, б) убывает, …, в) ...

Пример 2.Нацдите х, при котором функция пересекает ось ох: а) y = cos x при ,

б) y = sin x при ,в) у = tg x при .

Решение: а) ,б) ,в) .

Ответ: а) ,б) ,в) .

Пример 3. Определить принимает положительные или отрицательные значения функция:

а) y = cos x при ,б) б) y = sin x при ,в) у = tg x при .

Решение: а) положительные при , отрицательные при ,

б) положительные при , отрицательные при ,

в) положительные при , отрицательные при .

Ответ: а) положительные при , отрицательные при ,

б) положительные при , отрицательные при ,

в) положительные при , отрицательные при .

Пример 4.Построить график функции по таблице:

а) y = 2cos x , б) y = 6sinx .

х


0




у

0

2

0

2

0

х


0




у

6

0

6

6

0


Решение:

а) б)

hello_html_mab2a1ac.jpghello_html_m5dd79f1d.jpg


Пример 5.Сравнить а) и , б) и .

Решение: а) <, (0< < < ),б) >.Ответ: : а) <,б) >.


2)Решить задание ( по примерам):

  1. Определить возрастает или убывает функция: а) y = cos x при ,

б) y = sin x при ,в) у = tg x при.

  1. Нацдите х, при котором функция пересекает ось ох: а) y = cos x при ,

б) y = sin x при,в) у = tg x при .

  1. Определить принимает положительные или отрицательные значения функция:

а) y = cos x при ,б) б) y = sin x при ,в) у = tg x при.

  1. Построить график функции по таблице:

а) y = 6cos x .

x


0




y

0

6

0

6

0


б) y = 4sinx .

x

0





y

0

4

0

4

0


  1. Сравнить а) и , б) и .

3)Решить задание :

  1. Построить график функции y = ctg x ; запишите свойства этой функции, используя свойства функции y = tg x, и то что эти функции взаимо обратны.

  2. Сравнить числа: а) и, б) tg 2,3 и tg 3, в) и, г) tg 1 и tg 1,5.

  3. Построить график функции по таблице: y = sin 4x .


x

0





y

0

1

0

1

0


  1. Построить график функции по таблице: y = cos 4x.


x

0





y

1

0

1

0

1


  1. Построить график функции по таблице: y = tg 2x .


x

0





y

0

1


1

0


  1. Построить график функции по таблице: y = сtg 2x .


x







y

0

1

0

1

1

0


  1. Построить график функции:

а) y = sin 2x , б) y = 2sin x , в) y = cos 2x , г) y = 5sin x , д) y = 4cos x,е) y = 2сtg x .

  1. Построить график функции:

а) y = sin 4x, б) y = cos 4x, в) y = tg 2x, г) y = 2sin x, д) y = 2cos x, е) y = 2tg x.


Инструкционная карта

ПР № 43 «Решение иррациональных уравнений».   

Задание:

1)Перепишите и заполните пропуски:

Пример 1. Решить уравнениеhello_html_m31703e1.gif Решение: Уединим радикал hello_html_727bbaac.gif Это уравнение равносильно системе hello_html_mbbe3903.gif Решим уравнение (1): hello_html_m2e992f18.gif hello_html_m562ce331.gif hello_html_m11152785.gif hello_html_fa2cdec.gif х = … Найденное значение hello_html_m42749eb7.gif удовлетворяет условиям (2) и (3).

Ответ: –1. Пример 2. а) Найдите корень уравнения = 3 . Решение: Возведем в квадрат правую и левую части уравнения: )2 = 32, 15 – 2х = 9, –2х = 9 – 15, –2х = – 6, х = ... Сделаем проверку. Для этого подставим число 3 в исходное уравнение: = 3, 3 = 3 – верно.

Ответ: 3.

б) Решить уравнение = .

Решение: = => ˂=> => => х = ...

Ответ: 1. Пример 3. Решить уравнение = х -7 .

Решение: = х -7 => => => => => х = ...

Ответ: 14.

Пример 4. Решите уравнение   = .

Решение:  = => 7 х + х 2 2 = 2х 5 =>

5 – х = => 25 – 10х + х2 = х2 + 9х – 14 => 2 19х + 39 = 0,

D = (– 19)2 42 39= 361 – 312 = …, х1= (19 + 7) : 4 = …, х2 = (19 – 7) : 4 = …,

Проверка:  а)  х1= 6,5,   = ,

  = –  неверное равенство.

б) х2 = 3,   – = ,   = , –  верное равенство.

Ответ: 3.

Пример 5. Решить уравнение hello_html_m1160f3de.gif

Решение: Возводим в куб обе части уравнения hello_html_598c15c7.gif получим hello_html_m497b4a68.gif Учитывая, что выражение в скобках равно 1 (см. условие), получаем hello_html_625fcde.gif hello_html_52925630.gif hello_html_mdabf98.gif Возводим в куб: hello_html_30511dcf.gif hello_html_5282e7df.gif hello_html_33e13c73.gif Проверкой убеждаемся, что hello_html_m19405068.gif и hello_html_a6221c5.gif корни уравнения.

Ответ: 80, – 109.






2)Решить задание ( по примерам):

Решить уравнения:

  1. а) . б) .

  2. .

3) Решить задание:

  1. Решить уравнение:.

  2. Решить уравнение:.

  3. Решить уравнение:.

  4. Решить уравнение:.

  5. Решить уравнение: .

  6. Решить уравнение:3.

  7. Решить уравнение: .

  8. Решить уравнение:.

  9. Решить уравнение:.

  10. Решить уравнение:.

  11. Найдите корень уравнения:

  12. Найдите корень уравнения:

  13. Найдите корень уравнения:

Если уравнение имеет более одного корня, укажите меньший из них.

  1. Найдите корень уравнения:

  2. Найдите корень уравнения:

  3. Решить уравнение:

  4. Решить уравнение:

  5. Решить уравнение:

  6. Решить уравнение:

  7. Решить уравнение:

  8. Решить уравнение:

  9. Решить уравнение:

  10. Решить уравнение:

  11. Решить уравнение:

  12. Решить уравнение:

Инструкционная карта

ПР № 44 «Решение показательных уравнений ».
Задание:

1)Перепишите и заполните пропуски:

Пример 1. а)Найдите корень уравнения .

Решение: Чтобы решить это уравнение, вспомним свойства степени и приведем правую и левую части уравнения к степени с основанием 5: ,

Если степени с равными основаниями равны, то равны их показатели. Приравняем показатели степеней: х – 7 = - 3, х = 7 – 3, х = ...

Ответ: 4 .

б)Найдите корень уравнения .

Решение: Представим правую и левую части уравнения в виде степени с основанием ,

Приравняем показатели степеней: – 3 (– 3+ х) = 9, 9 – 3х = …, – 3х = 0, х = ...

Ответ: 0.

Пример 2. Решите уравнение.

Разделим обе части уравнения на : .

Пустьm,m > 0 , тогда 2m2 – 3m – 5 = 0, D = 9 – 42(– 5) = 9 + 40 = …, m1 = (3 + 7) : 4 = …,

m2 = (3 – 7) : 4 = – 4 : 4 = …, – не удовл. условию m > 0 .
Если m = 2,5 , то
Ответ:  – 1.

Пример 3. Решите уравнение 49x 8∙7x + 7 = 0.

Решение: Обозначим получим уравнение относительно у: у2 – 8у + 7 = 0,

D = (– 8)2 41 7= 64 – 28 = …, у1= (8 + 6) : 2 = …, у2 = (8 – 6) : 2 = ... Получим, что и , отсюда х1 = …, х2 = ... Ответ: х1 = 1, х2 = 0.

Пример 4. а)Решить уравнение .
Решение:

Ответ: 3.

б) Решите уравнение 

Решение:

Ответ: 1.

в)Решите уравнение

Решение:

Ответ: 4.

Пример 5. Решите уравнение а) 2х+1 + 2х-1 + 2х = 28, б) 9х – 8∙3х – 9 = 0, в) 8∙4х – 6∙2х + 1 = 0.

Решение:

а) 2х+1 + 2х-1 + 2х = 28, 2х-1 ∙ (22 + 1 + 2) = 28, 2х-1∙7 = 28, 2х-1 = 4, 2х-1 = 22, х – 1 = 2, х = ...

Ответ: 3.

б) 9х – 8∙3х – 9 = 0, (3х)2 – 8∙3х -9 = 0, Обозначим 3х = t, где t >0, тогда t2 – 8t – 9 = 0,

D = (–8)2 41 (–9) = 64 + 36 = …, t 1= (8 + 10) : 2 = …, t 2 = (8 – 10) : 2 = ... . t1 = 9, t2 = – 1, Возвращаемся к замене: 3х = 9, х = …, 3х = – 1, корней нет.

Ответ: 2.

в) 8∙4х – 6∙2х + 1 = 0, 8∙(2х)2 – 6∙2х + 1 = 0, Обозначим 2х = t, где t >0, тогда 8 t2 – 6t + 1 = 0, D = (–6)2 41 8= 36 – 32 = …, t1= (6 + 2) : 16 = …, t2 = (6 – 2) : 16 = ... t1 =, t2 = Возвращаемся к замене: 2х = , х 1= …, 2х = , х 2= ...

Ответ: – 1, – 2.

2)Решить задание ( по примерам):

  1. а) Найдите корень уравнения .б) Найдите корень уравнения: .

  2. Решите уравнение .

  3. Решите уравнение 25x 6∙5x + 5 = 0.

  4. а)Решить уравнение: б)Решите уравнение 

в)Решите уравнение.

Решите уравнение а) 3х+1 + 3х-1 + 3х = 117, б) 16х – 15∙4х – 16 = 0, в) 81х + 6∙9х + 9 = 0.

3)Решить задание :

  1. Решить уравнения: а) , б) в)

  2. Решите уравнение а) 4x 5∙2x + 4 = 0, б) 9x 4∙3x + 3 = 0.

  3. Решите уравнение  

  4. Найдите сумму корней уравнения :.

  5. Если - корень уравнения , то найдите значение выражения .

  6. Найдите произведение корней уравнения .

  7. Решите уравнение .

  8. Решите уравнение .

  9. Решите уравнение

  10. Решите уравнение 4х + 2х – 6 = 0;

  11. Решите уравнение 9х + 3х+1 = 4;

  12. Решите уравнение

  13. Решите уравнение .

  14. Решите уравнение: .

  15. Решите уравнение: 92х+1 – 9 = 72.

  16. Решите уравнение:

  17. Пусть х0 ─ наибольший корень уравнения . Найти 2х0 – 5.

  18. Решите уравнение: 23х+2 + 8х = 0,625.

  19. Пусть х0 ─ наименьший корень уравнения . Найти 3х0 + 2.

  20. Найти сумму корней уравнения: 4х – 40∙2х + 256 = 0.

  21. Решите уравнение:

  22. Решите уравнение :3∙ + 325 ∙ = 0.

  23. Решите уравнения:


  1. Решите уравнения:

  2. Решите уравнение: .

  3. Решите уравнения:


  1. Решите уравнение: .

  2. Решите уравнения:

  3. Найти корень (или сумму корней, если их несколько) уравнения: 7 · 8х+1 + 8х+3 = 71.

  4. Найти корень (или сумму корней, если их несколько) уравнения: 7 = 6 · 7х + 7.

  5. Решите уравнение: .

  6. Решите уравнение: .


Инструкционная карта

ПР № 45«Решение логарифмических уравнений».

Задание:

1)Перепишите и заполните пропуски:

Пример 1. Решите уравнение Решение: Используем метод - решение логарифмических уравнений заменой.

ОДЗ: х > 0. Введем замену , чтобы записать исходное уравнение в виде стандартного квадратного уравнения. Тогда уравнение примет вид: у2 – 4у + 4 = 0, ( у – 2)2 = 0, у – 2 = 0, у = ... Вернемся к  х : . Тогда по определению логарифма получаем, что х = 32, х = … - уд.ОДЗ.

Ответ: 9.

Пример 2. Решите уравнение:.  

Решение: Вновь начнем решение с определения области допустимых значений уравнения. Она определяется следующей системой неравенств:


Воспользовавшись правилом сложения логарифмов, переходим к равносильному в области допустимых значений уравнению:

Основания логарифмов одинаковы, поэтому в области допустимых значений можно перейти к следующему квадратному уравнению:

(х + 2) (х + 3) = 1 х , х2 + 6х + 5 = 0, D = (6)2 41 5= 36 – 20 = …,

х1= ( 6 4) : 2 = , х2 = ( 4) : 2 = ...  

Первый корень не входит в область допустимых значений уравнения, второй — входит.

Ответ: x = -1.

Пример 3. Решите уравнение:

Решение: Найдем ОДЗ по определению логарифма. ОДЗ:

.

Перепишем исходное уравнение, используя свойства суммы логарифмов и логарифма степени. Получим следующее уравнение:

Приравняем подлогарифмические выражения:

(3х ) (х) = ,

Найдем корни полученного квадратного уравнения:

D = (92)2 41 () = 8464 + 8436 = …,

х1= (92 + 130) : 6 = 222 : 6 = …, х2 = (92 130) : 6 = .

Учитывая ОДЗ, корнем исходного логарифмического уравнения будет только х = ...

Ответ: х = 37.

Пример 4. Решите уравнение:

Решение: В область допустимых значений входят только те x, при которых выражение, находящееся под знаком логарифма, больше нуля. Эти значения определяются следующей системой неравенств:


С учетом того, что получаем промежуток, определяющий область допустимых значений данного логарифмического уравнения:.

На основании теоремы 1, все условия которой здесь выполнены, переходим к следующему равносильному квадратичному уравнению:

  D = (5)2 41 () = 25 + 56 = …, х1= (5 + 9) : 2 = …, х2 = (5 9) : 2 =

В область допустимых значений входит только первый корень. Ответ: x = 7.

Пример 5. Решите уравнение:

Решение: Используем метод - решение логарифмических уравнений, переходя к одному основанию. ОДЗ: 

К логарифму по основанию x (второе слагаемое) вначале применим свойство логарифма степени, а затем по формуле замены основания логарифма приведем его к основанию 2:



Так как  то


Введем замену  тогда уравнение примет вид: у2 – 5у + 4 = 0.

Найдем корни полученного квадратного уравнения:

D = (5)2 41 = 25 = …, y1= (5 + 3) : 2 = …, y2 = (5 3) : 2 = ...

Вернемся к x, используя определения логарифма:

x = x = …, x = , x = …, Оба значения принадлежат ОДЗ.

Ответ: 16 и 2.

Пример 6. Решите уравнение:

Решение: Область допустимых значений уравнения определяется здесь легко: x > 0.

Используем подстановку: Уравнение принимает вид: 3у2 + 5у = 0,

D = (5)2 43 () = 25 + 24 = …, у1= (5 + 7) : 6 = 1/3, у2 = (5 7) : 6 =

Вернемся к x, используя определения логарифма:

x = , x = , x =... Оба ответа входят в область допустимых значений уравнения, поскольку являются положительными числами.

Ответ: и 4.

2)Решить задание ( по примерам):

  1. Решите уравнение

  2. Решите уравнение .

  3. Решите уравнение

  4. Решите уравнение:

  5. Решите уравнение:

  6. Решите уравнение:

3)Решить задание :

  1. Решите уравнение .

  2. Решите уравнение .

  3. Решите уравнение: а) б)

  4. Решите уравнение:

  5. Решите уравнение:

  6. Если - корень уравнения , то найдите значение выражения .

  7. Найдите произведение корней уравнения .

  8. Найдите сумму корней уравнения .

  9. Найдите больший корень уравнения .

  10. Решите уравнение: а) б)

  11. Решите уравнение:

  12. Решите уравнение:

  13. Решите уравнение:

  14. Найдите сумму корней уравнения .

  15. Найдите сумму корней уравнения .

  16. Найдите произведение корней уравнения .

  17. Если - корень уравнения , то найдите значение выражения .

  18. Решите уравнение: а) ,б).

  19. Решите уравнение: .

  20. Решите уравнение: .

  21. Решите уравнение:

Инструкционная карта

ПР № 46«Решение тригонометрических уравнений».

Задание:

1)Перепишите и заполните пропуски:

Пример 1.Решите уравнение sin6xcos3x = 0.

Решение: sin6x – cos3x = 0 , 2sin3x cos3x – cos3 x = 0 , сos3x(2sin3x – 1)=0 ,

сos3x = 0 или sin3x = 1/2 .

3x = π/2 + π k, k , 3x = (– 1) n π/6 + π n, n Z .

х1= π/6+ π k/3 , k Z, x2 = ( – 1)n π/18+ π n/3 , n Z .

Ответ: х1= π/6+ π k/3 , k Z; x2 = ( – 1)n π/18+ π n/3 , n Z .

Пример 2. Решите уравнение ( ctg x – 1) · (2sin + 1) = 0.

Решение: ( ctg x – 1) · (2sin + 1) = 0,

ctg x – 1 = 0 или 2sin + 1 = 0,

ctg x = 1 sin = – 1/2, х/6 = (– 1) n +1 π/6 + π n, n Z,

х1 = … + π k, k , х2 = (– 1) n +1 … + 2π n, n Z.

Ответ: х1 = π/4 + π k, k , х2 = (– 1) n +1 π + 6π n, n Z.

Пример 3. Решите уравнение . Решение: , cos (3x 2x) = , cos x = ,
x =

Ответ: x =

Пример 4. a)Решите уравнение sin2 x + 5sin x – 6 = 0.

Решение: sin2 x + 5sin x – 6 = 0. Делаем замену  sin x = t, t ,получаем

квадратное уравнение: t2 + 5t – 6 = 0, 

D = 52 41 (6 ) = 25 + 24 = …, t 1= ( 5 + 7) : 2 = 2 : 2 = …, t 2 = ( 5 7) : 2 = 12 : 2 = ... замечаем, что t2 = – 6 посторонний корень, поскольку t  ,

делаем обратную замену, т.е. решаем уравнение sin x = 1 , у которого корнями будут числа

x = +2πn, nZ .

Ответ: x = π/2 +2πn, nZ.

б) Решите уравнение tg2 x 3tg x + 2 = 0.

Решение: Делаем замену  tg x = t, получаем квадратное уравнение: t2 – 3t + 2 = 0, 

D = ( 3)2 41 2 = 9 8 = …, t 1= (3 + 1) : 2 = 4 : 2 = …, t 2 = ( 1) : 2 = 2 : 2 = ... делаем обратную замену: tg x = 1, x1 = + πn, nZ  или tg x = 2, x 2= arctg … + πk, kZ .

 Ответ: x1 = π/4 + πn, nZ  , x 2= arctg 2 + πk, kZ .

Пример 5. Решить уравнение 4 – cos2x = 4 sin x.

Решение: Вместо cos2x подставим тождественное ему выражение 1 – sin2x . Тогда исходное уравнение примет вид 4 – (1 –sin2x) = 4 sin x, 3 + sin2x = 4 sin x, sin2– 4 sin x + 3 = 0.

Если положить y = sin x, получим квадратное уравнение y2 – 4y + 3 = 0.

D = 42 41 3 = 16 12 = …, y 1= (4 + 2) : 2 = 6 : 2 = …, y 2 = (4 2) : 2 = 2 : 2 = ... Значит, исходное уравнение равносильно совокупности уравнений sin x = 1 или  sin x = 3.

Уравнение sin x = 1 имеет решение . Уравнение sin x = 3 решений не имеет. Ответ: .

Пример 6. Решите уравнение 6 sin2 x+7 cos x 1= 0 .

Решение: 6 sin2 x + 7 cos x – 1= 0 .

Вместо sin2x подставим тождественное ему выражение  1 – cos2x . Тогда исходное уравнение примет вид 6(1 – cos2 x) + 7 cosx – 1= 0; – 6 cos2 x + 7 cos x + 5= 0; 6 cos2 x – 7 cos x – 5= 0;

Замена cos x = t, |t|≤1, 6t2 – 7t – 5 = 0;D = (7)2 46 () = 49 + 120 = …, t1= (7 – 13) : 12 = , t2 = (7 13) : 12 = . - не удовлетворяет условию |t|≤1;

Делаем обратную замену cos x = ; x = ±arccos() + 2πk, kZ ,x = ± (π – π/3) + 2πk , kZ,

x = ± … + 2πk , kZ .

Ответ: x = ± 2π/3 + 2πk , kZ .

Пример 7. Решить уравнение .

Решение: .

Поделим обе части уравнения на cos x или sin x. Но предварительно надо доказать, что это выражение никогда не обращается в нуль. Предположим, что cos x= 0. Тогда 5sin x2∙0 = 0 , sin x = 0. Получается, что если sin x = 0, то и cos x = 0 , чего быть не может ввиду равенства . Значит можно поделить уравнение на cos x:

. Получим уравнение 5tg x 2 = 0, tg x = 2/5= 0,4.

Отсюда .

Ответ: .

Пример 8. Решите уравнение 2 sin2 х 3 sinх cos х 5 cos2 х = 0 .

Решение: 2 sin2 х 3 sinх cos х 5 cos2 х = 0 .

2 sin2 х 3 sinх cos х 5 cos2х = 0 | : cos2х ≠ 0,

2 tg 2x 3 tg x 5 = 0, замена tg x = t. , 2 t2 – 3t – 5 = 0,

D = (3)2 42 (– 5) = 9 + 40 = …, t1= (3 7) : 4 = 4 : 4 = …, t2 = (3 7) : 4 = 10 : 4 = …

Решением уравнения tg х = 1 являются числа вида х1 = … + πk , k Z.

Решением уравнение tg х = 2,5 являются числа вида х2 = arctg … + πn, n Z.

Ответ: х1 = – π/2 + πk , k Z, х2 = arctg 2,5+ πn, n Z.

2)Решить задание ( по примерам):

Решить задание ( по примерам):

  1. Решите уравнение sin8xcos4x = 0.

  2. Решите уравнение (2 sin x – ) ·tg x – ) = 0.

  3. Решите уравнение cos 4xcos4x + sin5xsin4x = / 2.

  4. a)Решите уравнение sin2 x + 8sin x – 9 = 0,б) Решите уравнение tg2 x 10tg x + 9 = 0.

  5. Решить уравнение 3 – cos2x = 3 sin x.

  6. Решите уравнение 4 sin2 x + 3 cos x– 3=0 .

  7. Решить уравнение .

  8. Решите уравнение hello_html_5b73cee4.gif.

3)Решить задание :

  1. Решить уравнение .

  2. Решите уравнение sin2xcosx = 0.

  3. Решите уравнение (2 cos x – 1)(ctg x – ) = 0.

  4. Решите уравнение ( tg x – 1)(2sin – ) = 0.

  5. Решите уравнение cos 4xcosxsin4xsinx = / 2.

  6. Решите уравнение 2cos(х + π/6) = – .

  7. Решите уравнение 5sin2 x – 7cos x + 1= 0.

  8. Решите уравнение cos 2x = 6cos x – 1.

  9. Решить уравнение : а) ; б) .

  10. Решить уравнение 5 sin x cos x – 3cos2 x = 0.

  11. Решить уравнение 3.

  12. Решите уравнение: .

  13. Решите уравнение: а). б) .

  14. Найдите сумму корней уравнения , принадлежащих промежутку

  15. Решите уравнение .

  16. Решите уравнение .

  17. Решите уравнение .

  18. Решите уравнение .

  19. Решите уравнение

  20. Решить уравнение 4 sin x cos xcos2 x = 0.

Инструкционная карта

ПР № 47 «Решение показательных неравенств».

Задание:

1)Перепишите и заполните пропуски:

Пример 1. Решите неравенства: а) . Решение: х ... Ответ: х 4.

б), Решение: , 2х 6 – 3х, х

Ответ: х

Пример 2. Решите неравенство: 36x 76x + 6 0.

Решение: Обозначим получим неравенство относительно у: у2 – 7у + 6 0, у2 – 7у + 6 = 0, D = (– 7)2 41 6= 49 – 24 = …, у1= (7 + 5) : 2 = 12 : 2 = …,

у2 = (7 – 5) : 2 = 2 : 2 = ...

+ 1 6 + у 1у 6, 1 6, …х ...


Ответ: 0х1.

Пример 3. Решите неравенство: .
Решение: Сделаем замену , тогда и неравенство перепишется в виде ,

откуда . Следовательно, решением данного неравенства являются числа х, удовлетворяющие неравенствам , и только такие числа. Но , , а функция убывает, поскольку < 1. Поэтому решением неравенств будут числа х, удовлетворяющие неравенствам … < х < ...

Ответ: ( 2; 1).

Пример 4. Решить неравенство: .

Решение: Так как 625 = 252 = , то заданное неравенство можно записать в виде .Так как 0 < 0,04 < 1, то, сравнивая показатели, запишем неравенство противоположного смысла х2 8 2. Имеем последовательно ,

,,

. + 2 3 + х

Решив последнее неравенство, получим … х ...

Таким образом множество решений заданного неравенства есть отрезок [2; 3].

Ответ: [2; 3].

Пример 5. Решите неравенства: а) , б) .

Решение: а) , , , , , , ,
,
функция возрастающая,

Ответ: .

б) .

Решение: , пусть,,,

,

и

, и , , ,функциявозрастающая,

, , , , ;

,,,,;

Ответ: .

2)Решить задание ( по примерам):

  1. Решите неравенства: а) , б).

  2. Решите неравенство 64x 9∙8x + 8 0.

  3. Решите неравенство .

  4. Решите неравенство

  5. Решите неравенства: а) , б) .

3)Решить задание:

  1. Найдите наименьшее целое решение неравенства .

  2. Найдите наименьшее целое решение неравенства .

  3. Решите неравенство < 1. 4.Решите неравенство ≥ 1.

  1. Решите неравенство ≤ 1.

  2. Решите неравенство.

  3. Решите неравенство.

  4. Найдите область определения функции;

  5. Решите неравенство: 9x 4∙3x + 3 0.

  6. Решите неравенства: а) , б).

  7. Решите неравенства:

  8. Найдите наибольшее целое значение х, удовлетворяющее неравенству:

  1. Решите неравенствоhello_html_29386776.gif.

  2. Решите неравенство .

  3. Решите неравенство .

  4. Решите неравенства: а) (1/3)2x – 6∙(1/3)x – 27 ≤ 0, б) (1/4)x – 3∙(1/2)x + 2 > 0.

  5. Решите неравенства: а) 2x+ 21x> 3, б) (1/3)x + 3x+3 ≤ 12.

  6. Решите неравенства: а) ,б) ,в) .

  7. Решите неравенства: а) , б) ,в) .

  8. Решите неравенства: а)


Инструкционная карта

ПР № 48 «Решение логарифмических неравенств».

Задание:

1)Перепишите и заполните пропуски:

Пример 1. Решить неравенство

Решение: По определению логарифма, область допустимых значений:

Решение данного неравенства найдем с помощью метода интервалов, для этого левую часть разложим на множители. Решим квадратное уравнение 

D = ()2 41 3 = 1612 = …, х1= ( + 2) : 2 = : 2 = …, х2 = ( 2) : 2 = : 2 = ...

Значит, левую часть неравенства можно представить в виде:

Отметим нули каждого множителя на числовой прямой и определим знаки неравенства в полученных интервалах:

hello_html_6b83020d.pngх

Учитывая знак неравенства, определим ОДЗ:

ОДЗ определили, теперь приступим к решению исходного логарифмического неравенства:


Представим правую часть неравенства как логарифм по основанию 2:

Перейдем от неравенства относительно логарифмов к неравенству для подлогарифмических функций: так как основание логарифма больше единицы ( 2 > 1 ), то знак неравенства не изменится: D = (4)2 41 () = 16 + 20 = …,

х1= (4 + 6) : 2 = 2 : 2 = …, х2 = (4 6) : 2 = : 2 = …

Таким образом, получили корни х1= 1, х2 = . Отметим точки на числовой оси и определим знаки неравенства в полученных интервалах.

hello_html_m641bb999.pngх

Учитывая, что нас интересуют все значения х, при которых данное неравенство принимает положительные значения, то получаем следующие интервалы:  Это ответ, так как данные интервалы полностью принадлежат ОДЗ. Ответ: 

Пример 2. Решить неравенство

Решение: Находим ОДЗ по определению логарифма.


Перейдем в неравенства от логарифмов к выражениям, стоящим под знаком логарифма, при этом, так как основание логарифма меньше единицы ( 0,5 < 1 ), знак неравенства поменяем на противоположный:

С учетом ОДЗ, окончательно имеем, что   Ответ: 

Пример 3. Решить неравенство

Решение: ОДЗ: х > 0. Логарифмируем левую и правую часть неравенства: .

По свойству логарифма степени получаем:

Ведем замену  Тогда наше неравенство принимает вид:

D = (1)2 41 () = 1 + 8 = …, y1= (1 + 3) : 2 = 4 : 2 = …, y2 = (1 3) : 2 = : 2 = ...

Неравенство примет вид: Отметим точки на числовой оси и определим знаки неравенства в полученных интервалах.

hello_html_43066964.pngх

Решением будет отрезок  Перейдем обратно к x:

.

В пересечении с ОДЗ получаем этот же промежуток  Ответ:

Пример 4. Решить неравенство

Решение: По определению логарифма, находим ОДЗ:

Используя свойство логарифма степени и формулы замены основания, приведем второй логарифм к основанию 3:

Введем замену   y + Перенесем 2 в левую часть и приводим к общему знаменателю: Данное неравенство равносильно следующему: y(y2) > 0.

 y2 D = (2)2 41 2 = 4 .

Дискриминант меньше нуля, и старший коэффициент a = 1 > 0, следовательно, при любом значении y выражение y2 > 0. А тогда произведение y(y2) положительно, когда y > 0. Перейдем к x, для этого делаем обратную замену: Пересекая с ОДЗ, окончательно имеем промежуток 

Ответ: 
Пример 5. Решите неравенство  .
Решение:


Ответ: (0,1;1).

2)Решить задание ( по примерам):

  1. Решить неравенство

  2. Решить неравенство

  3. Решить неравенство

  4. Решить неравенство

  5. Решить неравенство.

3)Решить задание:

  1. Решить неравенства: а)б)

  2. Решить неравенства: а) б)

  3. Решите неравенства: а)б)

  4. Найдите наибольшее целое решение неравенства .

  5. Найдите наибольшее целое решение неравенства .

  6. Найдите сумму целых решений неравенства .

  7. Укажите количество целых решений неравенства.

  8. Решите неравенства:

  9. Решить неравенства: .

  10. Решить неравенства:

  11. Решить неравенства:

  12. Решить неравенства:

  13. Укажите количество целых решений неравенства:   

Инструкционная карта

ПР № 49 «Решение тригонометрических неравенств».

Задание:

1)Перепишите и заполните пропуски:

Пример 1. Решить неравенство sin(t) 1/2.

Решение: Рисуем единичную окружность. Так как sin(t) по определению - это координата y, отмечаем на оси Оу точку у = 1/2. Проводим через неё прямую, параллельную оси Ох. В местах пересечения прямой с графиком единичной окружности отмечаем точки Pt1 и Pt2. Соединяем двум отрезками начало координат с точками Pt1 и Pt2

Решением данного неравенства будут все точки единичной окружности расположенные выше данных точек. Другими словами решением будет являться дуга l. Pt1 лежит в правой полуокружности, её ордината равна 1/2, hello_html_m203b787d.jpg

тогда t1= arcsin(1/2) = π/6. Для описания точки Pt1 можно записать следующую формулу: t2 = π – arcsin(1/2) = 7π/6. В итоге получаем

для t следующее неравенство:/6 t 7π/6,

Мы сохраняем знаки неравенств. А так как функция синус функция периодичная, значит решения будут повторяться через каждые 2π. Это условие добавляем к полученному неравенству для t и записываем ответ.

Ответ: /6+2πn t 7π/6 + 2πn, n Z.

Пример 2. Решить неравенство cos(t) < 1/2.

Решение: Нарисуем единичную окружность. Так как согласно определению cos(t) это координата х, отмечаем на грфике на оси Ох точку x = 1/2.
Проводим через эту точку прямую, параллельную оси Оу. В местах пересечения прямой с графиком единичной окружности отмечаем точки P
t1 и Pt2. Соединяем двум отрезками начало координат с точками Pt1 и Pt2.
hello_html_mecb7ec3.jpg

Решениями будут все точки единичной окружности, которые принадлежать

дуге l. Найдем точки t1 и t2t1 = arccos(1/2) = π/3 ,

t2 = 2π arccos(1/2) = 2π/3 = 5π/3.

Получили неравенство для t: π/3 < t < 5π/3.

Так как косинус - это функция периодичная, то решения будут повторяться через каждые 2π. Это условие добавляем к полученному неравенству для t и записываем ответ.hello_html_m174f9f6b.jpg

Ответ: π/3+2πn < t <5π/3+2πn, n Z.

Пример 3. Решить неравенство tg(t) 1.

Решение: Период тангенса равняется π. Найдем решения, которые принадлежат промежутку ( π/2; π/2) правая полуокружность. Далее воспользовавшись периодичностью тангенса, запишем все решения данного неравенства. Нарисуем единичную окружность и отметим на ней линию тангенсов.

Если t будет являться решение неравенства, то ордината точки Т = tg(t) должна быть меньше или равна 1. Множество таких точек будет составлять луч АТ. Множество точек Pt, которые будут соответствовать точкам этого луча – дуга l. Причем, точка P(-π/2) не принадлежит этой дуге. Найдем условие, при котором некоторая точка Pt будет принадлежать дуге l. t1 = arctg(1) = π/4.  Получаем неравенство /2 t/4. 

Учитывая период тангенса записываем ответ.

Ответ: /2 + πnt /4 + πn, n Z.

Пример 4. Решить неравенство:   sin x > 0.

Решение: В пределах одного оборота единичного радиуса это неравенство справедливо

при 0 < x < hello_html_3fac242e.gif. Теперь необходимо добавить период синуса  2hello_html_3fac242e.gif n :

0 + 2πn х π + 2πn, 2πn х π + 2πn, при любом целом n.

Ответ: 2πn х π+ 2πn, n Z.hello_html_m12869a62.pnghello_html_6e343a75.png

Пример 5. а) Решить неравенство:   sin x > 0.5 .

Решение: π/6 + 2πn < х < 5π/6 + 2πn, для любого целого n.

б) Решить неравенство cosх > /2.

Решение: /4 + 2πn х π/4 + 2πn,

для любого целого n.( по рис.)

Пример 6. Решить неравенство cos (x/4 – 1) ≤ ( / 2).

Решение: Обозначим x/4 – 1 = у. Решая неравенство cos у ≤ ( / 2), находим 
3π/4 + 2πn ≤ у ≤ 5π/4 + 2πn, n Z.

Заменяя у = x/4 – 1, получаем 3π/4 + 2πn ≤ x/4 – 1 ≤ 5π/4 + 2πn, откуда 
1 + 3π/4 + 2πn ≤ x/4 ≤ 1 + 5π/4 + 2πn, 4 + 3π + 8 πn ≤ х ≤ 4 + 5π + 8 πn, n Z.

Ответ: 4 + 3π + 8 πn ≤ х ≤ 4 + 5π + 8 πn, n Z.

2)Решить задание ( по примерам):

  1. Решить неравенство sin(t) 1/2.

  2. Решить неравенство cos(t) >1/2.

  3. Решить неравенство tg(t) 1.

  4. Решить неравенство sin x < 0.

  5. Решить неравенство sin x < 0.5 .

  6. Решить неравенство cos (x/4 – 1) ( / 2).

3)Решить задание:

  1. Решить неравенствa: a) .

  2. Решить неравенствa:a) tg x > 1, tg x ≤ 1, , сtg x ≤ 1.

  1. Решите неравенства: а) ,б) , в) .

  2. Решите неравенства: 1) ;2) ;3) ;

  3. Найдите какой-либо корень уравнения , удовлетворяющий неравенству .

  4. Решите неравенство: .

  5. Решите неравенство: .

  6. Решите неравенства: а)б) в)

  7. Решите неравенства: а) б)

  8. Решите неравенства: а) б)

  9. Решите неравенства: а)б)

  10. Решите неравенства: а) ,б) ,в) .

  11. Решите неравенства: а) ,б) ,в) .

  12. Решите неравенства: а), б) ,в) 1,г) ,
    д) ,е)

  13. Решите неравенства: а) 6sin2x – 5sinx + 1 ≥ 0,б) cos22x – 2cos2x ≥ 0, в) .

  14. Решите неравенства: а) б) в) .

  15. Решите неравенства: а)

  16. Определите все а, при каждом из которых неравенство 4sinx + 3cosxа имеет хотя бы одно решение.

Инструкционная карта

ПР № 50 «Решение систем уравнений и неравенств».

Задание:

1)Перепишите и заполните пропуски:

Пример 1. Решить систему уравнений:  .

Решение: ,

Выразим у через х из (2) -го уравнения системы и подставим это значение в (1) -ое уравнение системы. Решаем (2) -ое уравнение полученной системы: 2х+2x+2=10, применяем формулу: ax+y=ax∙ay.

2x+2x∙22=10, вынесем общий множитель 2х за скобки: 2х · (1+22)=10 или 2х∙5=10, отсюда 2х=2.

2х=21, отсюда х=... Возвращаемся к системе уравнений. у = х + 1 = 1 + 1 = ...

Ответ: (1; 2).

Пример 2. Решить систему уравнений:  .

Решение: Представляем левую и правую части (1) -го уравнения в виде степеней с основанием 2, а правую часть (2) -го уравнения как нулевую степень числа 5. Если равны две степени с одинаковыми основаниями, то равны и показатели этих степеней — приравниваем показатели степеней с основаниями 2 и показатели степеней с основаниями 5. Получившуюся систему линейных уравнений с двумя переменными решаем методом сложения.

,

Находим х = … и это значение подставляем вместо х во второе уравнение системы, находим у.

2(2 + у) = 7, 2 + у = 3,5 ; у = … Ответ: (2; 1,5).

Пример 3. Решить систему уравнений: .

Решение: ,

Сделаем замену , .

Выразим через . Подставим во 2 уравнение. Решим уравнение с переменной .

. По теореме Виета ,


Возвращаемся к х, у. , х = 2, у = 1. Ответ: (2; 1).

Пример 4. Решить систему уравнений:

Решение: ,

Подставим из 2 уравнения у в 1, решим с переменной х.

х· (4х 15) = 4, 4х2 15х 4 = 0. D = 152 4 = 225 64 = .


По свойству логарифмов х > 0, y > 0, поэтому х = 0,25. Найдем у: у = 4 0,25 15 = 1 + 15 = …

Ответ: (0,25;16).

Пример 5. а)Решить систему уравнений:

Решение: ,

Решаем способом сложения: 2 = 10, = 5, х = …, у = х21 = 25 21 = …

Ответ: (25,4).

б) Решить систему уравнений:

Решение: ,

Из 1 уравнения выразим х и подставим во 2: 2х = 5 у, х = 2,5 0,5у,

, , ,

, D = ()2 4 = 9 352 =


О.Д.З. : , у2 не уд. О.Д.З., поэтому

Ответ: (3;).

2)Решить задание ( по примерам):

  1. Решить систему уравнений:

  2. Решить систему уравнений: 

  3. Решить систему уравнений:

  4. Решить систему уравнений:

  5. а)Решить систему уравнений:

б) Решить систему уравнений:

3)Решить задание:

  1. Найти значение выражения ,если и - решение системы уравнений

а) ,б) в) г) ,д)

  1. Найти значение выражения ,если и - решение системы уравнений

а)б) в) г) ,д)

  1. Решить систему уравнений: 

а) б) в)

г)д)

  1. Решите систему уравнений , решение принадлежит отрезку [0;2].

  2. Решить систему уравнений:

 а)б)


Инструкционная карта

ПР № 51 «Решение неравенств с помощью метода интервалов».

Задание:

1)Перепишите и заполните пропуски:

Пример 1. Решите неравенства: а) (х ) (х) ( х , б) (х ) (х) ( х

Решение: а) (х ) (х) ( х , х = 0, х = 0, х х1 = …, х2 = …, х3 = ...





Ответ:

б) (х ) (х) ( х х = 0, х = 0, х х1 = …, х2 = …, х3 = ...






Ответ: .

Пример 2. Решите неравенства: а) x3 x < 0, б) (.

Решение: а) x3 x < 0, x· (x2 1) < 0, x· (x2 1 ) = 0, х1 = …, x2 1 = 0, х2 = 1, х3 = ...





Ответ: .

б) (, () = 0, 0,

D = (6)2 41 8 = 36 32 = …, x1= (6 2) : 2 = 8 : 2 = …, x2 = (6 2) : 2 = 4 : 2 = …,

x 1 = 0, x = ...






Ответ: .

Пример 3. Решите неравенства: а) б)

Решение: а) ,




hello_html_6f29e869.png


Ответ: .

б) ,




hello_html_4f115f52.png


Ответ: .

Пример 4. Решите неравенства: а) , б) ,

Решение: а) , H < 0, D < 0, a > 0 => P = ...

Ответ: P = .

б) , H ≥ 0, D < 0, a > 0 => P = ...

Ответ: P = R.

Пример 5. Решите неравенства: а)

б) .

Решение: а)

х = 0, х = 0, х

х1 = …, х2 = …, х3 = ...

n = 2,1петля n = 3,2петли






, x .

Ответ: .

б) ,

,

x1 = 0, n = 4, 3 петли, х = 0, х2 = …, n = 3, 2петли,

х = 0, х3 = …, n = 2, 1петля , х х4 = ...





Ответ: .

2)Решить задание ( по примерам):

  1. Решите неравенства: а) (х ) (х) ( х , б) (х ) (х) ( х

  2. Решите неравенства: а) x3 4x < 0, б) (.

  3. Решите неравенства: а) б)

  4. Решите неравенства: а) , б) ,

  5. Решите неравенства: а) б) .

3)Решить задание:

  1. Решите неравенства: а) (х ) (х) ( х , б) (х ) (х) ( х

  2. Решите неравенства: а) x3 x < 0, б) (.

  3. Решите неравенства: а) б)

  4. Решите неравенства: а) , б) ,

  5. Решите неравенства: а) б) .

  6. Решите неравенство . 7.

  7. Найти все значения параметра а, при которых неравенство х2+(2а+4)х+8а+1≤ 0 не имеет решений.

  8. Найдите все значения параметра а, при которых неравенство х2 + (2а+6)х7а15< 0 выполняется для любых х.

  9. Решите неравенство > 0.

  10. Решите неравенство (х21)2 (х+5) ≥ 0;

  11. Решите неравенство (х24) (х+7)3 ≤ 0;

  12. а) – (х1) (5х) (х+20) > 0, б) – (х2) (9х) (х+10) > 0.

  13. Решите неравенство

  14. Решите неравенство x3· (2x + 8) · (x − 3)2 > 0.



Инструкционная карта

ПР № 52 «Вычисление угловых коэффициентов. Составление уравнения касательной к графику функции».

Задание:

1)Перепишите и заполните пропуски:

Пример 1. Найти угловой коэффициент касательной к графику функции y = f(x) в точке с

абсциссой х: а) y(x) = x³, x = 1, б) y(x) = ln x, x = 1, в) y(x) = 3x² 4x, x = 2,

г) y(x) = х3 + 7x² 5x+3, x = 3, д) y(x) = ех, x = ln 7, e) y(x) = 7sinx, x = 0,ж) y(x) = е, x = ln 4.

Решение: угловой коэффициент k равен производной от функции в точке, т.е. k = y (x0) ,

найдем производные и вычислим их в точке x0

a)   бв)  

г)

д) е ln 7= …,е) 7cos x, 7 cos 0 = 7 1 = …,

ж) е3 ln 4 = 343 = 364 = …

Ответ: a)3, б)1, в)8,г) 64,д) 7,е)7,ж) 192.

Пример 2. а) Найти угловой коэффициент k, если α = arctg 6, α = arctg 8.

б) Найти α,если y(x) = х3, x = 2.

Решение: а) k = tgα = tg k = tgα = tg k = tgα = tg

k = tgα = tg

б)

Ответ: а)1, ,6,- 8, б) arctg 4.

Пример 3. Дана функция y = x3. Составить уравнение касательной к графику этой функции в точке x0 = 2.
Решение: Уравнение касательной: y = f  (x0) · (x − x0) + f(x0). Точка x0 = 2 нам дана, а вот значения f (x0) и f (x0) придется вычислять.

Для начала найдем значение функции. Тут все легко: f (x0) = f (2) = 23 = …;
Теперь найдем производную: f  (x) = (x3) = 3x2;
Подставляем в производную x0 = 2: f   (x0) = f  (2) = 3 · 22 = 34 = …;
Итого получаем: y = 12 · (x − 2) + 8 = 12x − 24 + 8 = 12x − 16. 
Это и есть уравнение касательной.

Ответ: y = 12x − 16. 
Пример 4. Составить уравнение касательной к графику функции f (x) = 2sin x + 5 в точке x0 = π/2.

Решение: f (x0) = f (π/2) = 2sin (π/2) + 5 = 2 + 5 = …; f  (x) = (2sin x + 5) = 2cos x;
f  (x0) = f  (π/2) = 2cos (π/2) = 0;

Уравнение касательной: y = 0 · (x − π/2) + 7  y = ...hello_html_44c9db7b.png

Ответ: y = 7.

Пример 5. Составьте уравнение касательной к графику функции  

в точке M(3; – 2).

Решение: Точка M(3; – 2) является точкой касания, так как

1. a = 3 – абсцисса точки касания.2. f(3) = – 2. 3. f '(x) = x2 – 4, f '(3) = 9 4 = …
y = – 2 + 5(x – 3), y = …x – 17 – уравнение касательной.

Ответ: y = 5x – 17.

Пример 6. Напишите уравнения всех касательных к графику функции y = – x2 – 4x + 2, проходящих через точку M(– 3; 6).

Решение: Точка M(– 3; 6) не является точкой касания, так как f(– 3)  6 (рис. 2).hello_html_m126a8a75.png

1. a – абсцисса точки касания.
2. f(a) = – a
2 – 4a + 2.
3. f '(x) = – 2x – 4, f '(a) = – 2a – 4.
4. y = – a2 – 4a + 2 – 2(a + 2)(x – a) – уравнение касательной.

Касательная проходит через точку M(– 3; 6), следовательно, ее координаты удовлетворяют уравнению касательной.

6 = – a2 – 4a + 2 – 2(a + 2)(– 3 – a),
a
2 + 6a + 8 = 0 , D = 62 41 8 = 36 32 = …,

а1= (6 2) : 2 = 8 : 2 = …, а2 = (6 2) : 2 = 4 : 2 = …,

Если a = – 4, то уравнение касательной имеет вид y = 4x + 18.

Если a = – 2, то уравнение касательной имеет вид y = 6.

Ответ: y = 4x + 18 или y = 6.
Пример 7. Напишите уравнения всех касательных к графику функции y = x3 – 3x2 + 3, параллельных прямой y = 9x + 1.

Решение: 1. a – абсцисса точки касания. 2. f(a) = a3 – 3a2 + 3.3. f '(x) = 3x2 – 6x, f '(a) = 3a2 – 6a.

Но, с другой стороны, f '(a) = 9 (условие параллельности). Значит, надо решить уравнение 3a2 – 6a = 9. 3a2 – 6a 9 = 0, hello_html_5a070ee0.png

D = (6)2 43 () = 36 108 = …, а1= (6 12) : 6 = 18 : 6 = …,

а2 = (6 12) : 6 = 6 : 6 = …,

Его корни a = – 1, a = 3 (рис. 3).

4. 1сл.) a = – 1; f(– 1) = – 1– 3 + 3 = …;  f '(– 1) = 3 + 6 = …;

 y = – 1 + 9(x + 1); y = 9x + 8 – уравнение касательной;

2сл.) a = 3; f(3) = 27–27 + 3 = …; f '(3) = 27 – 18 = …;
y = 3 + 9(x – 3); y = 9x – 24 – уравнение касательной.

Ответ: y = 9x + 8 и y = 9x – 24.

Пример 8. Напишите уравнение касательной к графику функции y = 0,5x2 – 3x + 1, проходящей под углом 45° к прямой y = 0 (рис. 4).hello_html_m32119c54.png

Решение: Из условия '(a) = tg 45°, найдем a:  a – 3 = 1 ,a = 3 + 1 = ...

1. a = 4 – абсцисса точки касания.
2. f(4) = 8 – 12 + 1 = ...
3. f '(4) = 4 – 3 = ...
4. y = – 3 + 1(x – 4). y = x – 7 – уравнение касательной.

Ответ: y = x – 7.

Пример 9. На параболе у = х2 взяты две точки с абсциссами 1 и 3. Через эти точки проведена прямая. В какой точке параболы касательная будет параллельна проведенной прямой?

Решение: у = х2 , (1;1), (3;9). Найдем уравнение прямой .

4х – 4 = у – 1. у = 4х – 3.

Прямые параллельны, если их угловые коэффициенты равны.

- угловой коэффициент касательной в точке с абсциссой х0.

 0 = 4. х0 = ... ,

Ответ: в точке (2;4) касательная параллельна заданной прямой.

Пример 10. При каких b и c прямые y = x и y = – 2x являются касательными к графику

функции y = x2 + bx + c?

Решение: Пусть t – абсцисса точки касания прямой y = x с параболой y = x2 + bx + c;

p – абсцисса точки касания прямой y = – 2x с параболой y = x2 + bx + c.

Тогда уравнение касательной y = x примет вид y = (2t + b)x + c – t2, а уравнение

касательной y = – 2x примет вид y = (2p + b)x + c – p2.

Составим и решим систему уравнений:

;

2t = 1,5; t = 0,75;

p = – t = …,

c = = = …,

b = 1 – 2t = 1 – 2 0,75 = 1– 1,5 = …

Ответ: b = – 0,5; c = 0,562 5.

2)Решить задание ( по примерам):

  1. Найти угловой коэффициент касательной к графику функции y = f(x) в точке с

абсциссой х: а) y(x) = x4, x = 1, б) y(x) = ln x, x = 2, в) y(x) = 3x² – 4x, x = 4,

г) y(x) = х3 + 7x² – 5x+3, x =5, д) y(x) = ех, x = ln 8, e) y(x) = 9sinx, x = 0,ж) y(x) = е, x = ln 6.

  1. а) Найти угловой коэффициент k, если α = arctg 9, α = arctg 11.

б) Найти α,если y(x) = х3, x = 4.

  1. Дана функция y = x3. Составить уравнение касательной к графику этой функции в точке x0 = 1.

  2. Составить уравнение касательной к графику функции f (x) = 4sin x + 5 в точке x0 = π/2.

  3. Составьте уравнение касательной к графику функции у(х) в точке M(3; – 1).
     

  4. Напишите уравнения всех касательных к графику функции y = – x2 – 4x + 2, проходящих через точку M(– 3; 9).

  5. Напишите уравнения всех касательных к графику функции y = x3 – 3x2 + 3, параллельных

прямой y = 24x + 1.

  1. Напишите уравнение касательной к графику функции y = 0,5x2 – 5x + 1, проходящей
    под углом 45° к прямой y = 0 .

  2. На параболе у = х2 взяты две точки с абсциссами 1 и 2. Через эти точки проведена прямая.
    В какой точке параболы касательная будет параллельна проведенной прямой?

  3. При каких b и c прямые y = x и y = – 2x являются касательными к графику
    функции y = x2 + 2bx + c?

3)Решить задание:

  1. Напишите уравнения касательных, проведенных к графику функции y = 2x2 – 4x + 3 в точках пересечения графика с прямой y = x + 3.

  2. При каких значениях a касательная, проведенная к графику функции y = x2 – ax в точке графика с абсциссой x0 = 1, проходит через точку M(2; 3)?

  3. Найдите все общие точки графика функции y = 3x – x3 и касательной, проведенной к этому графику через точку P(0; 16).

  4. На кривой y = x2 – x + 1 найдите точку, в которой касательная к графику параллельна
    прямой y – 3x + 1 = 0.

  5. Найдите угол q между касательными к графику функции y = x3 – 4x2 + 3x + 1, проведенными в точках с абсциссами 0 и 1.

  6. Прямая y = 2x + 7 и парабола y = x2 – 1 пересекаются в точках M и N. Найдите точку K пересечения прямых, касающихся параболы в точках M и N.

  7. При каких значениях b касательная, проведенная к графику функции y = bx3 – 2x2 – 4 в точке с абсциссой x0 = 2, проходит через точку M(1; 8)?

  8. Найти угол между касательными к графику функции , проведенными в точках с абсциссами 1 и 2.

  9. Является ли прямая у = х – 1 касательной к кривой у = х3 – 2х + 1?

  10. Найдите уравнение касательной к графику функции  в точке с абсциссой .

  11. К графику функции у = 3(х + 2) проведены две параллельные касательные, одна из которых проходит через точку графика с абсциссой х0 = – 1. Найдите абсциссу точки, в которой
    другая касательная касается графика данной функции.

  12. Напишите уравнение касательной к графику функции f(x) = x2 – 4x + 5, если эта касательная проходит через точку (0; 4) и абсцисса точки касания положительна.

  13. Напишите уравнение касательной к графику функции f(x) = x2 + 3x + 5, если эта касательная проходит через точку (0; 1) и абсцисса точки касания отрицательна.

  14. Найдите уравнение параболы f(x) = ax2 + bx + 1 касающейся прямой у = 7х + 2
    в точке М (1; 5).

  15. К графику функции провести касательную так, чтобы она была параллельна прямой у = 4х - 5.

  16. Из точки (0; 1) провести касательную к графику функции.

  17. Составить уравнение касательной к графику функции  в точке с абсциссой.

  18. Составить уравнение касательной к графику функции  в точке с абсциссой 

  19. Составить уравнение касательной к графику функции
      в точке с абсциссой  .

  20. Составить уравнение касательной к графику функции > 0, отсекающей от осей координат треугольник, площадь которого равна .












Инструкционная карта

ПР № 53 «Вычисление производных по правилам дифференцирования».

Задание:

1)Перепишите примеры:

Пример 1. Найти производную функции  y = .

Решение: По свойству дифференцирования произведения,

hello_html_m667262fc.png.

Используя формулу для нахождения производной показательной и степенной функций, получим: hello_html_m33ce9c10.png , hello_html_m7a155e45.png

Для нахождения производной использовались правила дифференцирования и таблица производных функций. Ответ: hello_html_m7259a439.png .

Пример 2. Найти производную функции  y = .

Решение: Воспользуемся правилом дифференцирования частного:

hello_html_m6c0be978.png.

Производная суммы/разности равна сумме/разности производных и константу можно выносить за знак производной, поэтому имеем:

hello_html_m6d9e2531.png,

hello_html_m4661b9f4.png,

hello_html_m25a8c90f.png, hello_html_303e1e97.png , hello_html_47c000ce.png .

Ответ: hello_html_6873cbb0.png .

Пример 3. Найти производную функции y =   .

Решение: По правилу дифференцирования частного:

hello_html_m2652334c.png,

Далее воспользуемся формулами из таблицы производных - формулам для производных степенной и тригонометрических функций, а также учитываем тот факт, что производная суммы равна сумме производных:

hello_html_6bfbb3b0.png,

hello_html_4f5a6440.png,

hello_html_345a5152.png, hello_html_3dd92d7d.png .

Ответ: hello_html_5f22ddde.png .

Пример 4. Найти производную функции  hello_html_mb62401c.png .

Решение: По свойству дифференцирования частного получаем:

hello_html_m1bbacdfd.png,

Далее пользуясь формулами для производных логарифмической и степенной функции, получим:

hello_html_m7e5e4019.png, hello_html_7fc6a7d9.png , hello_html_m68b092d7.png .

Ответ: hello_html_ecc0376.png . Пример 5. а) Найти производную функции  .

Решение: Примените таблицу основных производных и формулы производных линейной комбинации и отношения функций.




Ответ:  .

б) Вычислить производную функции y = cos ln ().

Решение: Примените таблицу основных производных и формулу производной сложной функции.

y / = sin ln (3x2 ) (ln (3x2)) / = sin ln (3x2 ) / =

= sin ln (3x2 ) . Ответ:  sin ln (3x2 ) .

2)Решить задание ( по примерам): Найти производную функции: 

1) y = . 2) y = . 3) y = . 4) .5) а) . б) y = cos ln (2x2).

3)Решить задание:

  1. Найдите производные следующих функций:

1) 2) 3) 4)

  1. Найдите производную функции в точке .

  2. Найдите производную функции в точке .

  3. Вычислите у ' ,если а) у(х) =, б) у(х) =.

  4. а)Вычислите у ', если у(х) = sin x · cos2 x .б)Вычислите у ', если у(х) = sin x · cos x .

  5. Решить неравенство у ' > 0, если у(х) = (3х – 1)10 · (2х + 5)7.

  6. Решить неравенство у ' > 0, если у(х) = (2х – 1)9 · (3х + 5)6.

  7. Решите уравнение: f ' (x) = 0, если f (x) = x –2cos x .

  8. Решите уравнение: f ' (x) = 0, если f (x) = x – tg x .

  9. Решите уравнение: f ' (x) = 0, если f (x) = x – ctg x.

Инструкционная карта

ПР № 54 «Вычисление производных элементарных функций».

Задание:

1)Перепишите и заполните пропуски:

Пример 1. Найдите производные функций: а) y = ex x7 ,б) у=3ех+cos2x, в) у = ехsinx,

г) у= ln2x ,д) , е) , ж)

Решение: а) б) в) = ех - cosx; г) ,

д)е)ж)

Ответ: а)б) в) = ех - cosx; г) ,д)

е)ж)

Пример 2. Вычислите значение производной функции:

а) у= в точке , б) у=ех sinx + x2 в точке ,

в) у = cos2x + 4x в точке ,г) в точке .

Решение: а)


б)

в)

г) Ответ: а)10,5; б)1;в)4; г)2.

Пример 3. Найдите производные функций: а) б)

в) г) д)
Решение: а) у  (x) = (x 2 + sin x) = (x 2) + (sin x) = …x + cos x;
б)
у  (x) = (x 3 · cos x) = (x 3) · cos x + x 3 · (cos x) = …x 2 · cos x + x 3· (− sin x) =

= x 2 · (3cos x  x · sin x),

в) у  (x) = ((x 2 + 7x − 7) · e x ) = (x 2 + 7x − 7) · e x + (x 2 + 7x − 7) · (e x ) = (2x + 7) · e x +

+ (x 2 + 7x − 7) · e x = e x · (2x + 7 + x 2 + 7x − 7) = (x 2 + …x) · e x = x(x + …) · e x .

г)
д)

По традиции, разложим числитель на множители — это значительно упростит ответ:


Ответ: а) у  (x) = 2x + cos x; б) у  (x) = x 2 · (3cos x  x · sin x), в) у  (x) = x(x + 9) · e x ,

г) д)

Пример 4. Найти производные функций:  f(x) = e 2x + 3g(x) = sin (x 2 + ln x).
Решение: Заметим, что если в функции f(x) вместо выражения 2x + 3 будет просто x, то получится элементарная функция f(x) = e x . Поэтому делаем замену: пусть 2x + 3 = tf(x) = f(t) = e t . Ищем производную сложной функции по формуле:

f  (x) = f  (t) · t  = (e t ) · t  = e t · t . Выполняем обратную замену: t = 2x + 3. Получим:

f  (x) = e t · t  = e 2x + 3 · (2x + 3) = e 2x + 3 · 2 = … · e 2x + 3

Теперь разберемся с функцией g(x). Очевидно, надо заменить x 2 + ln x = t. Имеем:

g  (x) = g  (t) · t  = (sin t) · t  = cos t · t . Обратная замена: t = x 2 + ln x. Тогда:

g  (x) = cos (x 2 + ln x) · (x 2 + ln x) = cos (x 2 + ln x) · (…x + 1/x).

Ответ: f  (x) = 2 · e 2x + 3; g  (x) = (2x + 1/x) · cos (x 2 + ln x).
Пример 5. Найти производную функции :

а)б)
Решение: а)

б)
Ответ: а) б)

2)Решить задание ( по примерам):

  1. Найдите производные функций: а) y = 2ex 3x7 ,б) у=5ех + cos3x, в) у = ехcosx,

г) у= – ln4х ,д) , е) , ж) .

  1. Вычислите значение производной функции:

а) у= в точке , б) у=2ех ·sinx +3 x2 в точке ,

в) у = cos2x + 8x в точке ,г) в точке .

  1. Найдите производные функций: а) б)

в) г) д)

  1. Найти производные функций:  f(x) = e 4x + 3; g(x) = sin (2x 2 + ln x).

  2. Найти производные функций :

а)б)

3)Решить задание:

  1. Найдите производную функции y = e x 2x7.

  2. Найдите производную функции у= 4х3+ е х.

  3. Найдите производную функции у = x2 + sinx в точке х0 =.
    Найдите производную функции у = sinх ex – 9x3 в точке xo=0.

  4. Найдите значение производной функции у = 5cos x – 7x в точке хо = 0.

  5. Вычислите значение производной функции y = ln(2x+11)+ 5x в точке хо= 5.

  6. Найдите производную функции: а) б)

Инструкционная карта

ПР № 55«Построение графиков функций с использованием производной ».

Задание:

1)Перепишите и заполните пропуски:

Пример 1. Исследовать и построить график функции:

Решение:

  1. D (f) = R, т.к. f -многочлен.

  2. Выясняем, является ли функция f четной или нечетной. - функция ни четная, ни нечетная.

  3. Функция непериодическая.

  4. Находим точки пересечения графика с осями координат:

а) с осью ОХ: у=0 получаем точки (0;0), (3;0)

б) с осью ОУ: х=0 получаем точки (0;0)

  1. Найдем производную функции:

  2. Найдем критические точки: , т.е. ,х = … или х = ...

Отмечаем эти точки 0 и 2 на числовой прямой, и определяем знак производной в каждом промежутке. −  +  

6(−  1) −  3(−  1)2 = −  6 −  3 = −  9 < 0

0 2 х

Значит, в промежутках и функция убывает и (0;2) – функция возрастает.

х = 0 - точка минимума, т.к. производная меняет знак с минуса на плюс.

Вычислим уmin=

х = 2 – точка максимума, т.к. производная меняет знак с плюса на минус.

Вычислим уmax= .

7.Составляем таблицу для внесения всех данных

x


0

(0;2)

2



− 

0

+

2

− 

f(x)


0


4




min


max


8. Строим график функции.

hello_html_6ac426ef.png


Пример 2. Сколько корней имеет уравнение: x4   4x3   9 = 0?

Решение: р (x) = x4   4x3   9, D(р) = ( hello_html_8781bd9.gif; hello_html_8781bd9.gif).

р ' (x) = 4 x 3 12x 2 = 4 x 2 3) = 0, x1 = 0; 1 петля; x2 = …, р ' (4) = 4 hello_html_m61765ba1.gif16 hello_html_m61765ba1.gif1 > 0






р(x) убывает на интервале (hello_html_8781bd9.gif ; 3]; р (x) возрастает на [3; +hello_html_8781bd9.gif).

x = 3 – min, р min= р (3) = 34   4 hello_html_m61765ba1.gif 33   9 = 81 4hello_html_m61765ba1.gif27 – 9 = 81   117= − < 0, в точке x = 0 график имеет точки перегиба (то есть меняет выпуклость), f(0) = 0   0  9 = ...

Строим эскиз графика

hello_html_331651eb.png

График пересекает ось 0Х в двух точках x1 и x2, следовательно, многочлен, а значит и данное уравнение имеет два корня.

Ответ: два.

Пример 3. Исследовать функцию f(x)= 3x5 3 + 2 и построим ее график.

Решение: 1.D (f ) = R, так как f (x) - многочлен.

2.Функция f не является ни четной, ни нечетной, так как

f (− x) = 3(− x)5 5(− x)3 + 2 = − 3x 5 + 5х3 + 2= −  ( 3x5 − 5х3 − 2) hello_html_382283fb.gif f(x)

3.Найдем координаты точек пересечения графика с осями координат:

а) с осью 0Х, для этого решим уравнение: 3x5 3 + 2 = 0.

Методом подбора можно найти один из корней (x = 1). Другие корни могут быть найдены только приближенно. Поэтому для данной функции остальные точки пересечения графика с осью абсцисс и промежутки знакопостоянства находить не будем.

б) с осью 0У: f(0) = 3hello_html_m61765ba1.gif05  5hello_html_m61765ba1.gif03 + 2 = …

Точка А (0; 2) - точка пересечения графика функции с осью 0У.

Отметили, что промежутки знакопостоянства не будем находить.

4.Найдем промежутки возрастания и убывания функции : а ) f '(x)= 15x4  15х2 = 15х2 hello_html_m61765ba1.gif2   1)

D (f ') = R, поэтому критических точек которых f '(x)не существует, нет.

б) f '(x) = 0, если х2hello_html_m61765ba1.gif2 1)=0 <=> x1 = 0 ; 1 петля ; x2  1= 0, x2 =  1, х2 = …, х3 = …

в) Получим три критические точки, они разбивают координатную прямую на четыре промежутка. Определим знак производной на этих промежутках: f ' (4) = 15 hello_html_m61765ba1.gif16 hello_html_m61765ba1.gif15 > 0






Так как функция непрерывна в точках – 1; 0; 1, то f   возрастает на (– hello_html_8781bd9.gif; – 1] и [1; +hello_html_8781bd9.gif);

f  убывает на [– 1; 0] и [0; 1].

5.Найдем точки экстремума функции и вычислим значения функции в этих точках.

x = − 1 - точка max, f (− 1) = 3hello_html_m61765ba1.gif (− 1) 5  5hello_html_m61765ba1.gif (− 1) 3 + 2 = − 3+ 5 + 2 = 7 – 3 = … ;

x = 1 - точка min, f (1) = 3hello_html_m61765ba1.gif15  5hello_html_m61765ba1.gif13 + 2 = 3− 5 + 2 = 5 – 5 = ...

Полученные результаты занесем в таблицу и построим график .


x


1

(− 1;0)

0

(0;1)

1



+

0

0

0

+

f(x)


4


2


0




max



min



Пример 4. Исследовать и построить график функции:hello_html_m46464d67.png

Решение: проведем исследование функции:

  1. Функция определена и непрерывна на всей числовой

прямой, D (f ) = R .
,

значит, данная функция является четной, ее график симметричен относительно оси ординат.
Очевидно, что функция непериодическая.

  1. Точки пересечения графика с координатными осями, интервалы знакопостоянства функции.
    График функции проходит через начало координат.

 на всей области определения.

  1. Возрастание, убывание, экстремумы функции.



х = 0
 – критическая точка. Определим знаки :
hello_html_5a5cd5d3.jpg

 возрастает на  и убывает на.
В точке х= 0 функция достигает минимума: .

  1. Найдем дополнительные точки и выполним чертёж:


х

0,5

1,5

2

2,5

3

4

5

6


0,08

0,43

0,57

0,68

0,75

0,84

0,89

0,93


hello_html_m1f9f2eb8.jpg


Пример 5. Сколько корней имеет уравнение: ?

Решение: Рассмотрим функцию р(x) =

1) Найдем область определения функции D(р) = (−hello_html_8781bd9.gif; hello_html_8781bd9.gif).

2) Найдем производную р' (x) = x 3 − 3x 2 – x + 3.

3) Найдем критические точки и промежутки возрастания и убывания функции:

р' (x) = 0 <=> x 3 − 3x 2 – х + 3= 0 <=> x 2 hello_html_m61765ba1.gif (х − 3) − (х − 3) = 0 <=> (х − 3) hello_html_m61765ba1.gif ( x 2 − 1) = 0 <=>

х1=3, х2= 1, х3= − 1. Получим три критические точки, они разбивают координатную прямую на четыре промежутка. Определим знак производной на этих промежутках:

р ' (4) = 1 ·15 > 0

hello_html_47ff2293.png




р(x) возрастает на интервалах [1; 1] и [3; +hello_html_8781bd9.gif);

р(x) убывает на (hello_html_8781bd9.gif ; 1] и [1; 3].

4) Найдем точки экстремума и экстремумы функции:

х = − 1 min р min= 1/4 + 1 − 1/2 − 3= 0,25 + 1 – 0,5 – 3 = 1,25 – 3,5 = − … < 0,

x = 1 max р max= 1/4 − 1 − 1/2 + 3 = 0,25 – 1 – 0,5 + 3 = 2 – 0,25=… > 0,

х = 3 min р min = 81/4 − 27 − 9/2 + 9 = 20,25 – 27 – 4,5 + 9 = 29,25 – 31,5 =

= − … < 0.

Строим эскиз графика.

Из рисунка видно, что многочлен имеет 4 корня, следовательно, уравнение имеет 4 решения.

Ответ: уравнение имеет 4 решения.

2)Решить задание ( по примерам):

  1. Исследуйте функцию и постройте ее график.

  2. Сколько корней имеет уравнение: x4 − 4x3 + 9 = 0?

  3. Исследовать функцию f(x)= 3x5 3 + 6 и построим ее график.

  4. Исследовать и построить график функции:.

  5. Сколько корней имеет уравнение: ?

3)Решить задание:

  1. Исследуйте функцию y = 1/3x3 − 3x2 + 8x и постройте ее график.

  2. Сколько корней имеет уравнение: x2 − x3/3− 1= 0?

  3. Исследовать и построить график функции:

а) y = 3x4 − 4x3 − 12x2 + 10; б) y = в) f (х) = x4 − 2х2.

  1. Исследуйте функцию и постройте ее график: f (x) = x4 −2х2 −3.

  2. Найти число корней уравнения: 2x 3 − 3x 2 − 12х − 11= 0.

  3. Исследуйте функцию и постройте ее график: а)б)

  4. Сколько корней имеет уравнение: а)б) в)

  5. Определите промежутки монотонности и экстремумы функции 

  6. Докажите, что функция f(x) = 4x — 3 sin x возрастает на всей числовой прямой.

  7. Исследуйте функцию f(x) = x4 + 4x2  5 и постройте ее график.

  8. Определите промежутки монотонности и экстремумы функции 

  9. Докажите, что функция f(x) = 5 cosx 7x убывает на всей числовой прямой.

  10. Исследуйте функцию f(x) = x4 + 8x2 − 9 и постройте ее график.

  11. Определите промежутки монотонности и экстремумы функции  .

  12. Докажите, что уравнение х5 + 2х3 + 8x + cos 3x = 0 имеет ровно один корень.

  13. Дана функция  
    а) постройте график функции 
    f(х);
    б) сколько корней имеет уравнение 
    f(х) = а?

  14. Дана функция 
    а) постройте график функции 
    f(x); б) сколько корней имеет уравнение f(х) = а?

  15. При каком наибольшем значении параметра а функция  убывает на всей числовой прямой?

  16. Докажите, что уравнение х5 + 4х3 + 7x sin2x = 0 имеет ровно один корень.

  17. При каком наибольшем значении параметра а функция  возрастает на всей числовой прямой?

  18. При каких значениях а функция f(х) = 8ac – a sin 6 7 sin 5х возрастает на всей числовой оси и не имеет стационарных точек?

  19. Проведите исследование и постройте график функции 

  20. Исследуйте функцию f(x) = х3 – 3x2 + 2 и постройте ее график.

  21. Исследуйте функцию f(х) = х3 3х + 2 и постройте ее график.

  22. При каких значениях параметра а функция f(x) = a sin 7x + 8 ax + sin 4 5x убывает на всей числовой оси и не имеет стационарных точек?

Инструкционная карта

ПР № 56 «Вычисление второй производной. Исследование функции на выпуклость, вогнутость, определение точек перегиба».

Задание:

1)Перепишите и заполните пропуски:

Пример 1. Найдите вторые производные функций :

а) , б) , в) , г) ,

д) , е) , ж) , з) .

Решение:

а)

б)

в)


г)

д)

е)


ж) з)

Ответ: а) б) в) г)

д) е) ж)

з)

Пример 2. Найти интервалы выпуклости, вогнутости и точки перегиба графика функции

Решение:

,,

, 6 hello_html_m61765ba1.gif 3 – 12 > 0hello_html_5a5cd5d3.jpg

точка перегиба (функция вогнутая)


Ответ: график функции  является вогнутым на интервале  и выпуклым на , точка перегиба: (2;12).

Пример 3.а) Найти интервалы выпуклости, вогнутости и точки перегиба графика

Решение: 1) Функция определена и непрерывна на R.
2) Найдём критические точки второй производной:

 – критические точки
3) Определим знаки второй производной на полученных интервалах: 8 hello_html_m61765ba1.gif (1 – 4) < 0hello_html_4bf8ac88.jpg
В точках  х= существуют перегибы графика.  
Ответ: график функции  является вогнутым на интервале  и выпуклым на , точки перегиба: .
б) Найти интервалы выпуклости, вогнутости и точки перегиба графика

Решение: 
1) Функция терпит бесконечный разрыв в точке х = 0.
2) Найдём критические точки второй производной:

,Критические точки отсутствуют.
3) Определим знаки второй производной на полученных интервалах: < 0.
hello_html_m1561b022.jpg
Ответ: график функции  является вогнутым на интервале  и
выпуклым на
, точки перегиба отсутствуют.

Пример 4. Найти интервалы выпуклости, вогнутости и точки перегиба графика

Решение: 
1) Функция определена и непрерывна на всей числовой прямой.

2) Найдём вторую производную: 


Найдём критические точки второй производной:

 – критическая точка

3) Определим знаки второй производной на полученных интервалах: 1,5 hello_html_m61765ba1.gif (2 – 1) > 0

hello_html_7f0e8469.jpg
При переходе через
 х = 1 вторая производная меняет знак, поэтому в данной точке существует перегиб графика.

Найдём ординату:  

Ответ: график функции выпукл на интервале   и вогнут на , в точке существует перегиб графика.

Пример 5. Исследовать функцию и построить ее график.

Решение: 

1. Областью определения функции являются все значения х, кроме х = 0.

2. Функция является функцией общего вида в смысле четности и нечетности.

3. Точки пересечения с координатными осями: c осью Ох: y = 0; x =

с осью Оу: x = 0; y – не существует.

4. Находим точки экстремума функции.

; y = 0 при х = 2, у = при х = 0.

y > 0 при х (, 0) – функция возрастает, y < 0 при х (0, 2) – функция убывает,

у > 0 при х (2, + ) – функция возрастает.

Таким образом, точка (2, 3) является точкой минимума.hello_html_1d6b91c9.jpg

Для определения характера выпуклости/вогнутости функции находим вторую производную.

> 0 при любом х 0, следовательно, функция вогнутая на всей области определения.

6. Построим график функции.

Пример 6. Исследовать функцию и построить график.

Решение: проведём исследование функции:
1) Функция определена и непрерывна на всей числовой прямой,D(f)=R .

значит, данная функция не является четной или нечетной. Функция непериодическая.

2) Точки пересечения графика с координатными осями, интервалы знакопостоянства функции.
График  проходит через начало координат.
С осью ox: Определим знаки : < 0.
hello_html_3bd04365.jpg

>0, если , < 0, если.

4) Возрастание, убывание, экстремумы функции.  – критические точки.
Определим знаки :< 0
hello_html_m1d1a2e40.jpg
 возрастает на  и убывает на .
В точке x = 3 функция достигает максимума:  

5) Выпуклость, вогнутость, перегибы графика.
  – критические точки.
Определим знаки : < 0
hello_html_m15d97516.jpg
График функции является выпуклым на  и вогнутым на. 
В обеих критических точках существуют перегибы графика.
hello_html_m4f092b07.png

6) Найдем дополнительные точки:

x

1,5

1

1

3,5

4,5

f(x)

4,6

1,3

0,7

5,4

11,4


Выполним чертёж.

2)Решить задание ( по примерам):

  1. Найдите вторые производные функций :

а) , б) , в) , г) ,

д) , е) , ж) , з) .

  1. Найти интервалы выпуклости, вогнутости и точки перегиба графика функции .

  2. а) Найти интервалы выпуклости, вогнутости и точки перегиба графика

б) Найти интервалы выпуклости, вогнутости и точки перегиба графика

  1. Найти интервалы выпуклости, вогнутости и точки перегиба графика

  2. Исследовать функцию и построить ее график.

  3. Исследовать функцию и построить график.

3)Решить задание:

  1. Найдите вторые производные функций :

а) , б) , в) , г) ,

д) , е) , ж) , з) .

и) к) л)

  1. Найти интервалы выпуклости, вогнутости и точки перегиба графика функций:

а) , б) , в) , г) ,

д) , е) , ж) , з) .

и) к) л)

  1. Исследовать функцию и построить график.

  2. Исследовать функцию и построить график.

  3. Исследовать функцию и построить график.

  4. Исследовать функцию и построить график.

  5. Исследуйте функцию f(х) = х3 3х + 5 и постройте ее график.

Инструкционная карта

ПР № 57 «Вычисление первообразных элементарных функций».

Задание:

1)Перепишите и заполните пропуски:

Пример 1. Найти общий вид первообразных данных функций :

1) f(x) = x2  cosx;   2) f(x) = 3; 3) f(x) = 10 sinx;  4) f(x) = 2sin 4x; 5) f(x) = 5x4x2  ; 6) f(x) = (3x – 1)2; 7) f(x) =. .

Решение: 1) F(x) = x3/3 – sinx  C; 2) F(x) =3x  C; 3) F(x) = 10  C; 

4) F(x) = 1/2 cos…x + C;5) F(x) =   x3/3 – 2;  6) F(x) = (3x – 1)3/… + C; 7) F(x) = /… + C.

Ответ: 1) F(x) = x3/3 – sinx  C; 2) F(x) =3x  C; 3) F(x) = 10cosx  C; 

4) F(x) = 1/2 cos 4x + C;5) F(x) = x5  x3/3 – 2;  6) F(x) = (3x – 1)3/9 + C; 7) F(x) =  /3 + C.

Пример 2. Для функции  f(x) = 4x + 1/x2 найти первообразную, график которой проходит через точку M(– 1; 4).

Решение: F(x) = 2x2– 1/x + C. , F(x) = 2()2– 1/() + C = 2 C = 4, 4 = 3 + C, C = …

Ответ: F(x) = 2х2 – 1/х + 1.

Пример 3. Докажите , что функция F(x) является первообразной для функции f(x).

a) f(x) = 2x; F(x) = x2 , б) f(x) = – sin x; F(x) = сos x ,
в) f(x) = 6x2 + 4; F(x) = 2x3 + 4x, г) f(x) = 1/cos2 x; F(x) = tg x .

Решение: a) f(x) = 2x; F(x) = x2 , F (x)= (x2) = … = f(x);

б) f(x) = – sin x; F(x) = сos x , F (x)= (cos x) = – … = f(x);

в) f(x) = 6x2 4; F(x) = 2x3 4x, F (x)= (2x3 4x) = …x2 4 = f(x);

г) f(x) = 1/cos2 x; F(x) = tg x , F (x)= (tg x) = 1/cos2 x= f(x).

Ответ: F(x) является первообразной для f(x). Пример 4. Найдите первообразные функций: a) f(x) = x4 x2 x ; б) g(u) = ;

в) h(x) = (x3 + 1)2 ; г) v(x) = cos (5x ). Решение: Для нахождения первообразных функций воспользуемся таблицей первообразных. а) x5/5 - одна из первообразных функции х4; x3/3 - одна из первообразных функции х2; x2/2 - одна из первообразных функции х; х - одна из первообразных функции 1. По правилу 1 нахождения первообразных F(x) =   - первообразная функции f(х);

б) функцию g(u) запишем в виде g(u) =  u - 1/3 u3/2.

3/2u2/3 - одна из первообразных функции u-1/3; 2/5u5/2 - одна из первообразных функции u3/2; 

G(x) = 3/2u2/3  2/5u5/2  - первообразная функции g(u);

в) h(x) = (x3 + 1)2 = x6 2x3 .

x7/7 - одна из первообразных функции х6; x4/4 - одна из первообразных функции х3; х - одна из первообразных функции 1.

По правилам 1 и 2 нахождения первообразных H(x) = …  1/2x4C - первообразная функции h(х);

г) v(x) = cos (5x ) , sinu - одна из первообразных функции cosu; V(x) = … sin(5x)  C - первообразная функции v(х).

Ответ: a) F(x) =   ; б) G(x) = 3/2u2/3  2/5u5/2  в) H(x) = 1/7x7  1/2x4xC; г) V(x) = 1/5 sin(5x)  C.

Пример 5. Найдите первообразную: a) g(x) = (47x)5 ; б) g(x)= x 2; в) t(x) = (5+ 2x)3.

Решение: a) G (x) = ; б) G(x) = ; в) T(x) = .


Ответ: a) G (x) = ; б) G(x) = ; в) T(x) = .

2)Решить задание ( по примерам):

  1. Найти общий вид первообразных данных функций :

1) f(x) = 4x3  2cosx;   2) f(x) = 5; 3) f(x) = 5 sinx;  4) f(x) = 3sin 4x;
5)
 f(x) = 6x5x4  ; 6) f(x) = (4x – 1)2; 7) f(x) =. .

  1. Для функции  f(x) = 6x + 1/x2 найти первообразную, график которой проходит через точку M(– 1; 8).

  2. Докажите , что функция F(x) является первообразной для функции f(x).

а)f(x) = 4x; F(x) = 2x2 , б) f(x) = – 2sin x; F(x) = 2сos x ,

в) f(x) = 6x2 7; F(x) = 2x3 7x, г) f(x) = 1/sin2 x; F(x) = ctg x .

  1. Найдите первообразные функций:
    a) f(x) = 10x4 4x2 2x ; б) g(u) = ; в) h(x) = (x3 3)2 ; г) v(x) = cos (7x ).

  2. Найдите первообразную: a) g(x)= (45x)5 ; б) g(x)= 2x- 2; в) t(x) = (54x)5.

3)Решить задание:

  1. Найдите первообразную функции f(x) = 4x3– 3x2 , график которой проходит через

точку M(–1; 2).

  1. Для функции f(х) = еx найти первообразную, график которой проходит через точку М(0; 2).

  2. Для функции y = –1–2x2 найдите первообразную, график которой проходит через

точку М(–3; 12).

  1. Известно, что F1, F2, F3– первообразные для f(x) = 4x3 –3x2 на R, графики которых проходят через точки M(–1; 2), N(1; 4), K(2; 5) соответственно. Перечислите, в каком порядке (сверху вниз) графики этих функций пересекают ось ординат?

  2. Материальная точка движется прямолинейно со скоростью v(t) = 12t + 4. Найдите закон движения точки, если в момент времени t = 1c пройденный путь составил 12 м.

  3. Найдите наименьшее значение первообразной функции y = 2x + 4, проходящей через

точку (2; 8) .

  1. Для функции f(х) = найти первообразную, график которой проходит через точку М(1;3).

  2. Найти общий вид первообразной для функции f:

а)f(x) = 1,б) f(x) = х+1,в) f(x) = х9 + 3, г) f(x) =3х32 ,д) f(x) = 5х + ,

е) f(x) = 1+ , ж) f(x) = 4 + 2cosx, з) f(x) =sin2x + x, и) f(x) = sinx + cosx.

  1. При каких значениях k и С функции kcosx + x - 4 является первообразной функции 3sinх + 1? 

  2. Найдите первообразную функции 2cosx, если график этой первообразной проходит

через точку М .

  1. При каком значении k и С функция   является первообразной функции  ?

  2. Найдите одну из первообразных функции f:

а) f(x) = 2х5 ,б) f(x) = 3х3+2х1, в) f(x) = 3 cosx 4 sinx ,г) f(x) = (х+1)4, д) f(x) = ,
е)
f(x) = sin(2x+3),ж) f(x) = cos(3x+4),з) f(x) = .

  1. Для функции f найдите первообразную, график которой проходит через точку М:

 f(x) =

  1. Найдите общий вид первообразной для функции f(x) = 2 cosx.

  2. Найдите общий вид первообразной для функции(x) = + 3 cosx.

  3.  Для функции f найдите первообразную, график которой проходит

через точку M: f(x) = 3

  1. Докажите, что функция F является первообразной для функции f на множестве R:

а) F(x) = х4-3 , f(x) = 4х3, б) F(x) = 5х- cosx, f(x) = 5+ sinx.

  1. Докажите, что функция F является первообразной для функции f на множестве R:

а) F(x) = 4хх3 , f(x) = 4 2, б) F(x) = 0,5 sinx, f(x) = cosx.

  1. Найдите первообразные функций: а) f(x) =,б)f(x) = 7 sinx, в) f(x) =г)f(x) =12cosx.

  2. При каком значении к и С функция является первообразной функции 5cosх + 2х?



Инструкционная карта

ПР № 58 «Вычисление площадей с помощью интегралов».

Задание:

1)Перепишите и заполните пропуски:

Пример 1. а)Вычислить площадь фигуры, ограниченной линиями у = х2 2, у = 0, х = 2, х = 1.

Решение: Выполним чертеж (обратите внимание, что уравнение у = 0  задает ось  ОХ): Штриховать криволинейную трапецию я не буду, здесь очевидно, о какой площади идет речь. Решение продолжается так:hello_html_m6ea081fc.png

На отрезке[– 2;1]    график функции у = х2 2  расположен над осью ОХ, поэтому:


Ответ: S = 9 eд2.

б)Вычислить площадь фигуры, ограниченной линиями у =  , х = 1  и координатными осями.

Решение: Выполним чертеж: Если криволинейная трапеция расположена под осью OX (или, по крайней мере, не выше данной оси), то её площадь можно найти по формуле: S =  .
В данном случае:
hello_html_m521974c9.png

Ответ: 

Пример 2.а)Найти площадь плоской фигуры, ограниченной линиями  у = 2х , у =  .hello_html_m34e0aa0e.png

Решение: Сначала нужно выполнить чертеж. Найдем точки пересечения параболы у = 2х   и

прямой у =   . Решаем уравнение: =  , 3х = 0, х(3) = 0,

х1 = …, х2 = ...

Значит, нижний предел интегрирования а = 0, верхний предел интегрирования b = 3 . x = a ,x = b , можно найти по формуле: S = .

Искомая фигура ограничена параболой y = 2х   сверху и прямой у =    снизу.
На отрезке
[0;3]  2х  , по соответствующей формуле

Ответ: S = 4,5 eд2.  . б)Вычислить площадь фигуры, ограниченной линиями , y = x  , y = 0  , x = 3 .

Решение: Сначала выполним чертеж: Площадь фигуры считается с помощью двух определенных интегралов. Действительно:hello_html_m2509795.jpg

1) На отрезке [– 1;1]  над осью OX расположен график прямой y = x   ;

2) На отрезке [1;3]   над осью OX  расположен график гиперболы

Совершенно очевидно, что площади можно (и нужно) приплюсовать, поэтому:

Ответ: .hello_html_dfee4aa.png

Пример 3.a) Вычислить площадь фигуры, ограниченной линиями

 ,2x  .


Решение: Представим уравнения в виде и выполним поточечный чертеж:
Из чертежа видно, что верхний предел у нас «хороший»:
  b = 1.
Найдем точки пересечения прямой
    и параболы
Для этого решаем уравнение:
3x2 = 2x 3x2 2x

D = 4 12 = …, = 4, x1 = , x2 = ... Действительно,a = .

На отрезке по соответствующей формуле: Ответ: .hello_html_3f5273e5.png

б)Найти площадь фигуры, ограниченной линиями y =  , y = 2x  .

Решение: Выполним чертеж:
На отрезке по соответствующей формуле:


Ответ: S = 10 eд2.  .

Пример 4.Найти площадь фигуры, ограниченной линиями x , xy = 3 .hello_html_m2e7c3126.png

Решение: Выполним чертеж . На отрезке  , по соответствующей формуле:
Ответ:  .
hello_html_78b890fb.jpg

Пример 5.a) Найти площадь фигуры, ограниченной параболой у = х2 +10 и касательными к этой параболе, проведёнными из точки (0;1). 

Решение: Неизвестна абсцисса точки касания х = а. Чтобы её найти, составим уравнение касательной:  y = f (x0) .

Имеем f(x) = x2 f (x) = 2x;значит, f(a) = a2 f (a) = 2a; уравнение касательной имеет вид:

y = a2 2 a(x ) = a2 2 ax ;

Уравнение касательной y = (1)

По условию касательная должна проходить через точку (0;1), то есть координаты точки (0;1) должны удовлетворять уравнению (1):

1 = 2a0 ; , a1 = a2 = ...

Подставим найденные значения в уравнение (1):


Если a =  то y = 9 10 Если a = 3 , то y =  .

Получили два уравнения касательных y =  . Параболы y = х2 + 10 они касаются в точках А(3;19) и В(3;19).

Найдём площадь фигуры DACB: SDACB = 2SDCB ,


hello_html_f9ffd32.gif






SDACB = 2 9 = ...

Ответ: 18.

б) Вычислить площадь фигуры, ограниченной линиями hello_html_35881468.jpg

у = 4/x, y = х, х = 4.

Решение: SABC = SMBAD SMBCD;

SMBAD = 1/2(MB ) MD = = 1/2 (2 ) 2 = 6;




hello_html_450e27c6.gif


Ответ: 6 – 4ln2.


2)Решить задание ( по примерам):

  1. а)Вычислить площадь фигуры, ограниченной линиями .

б)Вычислить площадь фигуры, ограниченной линиями и координатными осями.

  1. а)Найти площадь плоской фигуры, ограниченной линиями  .

б)Вычислить площадь фигуры, ограниченной линиями.

  1. a) Вычислить площадь фигуры, ограниченной линиями .

б)Найти площадь фигуры, ограниченной линиями y =  , y = 2x  .

  1. a)Найти площадь фигуры, ограниченной линиями  .

б) В каком отношении парабола делит площадь четырёхугольника, вершины которого находятся в точках с координатами (0;0); (2;0); (0;6); (2;6)? 

  1. a)Найти площадь фигуры, ограниченной параболой и касательными к этой параболе, проведёнными из точки (0;1). 

б) Вычислить площадь фигуры, ограниченной линиями .

3)Решить задание:

  1. a)Вычислить площадь фигуры, ограниченной линиями .

б)Вычислить площадь фигуры, ограниченной линиями и координатными осями.

  1. а)Найти площадь плоской фигуры, ограниченной линиями  .

б)Вычислить площадь фигуры, ограниченной линиями.

  1. a)Вычислить площадь фигуры, ограниченной линиями.

б)Найти площадь фигуры, ограниченной линиями .

  1. a)Найти площадь фигуры, ограниченной линиями.

б) В каком отношении парабола делит площадь четырёхугольника, вершины которого находятся в точках с координатами (0;0); (2;0); (0;6); (2;6)? 

  1. a)Найти площадь фигуры, ограниченной параболой и касательными к этой параболе, проведёнными из точки (0;1). 

б) Вычислить площадь фигуры, ограниченной линиями .

  1. Найти площадь фигуры, ограниченной функцией и осями координат.

  2. Найти площадь фигуры, ограниченной функциями и касательной к этой параболе, проведенной в точке (1/2;3/4).

  3. Найти площадь фигуры, ограниченной линиями.

  4. Найти площадь фигуры, ограниченной линиями.

  5. Найти площадь фигуры, ограниченной линиями.

  6. Найти площадь фигуры, ограниченной линиями.

  7. Найти площадь фигуры, ограниченной линиями..

  8. Найти площадь фигуры, ограниченной линиями .

  9. Найти площадь фигуры, ограниченной линиями.

  10. Найти площадь фигуры, ограниченную линиями .

  11. Найти площадь фигуры, ограниченной линиям .

  12. Вычислите площадь фигуры, ограниченной параболой и осью абсцисс.

  13. Вычислите площадь фигуры, ограниченной прямыми , .

  14. Вычислите площадь фигуры, ограниченной прямой и параболой .

  15. Вычислите площадь фигуры, ограниченной линиями и .


Инструкционная карта

ПР № 59 «Решение задач на подсчет числа размещений, перестановок, сочетаний. Решение задач на перебор вариантов».

Задание:

1)Перепишите и заполните пропуски:

Пример 1. а) Сколько трехзначных чисел можно записать, используя цифры 1,3,6,7,9, если каждая их них может быть использована в записи только один раз?

Решение: по формуле получаем: способов.

Ответ:60.

б) Из 20 учащихся надо выбрать старосту, его заместителя и редактора газеты. Сколькими способами это можно сделать?

Решение: по формуле получаем: способов.

Ответ:6840.

в)Сколькими способами четверо юношей могут пригласить четырех из шести девушек на танец?

Решение: Два юноши не могут одновременно пригласить одну и ту же девушку. И варианты,

при которых одни и те же девушки танцуют с разными юношами, считаются разными, поэтому:

Ответ: 360.

Пример 2. а)Сколькими способами можно представлять друг с другом цифры 1, 2, 3, 4?

Решение: Р4 = 4!= = …

Ответ: 24.

б)За столом пять мест. Сколькими способами можно расставить пятерых гостей?

Решение: Р5 = 5! =

Ответ:120.

в)Сколькими способами можно выложить в ряд красный, черный, синий и зеленый шарики?

Решение: На первое место можно поставить любой из четырех шариков (4 способа), на

второе – любой из трех оставшихся (3 способа), на третье место – любой из

оставшихся двух (2 способа), на четвертое место – оставшийся последний шар.

Всего 4 · 3 · 2 · 1 = 24 способа. Р4 = 4! = 1 · 2 · 3 · 4 = 24. Ответ: 24 способа.

г)Сколькими способами можно переставить буквы слова «ананас»?

Решение: всего букв 6. Из них одинаковы n1«а»=3, n2«н»=2, n3«с»=1. Следовательно, число различных перестановок равно

Ответ:60.

Пример 3. а) Из 15 членов туристической группы надо выбрать трех дежурных. Сколькими способами можно сделать этот выбор?

Решение: каждый выбор отличается от другого хотя бы одним дежурным. Значит, здесь речь идет о сочетаниях из 15 элементов по 3. Следовательно, по формуле получаем

Ответ:455.

б) На полке стоит 12 книг: англо-русский словарь и 11 художественных произведений на английском языке. Сколькими способами читатель может выбрать 3 книги, если :

1) словарь нужен ему обязательно; 2) словарь ему не нужен?

Решение:

1) 2)

Ответ: 1) 55,2) 165.

в) Учащимся дали список из 10 книг, которые рекомендуется прочитать во время каникул. Сколькими способами ученик может выбрать из них 6 книг?


Решение: Выбор 6 из 10 без учёта порядка: способов.

Ответ: 210 способов.

г) Сколько трехкнопочных комбинаций существует на кодовом замке (все три кнопки нажимаются одновременно), если на нем всего 10 цифр.

Решение: Так как кнопки нажимаются одновременно, то выбор этих трех кнопок – сочетание. Отсюда возможно вариантов.

Ответ:120.

Пример 4. а)Сколько двузначных чисел можно составить из цифр 1,4,7, используя в записи числа каждую из них не более одного раза?

Решение: для того, чтобы не пропустить и не повторить ни одно из чисел, будем записывать их в порядке возрастания. Сначала запишем числа, начинающиеся с цифры 1, затем с цифры 4, и, наконец, с цифры 7: 14, 17, 41, 47, 71, 74.

Ответ: 6.

б) На завтрак Вова может выбрать плюшку, бутерброд, пряник или кекс, а запить их он может кофе, соком или кефиром. Из скольких вариантов завтрака Вова может выбирать?

Решение: Составим таблицу:

плюшка бутерброд пряник кекс

кофе

КП

КБ

КПР

КК

сок

СП

СБ

СПР

СК

кефир

КЕП

КЕБ

КЕПР

КЕК


В ней три строки и четыре столбца, они образуют 12 клеток. Так как выбор еды и напитка происходит независимо, то в каждой клетке будет стоять один из возможных вариантов завтрака и, наоборот, любой вариант завтрака будет записан в одной из клеток. Значит, всего вариантов столько же, сколько клеток в таблице.

Ответ: 12.

Пример 5. а) Имеются 10 различных книг, три из которых – справочники. Сколькими способами

можно расставить эти книги на полке так, чтобы все справочники стояли рядом?

Решение: Т.к. в справочники должны стоять рядом, то будем рассматривать их как одну книгу. Тогда на полке надо расставить 10 – 3 + 1= … книг. Это можно сделать P8 способами. Для каждой из полученных комбинаций можно сделать P3 перестановок справочников.

Поэтому число способов расположения книг на полке равно произведению:

P8 · P3 = 8! · 3! = 40320 · 6 = ...

Ответ: 241920.

б) Сколько всего существует результатов опыта, заключающегося в подбрасывании двух одинаковых игральных костей?

Решение: Формула числа сочетаний из m элементов по n элементов с повторениями имеет вид:

,
Ответ: 21.

2)Решить задание ( по примерам):

  1. а) Сколько трехзначных чисел можно записать, используя цифры 1,2,4,6,7,9, если каждая их них может быть использована в записи только один раз?

б) Из 15 учащихся надо выбрать старосту, его заместителя и редактора газеты. Сколькими способами это можно сделать?

в)Сколькими способами четверо юношей могут пригласить четырех из пяти девушек на танец?

  1. а)Сколькими способами можно представлять друг с другом цифры 1, 2, 3, 4,5?

б)За столом семь мест. Сколькими способами можно расставить семерых гостей?

в)Сколькими способами можно выложить в ряд красный, черный, синий, белый и зеленый шарики?

г)Сколькими способами можно переставить буквы слова «Миссисипи»?

  1. а) Из 25 членов туристической группы надо выбрать трех дежурных. Сколькими способами можно сделать этот выбор?

б) На полке стоит 15 книг: англо-русский словарь и 14 художественных произведений на английском языке. Сколькими способами читатель может выбрать 3 книги, если :
1) словарь нужен ему обязательно; 2) словарь ему не нужен?

в) Учащимся дали список из 10 книг, которые рекомендуется прочитать во время каникул. Сколькими способами ученик может выбрать из них 7 книг?

г) Сколько четырехкнопочных комбинаций существует на кодовом замке (все три кнопки нажимаются одновременно), если на нем всего 10 цифр.

  1. а)Сколько двузначных чисел можно составить из цифр 1,4,5,7, используя в записи числа каждую из них не более одного раза?

б) На завтрак Вова может выбрать бутерброд, пряник или кекс, а запить их он может кофе, соком или кефиром. Из скольких вариантов завтрака Вова может выбирать?

  1. а)Имеются 10 различных книг, 6 из которых – справочники. Сколькими способами

можно расставить эти книги на полке так, чтобы все справочники стояли рядом?
б)
Сколько всего существует результатов опыта, заключающегося в подбрасывании трех одинаковых игральных костей?

3)Решить задание:
подсчет числа размещений, перестановок, сочетаний.

  1. «Вороне где-то Бог послал кусочек сыра», колбасы, хлеба и шоколада. «На ель Ворона взгромоздясь, позавтракать совсем уж было собралась, да призадумалась»: если есть кусочки по очереди, то из скольких вариантов придется выбирать?

  2. Сколькими способами можно из 25 учащихся выбрать 5 для участия в школьном марафоне?

  3. Сколькими способами могут быть распределены золотая и серебряная медали по итогам первенства по футболу, если число команд 12?

  4. В классе 7 человек успешно занимаются математикой. Сколькими способами можно выбрать из них двоих для участия в математической олимпиаде?

  5. Из 12 солдат нужно в разведку послать 5. Сколькими способами это можно сделать?

  6. Сколько пятизначных чисел можно составить, используя только цифры 3 и 5?

  7. «Проказница Мартышка, Осел, Козел и косолапый Мишка затеяли сыграть квартет». Сколькими способами они могут распределить четыре имеющихся у них инструмента?

  8. «Проказница Мартышка, Осел, Козел и косолапый Мишка затеяли сыграть квартет». На складе 12 музыкальных инструментов. Мишке поручили принести со склада 8 любых инструментов. Сколько вариантов выбора есть у мишки?

  9. Гера, Афина и Афродита попросили Париса не только назвать самую красивую из них, но и указать, кто «на втором и третьем местах». Сколько есть вариантов ответа?

  10. В магазине «Филателия» продается 8 различных наборов марок, посвященных «Дню Победы». Сколькими способами можно сформировать из них 3 набора?

  11. Сколько существует способов составить расписание уроков на один день из 6 предметов?

  12. Алфавит племени тумба-юмба состоит из букв А, У, С. Словом является любая последовательность из 4 букв. Сколько слов в языке этого племени?

  13. Сколькими способами можно выложить в ряд красный, зеленый, черный, синий кубики?

  14. Из колоды в 36 карт вынимают 5 карт. Найдите число всех возможных вариантов выбора.

  15. В классе 27 учеников, из которых нужно выбрать троих: первый ученик должен решить задачу, второй – сходить за мелом, третий – пойти дежурить в столовую. Сколькими способами это можно сделать?

  16. Сколькими способами можно из 6 человек составить комиссию, состоящую из двух человек?

  17. В соревновании участвуют 10 человек. Сколькими способами могут распределиться между ними места?

  18. Сколькими способами можно расставить на полке 4 различные книги?

  19. Сколько различных словарей необходимо переводчику, чтобы он мог переводить с любого из 5 языков – русского, английского, немецкого, французского, испанского – на любой другой из этих языков?

  20. Пять человек обменялись друг с другом фотографиями. Сколько всего фотографий было?

  21. На плоскости отмечены 6 точек. Каждые две точки соединили отрезком. Сколько получилось отрезков?

  22. Сколькими способами можно переставить 5 различных геометрических фигур?

  23. Сколькими способами можно выбрать гласную и согласную буквы из слова «здание»?

  24. За свои рисунки ученик получил две положительные оценки. Какими они могут быть? Сколько вариантов?

  25. Сколько флагов можно составить из трех разных цветов, если имеются полосы синего, белого, красного цветов?

  26. В понедельник в пятом классе 5 уроков. Сколькими способами можно составить расписание на понедельник?

  27. Из десяти учащихся надо выбрать старосту, физорга и культорга. Сколькими способами это можно сделать?

  28. В соревновании участвуют 10 человек. Сколькими способами могут распределиться между ними места?

  29. У одного человека 7 книг по математике, а у второго – 9. Сколькими способами они могут обменять друг у друга две книги на две книги.

  30. Имеется пять различных стульев и семь рулонов обивочной ткани различных цветов. Сколькими способами можно осуществить обивку стульев.

  31. На памятные сувениры в «Поле Чудес» спонсоры предлагают кофеварки, утюги, телефонные аппараты, духи. Сколькими способами 9 участников игры могут получить эти сувениры? Сколькими способами могут быть выбраны 9 предметов для участников игры?

  32. Сколькими способами можно расставить на шахматной доске 8 ладей так, чтобы на одна из них не могла бить другую?

  33. Сколько может быть случая выбора 2 карандашей и 3 ручек из пяти различных карандашей и шести различных ручек?

  34. На ферме есть 20 овец и 24 свиньи. Сколькими способами можно выбрать одну овцу и одну свинью? Если такой выбор уже сделан, сколькими способами можно сделать его еще раз?

  35. Сколько существует четных пятизначных чисел, начинающихся нечетной цифрой?

  36. В 9 классе учатся 7 учащихся, в 10 - 9 учащихся, а в 11 - 8 учащихся. Для работы на пришкольном участке надо выделить двух учащихся из 9 класса, трех – из 10, и одного – из 11 . Сколько существует способов выбора учащихся для работы на пришкольном участке?

  37. Сколько наборов из семи пирожных можно составить, если в продаже имеется четыре сорта пирожных?

перебором вариантов:

  1. 1. Запишите все двузначные числа, в записи которых используются только цифры 3, 5, 7, 9. Сколько двузначных чисел можно записать, если использовать при записи числа каждую цифру только один раз?

  2. 2. В четверг в первом классе должно быть три урока: русский язык, математика и физкультура. Сколько различных вариантов расписания можно составить на этот день?
    Указание: Перебирая варианты введите обозначения:
    Р – русский язык, М – математика, Ф – физкультура.

  3. 3. Саша выбрал в библиотеке 5 книг, но одновременно можно взять только две книги. Сколько вариантов выбора двух книг есть у Саши?

  4. 4. Школьники из Волгограда собрались на каникулы поехать в Москву, посетив по дороге Нижний Новгород. Сколькими различными способами могут ребята осуществить свое путешествие, если из Волгограда в Нижний Новгород можно отправиться на теплоходе ли поезде, а из Нижнего Новгорода в Москву – на самолете, теплоходе, поезде или автобусе?

  5. 5. Девять школьников, сдавая экзамены по математике и английскому языку, получили отметки «4» и «5». Можно ли утверждать, что по крайней мере двое из них получили по каждому предмету одинаковые отметки?

  6. 6. Сколько существует двузначных чисел, у которых первая цифра больше второй?

  7. 7. Несколько стран в качестве символа своего государства решили использовать флаг в виде четырех вертикальных полос, одинаковых по ширине, но разных по цвету: белый, синий, красный, зеленый. У каждой страны свой, отличный от других, флаг.
    а) Сколько всего стран могут использовать такую символику?
    б) Сколько всего стран могут использовать такую символику с верхней белой полосой?
    в) Сколько всего стран могут использовать такую символику с нижней зеленой полосой?
    г) Сколько всего стран могут использовать такую символику с синей и красной полосами, расположенными рядом?

  8. 8. а) Сколько двузначных чисел можно составить из цифр 1, 3, 5, 7, 9?

б) Сколько среди них чисел, кратных 5? в) Сколько среди них чисел, кратных 3?

Инструкционная карта

ПР № 60 «Вычисление числовых характеристик дискретной случайной величины».

Задание:

1)Перепишите и заполните пропуски:

Пример 1. а) М(Х) = 4, М(Y) = 5, Z = 3X – 2Y, М(Z) = ?

Решение: М(Z) = 34 – 2 5 = 12 – 10 = …,

б) D(Х) = 12, D (Y) = 10, Z = 2X + Y, D (Z) = ? D (Х – Y) =?

Решение: D (Z) = 22 12 + 10 = 412 + 10 = 48 + 10 = …, D (Х - Y) = 12 + 10 = …,

Пример 2. Найдите числовые характеристики Х и Y:

Х

10

20

P

0,3

0,7

Y

30

40

60

P

0,5

0,2

0,3


Решение:

М(Х) = р1х1 + р2х2 = 0,3 10 + 0,7 20 = 3 + 14 = …,

D(Х) = р1х12 + р2х22 – (р1х1 + р2х2)2 = 0,3100 + 0,7400 – 289 =30 + 280 – 289 = …,

(Х) = = …,

М(Y) = р1у1 + р2у2 + р3у3 = 0,5 30 + 0,2 40 + 0,3 60 = 15 + 8 + 18 = …, D(Y) = 0,5 900 + 0,2 1600 + 0,3 3600 – 1681= 450 + 320 + 1080 – 1681 = …,

(Y) = = = …,

Пример 3. а) По данным статистического распределения выборки найдите моду М0 :

xi

1

3

5

7

9

ni

1

5

6

5

3


Решение: Наибольшее n = 6 для Х= 5, поэтому М0 =…,

б) По данным вариационных рядов определить медиану Ме:
1) 23679, 2) 212866 , 3) 67543, 4)2134668553.

Решение: 1) 23679, n = 5, Ме = …, 2) 212866, n = 6, Ме = (2+8):2=…,

3) 67543, n = 5, Ме = …, 4) 2134668553. n =10, Ме = (6+6):2=…,

Пример 4. a)Закон распределения случайной величины X имеет вид:

X

1

0

1

2

P

0,1

0,2

0,3

0,4

Вычислить Dx   и x .

Решение: Найдем вначале математическое ожидание случайной величины X:

Mx = .

Вычислим дисперсию Dx :Dx = .

Тогда среднее квадратическое отклонение: Ϭx = Ответ: Dx = 1, Ϭx = 1.

б) Закон распределения случайной величины X имеет вид:

X

0

1

2

P

0,1

0,2

x

Найти x. Составить функцию распределения. Вычислить: P{X > 0,7} , Mx , Dx и x .

Решение. Согласно условию нормировки имеем уравнение: 0,1   Отсюда x = … 

Далее, воспользовавшись рядом распределения, найдем:

P{X > 0,7} = P {X = 1}P{X = 2} = 0,2 0,7 = … ; Mx =

Dx = ; x = .

Ответ: x = 0,7 ; P{X > 0,7} = 0, 9; Mx Dx ; x

Пример 5. Известно, что случайная величина X, принимающая два значения  x1 = 2 и x2 = 3 , имеет математическое ожидание, равное 2,2. Построить ряд распределения случайной величины X, найти дисперсию, среднее квадратическое отклонение и составить функцию распределения.

Решение. Пусть P{X = 2} = p . Тогда, согласно условию нормировки,P{X = 3} = 1  . Используя определение математического ожидания, получим Mx = 2p . Имеем уравнение 3 , откуда находим p = …  Ряд распределения имеет вид:

X

2

3

P

0,8

0,2

Теперь вычислим дисперсию и среднее квадратическое отклонение:

Dx = ; x =  .

Согласно определению функция распределения имеет вид

Fx(x) =

Ответ: Dx ; x =   Fx(x) =

2)Решить задачи ( по примерам):

  1. а) М(Х) = 5, М(Y) = 7, Z = 3X – 2Y, М(Z) = ?

б) D(Х) = 10, D (Y) = 14, Z = 2X + Y, D (Z) = ? D (Х – Y) =?

  1. Найдите числовые характеристики Х и Y:

Х

10

30

P

0,3

0,7

Y

10

20

40

P

0,5

0,2

0,3


  1. а) По данным статистического распределения выборки найдите моду М0 :

xi

1

4

7

10

13

16

ni

1

5

3

7

2

4


б) По данным вариационных рядов определить медиану Ме:
1) 12457, 2) 123761, 3) 35621, 4) 2235448997.

  1. a)Закон распределения случайной величины X имеет вид:

X

1

0

1

2

P

0,1

0,15

0,3

0,45


X

0

1

2

P

0,2

0,3

x

Вычислить Dx и x .

б) Закон распределения случайной величины X имеет вид:

Найти x. Составить функцию распределения.
Вычислить: P{X > 0,7} , Mx , Dx и x .

  1. Известно, что случайная величина X, принимающая два значения  x1 = 2 и x2 = 3 , имеет математическое ожидание, равное 2,4. Построить ряд распределения случайной величины X, найти дисперсию, среднее квадратическое отклонение и составить функцию распределения.

3)Решить задачи :

X

1

2

P

0,2

0,8

  1. Пусть задан закон распределения случайной величины X:

Найти математическое ожидание.


  1. Найти дисперсию случайной величины X со следующим законом распределения:

    X

    2

    3

    5

    P

    0,1

    0,6

    0,3

  2. Математическое ожидание и дисперсия СВ Х соответственно равны 0,5 и 5. Найти математическое ожидание и дисперсию случайной величины 2X 3 .

  3. Найти математическое ожидание числа очков, выпадающих при бросании игральной кости.

  4. Независимые дискретные случайные величины заданы законами распределения в табличной форме:

xi

1

2

4


yi

0,5

3

pi

0,2

0,5

0,3

pi

0,4

0,6

Найти математическое ожидание случайных величин Z = 2Х + 3Y и  U = 5XY.

  1. Функция распределения ДСВ Х имеет вид

hello_html_m1caf49ec.png

Найти: hello_html_16e94671.png





Просмотрено: 0%
Просмотрено: 0%
Скачать материал
Скачать материал "Практические работы обучающихся по дисциплине Математика для специальности Технология машиностроения."

Получите профессию

Бухгалтер

за 6 месяцев

Пройти курс

Рабочие листы
к вашим урокам

Скачать

Выбранный для просмотра документ ПР-ТЕХМАШ-2КУРС.docx

Инструкционная карта

ПР № 1 «Выполнение действий над матрицами». 

Цель:

  • повторить понятия: матрица;

  • формирование умения решать простейшие задачи на расчет матриц;

Теоретическая часть. Лекции.

Практическая часть. 1) Опорный конспект.

Пример 1. Даны матрицы

и .Построить матрицу С = 2А – 3В + АТ.

Решение:

-+=.

Пример 2.Найти произведение матрицы

на матрицу .

Решение:


т.е. .

Пример 3.Найти произведение С матрицы А на вектор – столбец .

.

Решение: Умножение возможно, т.к. вектор можно рассматривать как матрицу, имеющую 3 строки и 1 столбец, а матрица А имеет 3 столбца, и число ее столбцов равно числу строк вектора . Произведение С = А будет иметь порядок 4.1, т.е. будет вектором-столбцом с элементами с11, с21, с31, с41.

с11 = 1.4-1.2+0 = 2; с21 = 0.4+2.2-4.1 = 0;

с31 = 4+2 = 6; с41 = -4+4 = 0.

.

Таким образом, если умножение возможно, то произведение матрицы на вектор будет вектором.

Пример 4. Найти произведение AB, если

A= , B =.

Решение.

АВ = = .

Пример 5. Пусть , .

Найдем произведения AB:

.

2)Задание:

  1. Вычислить (3А 2В)·С, если hello_html_m143cc1a2.gif, hello_html_6ee9575b.gif, hello_html_m156e681.gif.

  2. Найти матрицу Х, если 3А+2Х=В, где hello_html_51ce4ad3.gif, hello_html_8bd19e7.gif.

  1. Даны матрицы А и В. Найти С = 2А + 3В. .

  2. Выполнить действия над матрицами.

а) ,

б) .

  1. Даны две матрицы

  1. Построить матрицу С.

  2. Найти произведение матриц А и С.

  3. Найти произведение матриц В и С.


вар

С

вар

С

1

3В АТ

11

2АВ2АТ

2

А2ВВТ

12

3А4В Т

3

В2АТ

13

А3В4АТ

4

3А ВВТ

14

4В ВТ

5

А 3В2АТ

15

3А ВАТ

6

ВТТ

16

2А5В Т

7

2АВ АТ

17

3А В Т

8

2А+3ВТ

18

4А2В АТ

9

4АВ Т

19

3В2АТ

10

4ВВТ

20

3В ВТ

Инструкционная карта

ПР № 2 «Вычисление определителей второго и третьего порядка».

Цель:
- сформировать навык вычисления определителей 2-го порядка;
- познакомить обучающихся с правилом треугольника для вычисления
определителей 3-го порядка и др..

Теоретическая часть. Лекции.

Практическая часть. 1) Опорный конспект.

Пример 1.а)Вычислить определитель матрицы: а) ,

б) Вычислить определитель : .

в) Вычислить определитель 4го порядка.

Решение: а)(А) = .

б) Вычислим по правилу Саррюса

= 1 · (1) . (5) + ( 2) · ( 4) ·0 + 4 · (3) ·3 0 · (1) ·3 4 · (2) · (5) (3) · (4) ·1=

= 5 + 0 36 + 0 40 12 = 83.

в) Найдем алгебраические дополнения А12, А13


= 0.

Пример 2. Вычислить определитель матрицы .

Решение: Находим миноры и алгебраические дополнения элементов 1-ой строки матрицы:


Вычисляем искомый определитель: (А) = 3.7 + (-2).(-35) + 4.(-7) = 63.

Пример 3.а)Найти обратную к матрице ,

б) Найти обратную к матрице .

Решение: а)Вычисляем алгебраические дополнения всех элементов данной матрицы:




Находим определитель матрицы А:  = 2.7 +( 1) . (10)+( 2) .11 = 2 .

Теперь записываем обратную матрицу.

Проверка:


Значит, матрица А-1 найдена верно.

б) Найти обратную к матрице

Вычислим алгебраические дополнения всех элементов данной матрицы

А11 = +( 4)=-4 А21 = ( 2)=2

А12 = 3 А22 = +1

Найдем определитель  = (А) = 1(-4)-3(-2)=-4+6=2,

Проверка:

2)Задание:

1. Контрольные вопросы

а) что такое определитель 2-го и 3-го порядка?

б) основные свойства определителей.

2. Вычислить определитель:

1) ; 2) ;

3) ; 4) ;

5) ; 6) ;

7) ; 8) ;

9) ; 10) ;

11) ; 12) ;

13) ; 14) ;

15) ; 16) .


  1. Вычислить определители:

а) hello_html_m36fef839.gif; б) hello_html_m54a3e356.gif; в) hello_html_4a34c43d.gif; г) hello_html_11c30ab7.gif; д) hello_html_2d8d71a6.gif; е) hello_html_m1543c416.gif; ж) hello_html_7a936dd7.gif;

  1. С помощью правила треугольников вычислить определители:

а) hello_html_3af4bca3.gif; б) hello_html_m11492ea1.gif; в) hello_html_c83f2f2.gif; г) hello_html_m495f8229.gif.

  1. Вычислить определитель разложением:

5.1. hello_html_33328dab.gif; 5.2. hello_html_5183d497.gif; 5.3. hello_html_1c395f7f.gif;



Инструкционная карта

ПР № 3 «Решение систем уравнений с помощью матричного метода».

Цель: сформировать навык решения систем уравнений с помощью матричного метода.

Теоретическая часть. Лекции.

Практическая часть. 1) Опорный конспект.

Пример 1. Решить систему линейных уравнений матричным методом


Решение: В матричной форме систему из n линейных уравнений c n неизвестными можно записать так: АХ=В, где А – основная матрица коэффициентов системы; Х – матрица-столбец неизвестных; В – матрица-столбец свободных членов. Умножив слева обе части равенства

АХ=В на А-1-1 существует, если detА=0), получим А-1АХ= А-1В; ЕХ= А-1В,

здесь Е – единичная матрица. Следовательно, Х= А-1В, т.е. чтобы найти решение системы n линейных уравнений с n неизвестными матричным методом, нужно матрицу, обратную матрице из коэффициентов системы, умножить на матрицу – столбец свободных членов. В результате получаем матрицу-столбец, которая и будет решением данной системы.

Найдем определитель матрицы А


Следовательно, матрица А имеет обратную матрицу.

Обратная матрица А-1 определяется по формуле


где Аij – алгебраические дополнения элементов ij данной матрицы А.

Найдем алгебраические дополнения для элементов матрицы:



.

Обратная матрица имеет вид

Необходимо сделать проверку: А-1А=Е.

, где


с11= 91+(16)(4)+1(1) =72; c12 = 92+(16)1+1(2) = 0; c13 = 93+(16)2+15 = 0;

с21=181+8(4)+(14)(1) = 0;c22 = 182+81+(14)(2) = 72; c23 = 183+82+(14)5 = 0;

с31 = 91+0(4)+9(1) = 0; c32 = 92+01+9(2) = 0; c33 = 93+02+95 = 72.

Т.е. .

Найдем теперь решение системы Х=А-1В


(Необходимо сделать проверку по исходной системе уравнений).

Ответ: х1= –2, х2= –1, х3= 4.

Пример 2. Решить систему матричным способом.

Решение:

  1. Введем обозначения:




Тогда в матричной форме система имеет вид , т.е.

А-1-обратная матрица, которая существует только тогда, когда исходная матрица А невырожденная, т.е.

  1. Найдем определитель матрицы по формуле:



Так как , то матрица А – невырожденная и обратная матрица А-1 существует и единственная.

  1. Найдем обратную матрицу по формуле: , где- присоеденненая матрица, элементы которой равны алгебраическим дополнениям элементов матрицы , и затем транспонированная.

    1. найдем алгебраического дополнения всех элементов матрицы:




Получается матрица .


    1. транспонируем матрицу (т.е. матрица AT, полученная из исходной матрицы заменой строк на столбцы)


    1. обратная матрица равна:


  1. Находим значение переменных х123:



х1 = –27, х2 = 36, х3 = –9

Ответ: х1 = –27, х2 = 36, х3 = –9.

Пример 3. Решить систему матричным способом.

hello_html_a87baa6.gif

Решение:

Решим систему линейных уравнений матричным методом.

Обозначим

Тогда данную систему можно записать в виде: АХ=В.

Т.к. матрица невырожденная (Δ= – 2), то X = A-1B.

Тогда A-1 = .

Получим X = A-1B =.

Ответ: х1 = –1, х2 = 4, х3 = 1.

2)Задание: Решить систему матричным способом:


1. 2. 3. 4.


5. 6. 7. 8.


9.

10.

11.

12.

13.

14.

15.

16.

17.

18.

19.

21.

23.

25.

20.

22.

24.

26.


Инструкционная карта

ПР № 4 «Решение систем линейных уравнений по правилу Крамера». 

Цель: сформировать навык решения систем уравнений с помощью определителя, по формулам Крамера.

Теоретическая часть. Лекции.

Практическая часть. 1) Опорный конспект.

Пример 1. Решить систему по формулам Крамера, выполнить проверку:

Решение: Имеем линейную неоднородную систему из 4 уравнений с 4 неизвестными (m=n=4)

Составим из коэффициентов при неизвестных основной определитель системы . Если 0, то согласно теореме Крамера система совместна и ее единственное решение находится по формулам : хi=, i=1,2,3,4, где i-всмогательный определитель для нахождения неизвестного хi, который получается из основного определителя системы  путем замены в нем i-го столбца столбцом свободных членов. Определители будем вычислять, образуя нули в каком - либо столбце (или строке), используя их свойство.







.

Проверка.

Подставим найденное решение в исходную систему, то есть во все четыре уравнения. Получим


Отсюда следует: решение найдено верно.

Ответ: х1 =6, х2 = 9, х3 =14, х4 = 9.

Пример 2. Решить систему по формулам Крамера.  

Решение: Решим систему по формулам Крамера.


, значит, система имеет единственное решение.










Ответ: x1 =  5, x2 =  -1, x3 =  1.

.

Пример 3. Решить методом Крамера систему уравнений:

                                              


Решение: Главный определитель этой системы

 

 

значит, система имеет единственное решение. Вычислим вспомогательные определители 

D i ( i = ), получающиеся из определителя D путем замены в нем столбца, состоящего из коэффициентов при xi, столбцом из свободных членов:







               

Отсюда x1 = D 1/D = 1, x2 = D 2/D = 2, x3 = D 3/D = 3, x4 = D 4/D = 1, решение системы - вектор С=(1, 2, 3, 1)T.

Ответ: x1 =  1, x2 =  2, x3 =  3, x4 = 1.


2)Задание: Решить систему по формулам Крамера:


1.

3.

5.

7.

2.

4.

6.

8.

9.

10.

11.

12.

13.

14.

15.

62.

17.

64.


16.

18.


19.

21.

94.


20.


22.

23. 24.


Инструкционная карта

ПР № 5 «Решение систем линейных уравнений методом Гаусса».

Цель: сформировать навык решения систем уравнений с помощью метода Гаусса.

Теоретическая часть. Лекции.

Практическая часть. 1) Опорный конспект.

Пример 1. Исследовать систему уравнений и решить ее методом Гаусса, если она совместна:

а) найти ее общее решение; б) базисное решение; в) частное решение; Сделать проверку.


Решение: Дана неоднородная линейная система из 4-х уравнений с 4-мя неизвестными (m=n=4).

1) Определим, совместна или нет система (*). Вычисляем для этого ранги расширенной и основной матриц системы: Rg(A,B) и RgA.



(привели матрицу (A,B) к матрице (A,B), имеющую ступенчатую форму).

Итак, Rg(A, B) = Rg(A, B) = 2, RgA= RgA = 2 RgA= Rg(A,B) = 2.

Следовательно система (*) совместна. Т.к. Rg A n (n = 4)  система имеет бесчисленное множество решений.

  1. Найдем все решения системы (*). Для этого перейдем к следующей эквивалентной системе

, (**)

где х1 и х3- базисные неизвестные,

х2 и х4- свободные неизвестные.

От системы (**) перейдем к другой эквивалентной системе : (***)

Решая систему (***) , как систему из 2-х уравнений с 2-мя неизвестными х1 и х3, найдем их. Из последнего уравнения имеем x3 = 1.

Тогда из первого уравнения найдем х1= 2х32 + 2х2 + x4 = 2х2 + x4.

Т.к. х2 и х4 – свободные неизвестные, то можно считать х2 = , х4 = b.

Тогда общее решение системы (***), а значит и (*) имеет вид

х1 = 2+ b ; x2 = , x3 = 1; х4= b

или

Х= = + b +

+ (*)

  1. Найдем частное решение системы (*), полагая например, =1,b = 1, тогда имеем

х1= 3, х2 = 1, х3 = 1, х4 = 1.

Проверка (по исходной системе).


общее решение найдено верно.

Частное решение, в котором все свободные переменные равны нулю, называют базисным решением: х1 = 0 , х2 = 0, х3 = 1 х4 = 0.

Ответ: а) х1 = 2+ b ; x2 = , x3 = 1; х4= b, б) х1 = 0, х2= 0, х3= 1 х4= 0,в) х1= 3, х2 = 1, х3 = 1, х4 = 1.

Пример 2. Исследовать систему и решить ее методом Гаусса, если она совместна


Решение: Дана неоднородная линейная система из 4-х уравнений с 4-мя неизвестными (m=n=4).

1) Определим, совместна или нет система (*). Вычисляем для этого ранги расширенной и основной матриц системы: Rg(A,B) и RgA.


(привели матрицу (A,B) к матрице (A,B), имеющую ступенчатую форму).

Итак, Rg(A, B) = Rg(A, B) = 4, RgA= RgA = 4 RgA= Rg(A,B) = 4. Следовательно система (*) совместна. Т.к. Rg A= n (n = 4)  система имеет единственное решение.

Найдем все решения системы (*). Для этого перейдем к следующей

эквивалентной системе.

,

где все неизвестные - базисные.

Решая систему (**), как систему из 4-х уравнений с 4-мя неизвестными, найдем x1, x2, x3, x4. Из последнего уравнения имеем x4 = 2.

Тогда из третьего уравнения найдем x3 = 26х454 = 5254= 2.

Из второго уравнения найдем x2 = 9х313x4 + 47 = 1826 + 47=44 + 47 = 3.

Из первого уравнения находим х1 = x2 + 4x39x4 + 22 = 3818 + 22 =1.

Проверка.

Подставим найденные значения неизвестных во все уравнения системы (*).

решение найдено верно.

Ответ: х1 =1, х2 = 3, х3 = 2, х4 = 2.

Пример 3. Решить однородную систему линейных уравнений и найти ее фундаментальную систему решений: (*)

Решение: Дана однородная линейная система из 3-х уравнений с 5-ю неизвестными (m=3, n=5).

Однородная система (*) всегда совместна. Определим теперь ранг основной матрицы

системы: RgA.

~ ~ = A.

(привели матрицу A к матрице A, имеющую ступенчатую форму).

RgA =Rg A= 2  Rg A  n (n=5)  система (*) имеет бесчисленное множество решений.

Найдем все решения системы (*). Перейдем к следующей эквивалентной системе уравнений.

(**),

где х1, х2 - базисные неизвестные;

х3, х4, х5- свободные неизвестные.

От системы (**) переходим к следующей эквивалентной системе

 (***)

Решая систему (***) относительно х1 и х2 найдем: х2 = .

х1 = х3 + х4 5 + 2х2 = .

Т.к. х3 , х4 , х5- свободные неизвестные, то можно считать х3 =, х4 = b, x5 = c.

Тогда общее решение системы (***), а значит и (*) есть

х1 = x2 = , х3 =, x4 = b , x5 = c или Х =

Найдем частное решение системы (*), полагая, что = 8, b = 8, c = 8

х1= 0, х2 = 12, х3 = 8, х4 = 8, х5 = 8

Проверка (по исходной системе).

.

общее решение найдено верно.

Перейдем теперь к поиску фундаментальной системы решений. Из уравнений (***) ,придавая свободным неизвестным значения х3 =1, х4 = 0, х5= 0, затем х3= 0, х4 = 1, х5 = 0, и затем

х3 = 0, х4 = 0, х5 = 1, получаем фундаментальную систему решений

х1

х2

х3

х4

х5

0


1

0

0


0

1

0


0

0

1

(0;1; 0; 0) , = (;0; 1; 0) , = (;0; 0; 1).

Тогда общее решение системы имеет вид

= с1 (0;1; 0; 0) + с2 (;0;1; 0) + с3 (;0; 0; 1)

или = (с2 с3; c1 c2 + c3; c1; c2; c3) (сравните с предыдущим).

Ответ: х1 = x2 = , x3 =, x4 = b , x5 = c

Или х = (,, b, c).

х1

х2

х3

х4

х5

0


1

0

0



0

1

0



0

0

1


2)Задание: Решить систему методом Гаусса:

1.2. 3.

4. 5. 6. 7.

8. 9. 10.

11. 12. 13.

14. 15. 16.

17. 18.

19. 20.



Инструкционная карта

ПР № 6 «Выполнение действий над комплексными числами в алгебраической форме».

Цель: формировать умение графического изображения комплексных чисел, выполнения арифметических операций с комплексными числами.

Теоретическая часть. Лекции.

Практическая часть. 1) Опорный конспект.

Пример 1. Найдите сумму и разность двух комплексных чисел : .

Решение:

=(2+3 i) + (1+2 i) = (2+1) + (3+2) i = 3 + 5 i.

= (2+3 i) – (1+2 i) = (2–1) + (3–2) i = 1 + i.

Ответ:=3 + 5 i. = 1 + i.

Пример 2. Найти произведение комплексных чисел z1 = 3+2i и z2 = –1–i.

Решение. z1z2 = (3+2i)( –1–i) = 3· (–1) +2 i · (–1) + 3(–i) +2 i · (– i) = – 3 –2 i – 3 i +2 = – 1–5i

Ответ: –1–5i.

Пример 3.Вычислите: .

Решение:

Ответ:

Пример 4. Произведите умножение комплексных чисел: а) (2 –i)(2 + i), б) (3 – 5i)2, в) (4 + 7i)2.

Решение: а) (2 –i)(2 + i) = 22 i2 = 4 + 1 = 5,

б) (3 – 5i)2 = 32 – 2 ·3 ·5 i + (5 i) 2 = 9 –30 i –25 = –16 –30 i,

в) (4 + 7i)2 = 42 + 2 ·4 ·7 i + (7 i)2=16 + 56 i – 49 = – 33 + 56 i .

Ответ: ) (2 –i)(2 + i) 5, б) (3 – 5i)2 = –16 –30 i, в) (4 + 7i)2 = – 33 + 56 i .

Пример 5. Вычислите модули заданных комплексных чисел Z = 8 + 15i .

Решение:

Ответ: 17.

Пример 6 Изобразим на комплексной плоскости числа

Z1 = 2 + i; z2 = 3i; z3 = 3 + 2i; z4 = 1 – i.

Решение:hello_html_m2b39151a.png



.






Пример 7. Решите уравнение: а),б) x2 – 4x + 29 = 0.

Решение: Имеем , , ,

.

Получаем

Извлечем квадратный корень из комплексного числа по формулам:

;

Так как , Тогда

Итак, , тогда

Где и

б) x2 – 4x + 29 = 0.

Ответ: а) ;б)

2)Задание:


Z1 = 4i , Z2 = 3 + i

Z 1= -5i , Z 2= 4 + i

Z 3= - 4 +3i , Z 4= - 2 -5i

Z3 = -7 + 2i , Z 4= -3 – 6i

2 . Вычислите модули заданных комплексных чисел

Z5 = 3 + 4i

Z5 = 8 + 6i

3. Произведите сложение и вычитание комплексных чисел:

а) (3 + 5i) + (7 – 2i).
б) (6 + 2i) + (5 + 3i). 

в) (– 2 + 3i) (7 – 2i).
г) (5 – 4i) (6 + 2i).

а) (3 – 2i) + (5 + i).
б) (4 + 2i) + (– 3 + 2i).
в) (– 5 + 2i) (5 + 2i).
г) (– 3 – 5i) (7 – 2i).

4. Произведите умножение комплексных чисел:

а) (2 + 3i)(5 – 7i). 
б) (6 + 4i)(5 + 2i).
в) (3 – 2i)(7 – i). 
г) (– 2 + 3i)(3 + 5i).

а) (1 –i)(1 + i).
б) (3 + 2i)(1 + i).
в) (6 + 4i)3i.
г) (2 – 3i)(– 5i).

5. Выполните действия:

а) (3 + 5i)2. б) (2 – 7i)2.
в) (6 +
 i)2. г) (1 – 5i)2.

а) (3 + 2i)2. б) (3 – 2i)2.

в) (4 + 2i)2. г) (5 – i)2.

6. Выполните действия:

А) (3 + 2i)(3 – 2i). 
б) (5 + i)(5 – i). 
в) (1 – 3i)(1 + 3i). 

а) (7 – 6i)(7 + 6i).
б) (4 + i)(4 – i).
в) (1 – 5i)(1 + 5i).

7. Решите уравнения:

а) x2 – 4x + 13 = 0.

б) 5x2 + 2x + 2 = 0

а) 2,5x2 + x + 1 = 0.
б) 4
x2 – 20x + 26 = 0.

8. На рисунке показано графическое изображение комплексных чисел. Перерисуйте рисунок в тетрадь. Обозначьте комплексные числа как z1, z2, zЗ. Запишите соответствующие аналитические формы.



Инструкционная карта

ПР № 7 «Выполнение действий над комплексными числами в тригонометрической форме».

Цель: формировать умение выполнения арифметических операций с комплексными числами, представленных в тригонометрической форме.

Теоретическая часть. Лекции.

Практическая часть. 1) Опорный конспект.

Пример 1. Запишите комплексные числа в тригонометрической форме:

а) ; б) ; в) ; г) .

Решение: Так как тригонометрическая форма комплексного числа имеет вид , тогда: а) В комплексном числе : .

Тогда

, поэтому

б) , где ,

в) , где ,

г) , а , то .

Поэтому


Ответ:  а);б);в) ; г).

Пример 2. Найдите тригонометрическую форму комплексного числа

.

Решение: Пусть , .

Тогда , , .

Поскольку и , , то , а

.

Следовательно, , поэтому

, где .

Ответ: , где .

Пример 3. Используя тригонометрическую форму комплексного числа, произвести указанные действия :

;

Решение: Представим сначала каждое из чисел в тригонометрической форме: 1 i = 2 ( cos (5 /3) + i sin (5 /3)) ; + i = 2 ( cos (5 /6) + i sin (5 /6) ) ;

1 + i = ( cos ( /4) + i sin ( /4) ) .

Поэтому (1i) + i) =2 (cos (5 /3) + i sin (5 /3) ) 2 ( cos (5 /6) + i sin (5 /6) ) =

= 4 ( cos (5 /2) + ism (5 /2)) ;

(1 i)( + i) = 4 ( cos (5 /2) + i sin (52) =

1+i (cos ( /4) + sin(/4))

= 2 ( cos ( /4) + i sin ( /4) ) = 2 ( (-/2/2) + i (/2) ) = 2(l+i)=2+2i.

Ответ: 2+2i.

Пример 4. Пользуясь корнями третьей степени из 1, вычислить.

Решение.

Известно, что все значения корня n-й степени из комплексного числа z можно получить, умножая одно из них на все значения корня n-степени из числа 1.

Одно из значений можно найти непосредственно. Оно равно 2i, так как= 8i .

Найдем теперь все значения :=

Где k принимает значения 0, 1 и 2.

Следовательно,

Таким образом, получаем три значения для :


Ответ: ;

Пример 5. Используя тригонометрическую форму комплексного числа, произведите указанные действия: .

Решение:

Представим числа и в тригонометрической форме.

1) , где тогда

Находим значение главного аргумента :


Подставим значения и в выражение , получим

2) , где тогда

Тогда

3) Найдем частное



Далее, применяя формулу (9) получим:


Полагая k=0, 1, 2, получим три различных значения искомого корня:

Если , то

если , то

если , то .

Ответ: :

:

: .

Пример 6. Вычислите: а) ()4 , если ,

б) , если , ,в) Перевести комплексное

число z = r() из тригонометрической формы в алгебраическую z = a + bi(п.а,б) .

Решение: а)Так как тригонометрическая форма комплексного числа имеет вид , тогда: в комплексном числе : .

Тогда

, поэтому



б) Так как тригонометрическая форма комплексного числа имеет вид , тогда: в комплексном числе : .

Тогда

, поэтому

,


.

в)

.

Ответ: а) б)

в)

2)Задание:


а) z = 2i ,б) z = ,

а) z = 3i ,б) z = ,

2 Вычислите:

а) ()8 , если

+ ()

б) ()12 , если

,

а) ()6 , если

+cos()

б) ()12 , если

,

2 Найдите значение модуля комплексного числа z, если

а) z = + i,

б) z = 1 + i,

в) z = 16i ,

г) z = () + i ().

а) z = 1 + i,

б) z = 1 – i,

в) z = 12i ,

г) z = () + i ().

3 Найдите тригонометрическую форму комплексного числа

а).

б) .

а)

б) .

5 Используя тригонометрическую форму комплексного числа, произведите указанные действия:

.


6 Перевести комплексное число z = r() из тригонометрической формы в алгебраическую z = a + bi .

а) ,

б))

в)

а) ,

б))

в)

Инструкционная карта

ПР № 8 «Выполнение действий над комплексными числами в показательной форме».

Цель: формировать умение выполнения арифметических операций с комплексными числами, представленных в показательной форме.

Теоретическая часть. Лекции.

Показательной формой комплексного числа называется форма hello_html_m336e7ddb.gif, где. Действия над комплексными числами в показательной форме выполняются по правилам действий со степенями: ,, ,, .

Практическая часть. 1) Опорный конспект.

Пример 1. Запишите в показательной форме следующие комплексные числа:

а) z = + i, б) z = 1 + i,в) z = 6i ,г) z = 3.

Решение: а) z = + i,

Так как тригонометрическая форма комплексного числа имеет вид , тогда: в комплексном числе : . Тогда

, поэтому ,


б) z = 1 + i,

В комплексном числе : . Тогда

, поэтому ,


в) z = 6i ,

В комплексном числе : . Тогда

, поэтому ,

г) z = 3. В комплексном числе : . Тогда

, поэтому ,

Ответ: а) , б) ,в) ,г) .

Пример 2. Вычислите z1z2, z1 / z2, z14, , если z1 = – 1 + i, z2 = ,

Решение:

,

z1z2 =

z1 / z2 =

z14 =

.

Ответ: z1z2 = z1 / z2 = z14 = .

Пример 3. Найдите показательную форму комплексного числа.

Решение:

Ответ:

Пример 4. Используя показательную форму комплексного числа, произведите указанные действия: .

Решение: . В комплексном числе : . Тогда , поэтому

z1 / z2 =

2)Задание:

а) z = + i,

б) z = 1 + i,

в) z = 16i ,

г) z = 5.

а) z = 1 + i,

б) z = 1 – i,

в) z = 12i ,

г) z = 7.

2 Вычислите z1z2, z1 / z2, z14, .

а) z1 = 2 – 2i, z2 = ,

б) z1 = 1 + i, z2 = ,

в) z1 = 2, z2 = ,

г) z1 = 5i, z2 = ,

а) z1 = 2 +2i, z2 = ,

б) z1 = 1 –i, z2 = ,

в) z1 = 4, z2 = ,

г) z1 = 3i, z2 = ,

3 Перевести комплексное число z = r() из тригонометрической формы в показательную z = r.

а) ,

б))

в)

а) ,

б))

в)

4 Найдите в показательной форме , если

.

.

5 Найдите показательную форму комплексного числа

а).

б) .

а)

б) .

6 Используя показательную форму комплексного числа, произведите указанные действия:

.


Инструкционная карта

ПР № 9 «Решение простейших задач на определение вероятности».

Цель: вычисление вероятностей событий по классической формуле определения вероятности, развитие самостоятельной мыслительной деятельности, вычислительных навыков, творческого мышления студентов.

Теоретическая часть. Лекции.

Практическая часть. 1) Опорный конспект.

Пример 1.

a)В партии из 100 деталей имеется 5 бракованных. Определить вероятность того, что, взятая наугад, деталь окажется стандартной.

Решение:

А: взятая наугад деталь оказалась стандартной.

Число исходов, благоприятствующих наступлению события А, равно 95.Поэтому вероятность события равна P(A) = m/ n = 95/100 = 0,95 .hello_html_m27618eb7.gif

Ответ: 0,95.

б) Из пяти букв разрезной азбуки составлено слово «книга». Ребенок, не умеющий читать, рассыпал эти буквы, а затем собрал их в произвольном порядке. Найти вероятность того, что у него снова получится слово «книга».

Решение:

А: из рассыпанных букв сложится слово «книга»

Число всех возможных исходов равно n = Pn = 5! = 120.

Число исходов, благоприятствующих событию А равно m =1.

Вероятность события А равна P(A) = m/ n = 1/120 = 0,0083 .hello_html_m27618eb7.gif 

Ответ: 0,0083.

Пример 2.

a) В коробке лежат 8 зеленых, 7 синих и 15 красных карандашей. Вычислить вероятность того, что взятый наугад карандаш будет, синим или зеленым.

Решение:

А: взяли синий карандаш, В: взяли зеленый карандаш, С: взяли синий или зеленый карандаш. Событие С равно сумме событий А и В: С = А + В

Вероятность события А равна P(A) = m/ n = 7/30. 

Вероятность события В равна P(B) = m/ n = 8/30. 

Вероятность события С равна P(C) = P(A) = 7/30 8/30 = 15/30 = 0,5.

Ответ: 0,5. б) В урне лежат шары, двузначные номера которых составлены из цифр 1,2,3,4,5. Какова вероятность вынуть шар с номером 15? Решение:

А: вынут шар с номером 15.

Число всех возможных исходов равно n =

Число исходов, благоприятствующих событию А, m = 1.

Вероятность события А равна P(A) = m/ n = 1/20 = 0,05 .

Ответ: 0,05.

Пример 3.

a) Набирая номер телефона, абонент забыл последние две цифры и, помня лишь, что эти цифры различны, набрал их наугад. Найти вероятность того, что набраны нужные цифры.

Решение:

А: абонент наугад набрал нужные цифры.

Число всех возможных исходов равно n =

Число исходов, благоприятствующих событию А, m = 1

Вероятность события А равна P(A) = m/ n = 1/90 = 0,011. 

Ответ: 0,011.

б) Устройство содержит два независимо работающих элемента. Вероятности отказа элементов равны соответственно 0,05 и 0,08. Найти вероятности отказа устройства, если для этого достаточно, чтобы отказал хотя бы один элемент. 

Решение:

Пусть событие А — «устройство не работает», В1 — «отказал первый элемент», 

В2 — « отказал второй элемент». Событие А соответствует тому, что может отказать один из «цементов либо оба элемента. События  В1 и В2  независимы в совокупности, поэтому:

q1 = 10,05 = 0,95,   q2 = 10,08 = 0,92.

 P(A) = 1 q1q2= 10,950,92 = 10,874 = 0,126.

Ответ:  0,126.

Пример 4.

a)Вероятность хотя бы одного попадания в цель при четырех выстрелах равна 0,9984. Найти вероятность попадания в цель при одном выстреле.

Решение:

 Пусть p - вероятность попадания в цель при одном выстреле. Введем событие 

X = {при четырех выстрелах есть хотя бы одно попадание} и противоположное ему событие  

= {при четырех выстрелах нет ни одного попадания}.

Вероятность события  равна P(  ) = (1p)4, тогда вероятность события Х равна 

P(X) =1P(  ) = 1 (1p)4. По условию эта вероятность равна 0,9984, откуда получаем уравнение относительно p: 1 (1p)4 = 0,9984, (1p)4 = 0,0016, (1p) = 0,2, p = 0,8.

Таким образом, вероятность попадания в цель при одном выстреле равна 0,8.

Ответ: 0,8.

б)На полке в случайном порядке расставлено 40 книг, среди которых находится трехтомник Пушкина. Найти вероятность того, что эти тома стоят в порядке возрастания номера слева направо, но не обязательно рядом.

Решение: 

Используем классическое определение вероятности: P = m/n, где n- число всех равновозможных элементарных исходов, m - число элементарных исходов, благоприятствующих осуществлению события A = (Тома стоят в порядке возрастания номера слева направо, но не обязательно рядом),n=403938=59280, так как первый том можно поставить на любое из 40 мест, второй - на любое из 39 мест и третий - на любое из оставшихся 38 мест. А число

m= 40! / (37! 3!) = (403938) : (123) = 9880.

Тогда искомая вероятность P(A)= m/n = 9880/59280 = 1/6.
Ответ: 1/6.

Пример 5.

a)В коробке имеется 250 лампочек, из них 100 по 90Вт, 50 - по 60Вт, 50 - по 25Вт и 50 – по 15Вт. Определить вероятность того, что мощность любой наугад взятой лампочки не превысит 60Вт.

Решение:

1. Рассматриваем следующие события: А = {мощность лампочки равна 90Вт}, вероятность Р(А) = 100/250 = 0,4; В = {мощность лампочки равна 60Вт}; С = {мощность лампочки равна 25Вт}; D = {мощность лампочки равна 15Вт}.

2. События А, В, С, D образуют полную систему, так как все они несовместны и одно из них обязательно наступит в данном опыте (выборе лампочки). Вероятность наступления одного из них есть достоверное событие, тогда Р (А)Р (В)Р (С)Р (D) = 1.

3. События {мощность лампочки не более 60Вт} (т.е. меньше или равна 60Вт), и {мощность лампочки более 60Вт} (в данном случае – 90Вт) являются противоположными. По свойству противоположных чисел Р (В)Р (С)Р (D) = 1Р (А).

4. Учитывая, что Р (В)Р (С)Р (D) = Р (ВСD), получим

Р (В СD) = 1Р (А) = 10,4 = 0,6.

Ответ: 0,6. 

б) Вероятность поражения цели первым стрелком при одном выстреле равна 0,7, а вторым стрелком – 0,9. Найти вероятность того, что 

1) цель будет поражена только одним стрелком;

2) цель будет поражена хотя бы одним стрелком.

Решение:

1. Рассматриваем следующие события:
А
1 = {первый стрелок поражает цель}, Р (А1) = 0,7 из условия задачи;
= {первый стрелок промахнулся}, при этом Р (А
1)Р (А̄1) = 1, поскольку А1 и А̄1 – противоположные события. Отсюда Р () = 10,7 = 0,3;
А
2 = {второй стрелок поражает цель}, Р (А2) = 0,9 из условия задачи;
= {второй стрелок промахнулся}, при этом Р () = 10,9 = 0,1.

2. Событие А={цель поражена только одним стрелком} означает, что наступило одно из двух несовместных событий: либо А1, либо А2.
По правилу сложения вероятностей Р (А) = Р (А1) + Р (А2).По правилу умножения вероятностей независимых событий:
Р (А1) = Р (А1)Р () = 0,70,1= 0,07;
Р (А2) = Р ()Р (А2) = 0,30,9 = 0,27.
Тогда Р (А)= Р (А1) Р (А2) = 0,070,27 = 0,34.

3. Событие B ={цель поражена хотя бы одним стрелком} означает, что либо цель поразил первый стрелок, либо цель поразил второй стрелок, либо цель поразили оба стрелка.

Событие B̄ = {цель не поражена ни одним стрелком} является противоположным событию В, а значит Р(В) = 1Р ().
Событие означает одновременное появление независимых событий и , следовательно

Р () = Р () = Р ()Р () = 0,30,1 = 0,03.
Тогда Р (В) = 1Р () = 10,03 = 0,97.

Ответ: 1) 0,34; 2) 0,97.

2)Задание:

1 вариант.

  1. a)В партии из 100 деталей имеется 3 бракованных. Определить вероятность того, что, взятая наугад, деталь окажется стандартной.

б) Из 4 букв разрезной азбуки составлено слово «мама». Ребенок, не умеющий читать, рассыпал эти буквы, а затем собрал их в произвольном порядке. Найти вероятность того, что у него снова получится слово «мама».

  1. a)В коробке лежат 5 зеленых, 3 синих и 12 красных карандашей. Вычислить вероятность того, что взятый наугад карандаш будет, синим или зеленым.

б) В урне лежат шары, двузначные номера которых составлены из цифр 1,2,3,4. Какова вероятность вынуть шар с номером 123?

  1. a)Набирая номер телефона, абонент забыл последние 3 цифры и, помня лишь, что эти цифры различны, набрал их наугад. Найти вероятность того, что набраны нужные цифры.

б) Устройство содержит два независимо работающих элемента. Вероятности отказа элементов равны соответственно 0,04 и 0,09. Найти вероятности отказа устройства, если для этого достаточно, чтобы отказал хотя бы один элемент. 

  1. a)Вероятность хотя бы одного попадания в цель при четырех выстрелах равна 0,9919. Найти вероятность попадания в цель при одном выстреле.

б)На полке в случайном порядке расставлено 21 книга, среди которых находится трехтомник Пушкина. Найти вероятность того, что эти тома стоят в порядке возрастания номера слева направо, но не обязательно рядом.

  1. a)В коробке имеется 200 лампочек, из них 60 по 90Вт, 60 - по 60Вт, 40 - по 25Вт и 40 – по 15Вт. Определить вероятность того, что мощность любой наугад взятой лампочки не превысит 60Вт.

б) Вероятность поражения цели первым стрелком при одном выстреле равна 0,4, а вторым стрелком – 0,7. Найти вероятность того, что 

1) цель будет поражена только одним стрелком;

2) цель будет поражена хотя бы одним стрелком.

2 вариант.

  1. a)В партии из 100 деталей имеется 6 бракованных. Определить вероятность того, что, взятая наугад, деталь окажется стандартной.

б) Из 3 букв разрезной азбуки составлено слово «сон». Ребенок, не умеющий читать, рассыпал эти буквы, а затем собрал их в произвольном порядке. Найти вероятность того, что у него снова получится слово «сон».


  1. a)В коробке лежат 7 зеленых, 2 синих и 11 красных карандашей. Вычислить вероятность того, что взятый наугад карандаш будет, синим или зеленым.

б) В урне лежат шары, двузначные номера которых составлены из цифр 1,2,3,4. Какова вероятность вынуть шар с номером 42?

  1. a)Набирая номер телефона, абонент забыл последние 4 цифры и, помня лишь, что эти цифры различны, набрал их наугад. Найти вероятность того, что набраны нужные цифры.

б) Устройство содержит два независимо работающих элемента. Вероятности отказа элементов равны соответственно 0,03 и 0,07. Найти вероятности отказа устройства, если для этого достаточно, чтобы отказал хотя бы один элемент. 

  1. a)Вероятность хотя бы одного попадания в цель при четырех выстрелах равна 0,9744. Найти вероятность попадания в цель при одном выстреле.

б)На полке в случайном порядке расставлено 34 книга, среди которых находится трехтомник Пушкина. Найти вероятность того, что эти тома стоят в порядке возрастания номера слева направо, но не обязательно рядом.

  1. a)В коробке имеется 400 лампочек, из них 280 по 90Вт,40 - по 60Вт, 40 - по 25Вт и 40 – по 15Вт. Определить вероятность того, что мощность любой наугад взятой лампочки не превысит 60Вт.

б) Вероятность поражения цели первым стрелком при одном выстреле равна 0,5, а вторым стрелком – 0,8. Найти вероятность того, что 

1) цель будет поражена только одним стрелком;

2) цель будет поражена хотя бы одним стрелком.
























Инструкционная карта

ПР № 10 «Вычисление производных функций».

Цель:

  • повторить понятия: производная степенной функции, правила вычисления производных суммы, разности, произведения, частного, сложных функций, правила вычисления производных элементарных функций;

  • развитие умений и навыков работы с источником информации, с практическим материалом.

Теоретическая часть.
Ш.А. Алимов. Алгебра и начала математического анализа 10-11 класс, глава 8-9.

Практическая часть.

1)Опорный конспект.
Пример 1. Найти производную функции  y = 3x2 5.

Решение:

Для нахождения производной данной функции используем правила дифференцирования и таблицу производных. Так как производная суммы/разности равна сумме ,разности производных, то

y / = (3x2 5) / = (3x2) / ( 5) / = 3 2x 5 4 (x - 3) / = = 6x 12 = 6x . Ответ: 6x .

Пример 2. Найти производную функции  y = 8x5 6x4 5x3 7x2 4x 3.

Решение:

y / = (8x5 6x4 5x3 7x2 4x 3)/ = 8 5x4 6 4x3 5 3x2 7 2x 4 = = 40x4 24x3 15x2 14x 4.

Ответ: 40x4 24x3 15x2 14x 4.

Пример 3. Найти производную функции 

Решение: y / = () / = = x x2 x3.

Ответ: x x2 x3.

Пример 4. Найти производную функции  .

Решение:

y / = () / = = x 4 = x .

Ответ: x .

Пример 5. а) Найти производную функции  y = 5x3 3 .

Решение:

Преобразуем каждое слагаемое данной функции в степенную форму:

y = 5x3 3 .

Применяя линейные свойства производной и правило дифференцирования степенной функции, получаем:

hello_html_db8932d.png

б) Найти производную функции  y = .

Решение: По правилу дифференцирования степенной функции находим:

hello_html_406c558a.png

2)Задание: 1)

  1. Найти производную функции  y = 7x2 6.

  2. Найти производную функции  y = 4x5 8x4 2x3 9x2 3x 8.

  3. Найти производную функции 

  4. Найти производную функции  .

  5. а) Найти производную функции  y = 7x3 5 .

б) Найти производную функции  y = .

2)Тест: ЧастьА.

А1. Найдите производную функции y = ex x7 . А2. Найдите производную функции у = ехsinx.

1) = ех + cosx; 2) = ех - cosx; 3) = ½ е2x cosx; 4) = е2x cosx.

А3. Вычислите значение производной функции у=3ех+cos2x в точке хо=0. 1) 3; 2) -1; 3) 1; 4)2. А4. Вычислите значение производной функции у= в точке хо=2. 1) 11,5; 2)10,5; 3) 11; 4) 9,5.

А5. Вычислить значение производной функции у=ех sinx + x2 в точке xo=0. 1) 0; 2) 1; 3) 2; 4) 3. А6. Вычислите значение производной функции у = cos2x + 4x в точке хо=. 1) 2; 2) -2; 3) 4; 4) 0. А7. Вычислите значение производной функции у= ln2x в точке хо = 2. 1) 3; 2) 4; 3) 2; 4) 1. А8. Вычислите значение производной функции у= 3+ 25x2 – 24x +23 в точке хо = 1. 1) 15; 2)11; 3) 17; 4) 9. А9. Найдите производную функции . 1) ; 2) ; 3) ; 4) . А10. Вычислите значение производной функции в точке хо= . 1) 2; 2) 4; 3) -2; 4)1/2.

Часть В. В1.Найдите производную функции:

1) 2) ; 3) ; 4) ;

В2. а) К графику функции проведена касательная через точку с абсциссой . Вычислите тангенс угла наклона этой касательной к оси абсцисс.

б) Напишите уравнение касательной к кривой в точке с абсциссой х = 0.

В3. а) Найдите наибольшее и наименьшее значения функции на

отрезке .

б) Площадь прямоугольника равна 81 см2. Найдите наименьший возможный периметр этого прямоугольника.

В4.Найдите область определения, промежутки монотонности, точки экстремума, экстремумы функции:

В5.Найдите корень (или сумму корней, если их несколько) уравнения f ' (x) = 0, если
f(x) = 4x + +8/x.


Инструкционная карта

ПР № 11 «Вычисление неопределенных интегралов».

Цель:

  • Повторить знания о первообразной, таблицу интегралов.

  • Закрепить навыки нахождения табличных интегралов.

  • Развивать логическое мышление, память, внимание и самостоятельность.

Теоретическая часть. Лекции.

Практическая часть. 1) Опорный конспект.

Пример 1. Найти неопределённый интеграл, использую таблицу интегралов.

а)б)

Решение: ,

Ответ: ,

Пример 2. Найти неопределённый интеграл, использую таблицу интегралов.

а)б)

Решение: а) б)

Ответ: а) б)

Пример 3. Найти неопределённый интеграл методом подстановки.

а)б)в)

Решение:

а)б)

в)

Ответ: а)б)

в) .

Пример 4. Найти неопределённый интеграл методом подстановки.

а) ,б) .

Решение: а) , dz = 2x dx, ,

б) , dz = 5ex dx,

Ответ: а) ,б)

2)Задание:

  1. Найти неопределённый интеграл, использую таблицу интегралов.

  1. Найти неопределённый интеграл, использую таблицу интегралов.

2.11


2.2

2.7


2.12


2.3


2.8

2.13

2.4

2.9

2.14


2.5

2.10

2.15


  1. Найти неопределённый интеграл методом подстановки.

3.11


3.2


3.7

3.12

3.3

3.8


3.13

3.4

3.9

3.14

3.5


3.10

3.15


  1. Найти неопределённый интеграл методом подстановки.


4.1 , 4.2 , ,


4.3 , , 4.4 , 4.5 ,.



Инструкционная карта

ПР № 12 «Вычисление определенных интегралов».

Цель:

  • Повторить знания о первообразной, таблицу интегралов.

  • Овладеть умением применения первообразной функции при решении вычислительных задач.

  • Закрепить навыки нахождения табличных интегралов.

  • Развивать логическое мышление, память, внимание и самостоятельность.

  • проверить уровень усвоения обучающихся предыдущих знаний по темам: первообразная и определённый интеграл.

Теоретическая часть. Лекции.

Практическая часть. 1) Опорный конспект.

Пример 1. Вычислить интеграл :

a) , б) ,в) ,г) .

Решение: a) б)

в)

г)

Ответ: a) б) в) г)

Пример 2. Найдите площадь фигуры, ограниченной линиями:

а) параболой , прямыми x =2, x = 1 и осью ОX,

б) параболой и осью ОX.

Решение: а) параболой , прямыми x =2, x = 1 и осью ОX,


,

б) параболой и осью ОX.



Ответ: а) ,б) hello_html_m1e7af1bf.jpg

Пример 3. Найдите площадь фигуры, ограниченной линиями:

и

( по рис.).

Решение: Найдем пределы интегрирования:


Ответ:21,3.

Пример 4. Вычислите площадь заштрихованной фигуры.

 hello_html_271fed43.jpg

Решение: Ответ: 2.

2)Задание:

а) Вычислить интеграл :

1) , 2) , 3) , 4),

5) , 6) , 7) , 8) ,

9) , 10),11) .

б) Найдите площадь фигуры, ограниченной линиями:

  1. параболой , прямыми x = 1, x = 1 и осью ОX.

  2. графиком функции y = 2x +3 , прямыми x = 1, x = 2, осью OX.

  3. параболой и осью ОX.

  4. параболой и прямой y = 2x.

  5. параболой и осью ОX.

  6. параболой , прямыми x = 1, x = 2 и осью ОX.

  7. параболой и прямыми x = 0 и x = 2 и осью ОX.

  8. параболой и осью ОX.

  9. графиком функции y = 2x , прямыми x = 1, x = 4, осью OX.

  10. Найти площадь фигуры, изображенной на заданном рисунке:

1) 2) 3) 4)

hello_html_26eeeb0c.gifhello_html_m15882a6d.gifhello_html_241fddae.gifhello_html_m3ed3576e.gif


Инструкционная карта

ПР №13 «Исследование функций и построение графика с помощью производной».

Цель: научиться применять производную для исследований функций на монотонность и экстремумы, находить асимптоты, строить графики.

Теоретическая часть. Лекции.

Практическая часть. 1) Опорный конспект.

Пример 1. Исследовать и построить график функции

Решение:

1) Функция определена и непрерывна на всей числовой прямой: D (f) = R.

значит,  данная функция является нечетной, её график симметричен относительно начала координат.Очевидно, что функция непериодическая.

2) Асимптоты, поведение функции на бесконечности.

Так как функция непрерывна на R, то вертикальные асимптоты отсутствуют

 

Прямая y=0   (ось ox) является горизонтальной асимптотой графика  при  .

Из непрерывности на R и существования горизонтальной асимптоты следует тот факт, что функция ограничена сверху и ограничена снизу.

3) Точки пересечения графика с координатными осями, интервалы знакопостоянства.

График   проходит через начало координат.

Других точек пересечения с координатными осями нет. Более того, интервалы знакопостоянства очевидны, и ось можно не чертить:  > 0, а значит, знак функции зависит только от «икса»: > 0 , если x > 0; < 0, если x < 0.

4) Возрастание, убывание, экстремумы функции.

 
 – критические точки.
Определим знаки производной:
hello_html_m632d655f.jpg
Функция возрастает на интервале(-
;)  и убывает на интервалах  

В точке   функция достигает максимума:  .

В точке   функция достигает минимумa:

Поскольку функция убывает на интервале , то, очевидно, на «минус бесконечности» график расположен под своей асимптотой. На интервале  функция тоже убывает, но здесь всё наоборот – после перехода через точку максимума  линия приближается к оси ox уже сверху. Из вышесказанного также следует, что график функции является выпуклым на «минус бесконечности» и вогнутым на «плюс бесконечности».

После этого пункта исследования прорисовалась и область значений функции:

5) Выпуклость, вогнутость, перегибы графика.


 – критические точки.

Определим знаки :
hello_html_m17ac4cb6.jpg
График функции является выпуклым на
  и вогнутым на .

Во всех критических точках существуют перегибы графика. Найдём ординаты точек перегиба, при этом снова сократим количество вычислений, используя нечётность функции: hello_html_60b1fc94.png

6) Дополнительные точки целесообразно рассчитать только для правой полуплоскости:

x

0,5

1

1,5

2

f(x)

0,39

0,37

0,16

0,04

Пример 2. Исследовать функцию и построить график

Решение:

  1. Функция определена и непрерывна на всей числовой прямой, D (f) = R .

, значит, данная функция не является четной или нечетной. Функция непериодическая.

  1. Асимптоты графика, поведение функции на бесконечности.
    Так как функция непрерывна на R, то вертикальные асимптоты отсутствуют.

, значит,  наклонные асимптоты также отсутствуют.
, функция не ограничена снизу.

3) Точки пересечения графика с координатными осями, интервалы знакопостоянства функции.
График f(x) проходит через начало координат.
С осью ox:  
Определим знаки f(x)  :
hello_html_m6eff383d.jpg
f(x) > 0, если , f(x) <0,, если .

4) Возрастание, убывание, экстремумы функции.

  – критические точки.
Определим знаки:
hello_html_m337d988e.jpg
f(x)  возрастает на  и убывает на .
В точке x=3 функция достигает максимума:  

  1. Выпуклость, вогнутость, перегибы графика.

  – критические точки. hello_html_7058cb7b.png

Определим знаки :
hello_html_m15d97516.jpg
График функции является выпуклым на  и вогнутым на . 
В обеих критических точках существуют перегибы графика.

  1. Найдем дополнительные точки:

x

-1,5

-1

1

3,5

4,5

f(x)

-4,6

-1,3

0,7

5,4

-11,4

Пример 3. Исследовать функцию и построить график
Решение:

  1. Функция определена по всюду кроме точки в которой знаменатель превращается

 в ноль (x = 1). Область определения состоит из двух интервалов D (y) : .

2) При подстановке значения x = 0  получим

Такую же точку получим если приравняем функцию к нулю. Точка x = 0  - единственная точка пересечения с осями координат.

3) Проверяем функцию на четность

Итак функция ни четная, ни нечетная, непериодическая.

4) В данном случае имеем одну точку разрыва x = 1   . Вычислим границы слева и справа от этой точки


Итак x = 1    – точка разрыва второго рода.

5) Для отыскания интервалов монотонности вычисляем первую производную функции


Приравнивая ее к нулю получим точки подозрительные на экстремум x = 0  , x = 2   . Они разбивают область определения на следующие интервалы монотонности .

Исследуем поведение производной слева и справа от найденных точек разбиения

> 0, < 0, < 0,

> 0.hello_html_m225fa59e.gif

Графически интервалы монотонности будут иметь вид


Исследуемая функция возрастает на интервалах  и убывает .

Точка x = 0   – точка локального максимума, x = 2  – локального минимума. Найдем значение функции


6) Для отыскания интервалов выпуклости найдем вторую производную


Таких интервалов нет, поскольку вторая производная не принимает нулевых значений в области определения.

7) Точка x = 1    – вертикальная асимптота функции. Уравнение наклонной асимптоты  имеет вид

y = kx + b где k,b - границы которые вычисляются по правилу

,

Находим нужные границы


Конечный вид прямой следующийhello_html_m48166f3d.png

8) На основе проведенного анализа выполняем построение графика функции. Для этого сначала строим вертикальные и наклонные асимптоты, затем находим значение функции в нескольких точках и по них проводим построение.

2)Задание:

а) Исследовать функции и построить их графики.

  1. .

  2. .

б) Найти асимптоты графика функции

1. , 2. , 3. ,4. .

5. ,6. , 7.
, 8.


Инструкционная карта

ПР №14 «Вычисление площади плоских фигур с помощью определённого интеграла».

Цель:

  • Повторить знания о первообразной, таблицу интегралов.

  • Овладеть умением применения первообразной функции при решении вычислительных задач.

  • Закрепить навыки нахождения табличных интегралов.

Теоретическая часть. Лекции.

Практическая часть.

1)Опорный конспект.

Пример 1. а)Вычислить площадь фигуры, ограниченной линиями у = х2 2, у = 0, х = 2, х = 1.hello_html_m6ea081fc.png

Решение: Выполним чертеж (обратите внимание, что уравнение у = 0  задает ось  ОХ): Штриховать криволинейную трапецию я не буду, здесь очевидно, о какой площади идет речь. Решение продолжается так:

На отрезке[– 2;1]    график функции у = х2 2  расположен над осью ОХ, поэтому:


Ответ: S = 9 eд2.

б)Вычислить площадь фигуры, ограниченной линиями у =  , х = 1  и координатными осями.

Решение: Выполним чертеж: Если криволинейная трапеция расположена под осью OX (или, по крайней мере, не выше данной оси), то её площадь можно найти по формуле: S =  .
В данном случае:
hello_html_m521974c9.png

Ответ: 

Пример 2.а)Найти площадь плоской фигуры, ограниченной линиями  у = 2х , у =  .hello_html_m34e0aa0e.png

Решение: Сначала нужно выполнить чертеж. Найдем точки пересечения параболы у = 2х   и

прямой у =   . Решаем уравнение: =  , 3х = 0, х(3) = 0,

х1 = …, х2 = ...

Значит, нижний предел интегрирования а = 0, верхний предел интегрирования b = 3 . x = a ,x = b , можно найти по формуле: S = .

Искомая фигура ограничена параболой y = 2х   сверху и прямой у =    снизу.
На отрезке
[0;3]  2х  , по соответствующей формуле

Ответ: S = 4,5 eд2.  

б)Вычислить площадь фигуры, ограниченной линиями, y = x  , y = 0  , x = 3 .

Решение: Сначала выполним чертеж: Площадь фигуры считается с помощью двух определенных интегралов. Действительно:hello_html_m2509795.jpg

1) На отрезке [– 1;1]  над осью OX расположен график прямой y = x   ;

2) На отрезке [1;3]   над осью OX  расположен график гиперболы

Совершенно очевидно, что площади можно (и нужно) приплюсовать, поэтому:

Ответ: 

Пример 3.a) Вычислить площадь фигуры, ограниченной линиями

 ,2x  .hello_html_dfee4aa.png

Решение: Представим уравнения в виде и выполним поточечный чертеж:
Из чертежа видно, что верхний предел у нас «хороший»:
  b = 1.
Найдем точки пересечения прямой
    и параболы
Для этого решаем уравнение:
3x2 = 2x 3x2 2x

D = 4 12 = …, = 4, x1 = , x2 = ... Действительно,a = .

На отрезке по соответствующей формуле: Ответ: .hello_html_3f5273e5.png

б)Найти площадь фигуры, ограниченной линиями y =  , y = 2x  .

Решение: Выполним чертеж:
На отрезке по соответствующей формуле:


Ответ: S = 10 eд2. hello_html_m2e7c3126.png

Пример 4.Найти площадь фигуры, ограниченной линиями x , xy = 3 .

Решение: Выполним чертеж . На отрезке  , по соответствующей формуле:
Ответ:  .

Пример 5.a) Найти площадь фигуры, ограниченной параболой у = х2 +10 и касательными к этой параболе, проведёнными из точки (0;1). 

Решение: hello_html_78b890fb.jpg

Неизвестна абсцисса точки касания х = а. Чтобы её найти, составим уравнение касательной:  y = f (x0) .

Имеем f(x) = x2 f (x) = 2x;значит, f(a) = a2 f (a) = 2a; уравнение касательной имеет вид:

y = a2 2 a(x ) = a2 2 ax ;

Уравнение касательной y = (1)

По условию касательная должна проходить через точку (0;1), то есть координаты точки (0;1) должны удовлетворять уравнению (1):

1 = 2a0 ; , a1 = a2 = ...

Подставим найденные значения в уравнение (1):

Если a =  то y = 9 10

Если a = 3 , то y =  .

Получили два уравнения касательных y =  

Параболы y = х2 + 10 они касаются в точках А(-3;19) и В(3;19).

Найдём площадь фигуры DACB: SDACB = 2SDCB , hello_html_f9ffd32.gif





hello_html_35881468.jpg

SDACB = 2 9 = ...

Ответ: 18.

б) Вычислить площадь фигуры, ограниченной линиями

у = 4/x, y = х, х = 4.

Решение: SABC = SMBAD SMBCD;

SMBAD = 1/2(MB ) MD = = 1/2 (2 ) 2 = 6;




hello_html_450e27c6.gif


Ответ: 6 – 4ln2.

2)Задание: 1 вариант.

  1. а)Вычислить площадь фигуры, ограниченной линиями у = х2 2, у = 0, х = 3, х = 3.

б)Вычислить площадь фигуры, ограниченной линиями у =  , х = 2  и координатными осями.

  1. а)Найти площадь плоской фигуры, ограниченной линиями  у = 4х , у =  .

б)Вычислить площадь фигуры, ограниченной линиями , y = x  , y = 0  , x = 4 .

  1. a) Вычислить площадь фигуры, ограниченной линиями  , 5x  .

б)Найти площадь фигуры, ограниченной линиями y =  , y = 2x  .

  1. a)Найти площадь фигуры, ограниченной линиями x , xy = 5 .

б) В каком отношении парабола у = х2 3 делит площадь четырёхугольника, вершины которого находятся в точках с координатами (0;0); (2;0); (0;6); (2;6)? 

  1. a)Найти площадь фигуры, ограниченной параболой у = 3х2 4 и касательными к этой параболе, проведёнными из точки (0;1). 

б) Вычислить площадь фигуры, ограниченной линиями у = 4/x, y = х, х = 6.


2 вариант.

  1. a)Вычислить площадь фигуры, ограниченной линиями у = 3х2 4, у = 0, х = 1, х = 1.

б)Вычислить площадь фигуры, ограниченной линиями у =  , х = 3  и координатными осями.

  1. а)Найти площадь плоской фигуры, ограниченной линиями  у = 6х , у =  .

б)Вычислить площадь фигуры, ограниченной линиями , y = x  , y = 0  , x = 4 .

  1. a)Вычислить площадь фигуры, ограниченной линиями  , 4x  .

б)Найти площадь фигуры, ограниченной линиями y =  , y = 2x  .

  1. a)Найти площадь фигуры, ограниченной линиями x , xy = 7 .

б) В каком отношении парабола у = х2 4 делит площадь четырёхугольника, вершины которого находятся в точках с координатами (0;0); (2;0); (0;6); (2;6)? 

  1. a)Найти площадь фигуры, ограниченной параболой у = 3х2 13 и касательными к этой параболе, проведёнными из точки (0;1). 

б) Вычислить площадь фигуры, ограниченной линиями у = 4/x, y = х, х = 8.




Просмотрено: 0%
Просмотрено: 0%
Скачать материал
Скачать материал "Практические работы обучающихся по дисциплине Математика для специальности Технология машиностроения."

Получите профессию

Методист-разработчик онлайн-курсов

за 6 месяцев

Пройти курс

Рабочие листы
к вашим урокам

Скачать

Получите профессию

Няня

за 6 месяцев

Пройти курс

Рабочие листы
к вашим урокам

Скачать

Скачать материал

Найдите материал к любому уроку, указав свой предмет (категорию), класс, учебник и тему:

6 662 891 материал в базе

Скачать материал

Другие материалы

Вам будут интересны эти курсы:

Оставьте свой комментарий

Авторизуйтесь, чтобы задавать вопросы.

  • Скачать материал
    • 31.05.2016 8103
    • RAR 8.7 мбайт
    • 18 скачиваний
    • Оцените материал:
  • Настоящий материал опубликован пользователем Зайцева Светлана Егоровна. Инфоурок является информационным посредником и предоставляет пользователям возможность размещать на сайте методические материалы. Всю ответственность за опубликованные материалы, содержащиеся в них сведения, а также за соблюдение авторских прав несут пользователи, загрузившие материал на сайт

    Если Вы считаете, что материал нарушает авторские права либо по каким-то другим причинам должен быть удален с сайта, Вы можете оставить жалобу на материал.

    Удалить материал
  • Автор материала

    Зайцева Светлана Егоровна
    Зайцева Светлана Егоровна
    • На сайте: 8 лет и 2 месяца
    • Подписчики: 0
    • Всего просмотров: 344138
    • Всего материалов: 170

Ваша скидка на курсы

40%
Скидка для нового слушателя. Войдите на сайт, чтобы применить скидку к любому курсу
Курсы со скидкой

Курс профессиональной переподготовки

Интернет-маркетолог

Интернет-маркетолог

500/1000 ч.

Подать заявку О курсе

Курс повышения квалификации

Развитие предметных навыков при подготовке младших школьников к олимпиадам по математике

36 ч. — 144 ч.

от 1700 руб. от 850 руб.
Подать заявку О курсе
  • Сейчас обучается 44 человека из 17 регионов
  • Этот курс уже прошли 101 человек

Курс профессиональной переподготовки

Математика: теория и методика преподавания с применением дистанционных технологий

Учитель математики

300 ч. — 1200 ч.

от 7900 руб. от 3650 руб.
Подать заявку О курсе
  • Сейчас обучается 34 человека из 16 регионов
  • Этот курс уже прошли 42 человека

Курс профессиональной переподготовки

Математика и информатика: теория и методика преподавания в профессиональном образовании

Преподаватель математики и информатики

500/1000 ч.

от 8900 руб. от 4150 руб.
Подать заявку О курсе
  • Сейчас обучается 41 человек из 23 регионов
  • Этот курс уже прошли 53 человека

Мини-курс

Медико-педагогические аспекты обучения и тренировки

2 ч.

780 руб. 390 руб.
Подать заявку О курсе

Мини-курс

Особенности психологической помощи детям

6 ч.

780 руб. 390 руб.
Подать заявку О курсе
  • Сейчас обучается 587 человек из 75 регионов
  • Этот курс уже прошли 227 человек

Мини-курс

Эффективная самоорганизация и планирование

3 ч.

780 руб. 390 руб.
Подать заявку О курсе
  • Сейчас обучается 109 человек из 38 регионов
  • Этот курс уже прошли 45 человек